Download as pdf or txt
Download as pdf or txt
You are on page 1of 1359

Become

An Author or Editor
StatPearls is an academic teaching project developed by thousands of health professionals worldwide. Our
goal is to provide inexpensive, up to date, peer reviewed study questions for all healthcare specialties. These
questions are available in Apps, eBooks, and an online learning system that includes the opportunity to earn
CME/CE.

We need your help improving questions. Please identify questions that need revision by flagging a question.
In addition, please consider becoming an author or editor. In as little as 2-3 hours you can make a
substantial contribution to medical education and you will receive free CME/CE for your contribution!

Please go to www.statpearls.com to sign up.


Learn More
Want to learn more? Please visit our online learning system at www.statpearls.com for additional FREE
questions, color pictures, and more detailed explanations.
Introduction
NOTE: The intent of Stat Pearls review books is to identify knowledge deficits and assist you in the learning
process. Review books are not intended to be a source of the knowledge base of medicine. The authors or
editors do not warrant the information is complete or accurate. The reader is encouraged to verify each
answer in several references.

All drug indications and dosages should be verified before administration.

Copyright 2018 Stat Pearls Publishing, LLC.

All rights reserved, including the right of reproduction, in whole or in part, in any form.

The authors and editors would like to extend special thanks to Gerson Cordero Rubio and Beata Beaty for
their editorial support and Susan Oliver for editorial team management.

http://www.StatPearls.com

Congratulations! This Stat Pearls book will help improve your fund of knowledge. Our books are designed
to help you identify and resolve knowledge deficits. To achieve this goal the text is written in a multiple-
choice format with answers and explanations. Explanations contain additional information intended to
reinforce your knowledge. Emphasis has been placed on covering facts that are easily overlooked, quickly
forgotten, and often show up in specialty reviews. For further information, all explanations are
electronically linked to Stat Pearls (free), eMedicine (free), Google (free), or UpToDate (subscription). With
the exception of Stat Pearls, we have no affiliation with these websites.

We suggest you note all questions answered incorrectly to allow you to rapidly review the content prior to
your exam.

We appreciate your comments, suggestions, and criticisms. Some answers and explanations may be in
variance with your own knowledge. This is usually attributable to variability among sources; however,
please make us aware of any potential errors you find as we update our books often. We appreciate your
input in regard to format, content, or presentation. Please contact us at support@statpearls.com or better yet,
click on the flag button after each question to give us specific input on any errors or omissions. We look
forward to hearing from you.

Good luck with learning your specialty!

Stat Pearls eBooks

Stat Pearls books were created exclusively for use on eBook readers. Because we don't publish our books
in paper format , we have the luxury of optimizing them for eBook use.
How are our books optimized for the viewing on an eBook Reader?

1. Efficient User Interface: Each multiple choice question is on a page by itself so you can see the
question and four multiple choice answers, but you can't see the correct answer until you flip the
page. This would be a complete waste of paper in an ordinary book, but it makes for an easy to use
eBook. You don't have to cover up the correct answer with a piece of paper. Most devices have a
dedicated button for page turning so with the click of a button, you can see the correct answer.
2. Research Links : After each explanation we include web links to search for more information on the
key concepts for that question. If you miss a question, this is when you probably want to do more
research on the question concepts. This would be onerous and require a lot of typing using a paper
book, but is a single click on an eBook reader, PC, or Mac. Our customers have asked for many
search options, and we have chosen the following for " assisted research ":
Stat Pearls Knowledge Base - http://knowledge.statpearls.com
eMedicine (now Medscape/WebMD) - http://emedicine.medscape.com/
UpToDate - http://www.uptodate.com
Google - http://www.google.com/
We have no affiliation with eMedicine, UpToDate, or Google . To look at more than one page you
may have to sign up for WebMD and UpToDate; Google requires no signup.
3. Flag a Question: Medical standards and drug standards change. Also, human errors can occur
resulting in questions that have issues. Stat Pearls strongly believes in Continuous Quality
Improvement (CQI). Flagging a question gives you, the medical professional, the ability to tell us if
something is wrong with a question and we will correct it and get back to you with an updated book
(providing you give us a phone number or email address.) If you don't provide us with contact
information, you can check for a book update using the normal methods from the books store and
you will soon find an update with the question corrected by one of our health professionals.
4. Volume of Questions: When we make a book we publish every question we have on a topic (up to
2,000 per book). This would be ridiculously expensive to do with traditional book publishing, but in
the eBook world it costs no more to publish 2,000 questions than 1,000 questions. You will find that
in most of our specialties, we dwarf the competition with breadth of material. This makes our books
"just what the doctor ordered" for those wishing to go the extra mile when studying.

STAT PEARLS AUTHORS AND EDITORS

Over 5,000 physician, nursing, and allied health professionals authored a database of over 50,000 classified
multiple-choice questions and explanations. Each question has been peer reviewed by two health
professionals and a pharmacist. All questions are reviewed annually and updated through a peer review
process.

WE APPRECIATE YOUR COMMENTS!

We appreciate comments, suggestions, corrections and additional contributing authors and editors. Please e-
mail us at: support@statpearls.com.
Pediatric-Pulmonology Specialty
Review and Self-Assessment
Stat Pearls Book ID:
MD_PEDIATRIC_PULMONOLOGYSPECIALTY_REVIEW_AND_SELF_ASSESSMENT-
2018-03-31
Version: 3/31/2018
Editors in Chief
Barbara Stewart, MD; Assistant Professor; East Tennessee State University;
United States, Johnson, TN, USA

Mary Cataletto, MD; Professor; Winthrop University Hospital; United

States, Brookville, NY, USA


Contributing Editors
Ali Abdulkarim, MBBS; Faculty of Medicine, Cairo University; Cairo,
Egypt

Rotimi Adigun, MBBS, MPH; Associate Professor; Director of Pathology

Program; University of Health Sciences, Antigua; St John's, Antigua &

Deps

Sami Akram, MD, MHA; Southern Illinois University; Springfield, IL,


USA

Faysal Alghoula, MBBch; Creighton University; Omaha, NE, USA

Mohamed Alhajjaj, MBBS, FRCP(C); Professor; Cosultant Pulmonologist;

University of Sharajah; Dubai, United Arab Emirates

Catiele Antunes, MD; Private Practice; Philadelphia, PA, USA

Elisa Aponte, MD, RDMS; Attending Physician; Emergency Ultrasound

Director; Kaweah Delta Medical Center; Visalia, CA, USA


Usha Avva, MD; Division Chief, Pediatric ED; Joseph M. Sanzari

Children's Hospital; Hackensack, NJ, USA


Folusakin Ayoade, MD; Assistant Professor; University of Miami; Miami,

FL, USA
Matthew Ball, BMBS, MSc; NHS; Bristol, United Kingdom
Maria Bandres, MD; Private Practice; Philadelphia, PA, USA

Seth Baruffi, MD; Associate Prof. of Emergency Medicine; Core Teaching


Faculty; Inspira Medical Center; Vineland, NJ, USA

Steve Bhimji, BSci Pharm, MS, MD, PhD; Clinical Instructor (Retired); Un

of Toronto; Toronto, Canada

Kim Bird, MD, FCCP, FAASM; Associate Program Director Internal Med;
Danville Regional Medical Center; Danville, VA, USA

Syed Rizwan Bokhari, BS, MBBS, MD, FCPS; Consultant; Tulane Un, Un
Med Center New Orleans; New Orleans, LA, USA

Sameh Boktor, MD, MPH; Penn State College of Medicine;

Mechanicsburg, PA, USA

Daniel Bouchette, BS; St.Martinus Faculty of Medicine; New Orleans, LA,

USA

Darrell Brackett, BA; Respiratory Director; Continue Care Hospital;

Corbin, KY, USA


Daniel Brito, MD; Internal Medicine; Universidad Central de Venezuela;

Philadelphia, PA, USA

Falon Brown, DO; CUSOM/Sampson Regional Medical Center;


Wilmington, NC, USA

Kristin Brown, MD; University of Tennessee; Nashville, TN, USA


Joshua Bucher, MD; Assistant Professor; Assistant EMS Medical Director;

Rutgers - RWJMS; New Brunswick, NJ, USA


Lauren Burton, MD; UTHSC; Murfreesboro, TN, USA

Lauren Camaione, BS, PharmD; Clinical Pharmacist; Pediatric


Pharmacotherapy Specialist; The Brooklyn Hospital Center; Brooklyn, NY,
USA

Danny Castro, DO, MEd; Assistant Professor; Dir of Fellow Ed, Med Dir

of Resp Care; Baylor College of Med, TX Children Hosp; Houston, TX,


USA

Mary Cataletto, MD; Professor; Winthrop University Hospital; Brookville,


NY, USA

Terry Cates, AS; Manager Respiratory Care; Arkansas Continued Care

Hospital; Jonesboro, AR, USA

Raheel Chaudhry, MD; Ross University School Of Medicine; Johnson City,

CA, USA

Krati Chauhan, MD, MPH; Assistant Professor Rheumatology; Southern

Illinois University; Springfield, IL, USA


Jeffrey Cooper, MD; Associate Professor; Director; University of Nebraska

Medical Center; Omaha, NE, USA

John Costumbrado, MD, MPH; Riverside Community Hospital/UCR EM;


Riverside, CA, USA

Rajeev Dalal, BS; NYIT College of Osteopathic Medicine; York, PA, USA
Maureen Davis, MD; Advocate Christ Medical Center; Chicago, IL, USA

Richard de la Cruz, MD, MS; Prof; Orange Park Medical Center; Green
Cove Springs, FL, USA

Daniela de Lima Corvino, MD; Einstein Medical Center Philadelphia;


Jenkintown, PA, USA
Bradley DeVrieze, MD; Assistant Professor of Medicine; Academic

Hospitalist; Creighton University School of Medicine; Omaha, NE, USA

Christopher Dion, MS; Midwestern University; Phoenix, AZ, USA


Joseph Dougherty, DO; CA Professor: WVSOM, MSUCOM; Program

Director; Ohio Valley Medical center WVSOM OTI; Wheeling, WV, USA
Peter Edemekong, MD, MPH; Faculty; NOVA Southeastern University

College of Osteopathic Medicine; Jupiter, FL, USA

Allen Elster, MD; Louisiana State University Health Sciences Center;

Shreveport, LA, USA

Evelyn Erickson, MD; Attending; Lincoln Medical Center; New York, NY,

USA

Stacey Ernest, MD; Loma Linda Unversity Medical Center; Rancho


Cucamonga, CA, USA

Avram Estes, MD; Private Practice; Chicago, IL, USA

Mario Fadila, MD; University of Missouri; Columbia, MO, USA


Jeffrey Florek, MD; Advocate Christ Medical Center; Munster, IN, USA

Thomas Folino, DO; NYIT College Of Osteopathic Medicine; Brooklyn,


NY, USA

Michaelia Fosses Vuong, DO; Southern Illinois University; Springfield, IL,


USA

Amanda Friend, MD, MS; Advocate Christ Medical Center; Chicago, IL,
USA
Justin Fuehrer, DO; Private Practice; Glen Oaks, NY, USA

Evan Fusco, BS, MD, MHA; Medical Director; Springfield, MO, USA

Veronica Garvia, MD; Einstein Medical Center; Philadelphia, PA, USA


Yana Gelman, MD; University of Illinois COM Peoria; Peoria, IL, USA

Namratta Ghatehorde, BSc, MD; Univeristy of Missouri Columbia;


Columbia, MO, USA

Dulce Gonzalez, MD; Lincoln Hospital; New York, NY, USA

William Gossman, MD; Professor; Creighton University; Elkhorn, NE,

USA

Charles Graham, MD; Clinical Instructor; Mayo Clinic Florida; Neptune

Beach, FL, USA

Paul Gruber, MD; Resident Physician; Saint Louis, MO, USA


Yazan Haddadin, MD, MBBS; Un of Missouri School of Medicine;

Columbia, MO, USA

Wissem Hafsi, MD; University Assistant; Dermatologist and Venereologist;


Faculty of Medicine of Tunis, Tunisia; Bardo, Tunisia

Mary Hanley, DO; Associate Professor; Pro Dir Undersea and Hyperbaric;
Un of N En Coll of Osteo Med; Charleston, SC, USA

Taha Haque, DO; Private Practice; Lorton, VA, USA


Hannah Hays, MD; Assistant Professor; Ohio State University COM;

Delaware, OH, USA


Alan Heffner, MD; Director of Critical Care; Charlotte, NC, USA
Martin Huecker, MD; Assistant Professor; Research Director; University of

Louisville; Louisville, KY, USA

Katie Huynh, PA-C, MS, AAHIVM-S; Private Practice; Philadelphia, PA,


USA

Michael Ibrahim, MD, PhD; Associate Professor; Biochem, Pharma, Phys


& Medical Imaging; California Northstate University College of Medicine,

Elk Grove, CA 95757; Elk Grove, CA, USA

Ashika Jain, MD; Associate Professor; Director- Division of Emergency

Trauma; New York University Langone Health; New York, NY, USA

Jacob Jensen, DO, DiMM, FAWM; OSF St Francis Med Center EM

Residency; Peoria, IL, USA

Nathaniel Justice, MD, MBA; Assistant Professor; East Tennessee State


University; Johnson City, TN, USA

Matthew Kerrigan, BS; University of Nebraska Medical Center; Omaha,

NE, USA
Noah Kondamudi, MD, MBA; Associate Professor; Medical Director;

Rutgers New Jersey Medical School; West Orange, NJ, USA


Ioannis Koutroulis, MD, PhD, MBA; Assistant Professor; Children's

National Medical Center; Washington, DC, USA


Rachana Krishna, MD; Private Practice; Rochester, MN, USA

Jacqueline Le, MD; Assistant Medical Director; Desert Regional Medical


Center; Palm Springs, CA, USA
Daniella Lent-Schochet, BS; California Northstate University College of

Medicine; Elk Grove, CA, USA

Tian Liang, MD; SUNY Downstate Medical Center; Brooklyn, NY, USA
Angela Macri, DO, MMS; Sampson Regional Medical Center; Wilmington,

NC, USA
Sidharth Mahapatra, MD, PhD; Assistant Professor; University of Nebraska

Medical Center; Omaha, NE, USA

Naser Mahmoud, MD; Louisiana State University/Shreveport; Shreveport,

LA, USA

Sohaib Mandoorah, MD; Private Practice; Saginaw, MI, USA

Rupinder Mangat, MD; Private Practice; Syracuse, NY, USA

Samip Master, MD; Assistant Professsor; Louisiana State University HSC;


Shreveport, LA, USA

George Mathew, MS, MD; Lecturer; Executive Dean; University of Pelita

Harapan; Tangerang, Indonesia


Venkata Vinod Matli, MD; SUNY Upstate Medical University; Syracuse,

NY, USA
Janelle McCall, MD; University of Florida; Gainesville, FL, USA

Catherine McKnight, MD; Instructor; University of TN, UT Medical


Center; Knoxville, TN, USA

Alyssa Miceli, DO; Private Practice; Jacksonville, FL, USA


Justine Moe, MD, DDS; Emory University; Atlanta, GA, USA
Andres Mora Carpio, MD; Private Practice; Philadelphia, PA, USA

Taif Mukhdomi, MD; University of California Riverside; Columbus, OH,

USA
Sean Oldroyd, DO; Associate Program Director; Kaweah Delta Health Care

District; Visalia, CA, USA


Parul Pahal, MBBS; Private Practice; Florence, KY, USA

Pujan Patel, MD; Saint Louis University Hospital; Saint Louis, MO, USA

Milan Patel, MD; SUNY Upstate Medical University; Syracuse, NY, USA

Shivani Patel, BS; NYITCOM; Brooklyn, NY, USA

Furquan Pathan, MD; Orange Park Medical Center; Orange Park, FL, USA

Pallavi Patwari, MD; Assistant Professor; Director, Pediatric Sleep

Medicine; Rush University Medical Center; Chicago, IL, USA


Hector Peniston Feliciano, MD; Volunteer Faculty; FIU Herbert Wertheim

College of Medicine; San Juan, PA, USA

Louisdon Pierre, MD, MBA; Assistant Professor Clinical Pediatrics;


Director, Pediatric Critical Care; Ichan School of Medicine at Mount Sinai;

Suffern, NY, USA


Venessa Pinto, MD; Assistant Professor; Baylor College of Medicine;

Houston, TX, USA


Mario Ponce, MD; Medical Unviersity of South Carolina; Charleston, SC,

USA
Timothy Prince, MD, MSPH; Associate Professor; University of Kentucky;
Lexington, KY, USA

Michael Quinn, DO; NYIT College of Osteopathic Medicine; Marlboro,

NJ, USA
Chandni Ravi, MD; Rutgers University; Jersey City, NJ, USA

Hariharan Regunath, MD; Assistant Professor; University of Missouri;


Columbia, MO, USA

Felix Reyes, MD; State University of New York; Brooklyn, NY, USA

Mary Rodriguez Ziccardi, MD; Private Practice; Philadelphia, PA, USA

Thomas Rodziewicz, MD, JD; Assistant Professor; Michigan State

University COM; Okemos, MI, USA

Steven Rosenbaum, MD; Livingston, NJ, USA

Mohan Rudrappa, MD, FACP; Assistant Professor; Lousiana State


University; Shreveport, LA, USA

Moiz Salahuddin, MD; University of Texas Houston; Houston, TX, USA

Taimur Saleem, MD; Senior Fellow Physician; David Geffen School of


Medicine UCLA; Los Angeles, CA, USA

Raghavendra Sanivarapu, MD; Nassau University Medical Center;


Levittown, NY, USA

John Schweitzer, MD; Assistant Professor; East Tennessee State University;


Johnson City, TN, USA

Aziza Sedrak, MD; Private Practice; Brooklyn, NY, USA


Marc Seligson, BS; SUNY Upstate Medical University; Syracuse, NY, USA
Rouhin Sen, MD; Creighton University; Omaha, NE, USA

Sandeep Sharma, MD; Adjacent Faculty, Attending; Baptist Regional

Medical Center; London, KY, USA


Girish Sharma, MD; Professor of Pediatrics; Chief Peds. Pulmonology,

Director CF Cr.; Rush University Medical Center; Willowbrook, IL, USA


Emad Shehada, MD; Clinical Assistant Professor; Fellowship Program

Director (Pulm /CC); MSU / Mclaren Oakland; Rochester Hills, MI, USA

Faraz Siddiqui, MD; Private Practice; Staten Island, NY, USA

Leslie Simon, DO; Assistant Professor, Emergency Medicine; Mayo Clinic

Florida; Jacksonville, FL, USA

Kari Simonsen, MD; Associate Professor; University of Nebraska Medical

Center; Omaha, NE, USA


Subhas Sitaula, MD; SUNY Upstate Medical University; Syracuse, NY,

USA

Jennifer Slowik, DO, MS, BS; Madigan Army Medical Center; Dupont,
WA, USA

Jessica Snowden, MD; Associate Professor; University of Nebraska


Medical Center; Omaha, NE, USA

Alison Southern, MD, MS; Associate Professor- NEOMED; Attending


physician; Western Reserve Hospital; Clinton, OH, USA

Francis Squadrito, MD; Temple University Hospital; Devon, PA, USA


Brittany Stern, MD; Rush University Medical Center; Chicago, IL, USA
Barbara Stewart, MD; Assistant Professor; East Tennessee State University;

Johnson, TN, USA

Jolee Suddock, DO; University of New Mexico; Albuquerque, NM, USA


Justina Truong, DO; KRMC; Kingman, AZ, USA

Jake Turner, BSc; University of Warwick Medical School; Warwick, United


Kingdom

Ngoc Van Horn, MD; Clinical-Educator Instructor; University of Texas-

Southwestern/Dallas Childrens Medical Center; Dallas, TX, USA

Priyanka Vashisht, MBBS; Private Practice; Omaha, NE, USA

Ray Walther, MD; Assistant Professor; Methodist University Hospital;

Germantown, TN, USA

Stacey Watt, MD; Associate Professor; Program Dir Pediatric


Anesthesiology; University at Buffalo; Grand Island, NY, USA

Grant Wei, MD; Assistant Professor; Assistant Program Director; Rutgers

Robert Wood Johnson MS; New Brunswick, NJ, USA


Lacey Whited, MD; Assistant Professor; LSUHSC; Shreveport, LA, USA

Michael Young, MD; Indiana University; Indianapolis, IN, USA


Elaine Yu, BA, MS; Western University of Health Sciences; Fremont, CA,

USA
Sikander Zulqarnain, MD; Clinical Assistant Professor of Medicine;

Physician Specialist; SUNY Downstate Medical Center; Woodmere, NY,


USA
Authors
Maged Abbas, MD; Fellow; Sana'a, Yemen
Tony Abdelmaseeh, MD; Private Practice; Bronx, NY, USA

Heath Ackley; Private Practice; , NY, USA

Rotimi Adigun, MBBS, MPH; Associate Professor; Director of Pathology

Program; University of Health Sciences, Antigua; St John's, Antigua &

Deps
Sami Akram, MD, MHA; Southern Illinois University; Springfield, IL,

USA

Faysal Alghoula, MBBch; Creighton University; Omaha, NE, USA

Mohamed Alhajjaj, MBBS, FRCP(C); Professor; Cosultant Pulmonologist;

University of Sharajah; Dubai, United Arab Emirates

John Ashurst, DO, MSc; Program Director; Kingman Regional Medical

Center; Kingman, AZ, USA


Usha Avva, MD; Division Chief, Pediatric ED; Joseph M. Sanzari

Children's Hospital; Hackensack, NJ, USA


Folusakin Ayoade, MD; Assistant Professor; University of Miami; Miami,

FL, USA
Byung Ban, DO; University of Tennessee Chattanooga; Chattanooga, TN,
USA

Maria Bandres, MD; Private Practice; Philadelphia, PA, USA


Whitney Barton, RN, MSN, FNP-C; Baptist Health System; Corbin, KY,
USA

Seth Baruffi, MD; Associate Prof. of Emergency Medicine; Core Teaching

Faculty; Inspira Medical Center; Vineland, NJ, USA


Amanda Battisti, DO; Professor; Grand Strand Medical Ctr; Myrtle Beach,

SC, USA
Steve Bhimji, BSci Pharm, MS, MD, PhD; Clinical Instructor (Retired); Un

of Toronto; Toronto, Canada

Edgar Billowitz, MD, MA; Assistant Professor; UMN; Santa Fe, NM, USA

Syed Rizwan Bokhari, BS, MBBS, MD, FCPS; Consultant; Tulane Un, Un

Med Center New Orleans; New Orleans, LA, USA

Daniel Bouchette, BS; St.Martinus Faculty of Medicine; New Orleans, LA,

USA
Darrell Brackett, BA; Respiratory Director; Continue Care Hospital;

Corbin, KY, USA

Daniel Brito, MD; Internal Medicine; Universidad Central de Venezuela;


Philadelphia, PA, USA

Falon Brown, DO; CUSOM/Sampson Regional Medical Center;


Wilmington, NC, USA

Kristin Brown, MD; University of Tennessee; Nashville, TN, USA


Joshua Bucher, MD; Assistant Professor; Assistant EMS Medical Director;

Rutgers - RWJMS; New Brunswick, NJ, USA


Lauren Burton, MD; UTHSC; Murfreesboro, TN, USA
Lauren Camaione, BS, PharmD; Clinical Pharmacist; Pediatric

Pharmacotherapy Specialist; The Brooklyn Hospital Center; Brooklyn, NY,

USA
Sarah Campbell, MS; Assistant Professor; Clinical Services Director;

University of Kentucky; Lexington, KY, USA


Gregory Caputy, MD, PhD; Professor; University of Hawaii; Honolulu, HI,

USA

Danny Castro, DO, MEd; Assistant Professor; Dir of Fellow Ed, Med Dir

of Resp Care; Baylor College of Med, TX Children Hosp; Houston, TX,

USA

Mary Cataletto, MD; Professor; Winthrop University Hospital; Brookville,

NY, USA
Terry Cates, AS; Manager Respiratory Care; Arkansas Continued Care

Hospital; Jonesboro, AR, USA

Deepak Chandra, MBBS, MD; Uni of Nebraska Medical Center; Omaha,


NE, USA

Raheel Chaudhry, MD; Ross University School Of Medicine; Johnson City,


CA, USA

Rebecca Chu, PharmD; Albany College of Pharmacy; Albany, NY, USA


Casey Ciresi, BSN; Clinical Instructor; University of Louisville; Louisville,

KY, USA
Jeffrey Cooper, MD; Associate Professor; Director; University of Nebraska
Medical Center; Omaha, NE, USA

John Costumbrado, MD, MPH; Riverside Community Hospital/UCR EM;

Riverside, CA, USA


Maureen Davis, MD; Advocate Christ Medical Center; Chicago, IL, USA

Richard de la Cruz, MD, MS; Prof; Orange Park Medical Center; Green
Cove Springs, FL, USA

Daniela de Lima Corvino, MD; Einstein Medical Center Philadelphia;

Jenkintown, PA, USA

Timothy Depp, MD; Clinical Assistant Professor; University of South

Carolina; Greenville, SC, USA

Bradley DeVrieze, MD; Assistant Professor of Medicine; Academic

Hospitalist; Creighton University School of Medicine; Omaha, NE, USA


Nagesh Dhadge, MD ( Respiratory Medicine ), MRCPI; Consultant;

Central Health Services; Pune, India

Chaddie Doerr, BSN, MS, CRNA; Clinical Instructor; Norton Health


Systems; Treasure Island, FL, USA

Scott Dulebohn, MD; Associate Professor; ETSU, Meharry, LMU; Johnson


City, TN, USA

Jessie Dunne, PharmD; Assistant Professor; Clinical Pharmacy Specialist;


University of Kentucky; Lexington, KY, USA

Moavia Ehsan, MBBS; Medical Officer; Allama Iqbal Medical College;


Khushaab, Pakistan
Matthew Ellison, MD; Associate Professor; WVU Medicine; Morgantown,

WV, USA

Allen Elster, MD; Louisiana State University Health Sciences Center;


Shreveport, LA, USA

Evelyn Erickson, MD; Attending; Lincoln Medical Center; New York, NY,
USA

Stacey Ernest, MD; Loma Linda Unversity Medical Center; Rancho

Cucamonga, CA, USA

Avram Estes, MD; Private Practice; Chicago, IL, USA

Thomas Folino, DO; NYIT College Of Osteopathic Medicine; Brooklyn,

NY, USA

Amanda Friend, MD, MS; Advocate Christ Medical Center; Chicago, IL,
USA

Justin Fuehrer, DO; Private Practice; Glen Oaks, NY, USA

Evan Fusco, BS, MD, MHA; Medical Director; Springfield, MO, USA
Veronica Garvia, MD; Einstein Medical Center; Philadelphia, PA, USA

Yana Gelman, MD; University of Illinois COM Peoria; Peoria, IL, USA
Namratta Ghatehorde, BSc, MD; Univeristy of Missouri Columbia;

Columbia, MO, USA


Daniel Girzadas, MD; Director of Faculty Development; Advocate Christ

Medical Center; Palos Heights, IL, USA


Kiran Godse, MD, DVD, FRCP, FAAD; Professor; D Y Patil University;
Navi Mumbai, India

Dulce Gonzalez, MD; Lincoln Hospital; New York, NY, USA

William Gossman, MD; Professor; Creighton University; Elkhorn, NE,


USA

Sherry Gossman, BSN; Clinical Instructor (Retired); Elkhorn, NE, USA


Charles Graham, MD; Clinical Instructor; Mayo Clinic Florida; Neptune

Beach, FL, USA

Ali Gunes, MD; Assistant Professor; Harran University; Sanliurfa, Turkey

Farhan Hameed, MD, MS; Director, Med Informatics & Editorial; NE Uni,

Chicago State Uni, MW Uni; Phoenix, AZ, USA

Jaron Hammons, BSN; Private Practice; Barbourville, KY, USA

Mary Hanley, DO; Associate Professor; Pro Dir Undersea and Hyperbaric;
Un of N En Coll of Osteo Med; Charleston, SC, USA

Jackie Hayes, MD; Private Practice; Schertz, TX, USA

Hannah Hays, MD; Assistant Professor; Ohio State University COM;


Delaware, OH, USA

Alexander Heckman, MD; Mayo Clinic Florida; Ponte Vedra Beach, FL,
USA

Alan Heffner, MD; Director of Critical Care; Charlotte, NC, USA


Suezette Hicks, AA, BA, MA; Director Respiratory Care Program;

Pocahontas, AR, USA


Tamer Hudali, MD; Assistant Professor; Southern Illinois University;
Springfield, IL, USA

Martin Huecker, MD; Assistant Professor; Research Director; University of

Louisville; Louisville, KY, USA


James Hughes, MD; Assistant Professor (Retired); MD; Skokie, IL, USA

Katie Huynh, PA-C, MS, AAHIVM-S; Private Practice; Philadelphia, PA,


USA

Phillip Hynes, MD; Radiation Oncology; Lincoln, NE, USA

Michael Ibrahim, MD, PhD; Associate Professor; Biochem, Pharma, Phys

& Medical Imaging; California Northstate University College of Medicine,

Elk Grove, CA 95757; Elk Grove, CA, USA

Ashika Jain, MD; Associate Professor; Director- Division of Emergency

Trauma; New York University Langone Health; New York, NY, USA
Sidney James, DO; Western University; Visalia, CA, USA

Jacob Jensen, DO, DiMM, FAWM; OSF St Francis Med Center EM

Residency; Peoria, IL, USA


David Johnson, MD; Chairman, Dept of Emergency Services; Mercy St

Vincents Medical Center; Perrysburg, OH, USA


Jijoe Joseph, MD, DO, MBA; Nassau University Medical Center; Long

Island City, NY, USA


Nathaniel Justice, MD, MBA; Assistant Professor; East Tennessee State

University; Johnson City, TN, USA


Matthew Kerrigan, BS; University of Nebraska Medical Center; Omaha,
NE, USA

Ryan King, DO; University of Louisville; Louisville, KY, USA

Keith Knigge, DO; Mercy St Vincent; Perrysburg, OH, USA


GoleNaz Kohbodi, MD; NYU Winthrop University; Mineola, NY, USA

Janak Koirala, MD, MPH; Professor; Division Chief; Southern IL Un


School of Medicine; Springfield, IL, USA

Noah Kondamudi, MD, MBA; Associate Professor; Medical Director;

Rutgers New Jersey Medical School; West Orange, NJ, USA

Ioannis Koutroulis, MD, PhD, MBA; Assistant Professor; Children's

National Medical Center; Washington, DC, USA

Rachana Krishna, MD; Private Practice; Rochester, MN, USA

Lauren Krowl, MD, MBA; Private Practice; Syracuse, NY, USA


Jacqueline Le, MD; Assistant Medical Director; Desert Regional Medical

Center; Palm Springs, CA, USA

Daniella Lent-Schochet, BS; California Northstate University College of


Medicine; Elk Grove, CA, USA

Stephen Leslie, MD; Associate Professor of Surgery, Urology; Creighton


University Medical Center; Omaha, NE, USA

Claudia Lopez, MD; Private Practice; Bronx, Azerbaijan


Angela Macri, DO, MMS; Sampson Regional Medical Center; Wilmington,

NC, USA
Naser Mahmoud, MD; Louisiana State University/Shreveport; Shreveport,
LA, USA

Sohaib Mandoorah, MD; Private Practice; Saginaw, MI, USA

Julianna Martel, BA; Tulane University School of Medicine; New Orleans,


LA, USA

Samip Master, MD; Assistant Professsor; Louisiana State University HSC;


Shreveport, LA, USA

Venkata Vinod Matli, MD; SUNY Upstate Medical University; Syracuse,

NY, USA

Catherine McKnight, MD; Instructor; University of TN, UT Medical

Center; Knoxville, TN, USA

Kenneth Metcalf, MD, PhD; Professor Emeritus; Dept of Anatomy,

Embryology, & Genetics; UNMC; Omaha, NE, USA


Jessica Meyer, BS; Staff Nurse; Naval Hospital Camp Lejeune;

Jacksonville, NC, USA

Alyssa Miceli, DO; Private Practice; Jacksonville, FL, USA


Penchala Mittadodla, MD; Adjunct Clinical Assistant Professor;

Pulmonology / Critical Care Physician; Mercy Hospital; Rogers, AR, USA


Justine Moe, MD, DDS; Emory University; Atlanta, GA, USA

Mubeen Khan Mohammed Abdul, MD; Private Practice; Memphis, TN,


USA

Andres Mora Carpio, MD; Private Practice; Philadelphia, PA, USA


Taif Mukhdomi, MD; University of California Riverside; Columbus, OH,
USA

Ahmed Munir, MBBS; Resident; Creighton University Medical Center;

Omaha, NE, USA


Susan Murin, BA, MD, MSc, MBA; Professor; UC Davis School of

Medicine; Sacramento, CA, USA


Rodney Nyland, PhD; Assistant Professor; Oakland Un William Beaumont

Sch of Med; Rochester, MI, USA

Sean Oldroyd, DO; Associate Program Director; Kaweah Delta Health Care

District; Visalia, CA, USA

Danny Opperman, AAS, BS; EMS Educator/Clinical Coordinator; Atlantic

Cape Community College; Atlantic City, NJ, USA

Parul Pahal, MBBS; Private Practice; Florence, KY, USA


Jorge Palacios, MD; Faculty; Chair of Anesthesia; Kaweah Delta Medical

Center; Clovis, CA, USA

Beckie Palmer; Private Practice; Culpeper, VA, USA


Pallavi Patwari, MD; Assistant Professor; Director, Pediatric Sleep

Medicine; Rush University Medical Center; Chicago, IL, USA


Hector Peniston Feliciano, MD; Volunteer Faculty; FIU Herbert Wertheim

College of Medicine; San Juan, PA, USA


Thomas Perera, MD; Associate Professor; Hofstra Northwell; Manhasset,

NY, USA
Yomayra Perez, MD; Private Practice; Bronx, NY, USA
Meghana Phiske, MD, DNB, DVD, DDV; Assisstant Professor; Navi

Mumbai, India

Venessa Pinto, MD; Assistant Professor; Baylor College of Medicine;


Houston, TX, USA

S Plantz, MD; Associate Professor (Retired); MD; University of Louisville;


Louisville, KY, USA

Pooja Poudel, MD; Suny Upstate Medical University; Syracuse, NY, USA

Michael Quinn, DO; NYIT College of Osteopathic Medicine; Marlboro,

NJ, USA

Chandni Ravi, MD; Rutgers University; Jersey City, NJ, USA

Deepa Rawat, MD; Baptist Regional Medical Center; Iondon, PA, USA

Hariharan Regunath, MD; Assistant Professor; University of Missouri;


Columbia, MO, USA

Hamidreza Reihani, MD; Assistant Professor; Mashhad, Iran

Felix Reyes, MD; State University of New York; Brooklyn, NY, USA
Kenneth Robinson, MD, MHCM; Professor; Chair, Dept. of Emergency

Medicine; University Of Connecticut; Hartford, CT, USA


Omara Rodriguez; Private Practice; houston, TX, USA

Mary Rodriguez Ziccardi, MD; Private Practice; Philadelphia, PA, USA


Mohan Rudrappa, MD, FACP; Assistant Professor; Lousiana State

University; Shreveport, LA, USA


Moiz Salahuddin, MD; University of Texas Houston; Houston, TX, USA
Taimur Saleem, MD; Senior Fellow Physician; David Geffen School of

Medicine UCLA; Los Angeles, CA, USA

Raghavendra Sanivarapu, MD; Nassau University Medical Center;


Levittown, NY, USA

Erez Schori, MD; Private Practice; Bronx, NY, USA


John Schweitzer, MD; Assistant Professor; East Tennessee State University;

Johnson City, TN, USA

Brian Seavey, DO; Attending Physician; Grandview Medical Center;

Centerville, OH, USA

Marc Seligson, BS; SUNY Upstate Medical University; Syracuse, NY, USA

Rouhin Sen, MD; Creighton University; Omaha, NE, USA

Girish Sharma, MD; Professor of Pediatrics; Chief Peds. Pulmonology,


Director CF Cr.; Rush University Medical Center; Willowbrook, IL, USA

Sandeep Sharma, MD; Adjacent Faculty, Attending; Baptist Regional

Medical Center; London, KY, USA


Kinjel Shastri, DO; McLaren Oakland Hospital; Pontiac, MI, USA

Emad Shehada, MD; Clinical Assistant Professor; Fellowship Program


Director (Pulm /CC); MSU / Mclaren Oakland; Rochester Hills, MI, USA

Faraz Siddiqui, MD; Private Practice; Staten Island, NY, USA


Leslie Simon, DO; Assistant Professor, Emergency Medicine; Mayo Clinic

Florida; Jacksonville, FL, USA


Kari Simonsen, MD; Associate Professor; University of Nebraska Medical
Center; Omaha, NE, USA

Virteeka Sinha, MD; Assistant Professor; Rutgers New Jersey Medical

School; New York, NY, USA


Subhas Sitaula, MD; SUNY Upstate Medical University; Syracuse, NY,

USA
Jennifer Slowik, DO, MS, BS; Madigan Army Medical Center; Dupont,

WA, USA

Frank Smeeks, MS, MD, MBA; Private Practice; Hickory, NC, USA

Jessica Snowden, MD; Associate Professor; University of Nebraska

Medical Center; Omaha, NE, USA

Isaac Soliman, MD; Chief Resident; Hackensack Meridian Mountainside

Medical Center; Bloomfield, NJ, USA


Alison Southern, MD, MS; Associate Professor- NEOMED; Attending

physician; Western Reserve Hospital; Clinton, OH, USA

Francis Squadrito, MD; Temple University Hospital; Devon, PA, USA


Brittany Stern, MD; Rush University Medical Center; Chicago, IL, USA

Britni Sternard, MD; Assistant Professor; Assistant Program Director;


Louisiana State University; Shreveport, LA, USA

Terrell Swanson, MD; Orange Park Medical Center; Ponte Vedra, FL, USA
Francisco Talavera, PharmD, PhD; Assistant Professor; University of

Nebraska Medical Center; Omaha, NE, USA


Imani Thornton, MD; Clinical Professor; Wayne State University Physician
Group; West Bloomfield, MI, USA

Tammy Toney-Butler, AS, RN, CEN, TCRN, CPEN; Clinical

Instructor/Nurse Educator; Naples, FL, USA


Justina Truong, DO; KRMC; Kingman, AZ, USA

Jake Turner, BSc; University of Warwick Medical School; Warwick, United


Kingdom

Ngoc Van Horn, MD; Clinical-Educator Instructor; University of Texas-

Southwestern/Dallas Childrens Medical Center; Dallas, TX, USA

Priyanka Vashisht, MBBS; Private Practice; Omaha, NE, USA

Julian Vega, DO; CPT; Martin Army Community Hospital; Fort Mitchell,

AL, USA

Charlie Vidal, MD, MBA, MPH; Private Practice; Utuado, CT, USA
Ahmed Virani, MD; LSUHSC-Shreveport; Shreveport, LA, USA

Jason Wallen, MD; Assistant Professor; Upstate Medical University;

Syracuse, NY, USA


Ray Walther, MD; Assistant Professor; Methodist University Hospital;

Germantown, TN, USA


Cassandra Wasson, DO; Riverside University Health System; Los Angeles,

CA, USA
Stacey Watt, MD; Associate Professor; Program Dir Pediatric

Anesthesiology; University at Buffalo; Grand Island, NY, USA


James Waymack, MD; Assistant Professor; Program Director; Southern
Illinois University; Springfield, IL, USA

Grant Wei, MD; Assistant Professor; Assistant Program Director; Rutgers

Robert Wood Johnson MS; New Brunswick, NJ, USA


Benjamin Wiederhold, MD; Professor; Chair & Med Dir., Dept of

Emergency Med; St Joseph Medical Center; Stockton, CA, USA


Michael Young, MD; Indiana University; Indianapolis, IN, USA

Marianna Zagurovskaya, MD; Assistant Professor; University of Kentucky;

Lexington, KY, USA

Sikander Zulqarnain, MD; Clinical Assistant Professor of Medicine;

Physician Specialist; SUNY Downstate Medical Center; Woodmere, NY,

USA
Grammar and Illustration Editors
Susan Oliver, BA
Beata Beaty, BA

Gerson Cordero Rubio


Table of Contents
Please Contribute

Learn More

Introduction

Editors in Chief

Contributing Editors

Authors

Section 1 ( Questions 1 - 65 )

Section 2 ( Questions 66 - 130 )

Section 3 ( Questions 131 - 195 )

Section 4 ( Questions 196 - 260 )

Section 5 ( Questions 261 - 325 )

Section 6 ( Questions 326 - 390 )

Section 7 ( Questions 391 - 455 )

Section 8 ( Questions 456 - 520 )

Section 9 ( Questions 521 - 585 )

Section 10 ( Questions 586 - 650 )

About Stat Pearls


Section 1

Question 1: A patient suffers a stab wound to the chest. In the emergency


department, he is found to have an 80% pneumothorax with a midline shift of
the trachea to the contralateral chest. There is evidence of circulatory and
respiratory dysfunction. What is the next step in management?

Choices:
1. Endotracheal tube placement
2. Fluid bolus
3. Intravenous epinephrine
4. Needle decompression with a large bore needle
Answer: 4 - Needle decompression with a large bore needle
Explanations:
There is no other acceptable method to treat a tension pneumothorax. The
condition is life-threatening and needs urgent treatment. Needle
decompression in an adult should be performed with a 14 to 16 gauge
needle approximately 3.5 inches in length.
After providing 100% oxygen, emergency needle decompression can be
life-saving. Following patient stabilization, a chest tube can be inserted.
A chest x-ray should always be obtained after chest tube placement to
ensure lung re-expansion. Sometimes an arterial blood gas may be required.
The most common complication is from mistaking an ordinary
pneumothorax for a tension pneumothorax. In any case, the treatment is the
same.

Go to the next page if you knew the correct answer, or click the link image(s)
below to further research the concepts in this question (if desired).

Research Concepts:
Pneumothorax, Tension And Traumatic

We update eBooks quarterly and Apps daily based on user feedback.


Please tap flag to report any questions that need improvement.
Question 2: A 17-year-old female is admitted with status asthmaticus. She is
treated with continuous albuterol nebulization, intravenous corticosteroids, and
bilevel positive airway pressure mechanical respiration. The patient's respiratory
status improves, but she develops generalized weakness, fatigue, and myalgias.
The cardiac monitor shows prolonged QT interval, flattened T waves, and ST
depression. What is the most likely etiology of these changes?

Choices:
1. Central pontine myelitis
2. Hypokalemia
3. Corticosteroid-induced myopathy
4. Myocardial infarction
Answer: 2 - Hypokalemia
Explanations:
Beta-agonists, such as albuterol, can cause hypokalemia.
This is secondary to insulin secretion and cellular uptake of potassium.
Initially, this can cause fatigue, muscle weakness, and myalgias, but can
progress to complete paralysis and hypoventilation.
Intravenous replacement of potassium is limited by pain and sclerosing of
veins.

Go to the next page if you knew the correct answer, or click the link image(s)
below to further research the concepts in this question (if desired).

Research Concepts:
Asthma, Status Asthmaticus

We update eBooks quarterly and Apps daily based on user feedback.


Please tap flag to report any questions that need improvement.
Question 3: A 17-year-old competitive swimmer complains of increased
coughing after a race. You suspect exercise-induced bronchospasm (EIB) and
perform an exercise challenge. In order to make the diagnosis, which of the
following measurements would show at least ten percent drop?

Choices:
1. Forced vital capacity (FVC)
2. Forced expiratory volume at 1 second (FEV1)
3. Peak expiratory flow rate (PEFR)
4. FEF 25-75
Answer: 2 - Forced expiratory volume at 1 second (FEV1)
Explanations:
Exercise induced bronchoconstriction is defined as a transient airway
obstruction following exercise.
It is measured by a fall in FEV1.
A fall in FEV1 of 10 percent post exercise is generally considered
diagnostic.
Post exercise cough is a common feature of exercise induced
bronchospasm. Children may also have shortness of breath, chest tightness
or wheezing.

Go to the next page if you knew the correct answer, or click the link image(s)
below to further research the concepts in this question (if desired).

Research Concepts:
Asthma, Exercise Induced

We update eBooks quarterly and Apps daily based on user feedback.


Please tap flag to report any questions that need improvement.
Question 4: A patient presents with exertional dyspnea, nonproductive
cough, and fever. A chest x-ray shows perihilar infiltrates, and a methenamine
silver stain shows cysts. The patient's symptoms worsen with erythromycin
therapy. What is the most likely diagnosis?

Choices:
1. Mycoplasma pneumoniae
2. Pneumocystis jiroveci pneumonia
3. Acute pulmonary histoplasmosis
4. Tuberculosis
Answer: 2 - Pneumocystis jiroveci pneumonia
Explanations:
Pneumocystis jiroveci pneumonia (PJP) causes an interstitial infiltrate on
chest x-ray.
Patients with PJP typically present with worsening exertional dyspnea,
nonproductive cough, and fever.
If PJP is strongly suspected, a sputum sample by sputum induction should
be obtained.
Visualization of cysts with methenamine silver staining of a sputum
specimen is diagnostic.

Go to the next page if you knew the correct answer, or click the link image(s)
below to further research the concepts in this question (if desired).

Research Concepts:
Pneumonia, Pneumocystis (Carinii) Jiroveci

We update eBooks quarterly and Apps daily based on user feedback.


Please tap flag to report any questions that need improvement.
Question 5: A 2-year-old presents to the emergency department with a
barking cough, inspiratory stridor, and sternal wall retractions at rest. She
appears minimally agitated. Racemic epinephrine is administered via a nebulizer.
For how long is it appropriate to monitor this child for possible rebound of
stridor?

Choices:
1. At least 30 minutes
2. At least 4 hours
3. At least 1 hour
4. At least 2 hours
Answer: 2 - At least 4 hours
Explanations:
This patient has moderately severe croup. Nebulized epinephrine is
associated with a reduction in croup symptoms 30 minutes following
administration.
If epinephrine is given for croup, monitor for at least 4 hours for rebound of
the condition.
Five milliliters of L-adrenaline 1:1000 is considered as safe as the racemic
form.
Epinephrine also is used to treat anaphylaxis, asthma, bradycardia, cardiac
arrest and shock, toxins, and overdoses.

Go to the next page if you knew the correct answer, or click the link image(s)
below to further research the concepts in this question (if desired).

Research Concepts:
Epinephrine

We update eBooks quarterly and Apps daily based on user feedback.


Please tap flag to report any questions that need improvement.
Question 6: A 21-year-old presents with shortness of breath. After a detailed
workup diagnosis of primary pulmonary hypertension was made. Which class of
medication has shown improvement in mortality in primary pulmonary
hypertension?

Choices:
1. Calcium channel blocker
2. Prostacyclin
3. Guanylate cyclase stimulators
4. Endothelin receptor antagonists
Answer: 2 - Prostacyclin
Explanations:
Continuous long-term intravenous epoprostenol infusion for which semi-
permanent central venous catheter is required is considered the most
effective therapy. It has shown to improve mortality, but short half-life and
high cost are the limitations.
For those who cannot tolerate intravenous infusion, inhaled or subcutaneous
prostanoids can be considered. Treprostinil can be used by intravenously,
subcutaneously, and by inhalation. Oral prostanoids are still under clinical
trials.Benefits include vasodilation, platelet inhibition, antiproliferative, and
inotropic effects.
Soluble guanylate cyclase stimulators are under clinical trials and are
beneficial in pulmonary artery hypertension as they have dual mode of
action.They stimulate the receptor to mimic nitric oxide action and increase
the sensitivity of guanylyl cyclase to endogenous nitric oxide. Riociguat has
shown to improve exercise capacity and decrease in pulmonary vascular
resistance in the studies done so far.
Oral high dose calcium channel blockers such as diltiazem and nifedipine
are the first line treatment but used only in those with vasoreactivity testing
positive for acute vasodilator response with short-acting pulmonary
vasodilators such as adenosine, nitric oxide, or epoprostenol. The criteria
for testing positive is fall in pulmonary artery pressure to more than 10
mmHg with an increase or no change in cardiac output. Although first line
but useful only in 5% patients with idiopathic pulmonary artery
hypertension and should not be used in nonresponders to vasoreactivity test
due to the risk of harm rather than any improvement.

Go to the next page if you knew the correct answer, or click the link image(s)
below to further research the concepts in this question (if desired).

Research Concepts:
Pulmonary Hypertension, Primary
We update eBooks quarterly and Apps daily based on user feedback.
Please tap flag to report any questions that need improvement.
Question 7: A 17-year-old male begins inhaled corticosteroid therapy for
mild persistent asthma. During the office visit, he is counseled on proper inhaler
technique. He also is advised to rinse his mouth and throat with water after each
use, expectorating the rinsate. This is intended to prevent overgrowth by which
of the following organisms?

Choices:
1. Blastomyces dermatitidis
2. Candida albicans
3. Histoplasma capsulatum
4. Pneumocystis jirovecii
Answer: 2 - Candida albicans
Explanations:
Oral thrush or candidiasis, caused by Candida overgrowth, is a common
local adverse effect associated with inhaled corticosteroids.
Effects of local deposition in the oropharynx and larynx include oral thrush,
dysphonia, cough, and throat irritation. Systemic effects result from
ingestion as well as absorption from the lung.
The majority of inhaled corticosteroid doses delivered by metered dose
inhalers are deposited in the oropharynx. This drug can then be swallowed,
undergo first-pass metabolism, and enter the systemic circulation. Methods
to reduce the amount of drug being ingested include rinsing the mouth after
each use, using a spacer, and proper technique. Dry powder inhalers have
reduced deposition in the oropharynx and larynx.
In addition to prevention of oral thrush, rinsing the mouth after each use of
inhaled corticosteroids can reduce systemic adverse effects.

Go to the next page if you knew the correct answer, or click the link image(s)
below to further research the concepts in this question (if desired).

Research Concepts:
Corticosteroids, Inhaled

We update eBooks quarterly and Apps daily based on user feedback.


Please tap flag to report any questions that need improvement.
Question 8: An Egyptian male is undergoing anesthesia for surgery of his
appendix. Just after induction of anesthesia, a large parasite is seen wriggling in
the oral cavity. This parasite is most likely which of the following?

Choices:
1. Wucheria bancrofti
2. Paragonimus
3. Ascaris lumbricoides
4. Strongyloides
Answer: 3 - Ascaris lumbricoides
Explanations:
Ascaris lumbricoides tends to accumulate in the lung and intestine.
These parasitic worms grow very large and have an aversion to anesthetic
gasses.
In rare instances, the parasite will be seen crawling out of the mouth during
anesthesia.
Ascariasis also can sensitize an individual to shrimp and dust.

Go to the next page if you knew the correct answer, or click the link image(s)
below to further research the concepts in this question (if desired).

Research Concepts:
Ascariasis

We update eBooks quarterly and Apps daily based on user feedback.


Please tap flag to report any questions that need improvement.
Question 9: Which of the following is most likely to lead to lung injury
when using mechanical ventilation in a child?

Choices:
1. FIO2 of 0.50
2. PEEP of 15 cmH2O
3. Tidal volumes greater than 8 ml/kg
4. High-frequency oscillatory support
Answer: 3 - Tidal volumes greater than 8 ml/kg
Explanations:
The goal with ventilator support is to provide acceptable gas exchange
without causing lung damage.
Reducing tidal volumes to less than 6 to 8 ml/kg is desirable.
Reducing FIO2 to less than 0.60 is also important.
One major causative factor is the overstretching of the airways and alveoli
volutrauma. During mechanical ventilation, the flow of gas into the lung
takes the path of least resistance. Areas of the lung that are collapsed or
filled with secretions and will be underinflated, while those areas that are
relatively normal will be overinflated. These areas will become
overdistended and then injured. This may be reduced by using smaller tidal
volumes.

Go to the next page if you knew the correct answer, or click the link image(s)
below to further research the concepts in this question (if desired).

Research Concepts:
Ventilation, Mechanical

We update eBooks quarterly and Apps daily based on user feedback.


Please tap flag to report any questions that need improvement.
Question 10: An adolescent with mild, persistent asthma has been well
controlled with low-dose inhaled corticosteroids. He presents for follow up
complaining of some limitation with normal activities and needing albuterol for
symptom relief 3 to 4 days per week over the past month. Which of the
following is the preferred next step?

Choices:
1. Begin an oral corticosteroid burst
2. Medium-dose inhaled corticosteroid
3. Add a leukotriene receptor antagonist
4. Begin omalizumab
Answer: 2 - Medium-dose inhaled corticosteroid
Explanations:
This patient is currently on step 2 of asthma controller therapy, and his level
of control for this scenario is "not well controlled." Therefore the next step
(step 3) is a medium-dose inhaled corticosteroid.
A less preferred option is adding a leukotriene receptor antagonist, such as
montelukast, to the low-dose inhaled corticosteroid.
Inhaled corticosteroids are considered to be superior to leukotriene receptor
antagonists when used as monotherapy.
If good control is demonstrated for at least 3 months, a trial of stepped-
down treatment is appropriate. Close follow-up is needed to assess the
effectiveness of the change in treatment.

Go to the next page if you knew the correct answer, or click the link image(s)
below to further research the concepts in this question (if desired).

Research Concepts:
Asthma

We update eBooks quarterly and Apps daily based on user feedback.


Please tap flag to report any questions that need improvement.
Question 11: A 17-year-old male with AIDS presents with worsening
exertional dyspnea, fever, and a nonproductive cough. His temperature is 102 F,
heart rate 120 beats/min, and respiratory rate 19. A physical exam reveals mild
crackles and rhonchi bilaterally. A chest x-ray shows diffuse bilateral infiltrates.
He has no known drug allergies. What is the best initial treatment?

Choices:
1. Intravenous trimethoprim/sulfamethoxazole
2. Intravenous pentamidine
3. Intravenous corticosteroids followed by IV trimethoprim/sulfamethoxazole
4. Intravenous corticosteroids followed by IV levofloxacin
Answer: 3 - Intravenous corticosteroids followed by IV
trimethoprim/sulfamethoxazole

Explanations:
Corticosteroids decrease mortality in patients with severe Pneumocystis
jiroveci pneumonia (PCP).
Trimethoprim/sulfamethoxazole is the drug of choice, but pentamidine is
used for those with sulfa allergy.
Levofloxacin does not cover PCP. However, if there are infiltrates, it can be
added for possible bacterial pneumonia.
Adjunctive corticosteroids can decrease the inflammatory response
associated with PCP. Also, they can reduce the decline of oxygenation and
reduce the incidence of respiratory failure.

Go to the next page if you knew the correct answer, or click the link image(s)
below to further research the concepts in this question (if desired).

Research Concepts:
Pneumonia, Pneumocystis (Carinii) Jiroveci

We update eBooks quarterly and Apps daily based on user feedback.


Please tap flag to report any questions that need improvement.
Question 12: A 17-year-old male who is involved in a fight and is stabbed
in the right chest at the eighth intercostal space in the midclavicular line. In the
emergency department, he has a blood pressure of 100/60 mmHg, respiratory
rate of 22, and a pulse of 120 beats/min. At room air, his pulse oximeter is 94%.
Auscultation reveals bilateral air entry. What is the next best step in
management?

Choices:
1. Place a right-sided chest tube
2. Explore the stab wound in the emergency department
3. Send patient for a CT scan
4. Diagnostic laparoscopy
Answer: 4 - Diagnostic laparoscopy
Explanations:
In any patient with an injury below the seventh rib, one should always be
cognizant of an abdominal injury.
In this patient, a diagnostic laparoscopy is worthwhile and it can also be
converted to an open procedure if there is suspicion of injury.
The abdominal contents cannot always be fully explored with a
laparoscope, but the presence of blood, bile, or tissue trauma should raise
suspicion for organ injury.
A CT scan has low sensitivity for assessing penetrating abdominal trauma.

Go to the next page if you knew the correct answer, or click the link image(s)
below to further research the concepts in this question (if desired).

Research Concepts:
Trauma, Chest, Penetrating

We update eBooks quarterly and Apps daily based on user feedback.


Please tap flag to report any questions that need improvement.
Question 13: A patient has edema and a dilated cardiomyopathy due to a
dietary deficiency. What is the most likely etiology?

Choices:
1. Thiamine deficiency
2. Vitamin C deficiency
3. Niacin deficiency
4. Pyridoxine deficiency
Answer: 1 - Thiamine deficiency
Explanations:
This patient has wet-beriberi, which results from thiamine (B1) deficiency.
In the developed world, thiamine deficiency most commonly occurs in
alcoholics.
Thiamine deficiency also causes Wernicke-Korsakoff and dry beriberi.
Dry beriberi manifests as polyneuritis and symmetrical muscle wasting.

Go to the next page if you knew the correct answer, or click the link image(s)
below to further research the concepts in this question (if desired).

Research Concepts:
Vitamin, B1, Thiamine Deficiency (Beriberi)

We update eBooks quarterly and Apps daily based on user feedback.


Please tap flag to report any questions that need improvement.
Question 14: A 1-year old boy presents with a persistent cough for the past
5 days. He has had several episodes of pneumonia and upper respiratory
infections since he was born. He also has several skin infections. He is in the
10th percentile for weight and 5th percentile for height, but at birth, he was in
the 50th percentile. A dihydrorhodamine test is abnormal and peripheral blood
smear does not indicate hemolytic anemia. What is the most likely cause of this
patient’s disease?

Choices:
1. Deficiency in NADPH oxidase
2. Glutathione synthase deficiency
3. CFTR gene mutation
4. Glucose-6-phosphate dehydrogenase
Answer: 1 - Deficiency in NADPH oxidase
Explanations:
Patients with chronic granulomatous disease (CGD) have an NADPH
oxidase deficiency, which makes them susceptible to recurrent infections,
typically of the respiratory tract.
At the cellular level, CGD can be diagnosed by measuring the capacity of
phagocytic leucocytes to form superoxide or hydrogen peroxide.
Neutrophilic granulocytes are usually used to detect NADPH oxidase
activation in the cells. NADPH oxidase assays include cytochrome c
reduction assay and the nitroblue tetrazolium slide assay, both of which
measure superoxide. Other assays exist to measure hydrogen peroxide
including the dihydrorhodamine-1,2, 3 (DHR) assay, and the hydrogen
peroxide (H2O2) or peroxidase activity assay. An abnormal
dihydrorhodamine test is significant for a CGD diagnosis.
While clinical history may help indicate the patient’s genetic inheritance
pattern, genetic testing is available to identify genetic mutations.

Go to the next page if you knew the correct answer, or click the link image(s)
below to further research the concepts in this question (if desired).

Research Concepts:
Chronic Granulomatous Disease

We update eBooks quarterly and Apps daily based on user feedback.


Please tap flag to report any questions that need improvement.
Question 15: A patient with severe neutropenia presents with pneumonia.
Bronchial alveolar fluid reveals dichotomously branching, generally with acute
angles, and septate hyphae. What is the most likely etiology?

Choices:
1. Cryptococcus
2. Candida
3. Aspergillus
4. Malassezia
Answer: 3 - Aspergillus
Explanations:
Aspergillus spores are commonly airborne. These asexual spores are
responsible for initiation of aspergillosis. Inhalation of infectious conidia is
a frequent event.
Invasive infections with Aspergillus are controlled by phagocytic cells such
as airway epithelial cells and alveolar macrophages. Activation of cellular
immunity is important in killing invasive hyphae.
The most common causal agent of invasive aspergillosis, Aspergillus
fumigatus, responds to low-oxygen environments for pathogenesis and
disease progression. It is one of the major microbes in cystic fibrosis.
For the earliest stages of the disease, a chest x-ray is less sensitive than a
chest CT. The pulmonary form of the disease classically presents as single
or multiple nodules that may or may not be associated with cavitation,
consolidations, or peribronchial infiltrates. Tree-in-bud patterns may be
seen in some cases.

Go to the next page if you knew the correct answer, or click the link image(s)
below to further research the concepts in this question (if desired).

Research Concepts:
Aspergillus Fumigatus

We update eBooks quarterly and Apps daily based on user feedback.


Please tap flag to report any questions that need improvement.
Question 16: A 12-year-old with perennial aeroallergies has severe
persistent asthma which is not well controlled with step 5 therapy with high dose
inhaled corticosteroids and long-acting beta agonist. Which of the following
options should be considered?

Choices:
1. Addition of montelukast
2. Addition of theophylline
3. Addition of omalizumab
4. Discontinuation of the long-acting beta2 agonist
Answer: 3 - Addition of omalizumab
Explanations:
Omalizumab should be considered in patients with documented perennial
aeroallergen sensitivity and moderate to severe asthma which is not well
controlled.
Omalizumab is an anti IgE monoclonal antibody.
Omalizumab is now FDA approved for patients ages 6 years and older.
Safety and efficacy in children less than 6 years of age have not been
established.

Go to the next page if you knew the correct answer, or click the link image(s)
below to further research the concepts in this question (if desired).

Research Concepts:
Asthma, Pediatric

We update eBooks quarterly and Apps daily based on user feedback.


Please tap flag to report any questions that need improvement.
Question 17: A 5-year-old female is brought in with a 3-day history of fever
and cough. The temperature is 39.2 C and respiratory rate is 42. There are
decreased breath sounds and dullness to percussion on the right. Chest
radiograph shows opacity of the right side. What is the appropriate initial
treatment?

Choices:
1. Triple antibiotics including vancomycin
2. Tube thoracostomy
3. Bronchoscopy
4. Nebulized N-acetylcysteine
Answer: 1 - Triple antibiotics including vancomycin
Explanations:
The patient most likely has pneumonia with empyema.
The most probable causative organisms include Staphylococcus aureus,
Streptococcus pyogenes, and Streptococcus pneumoniae.
Vancomycin should be included in the antibiotics due to the high rate of
resistance.
Chest tube placement may be needed.

Go to the next page if you knew the correct answer, or click the link image(s)
below to further research the concepts in this question (if desired).

Research Concepts:
Empyema

We update eBooks quarterly and Apps daily based on user feedback.


Please tap flag to report any questions that need improvement.
Question 18: A 65-year-old male, with medical history significant for
chronic obstructive pulmonary disease Global Initiative for Chronic Lung
Disease (GOLD) class C, is evaluated for worsening exertional shortness of
breath which has been progressing over the last 6 months. The patient denied
any new change in his cough, nor the amount or the color of the sputum. On
examination, the patient’s oxygen saturation was 89%, otherwise, he was stable.
Chest exam revealed polyphonic wheezes allover the chest. Cardiac exam shows
normal S1 and loud S2 with no murmurs. Arterial blood gas evaluation shows
PaO2 of 56 mmHg, PaCO2 of 47 mmHg, and PH of 7.39. Echocardiogram
shows normal left ventricular ejection fraction with a right ventricular systolic
pressure of 45 mmHg. What is the appropriate treatment?

Choices:
1. Start long-term corticosteroid therapy.
2. Start long-term oxygen therapy
3. Start long-term azithromycin therapy.
4. Start roflumilast
Answer: 2 - Start long-term oxygen therapy
Explanations:
Among the mentioned options; long-term oxygen therapy (LTOT) is the
only option that reduces the mortality in patients with chronic obstructive
pulmonary disease (COPD).
LTOT is indicated in these patient populations: (1) Group I (Absolute)
PaO2 55 mm Hg or SaO2Â 88%, (2) Group II (In the presence of cor
pulmonale) PaO2Â 55 to 59 mm Hg or oxygen saturation (SaO2) 89%,
ECG evidence of right atrial enlargement, hematocrit greater than 55%, and
congestive heart failure
This patient has PaO2 of 56 mmHg with SaO2 of 89 with evidence of
pulmonary hypertension on echocardiogram, putting his in the group II
indication of LTOT.
Long-term corticosteroid therapy is associated with significant side
effects with no proven benefit over the current standards of care. Also, the
rationale behind using roflumilast and azithromycin is to reduce the rate of
exacerbations no proven mortality benefit.

Go to the next page if you knew the correct answer, or click the link image(s)
below to further research the concepts in this question (if desired).

Research Concepts:
Anoxia (Hypoxic Hypoxia)

We update eBooks quarterly and Apps daily based on user feedback.


Please tap flag to report any questions that need improvement.
Question 19: A patient with diabetes mellitus presents complaining of
chills, fevers, and malaise for 1 day. A chest x-ray reveals a large mass. CT
shows pleural effusion, consolidation, a nodule, and ground-glass opacities.
During exploration, a dimorphic fungal organism is seen adjacent to thrombosed
vessels. What is the treatment of choice?

Choices:
1. Intravenous amphotericin
2. Limited resection
3. Intravenous amphotericin for 6 weeks followed by thoracotomy
4. Immediate radical debridement
Answer: 4 - Immediate radical debridement
Explanations:
Mucormycosis is a lethal infection and can erode into all tissue planes. It is
a rare infection usually involving the rhinocerebral sinuses.
Risk factors for developing mucormycosis are diabetes mellitus and
immunosuppression including patients undergoing chemotherapy or with
leukemia.
The organism causes necrosis and is found adjacent to thrombosed blood
vessels. These patients need immediate surgery with radical debridement.
Mucormycosis carries a high mortality.

Go to the next page if you knew the correct answer, or click the link image(s)
below to further research the concepts in this question (if desired).

Research Concepts:
Mucormycosis

We update eBooks quarterly and Apps daily based on user feedback.


Please tap flag to report any questions that need improvement.
Question 20: A 7-year old is seen in the emergency room with a wet cough,
fever, and general malaise. The work-up reveals that he has lobar pneumonia. He
has otherwise been healthy, and he has had all his vaccinations to date. Based on
the 2011 guidelines released by the Infectious Disease Society of America, the
child should be treated with which of the following?

Choices:
1. Observation only
2. Ampicillin
3. Ceftriaxone
4. Chloramphenicol
Answer: 2 - Ampicillin
Explanations:
According to the 2011 national guidelines released by the PIDS and IDSA
for the treatment of vaccinated children hospitalized with uncomplicated
community-acquired pneumonia, the use of single, narrow-spectrum
antibiotics, such as penicillin or ampicillin was emphasized.
According to experts, the 2011 national guidelines released by the PIDS
and IDSA for the treatment of vaccinated children hospitalized with
uncomplicated community-acquired pneumonia was based on evidence that
Streptococcus pneumoniae most commonly caused the illness and that the
incidence of penicillin-resistant pneumococcal disease has dropped after the
vaccines' introduction.
Community-acquired pneumonia is a serious infection in children and often
requires hospitalization. The diagnosis can be based on the history and
physical examination. Chest radiography and rapid viral testing may be
helpful. Viral and Streptococcus pneumoniae infections are most common
in preschool-aged children. Mycoplasma pneumoniae is more common in
older children. The decision to treat with antibiotics is challenging,
especially with the increasing prevalence of viral and bacterial coinfections.
Preschool-aged children with uncomplicated bacterial pneumonia should be
treated with amoxicillin or ampicillin. Macrolides are first-line agents in
older children. Immunization with the 13-valent pneumococcal conjugate
vaccine is important to reduce the severity of childhood pneumococcal
infections.
Community-acquired pneumonia is a significant cause of respiratory
morbidity and mortality in children, particularly in developing countries. It
is the leading cause of death in children younger than five years. Factors
that increase the severity of pneumonia in children include prematurity,
malnutrition, low socioeconomic status, tobacco smoke exposure, and child
care attendance.

Go to the next page if you knew the correct answer, or click the link image(s)
below to further research the concepts in this question (if desired).

Research Concepts:
Pneumonia, Community-Acquired

We update eBooks quarterly and Apps daily based on user feedback.


Please tap flag to report any questions that need improvement.
Question 21: A 17-year-old female presents with fever, cough with sputum,
and rigors. She has a past medical history significant for AIDS with a CD4 count
of 55 cells/mm3. An exam reveals coarse breath sounds in the right lower lobe,
and a chest radiograph reveals an opacity in the same area. What is the most
common organism to cause this patient's condition?

Choices:
1. Mycoplasma pneumoniae
2. Pneumocystis jiroveci
3. Staphylococcus aureus
4. Streptococcus pneumoniae
Answer: 4 - Streptococcus pneumoniae
Explanations:
Mycoplasma pneumoniae is an atypical bacteria that can cause pneumonia.
However, it is not the most common cause of pneumonia.
Pneumocystis jiroveci is a common cause of pneumonia in AIDS patients
with a CD4 count less than 200 cells/mm3. However, it is not the most
common cause of pneumonia.
Although Staphylococcus aureus can cause pneumonia, it typically is seen
following a viral infection.
In all patients, Streptococcus pneumoniae is the most common cause of
pneumonia.

Go to the next page if you knew the correct answer, or click the link image(s)
below to further research the concepts in this question (if desired).

Research Concepts:
Pneumonia, Streptococcus Pneumoniae

We update eBooks quarterly and Apps daily based on user feedback.


Please tap flag to report any questions that need improvement.
Question 22: A diver with missed decompression presents with vertigo and
tinnitus 4 hours post dive. There was no difficulty in equalizing the ears or any
ear pain. Neurological exam reveals nystagmus and decreased hearing on the left
but is otherwise normal. The diver reports no pain or other symptoms. What is
the most likely diagnosis?

Choices:
1. Otic barotrauma
2. Type 1 decompression sickness
3. Inner ear decompression sickness
4. Cerebral gas embolism
Answer: 3 - Inner ear decompression sickness
Explanations:
Otic barotrauma would likely present immediately or in the water.
Neurological decompression sickness (DCS), not otherwise involving the
inner ear, unlikely would present with both hearing and balance issues but
no other neurological findings.
Although inner ear DCS may be a form of arterial gas embolism, cerebral
gas embolism unlikely would present with both hearing and balance issues
but no other neurological issues.
Benign positional vertigo also may present like this. However, the history
of diving and missed decompression makes this unlikely.

Go to the next page if you knew the correct answer, or click the link image(s)
below to further research the concepts in this question (if desired).

Research Concepts:
Diving, Inner Ear Decompression Sickness

We update eBooks quarterly and Apps daily based on user feedback.


Please tap flag to report any questions that need improvement.
Question 23: A 6-year-old child presents with a hoarse and brassy cough
but no barky cough. She was diagnosed with a viral upper respiratory infection
days before. A neck x-ray demonstrates a steeple sign with tracheal lining
irregularities. Which of the following is the most common etiology of this child's
probable condition?

Choices:
1. Respiratory syncytial virus
2. Staphylococcus aureus
3. Group A Streptococcus
4. Haemophilus influenzae
Answer: 2 - Staphylococcus aureus
Explanations:
A brassy cough most likely indicates bacterial tracheitis. A barky cough
indicates croup. However, croup can precede bacterial tracheitis.
Bacterial tracheitis is most commonly caused by Staphylococcus aureus.
Streptococcus pneumonia is the most common cause of acute ear infections
in children.
Epiglottitis will demonstrate a thumbprint sign on lateral neck x-ray.

Go to the next page if you knew the correct answer, or click the link image(s)
below to further research the concepts in this question (if desired).

Research Concepts:
Tracheitis, Bacterial

We update eBooks quarterly and Apps daily based on user feedback.


Please tap flag to report any questions that need improvement.
Question 24: What is the most common skin manifestation of Osler-Weber-
Rendu syndrome?

Choices:
1. Telangiectasia
2. Purpura
3. Clubbing
4. Cyanosis

Photo:Contributed by Wikimedia Commons, Herbert L. Fred, MD and Hendrik A. van Dijk (CC by 2.0)
https://creativecommons.org/licenses/by/2.0/
Answer: 1 - Telangiectasia
Explanations:
The most common skin finding in patients with Osler-Weber-Rendu
syndrome is telangiectasias.
These lesions are usually seen on the oral mucosa, skin, and conjunctiva.
The skin changes can include clubbing and cyanosis.
The telangiectasias usually develop 12 months after the first nose bleed.

Go to the next page if you knew the correct answer, or click the link image(s)
below to further research the concepts in this question (if desired).

Research Concepts:
Osler-Weber-Rendu Disease (Hereditary Hemorrhagic Telangiectasia,
HHT)

We update eBooks quarterly and Apps daily based on user feedback.


Please tap flag to report any questions that need improvement.
Question 25: An 18-month-old, previously healthy child is admitted for
pneumonia with empyema. After fluid drainage using video-assisted
thoracoscopic surgery, a 2 cm lung abscess is seen. Which of the following
would be the most appropriate therapy?

Choices:
1. CT-guided needle drainage
2. Open thoracostomy urgently
3. Open thoracostomy after resolution of fever
4. Long-term intravenous antibiotics
Answer: 4 - Long-term intravenous antibiotics
Explanations:
Most lung abscesses can be treated with long-term parenteral antibiotics.
Staphylococcus aureus is the primary cause of a lung abscess in children
who are born and raised in the United States.
If methicillin-resistant S. aureus is determined to be the source of a lung
abscess, vancomycin and linezolid should be considered.
Thoracostomy can result in contamination of uninfected tissue.

Go to the next page if you knew the correct answer, or click the link image(s)
below to further research the concepts in this question (if desired).

Research Concepts:
Abscess, Lung

We update eBooks quarterly and Apps daily based on user feedback.


Please tap flag to report any questions that need improvement.
Question 26: A 5-year-old girl presents to the clinic with her parents with
complaints of bloody diarrhea, abdominal pain, and pallor. Her parents also
report a 3-year history of recurrent pneumonia, which typically resolves with
antibiotic or anti-fungal medications. Her laboratory results show signs of
malabsorption and anemia. Liver and thyroid functions test are normal. A
dihydrorhodamine test is abnormal. Chronic granulomatous disease is strongly
suspected. Given the patient’s likely diagnosis, with what condition are her
gastrointestinal symptoms are most similar?

Choices:
1. Celiac disease
2. Colon cancer
3. Gastric ulcers from a Helicobacter pylori infection
4. Inflammatory bowel disease
Answer: 4 - Inflammatory bowel disease
Explanations:
A small subset of patients suffering from chronic granulomatous disease
(CGD) experiences gastrointestinal symptoms similar to symptoms seen in
inflammatory bowel disease (IBD), especially Crohn disease.
Symptoms can include mild to bloody diarrhea and malabsorption.
Granulomata in CGD IBD have sharply defined histiocyte aggregates with
surrounding lymphocytic inflammation, unlike the poorly formed
granulomata often seen in Crohn disease.
The CGD genotype seems to accentuate the standard genetic risk associated
with IBD. Recent studies suggest that there may also be a relationship
between discoid lupus erythematosus with female carriers of X-linked
CGD.
While patients with CGD granulomatous colitis respond well to tumor
necrosis factor-alpha inhibition, such as infliximab, they often have
significant infectious complications, sometimes fatal. This is more common
than typical patients with Crohn disease.

Go to the next page if you knew the correct answer, or click the link image(s)
below to further research the concepts in this question (if desired).

Research Concepts:
Chronic Granulomatous Disease

We update eBooks quarterly and Apps daily based on user feedback.


Please tap flag to report any questions that need improvement.
Question 27: A macrosomic neonate is being evaluated for difficulty
breathing and is noted to have persistent grunting. Lung sounds are clear on
examination. Which of the following tests is most sensitive for detecting a
phrenic nerve injury?

Choices:
1. Chest CT
2. Chest radiograph
3. Diaphragmatic ultrasound
4. Fiberoptic bronchoscopy
Answer: 3 - Diaphragmatic ultrasound
Explanations:
Phrenic nerve injuries may lead to diaphragmatic dysfunction or paralysis.
An elevated hemidiaphragm on chest radiograph is suggestive of
diaphragmatic paralysis. Bilateral diaphragmatic paralysis can be easily
missed on chest radiograph.
Diaphragmatic ultrasound is more sensitive for detecting diaphragmatic
paralysis than a chest radiograph.
Diaphragmatic ultrasound provides direct visualization of each
hemidiaphragm and its function.

Go to the next page if you knew the correct answer, or click the link image(s)
below to further research the concepts in this question (if desired).

Research Concepts:
Phrenic Nerve Injury

We update eBooks quarterly and Apps daily based on user feedback.


Please tap flag to report any questions that need improvement.
Question 28: A 17-year-old patient was involved in a motor vehicle
accident. He was found to be hypotensive at the scene of the accident and had to
be resuscitated. He suffered multiple injuries to his lower extremities and
required numerous surgeries and prolonged mechanical ventilation. He was
started on a high concentration of enteral glucose feeds on a 24-hour protocol.
Four weeks later, he is still dependent on the mechanical ventilator, but his chest
x-ray remains clear. He has profound respiratory muscle weakness and the MRI
shows significant thinning of the diaphragm. Which of the following is the most
likely cause of his muscle weakness?

Choices:
1. Hypocalcemia
2. Hypophosphatemia
3. Zinc deficiency
4. Hypernatremia
Answer: 2 - Hypophosphatemia
Explanations:
Low phosphate levels can cause significant weakness of skeletal and
smooth muscle, which can affect the eyes as well as the diaphragm.
Respiratory insufficiency in patients on ventilators is common after
hypophosphatemia. Impaired cardiac contractility can also occur and
usually reverses when phosphate is reinstituted in the diet.
Administering high carbohydrate diets can lead to a lowering of phosphate
by stimulating insulin release. Insulin moves phosphate into the cells.
Diabetic ketoacidosis is another important cause of hypophosphatemia.
However, routine replacement of phosphate in these patients has not been
shown to reduce morbidity or mortality.

Go to the next page if you knew the correct answer, or click the link image(s)
below to further research the concepts in this question (if desired).

Research Concepts:
Hypophosphatemia

We update eBooks quarterly and Apps daily based on user feedback.


Please tap flag to report any questions that need improvement.
Question 29: A 17-year-old girl has a cough associated with exercise for
four months. She occasionally has white sputum. Her physical exam is normal.
What is the best next diagnostic test?

Choices:
1. Chest radiograph
2. Spirometry
3. Serum IgE
4. Trial of albuterol
Answer: 2 - Spirometry
Explanations:
The patient may have exercise-induced asthma.
Spirometry before exercise, five minutes after exercise, and ten minutes
after exercise will confirm the diagnosis.
Treatment is with albuterol before exercise.
Exercise-induced bronchoconstriction better reflects the underlying
pathophysiology. In a patient with EIB, bronchodilation develops and peaks
at around 10–15 minutes, and usually resolves itself within an hour. During
an episode of bronchoconstriction, the levels of inflammatory mediators,
particularly leukotrienes, histamine, and interleukin may increase. TH2-
type lymphocytes are activated, with an increase in T cells expressing
CD25 (IL-2R), and B cells expressing CD 23, causing increased production
of IgE. After exercise, the conditions fades in one to three minutes. In EIB,
this is followed by a refractory period less than four hours. If exercise is
repeated, the bronchoconstriction is usually not as severe. The underlying
cause of EIB bronchoconstriction appears to be a large volume of cool, dry
air inhaled during strenuous exercise. The condition improves when the air
inhaled is more fully humidified and closer to body temperature.

Go to the next page if you knew the correct answer, or click the link image(s)
below to further research the concepts in this question (if desired).

Research Concepts:
Asthma, Exercise Induced

We update eBooks quarterly and Apps daily based on user feedback.


Please tap flag to report any questions that need improvement.
Question 30: An HIV positive male, admitted to the intensive care unit with
septic shock secondary to lobar pneumonia, is being mechanically ventilated.
After 2 days of intravenous ceftriaxone, trimethoprim-sulfamethoxazole, and
erythromycin, his fever has resolved. Blood cultures are positive for
Streptococcus pneumoniae. Oxygen saturation is 92% on room air and blood
pressure is 80/40 mm Hg, despite adequate fluids and 2 micrograms/kg/min of
norepinephrine. Further examination reveals warm skin and full peripheral
pulses. What is the best next step in management?

Choices:
1. Order a CT scan of the sinuses
2. Cosyntropin stimulation test followed by hydrocortisone 100 mg IV every 6
hours
3. Place a pulmonary artery catheter
4. Check sensitivity of the bacteria to antibiotics
Answer: 2 - Cosyntropin stimulation test followed by hydrocortisone 100 mg
IV every 6 hours

Explanations:
Always consider adrenal insufficiency in the differential diagnosis of
distributive shock, particularly when no other cause is present or an original
cause appears to be under effective treatment, as in this patient.
Refractory hypotension suggests adrenal insufficiency, especially with
concomitant HIV infection in the face of effective treatment of sepsis.
Searching for a second source of infection would be appropriate in an HIV-
infected patient.
Diagnosis of primary adrenal insufficiency requires suspicion.
Hyponatremia with hyperkalemia and hypoglycemia may be present. Serum
cortisol, adrenocorticotropic hormone (ACTH), renin, aldosterone, and
chemistry panel should be obtained early. Serum cortisol levels can help
make a diagnosis in the presence of elevated ACTH and plasma renin
activity. The ACTH test can be performed to determine if the cause is
central or peripheral. Patients with secondary adrenal insufficiency are
more likely to have hypoglycemia, but will not have dehydration,
hyperkalemia, or skin hyperpigmentation. Consider testing for HIV and
tuberculosis in patients with unclear etiology.

Go to the next page if you knew the correct answer, or click the link image(s)
below to further research the concepts in this question (if desired).

Research Concepts:
Adrenal, Insufficiency

We update eBooks quarterly and Apps daily based on user feedback.


Please tap flag to report any questions that need improvement.
Question 31: An 8-month old previously healthy male who is fully
vaccinated is brought to the emergency department for lethargy. The vital signs
are temperature 99.6 F, respiratory rate 9/minute, pulse 106 bpm, blood pressure
95/50 mmHg, and pulse oximetry 75% on room air. His eyes are dilated, and his
breathing is shallow. There are no rashes or signs of trauma. The parents deny
that he could have gotten into anything but do state that they recently gave the
child some canned peaches from last year’s harvest. The patient has minimal
response to painful stimuli and when he has a weak gag reflex. After the patient
is stabilized and placed on mechanical ventilation, what is the definite treatment
that can decrease hospital stay and days requiring mechanical ventilation?

Choices:
1. Antibiotics
2. Corticosteroids
3. Intravenous immunoglobulin
4. Human-derived botulism immune globulin
Answer: 4 - Human-derived botulism immune globulin
Explanations:
This patient has a diagnosis of infantile botulism that is due to the
contaminated canning process of the peaches. The gram-positive anaerobic
Clostridium botulinum spore causes decrease release of acetylcholine in the
presynaptic cleft caused flaccid paralysis causes infantile botulism.
Symptoms are consistent with increased lethargy, constipation, depressed
respiration, poor feeding, and suck. The first line treatment is supportive
care since most of these patients will experience respiratory failure that will
require mechanical ventilation.
However in terms of what will decrease hospital length of stay and decrease
the number of overall mechanical ventilation days is human-derived
botulism immune globulin. There is no indication to start antibiotics in
infantile botulism.
Corticosteroids and intravenous immunoglobulin have not been shown to
be of benefit for infantile botulism.

Go to the next page if you knew the correct answer, or click the link image(s)
below to further research the concepts in this question (if desired).

Research Concepts:
Infantile Botulism

We update eBooks quarterly and Apps daily based on user feedback.


Please tap flag to report any questions that need improvement.
Question 32: A 28-year-old female presents with shortness of breath for 3
days which is progressively getting worse. In the emergency department, the
blood workup shows white cell count of 14,000 with 78% neutrophils 78% and
11% eosinophils. Basic metabolic profile shows sodium of 136 mmol/L,
potassium 4.1 mmol/L, chloride 100 mmol/L, and bicarbonate 20 mmol/L. The
patient has seasonal allergies, and family history is positive for asthma in her
mother. Chest x-ray is normal, EKG shows sinus tachycardia. Arterial blood gas
(ABG) shows pH of 7.2, PCO2 of 60, and bicarbonate of 20. Initially, the patient
was given albuterol and Ipratropium nebulization and intravenous
corticosteroids, was put on BiPAP, but she did not improve. Her shortness of
breath increased, and she was using accessory muscles. She is intubated and
placed on a ventilator with settings AC mode tidal volume of 400, rate of 24,
PEEP of 5, and FiO2 40%. ABG was drawn after 1 hour which shows pH 7.16,
PCO2 66, and bicarbonate of 25. What is the acid-base disorder patient has on
last ABG and how should the ventilator settings be adjusted?

Choices:
1. The patient has metabolic acidosis and the respiratory rate should be increased
to 26, flow should be decreased
2. The patient has respiratory acidosis with metabolic compensation and
respiratory rate should be lowered to 20, flow should be increased
3. The patient has respiratory and metabolic acidosis. The respiratory rate and
flow rate should not be changed
4. The patient has metabolic alkalosis with respiratory acidosis and respiratory
rate should be increased to 30 and flow should be increased
Answer: 2 - The patient has respiratory acidosis with metabolic compensation
and respiratory rate should be lowered to 20, flow should be increased

Explanations:
The patient has respiratory acidosis with metabolic compensation. The
respiratory rate should be lowered to 20, flow should be increased.
Respiratory acidosis is a state in which there is usually a failure of
ventilation and an accumulation of carbon dioxide. The primary disturbance
of elevated arterial PCO2 decreases the ratio of bicarbonate to arterial
PCO2, which leads to a lowering of the pH. In the presence of alveolar
hypoventilation, two features commonly seen are respiratory acidosis and
hypercapnia.Â
The patient's acidosis worsened as she developed auto peep.
To correct auto peep her I:E ratio has to be adjusted. Lower the respiratory
rate which will give her more expiratory time. Increasing the flow will also
help in getting more expiratory time and will help in acidosis. ABG should
be repeated in 30 to 45 minutes.
Carbon dioxide plays a remarkable role in the human body mainly through
its role in pH regulation of the blood. It is the primary stimulus to initiate
ventilation. In its normal state, the body maintains CO2 in a well-controlled
range from 38 to 42 mmHg by balancing its production and elimination.

Go to the next page if you knew the correct answer, or click the link image(s)
below to further research the concepts in this question (if desired).

Research Concepts:
Physiology, Acidosis, Respiratory

We update eBooks quarterly and Apps daily based on user feedback.


Please tap flag to report any questions that need improvement.
Question 33: A 2-year-old female presents to the emergency department
with a dry cough and a low-grade fever. Over the last 2 days, she has become
very irritable and refused to eat. The mother reports that the girl also had a runny
nose. On examination, the child has significant wheezing and is tachycardic. A
test of the nasal discharge shows a respiratory syncytial virus infection. What
condition is she at risk for developing later in life?

Choices:
1. Lung cancer
2. Lung abscess
3. Bronchiectasis
4. Asthma
Answer: 4 - Asthma
Explanations:
Infants hospitalized with respiratory syncytial virus (RSV) have a higher
risk for subsequent wheezing later in life.
Some studies question whether reactive airway disease is already present in
patients wheezing in the presence of RSV at age less than 2 years.
Some studies show that fewer infants treated with palivizumab develop
wheezing later in life than those who do not receive palivizumab,
supporting the idea that RSV predisposes to later asthma and atopy.
Current evidenced-based practice does not support the use of ribavirin.
Ribavirin is an antiviral with limited positive effects that has been linked to
adverse effects for patients. The risk of adverse effects to the patient and
provider likely outweigh any benefits of the drug, however, in the case of
immunocompromised patients with severe RSV, it may be considered.
Palivizumab is an immunization recommended for RSV in high-risk infants
and children no older than 24 months. Its efficacy has been established.

Go to the next page if you knew the correct answer, or click the link image(s)
below to further research the concepts in this question (if desired).

Research Concepts:
Respiratory Syncytial Virus Infection (RSV)

We update eBooks quarterly and Apps daily based on user feedback.


Please tap flag to report any questions that need improvement.
Question 34: A patient has a sore throat, cough, fever, and general malaise
for 5 days. Examination reveals that he has a thick, silvery-grey exudate on the
tonsils and bilateral lymphadenopathy. Scraping the tonsils leads to bleeding.
What is the drug of choice for this condition?

Choices:
1. Vancomycin
2. Erythromycin
3. Sulfadiazine
4. Amikacin
Answer: 2 - Erythromycin
Explanations:
Diphtheria can cause severe enlargement of lymph nodes in the neck.
Moreover, the patients have to be observed as abnormal heart rhythms can
occur.
Classic treatment involves the use of erythromycin or procaine penicillin.
Diphtheria antitoxin does not neutralize the toxin that has already become
bound to the nerves.
Bronchoscopy can be performed to remove pseudomembranes that have
obstructed the pharynx.

Go to the next page if you knew the correct answer, or click the link image(s)
below to further research the concepts in this question (if desired).

Research Concepts:
Diphtheria

We update eBooks quarterly and Apps daily based on user feedback.


Please tap flag to report any questions that need improvement.
Question 35: A 3-year-old child is seen in the clinic with recurrent
respiratory tract infections. A lung examination reveals an empty right upper
quadrant and a full left lower quadrant. Chest x-ray reveals that the base of the
heart is in the right chest cavity. What is the most likely diagnosis?

Choices:
1. Congenital lobar emphysema
2. Kartagener disease
3. Cystic fibrosis
4. DiGeorge syndrome
Answer: 2 - Kartagener disease
Explanations:
Kartagener disease usually presents with recurrent sinopulmonary
infections due to impaired ciliary tract function.
Other features of the disorder include situs inversus, due to impaired ciliary
motion during embryogenesis. There is a lateral transposition of lungs,
abdominal and thoracic viscera are on opposite sides of the body from
normal.
Kartagener disease can also present with dextrocardia and male sterility.
It has an autosomal recessive inheritance pattern.

Go to the next page if you knew the correct answer, or click the link image(s)
below to further research the concepts in this question (if desired).

Research Concepts:
Ciliary Dysfunction (Kartagener Syndrome, Primary Ciliary
Dyskinesia)

We update eBooks quarterly and Apps daily based on user feedback.


Please tap flag to report any questions that need improvement.
Question 36: A 3-year-old child develops a staphylococcal infection and
soon after develops a parapneumonic effusion. A chest tube is placed. Despite
antibiotics, the effusion enlarges. He continues to have a fever. At this point,
what is the appropriate management?

Choices:
1. Decortication
2. Change antibiotics
3. Video-assisted thoracoscopic surgery
4. Place a second chest tube
Answer: 4 - Place a second chest tube
Explanations:
In children, unlike adults, most empyema resolves with chest tube drainage
and seldom requires an open procedure.
Children improve with adequate drainage, and this patient needs a second
chest tube.
Staphylococcal empyema requires drainage.
In the rare case that the empyema is loculated, a surgical procedure is
required.

Go to the next page if you knew the correct answer, or click the link image(s)
below to further research the concepts in this question (if desired).

Research Concepts:
Empyema

We update eBooks quarterly and Apps daily based on user feedback.


Please tap flag to report any questions that need improvement.
Question 37: A 17-year-old is brought to the emergency department by
ambulance after developing sudden shortness of breath. A chest radiograph
shows a 10% pneumothorax. The patient remains stable, is in no respiratory
distress, pulse oximetry is 100% on room air, and his blood pressure is normal.
What would appropriate management include?

Choices:
1. Immediate chest tube placement
2. Intubation and mechanical ventilation
3. Pulmonary function testing
4. Observation
Answer: 4 - Observation
Explanations:
Pneumothorax is the accumulation of air within the pleural space. The usual
cause of pneumothorax is a penetrating wound such as a stabbing, gunshot
wound, or deceleration-type injury as seen in motor vehicle accidents.
Spontaneous pneumothorax also can occur as a result of a ruptured bleb.
This typically affects tall, thin men or smokers. Clinical findings include
decreased breath sounds on the side affected, shortness of breath, chest
pain, and cough. A chest radiograph is the initial diagnostic test.
Treatment may require immediate intervention but depends on the extent of
the pneumothorax and clinical symptoms of the patient. If the
pneumothorax involves less than 15% to 20% of lung volume, observation
is the only treatment necessary with supplemental oxygen administration.
For larger pneumothoraces, chest tube thoracostomy is necessary. Tension
pneumothoraces require emergent decompression with a large-bore needle
placed in the second intercostal space, followed by chest tube placement.

Go to the next page if you knew the correct answer, or click the link image(s)
below to further research the concepts in this question (if desired).

Research Concepts:
Pneumothorax, Spontaneous

We update eBooks quarterly and Apps daily based on user feedback.


Please tap flag to report any questions that need improvement.
Question 38: A 10-year-old female with asthma comes in with a complaint
of a low-grade fever and increased wheezing. Sputum examination shows
Curschmann spirals. There is peripheral eosinophilia on the CBC. Which of the
following is the most likely cause of the exacerbation?

Choices:
1. Tree pollen
2. Aspergillus fumigatus
3. Staphylococcus aureus
4. Streptococcus pneumonia
Answer: 2 - Aspergillus fumigatus
Explanations:
Aspergillus fumigatus causes eosinophilic pneumonia in those with asthma
and those with cystic fibrosis. Chest CT has become the imaging method of
choice in evaluating patients suspected of having allergic
bronchopulmonary aspergillosis (ABPA).
The diagnosis of ABPA is based on a serum IgE level greater than 1000
IU/L, a positive specific IgG to Aspergillus, and a positive prick skin test
for Aspergillus antigen.
Tree pollen, Streptococcus pneumonia, and Staphylococcus aureus would
not cause eosinophilic pneumonia. Common causes of eosinophilic
pneumonia are fungal, parasitic, drug-induced, vasculitic, or cryptogenic.
ABPA is estimated to occur in 2.5% of patients with asthma. Clinical
presentation of ABPA is characterized by asthma and recurrent
exacerbations. In severe cases, episodes of bronchial obstruction, fever,
malaise, expectoration of brownish mucus plugs, and hemoptysis may
occur.

Go to the next page if you knew the correct answer, or click the link image(s)
below to further research the concepts in this question (if desired).

Research Concepts:
Aspergillus Fumigatus

We update eBooks quarterly and Apps daily based on user feedback.


Please tap flag to report any questions that need improvement.
Question 39: A 4-year old boy from the United States was recently
diagnosed with chronic granulomatous disease. His provider advises the patient’s
parents to begin prophylactic treatment to prevent the rate of severe infections.
Which drug regimen is most likely to be recommended?

Choices:
1. Amphotericin B and penicillin
2. Trimethoprim-sulfamethoxazole only
3. Trimethoprim-sulfamethoxazole and itraconazole
4. There is no prophylactic treatment given. Children are only treated once
symptoms of an infection are present
Answer: 3 - Trimethoprim-sulfamethoxazole and itraconazole
Explanations:
When chronic Granulomatous Disease (CGD) was first identified, affected
children had near certain fatality, but now CGD can be managed with a
higher survival rate.
Management of CGD is based lifelong antibacterial and antifungal
prophylaxis, early diagnosis of infection, and aggressive management of
infectious complications. The standard prophylactic treatment for children
with CGD is trimethoprim-sulfamethoxazole and itraconazole.
Some countries also include interferon-gamma but this is not a globally
accepted treatment. Clinicians practicing outside the United States often
omit interferon-gamma.
Studies have shown that these drugs dramatically reduce the rate of severe
infections in children with CGD.

Go to the next page if you knew the correct answer, or click the link image(s)
below to further research the concepts in this question (if desired).

Research Concepts:
Chronic Granulomatous Disease

We update eBooks quarterly and Apps daily based on user feedback.


Please tap flag to report any questions that need improvement.
Question 40: A 2-year-old patient with presumed laryngotracheobronchitis
is treated with nebulized epinephrine and oral dexamethasone. Two hours later
the patient's oxygen saturation is 93% on room air. The stridor and retractions
are improved but not resolved. What is the appropriate management of this
patient?

Choices:
1. Discharge to home with follow up in a 2 to 3 days
2. Discharge to home on cool mist humidified air
3. Admit the patient for further treatment
4. Observe the patient for an additional 4 hours
Answer: 3 - Admit the patient for further treatment
Explanations:
The patient still has symptoms and the oxygen saturation should be over
95%.
The patient should be admitted for continued nebulized epinephrine.
If the patient is not improving, one also must consider a foreign body,
epiglottitis, or bacterial tracheitis.
An otolaryngologist must be consulted to assess the patient. If the patient is
not improving, an alternate diagnosis must be entertained.

Go to the next page if you knew the correct answer, or click the link image(s)
below to further research the concepts in this question (if desired).

Research Concepts:
Laryngotracheobronchitis

We update eBooks quarterly and Apps daily based on user feedback.


Please tap flag to report any questions that need improvement.
Question 41: A patient has been receiving mechanical ventilation with a
balloon-cuffed tracheostomy tube for the past 3 weeks after suffering a severe
blunt laryngeal injury. On two occasions within 24 hours, about 30 ml of blood
have been suctioned from the tracheostomy tube. What is the most likely source
of the bleeding?

Choices:
1. Lung pathology
2. Granulation tissue at the tracheostomy site
3. Erosion into the internal jugular vein
4. Erosion into the innominate artery
Answer: 4 - Erosion into the innominate artery
Explanations:
Any patient with tracheal bleeding that amounts to more than blood
streaking of the tracheal aspirate, 48 hours or more after tracheostomy, must
be assumed to have tracheal erosion into the innominate artery until proven
otherwise.
Episodes of such minimal blood loss tend to precede or herald the
development of a massive and frequently fatal hemorrhage. Recognition of
the significance of such seemingly minor bleeding episodes, with
appropriate subsequent management, offers the best chance for survival.
Patients with severe bleeding should be immediately re-intubated and
corrective surgery should be undertaken if the diagnosis is confirmed.
Massive bleeding from the tracheostomy site invariably comes from an
erosion of the innominate artery and is usually fatal unless rapidly
controlled.
In most cases, bleeding stops temporarily with further inflation of the
balloon cuff until the patient is taken to the operating room. Sometimes the
innominate artery can be compressed against the back of the sternum with a
finger introduced through the tracheostomy incision, between the trachea
and the artery.

Go to the next page if you knew the correct answer, or click the link image(s)
below to further research the concepts in this question (if desired).

Research Concepts:
Fistula, Tracheo Innominate Artery

We update eBooks quarterly and Apps daily based on user feedback.


Please tap flag to report any questions that need improvement.
Question 42: A patient was started on an antibiotic for the treatment of a
urinary tract infection. She has continued taking the antibiotic for 4 months
because of recurrent dysuria. She now presents with a 3-week history of dyspnea
and a dry cough. She denies any weight loss or smoking. What is the most likely
etiology?

Choices:
1. Sulfonamide
2. Nitrofurantoin
3. Congestive heart failure
4. Goodpasture syndrome
Answer: 2 - Nitrofurantoin
Explanations:
Nitrofurantoin can cause pulmonary fibrosis when used for long periods.
Patients may present with a gradual cough and flu like symptoms.
The pulmonary fibrosis is often not reversible even if the drug is stopped.
The mechanism of pulmonary toxicity due to nitrofurantoin is not known.

Go to the next page if you knew the correct answer, or click the link image(s)
below to further research the concepts in this question (if desired).

Research Concepts:
Pulmonary Fibrosis, Interstitial (Nonidiopathic)

We update eBooks quarterly and Apps daily based on user feedback.


Please tap flag to report any questions that need improvement.
Question 43: A 4-month-old female is brought to the emergency room after
she stops breathing for about 30 seconds. The mother reports her lips became
blue and she didn't resume breathing until she blew in her face. The patient had
no symptoms before the event. She is breastfed and has had all recommended
immunizations. The mother's first child died of SIDS. On exam, the patient is
irritable and alert. There is dried blood at both nares. Oxygen saturation is 97
percent on room air. The rest of the exam is normal. What is indicated by the
physical exam and the fact that the first child died from SIDS?

Choices:
1. There is an increased risk of SIDS in this child
2. There is suspicion for non-accidental trauma
3. There is need for extensive evaluation
4. There is risk for apparent life-threatening events (ALTE)
Answer: 2 - There is suspicion for non-accidental trauma
Explanations:
Babies who've had siblings or cousins die of SIDS are at higher risk of
SIDS, but the exam findings do not suggest SIDS.
The previous reported history of SIDS in the family with a child presenting
with an apparent life-threatening event (ALTE) is suspicious, given the
blood at the nares.
Most patients with ALTE are not at increased risk for SIDS.
SIDS is a sudden death of a child or infant that is unexplained by both
history and autopsy.

Go to the next page if you knew the correct answer, or click the link image(s)
below to further research the concepts in this question (if desired).

Research Concepts:
Apnea, Children

We update eBooks quarterly and Apps daily based on user feedback.


Please tap flag to report any questions that need improvement.
Question 44: Which of the following medications is found to be helpful in
patients with cystic fibrosis?

Choices:
1. Dornase alfa
2. Ipratropium
3. Albuterol
4. Theophylline
Answer: 1 - Dornase alfa
Explanations:
Patients with cystic fibrosis suffer from thick mucus secretions and repeated
chest infections.
Dornase alfa is helpful to thin the unusually thick respiratory secretions in
cystic fibrosis. N-acetylcysteine was used in the past.
Its mucolytic action works by breaking the sulfhydryl groups found in
mucus secretion.
Bronchodilators may not be of much help in cystic fibrosis.

Go to the next page if you knew the correct answer, or click the link image(s)
below to further research the concepts in this question (if desired).

Research Concepts:
Dornase Alfa

We update eBooks quarterly and Apps daily based on user feedback.


Please tap flag to report any questions that need improvement.
Question 45: At what point should hyperoxygenation be administered when
performing tracheal suctioning on a mechanically ventilated patient?

Choices:
1. Before the procedure
2. After the procedure
3. Before and after the procedure
4. During the procedure
Answer: 3 - Before and after the procedure
Explanations:
Mechanically ventilated patients should be hyperoxygenated before the
performance of tracheal suctioning.
Mechanically ventilated patients should be hyperoxygenated after the
performance of tracheal suctioning.
100% oxygen should be delivered for >30 seconds prior to the procedure.
100% oxygen should be delivered for >1 minute after the procedure.

Go to the next page if you knew the correct answer, or click the link image(s)
below to further research the concepts in this question (if desired).

Research Concepts:
Ventilation, Mechanical

We update eBooks quarterly and Apps daily based on user feedback.


Please tap flag to report any questions that need improvement.
Question 46: Which of the following is false about tuberculosis of the lung?
Choices:
1. If sputum is still positive after 4 months of chemotherapy, patients should
have surgery
2. Tuberculin skin tests are read after 48 to 72 hours
3. Induration of 10 to 15 mm is considered positive depending on risk factors
4. Pure tuberculous empyema is preferably treated with thoracentesis rather than
thoracostomy
Answer: 1 - If sputum is still positive after 4 months of chemotherapy,
patients should have surgery

Explanations:
If there are no complications with medical therapy, patients with
tuberculosis need no surgery.
Medical treatment for at least 6 months with triple-drug therapy is required.
Tuberculosis empyemas are treated with chemotherapy and thoracentesis.
Indications for surgery in tuberculosis include bronchopleural fistulas,
hemoptysis, bronchial stenosis, suspicion of cancer, and empyema.
All non-functioning lung should be resected and a double lumen tube is
recommended.

Go to the next page if you knew the correct answer, or click the link image(s)
below to further research the concepts in this question (if desired).

Research Concepts:
Tuberculosis

We update eBooks quarterly and Apps daily based on user feedback.


Please tap flag to report any questions that need improvement.
Question 47: What device is utilized to deliver the proper oxygen percent
from the ventilator?

Choices:
1. Gas tank
2. Oxygen analyzer
3. Tube patency
4. Machine power
Answer: 2 - Oxygen analyzer
Explanations:
A properly calibrated oxygen analyzer should be routinely used to verify
that the ventilator is providing the correct oxygen percent.
Oxygen analyzers continuously measure and display the FIO2 in the
breathing system.
Oxygen analyzers indicate when inspired oxygen deviates from desired
limits. The oxygen analyzer is the last line of defense against hypoxic
mixtures being delivered to the patient.
An oxygen analyzer is necessary and readings must be charted for every
patient receiving a general anesthetic.

Go to the next page if you knew the correct answer, or click the link image(s)
below to further research the concepts in this question (if desired).

Research Concepts:
Ventilation, Mechanical

We update eBooks quarterly and Apps daily based on user feedback.


Please tap flag to report any questions that need improvement.
Question 48: During a thoracentesis, the needle should be introduced at
what location?

Choices:
1. The upper intercostal space
2. The middle intercostal space
3. The lower intercostal space
4. The subxiphoid space
Answer: 3 - The lower intercostal space
Explanations:
The intercostal nerve, artery, and vein are in the high intercostal space.
The needle should be advanced just above the rib in the low intercostal
space.
A posterior approach is generally used.
The subxiphoid space is used for pericardiocentesis.

Go to the next page if you knew the correct answer, or click the link image(s)
below to further research the concepts in this question (if desired).

Research Concepts:
Thoracentesis

We update eBooks quarterly and Apps daily based on user feedback.


Please tap flag to report any questions that need improvement.
Question 49: A patient has persistent dyspnea and mild stridor several days
following extubation after a prolonged period of intubation and mechanical
ventilation for management of ARDS. What is the most likely cause?

Choices:
1. Vocal cord damage
2. Tracheal stenosis
3. Tracheoesophageal fistula
4. Normal process
Answer: 2 - Tracheal stenosis
Explanations:
Tracheal stenosis is a result of tracheal scar formation from cuff.
Plain AP and lateral radiographic images can be done initially.
High-resolution computed tomography, or CT, scanning of the neck and
thorax is more specific.
The definitive test is bronchoscopy.

Go to the next page if you knew the correct answer, or click the link image(s)
below to further research the concepts in this question (if desired).

Research Concepts:
Trachea, Laryngotracheal Stenosis

We update eBooks quarterly and Apps daily based on user feedback.


Please tap flag to report any questions that need improvement.
Question 50: What anti-asthmatic drug can cause convulsions and
arrhythmias?

Choices:
1. Prednisone
2. Salmeterol
3. Zafirlukast
4. Theophylline
Answer: 4 - Theophylline
Explanations:
Theophylline toxicity can cause abdominal pain, seizures, confusion,
tremors, change in behavior, convulsions, dark or bloody vomit, dizziness,
diarrhea, arrhythmias, nervousness, and restlessness.
The side-effects of theophylline limit its use for long-term therapy in
asthma.
Theophylline is very effective at relaxing bronchial smooth muscles. It also
can increase myocardial contractility and act as a mild inotropic agent.
Theophylline toxicity is treated with beta-blockers. Most people are not
able to tolerate the tachyarrhythmias associated with the agent.

Go to the next page if you knew the correct answer, or click the link image(s)
below to further research the concepts in this question (if desired).

Research Concepts:
Toxicity, Theophylline

We update eBooks quarterly and Apps daily based on user feedback.


Please tap flag to report any questions that need improvement.
Question 51: Which is true about surgery for the treatment of
pneumothorax?

Choices:
1. Is never indicated for a primary spontaneous pneumothorax
2. Has remained unchanged over the past 25 years
3. Always involves resection of apical blebs
4. Is highly effective at preventing recurrence
Answer: 4 - Is highly effective at preventing recurrence
Explanations:
Surgical treatment of pneumothorax in numerous studies is found to be
effective in more than 90% of patients. Video-assisted thoracoscopic
surgery (VATS) for this disease has become the standard.
Several indications for surgical treatment of a primary pneumothorax exist.
Surgery often involves resection of apical blebs, but blebs may be found
elsewhere in the lung, necessitating a thorough search at the time of
surgery.
Occasionally blebs cannot be identified at surgery and some form of
pleurodesis should be carried out.

Go to the next page if you knew the correct answer, or click the link image(s)
below to further research the concepts in this question (if desired).

Research Concepts:
Pneumothorax

We update eBooks quarterly and Apps daily based on user feedback.


Please tap flag to report any questions that need improvement.
Question 52: Which of the following is not commonly affected by
Pneumocystis jiroveci?

Choices:
1. Eye
2. Heart
3. Bone marrow
4. Lymph node
Answer: 2 - Heart
Explanations:
Several extrapulmonary manifestations of Pneumocystis jiroveci may occur
in the absence of lung involvement.
The brain often is infected with Pneumocystis jiroveci, followed by the
bone marrow, lymph nodes, eyes, thyroid, and gut.
In rare cases, there may be generalized lymphadenopathy.
Some patients may present with an enlarging thyroid mass.

Go to the next page if you knew the correct answer, or click the link image(s)
below to further research the concepts in this question (if desired).

Research Concepts:
Pneumonia, Pneumocystis (Carinii) Jiroveci

We update eBooks quarterly and Apps daily based on user feedback.


Please tap flag to report any questions that need improvement.
Question 53: Which of the following is FALSE about atypical
mycobacterial infections?

Choices:
1. In immunocompetent patients it is more common in females.
2. Often associated with cavitations
3. They are often diagnosed by PCR
4. Resistance to drugs is rare
Answer: 4 - Resistance to drugs is rare
Explanations:
Mycobacterium avium complex (MAC) can cause several syndromes.
HIV disease is associated with disseminated infections. Isolated pulmonary
disease can be seen in immunocompetent patients.
Cervical lymphadenopathy is seen in children. Postmenopausal women are
more commonly infected than men.
Cavitary disease is seen especially in patients with underlying pulmonary
disease. PCR is often used for diagnosis. Multiple drug resistance is
common.

Go to the next page if you knew the correct answer, or click the link image(s)
below to further research the concepts in this question (if desired).

Research Concepts:
Atypical Mycobacterial Disease

We update eBooks quarterly and Apps daily based on user feedback.


Please tap flag to report any questions that need improvement.
Question 54: A patient is diagnosed with subacute hypersensitivity
pneumonitis secondary to exposure to feathers and bird droppings at work in a
pet store. Select the best treatment.

Choices:
1. Corticosteroids
2. Avoidance of the allergen and corticosteroids
3. Doxycycline and corticosteroids
4. Doxycycline
Answer: 2 - Avoidance of the allergen and corticosteroids
Explanations:
Treatment of subacute hypersensitivity pneumonitis is corticosteroids and
avoidance of the allergen.
If exposure is unavoidable, a mask may be of help.
Acute hypersensitivity pneumonitis is characterized by fevers, chills, and
shortness of breath 6 to 8 hours after exposure.
The subacute presentation is worsening of dyspnea on exertion and dry
cough developing over weeks to months.

Go to the next page if you knew the correct answer, or click the link image(s)
below to further research the concepts in this question (if desired).

Research Concepts:
Hypersensitivity Pneumonitis

We update eBooks quarterly and Apps daily based on user feedback.


Please tap flag to report any questions that need improvement.
Question 55: After an aortopexy is performed for tracheomalacia at age 2 in
a child, what is a major risk in future?

Choices:
1. Sternal dehiscence
2. Aneurysm formation
3. Aortic dissection
4. Subclavian steal
Answer: 2 - Aneurysm formation
Explanations:
Aortopexy is not a benign procedure.
Once the aorta is tacked on to the posterior sternum, bleeding can occur
through the sutures and in the long term, aneurysm development has been
observed.
Operative failures are very common with this procedure.
Aortopexy should generally be a treatment of last resort.

Go to the next page if you knew the correct answer, or click the link image(s)
below to further research the concepts in this question (if desired).

Research Concepts:
Tracheobronchomalacia

We update eBooks quarterly and Apps daily based on user feedback.


Please tap flag to report any questions that need improvement.
Question 56: Which of the following is the best treatment for
histoplasmosis?

Choices:
1. Sulfamethoxazole
2. Erythromycin
3. Fluconazole
4. Nystatin
Answer: 3 - Fluconazole
Explanations:
Fluconazole, itraconazole, and amphotericin B are the most commonly used
medications.

Go to the next page if you knew the correct answer, or click the link image(s)
below to further research the concepts in this question (if desired).

Research Concepts:
Histoplasmosis

We update eBooks quarterly and Apps daily based on user feedback.


Please tap flag to report any questions that need improvement.
Question 57: Which is the most likely causal organism of pneumonia in a
patient with cystic fibrosis?

Choices:
1. Pseudomonas
2. H influenza
3. Influenza
4. Streptococcus
Answer: 1 - Pseudomonas
Explanations:
Streptococcus pneumonia is responsible for most community-acquired
pneumonia.
Pseudomonas is most common in patients with cystic fibrosis.
Staphylococcus aureus is common in elderly.

Go to the next page if you knew the correct answer, or click the link image(s)
below to further research the concepts in this question (if desired).

Research Concepts:
Pseudomonas Aeruginosa

We update eBooks quarterly and Apps daily based on user feedback.


Please tap flag to report any questions that need improvement.
Question 58: Which of the following statements about diaphragmatic
eventration is false?

Choices:
1. Most eventration are asymptomatic
2. Phrenic nerve injury may be the cause
3. All eventration require treatment
4. Fluoroscopy can be diagnostic
Answer: 3 - All eventration require treatment
Explanations:
Eventrations can be either acquired or congenital. Most acquired cases are
due to unilateral phrenic nerve injury. Other causes include trauma, surgery,
and birth injury.
Eventrations result in a floppy diaphragm. Most cases are due to injury to
the phrenic nerve. In adults, the most common cause of phrenic nerve
paralysis is a viral infection.
The abdominal organs push into the thorax and interfere with lung function.
In the adult, most do not require treatment. In children, respiratory tract
infections from lower lobe atelectasis and failure to be weaned off
mechanical ventilation may require plication.
Both ultrasound and fluoroscopy are diagnostic of eventration.

Go to the next page if you knew the correct answer, or click the link image(s)
below to further research the concepts in this question (if desired).

Research Concepts:
Diaphragm, Eventration

We update eBooks quarterly and Apps daily based on user feedback.


Please tap flag to report any questions that need improvement.
Question 59: Which is not a proper management step for an inverse
inspiratory:expiratory ratio?

Choices:
1. Decreased tidal volume
2. Increased flow rate
3. Decreased respiratory rate
4. Increased FIO2
Answer: 4 - Increased FIO2
Explanations:
Increasing the FIO2 does not alter inspiratory:expiratory (I:E) ratio.
Inverse ratio ventilation (IRV) is a subset of pressure-controlled ventilation
in which inflation time is prolonged (inspiratory reserve volume, 1:1, 2:1,
or 3:1 may be used). A normal I:E is 1:3.
This will lower peak airway pressure but increase mean airway pressures.
The result may be improved oxygenation but at the expense of
compromised cardiac output and venous return. It is not clear if this mode
of ventilation leads to improved survival.
IRV's major indication is acute respiratory distress syndrome with
refractory hypoxemia or hypercapnia in other modes of ventilation.

Go to the next page if you knew the correct answer, or click the link image(s)
below to further research the concepts in this question (if desired).

Research Concepts:
Ventilation, Inverse Ratio

We update eBooks quarterly and Apps daily based on user feedback.


Please tap flag to report any questions that need improvement.
Question 60: A 2-year-old is diagnosed with laryngotracheobronchitis and
treated with nebulized epinephrine. Two hours later, he is doing well and the
fever has resolved with acetaminophen. The patient is able to drink and has no
other findings. Which of the following is the appropriate treatment?

Choices:
1. Amoxicillin/clavulanate
2. Albuterol home nebulizer
3. Dexamethasone 0.2 mg/kg IV one time
4. Dexamethasone 0.6 mg/kg by mouth one time
Answer: 4 - Dexamethasone 0.6 mg/kg by mouth one time
Explanations:
Laryngotracheobronchitis is a viral illness, most commonly due to
parainfluenza.
Albuterol nebulization is not helpful.
Oral dexamethasone at 0.6 mg/kg, up to a maximum dose of 10 mg orally,
has been shown to be helpful.

Go to the next page if you knew the correct answer, or click the link image(s)
below to further research the concepts in this question (if desired).

Research Concepts:
Laryngotracheobronchitis

We update eBooks quarterly and Apps daily based on user feedback.


Please tap flag to report any questions that need improvement.
Question 61: A patient was given a purified protein derivative on the inside
of the forearm 48 hours ago. A 12-mm induration has developed. What is the
indication?

Choices:
1. Active tuberculosis
2. Immunity to tuberculosis
3. Advanced tuberculosis
4. Exposure to tuberculosis
Answer: 4 - Exposure to tuberculosis
Explanations:
The client has been exposed to tuberculosis. A 12-mm induration indicates
that the client has been exposed to Mycobacterium tuberculosis recently or
in the past. It does not necessarily suggest that the client has an active
tubercular disease.
This reaction might be due to the actual disease or exposure to the disease.
Diagnosis of pulmonary tuberculosis is usually confirmed by chest x-ray
and sputum tests. Active tuberculosis is not diagnosed by the tubercle
extract injection.
A significant skin-test reaction (10 mm or more) would be followed up by
sputum testing and chest x-ray. A positive result of the tubercle bacillus
extract injection does not imply immunity to tuberculosis.
Advanced tuberculosis is not diagnosed by the tubercle bacillus extract
injection.

Go to the next page if you knew the correct answer, or click the link image(s)
below to further research the concepts in this question (if desired).

Research Concepts:
Tuberculosis

We update eBooks quarterly and Apps daily based on user feedback.


Please tap flag to report any questions that need improvement.
Question 62: Which of the following is the most common presenting
symptom of pulmonary embolism in adolescents?

Choices:
1. Productive cough
2. Pleuritic chest pain
3. Hemoptysis
4. Tachycardia
Answer: 2 - Pleuritic chest pain
Explanations:
Pleuritic chest pain is the presenting symptom in approximately 84% of
teens with pulmonary embolism (PE).
Unexplained, persistent tachypnea is suggestive of PE in the pediatric age
group overall.
Unfortunately, none of the physical signs are indicative of a PE.
The definitive test to make the diagnosis of a PE is a CT scan.

Go to the next page if you knew the correct answer, or click the link image(s)
below to further research the concepts in this question (if desired).

Research Concepts:
Embolism, Pulmonary

We update eBooks quarterly and Apps daily based on user feedback.


Please tap flag to report any questions that need improvement.
Question 63: A 43-year-old male with a history of hypertension and chronic
kidney disease stage 5 was admitted to the intensive care unit for sepsis from
urinary source. He is placed on volume cycled assist control ventilation with an
initial tidal volume of 500 and respiratory rate of 12. On day two after
admission, he is found to be over breathing the vent to 25 bpm. Which of the
following complications is most likely to occur in patients who become
tachypneic while on volume-cycled assist control ventilation?

Choices:
1. Hypercapnia
2. Dynamic hyperinflation (auto-PEEP)
3. Respiratory acidosis
4. Ventilator induced lung injury (VILI)
Answer: 2 - Dynamic hyperinflation (auto-PEEP)
Explanations:
Hyperventilation and breath staking can lead to dynamic hyperinflation,
also called auto-PEEP. This is a process in which the full volume of inhaled
air is not exhaled at the end of the breath cycle. If this happens repeatedly, a
great amount of air can be trapped in the lungs generating increased
pulmonary pressure (auto-PEEP) which can lead to hypotension and
cardiovascular collapse.
Hypocapnia and respiratory alkalosis are likely to occur secondary to
hyperventilation, not hypercapnia.
Respiratory alkalosis, not respiratory acidosis will happen in a patient who
is hyperventilating.
Although VILI may happen in patients who have increased intrapulmonary
pressures, this is more commonly related to shear stress from high tidal
volume ventilation or to plateau pressures above 30 cmH2O. In this patient,
the plateau pressures will increase secondary to auto-PEEP, but the total
pressure will depend on the lung compliance.

Go to the next page if you knew the correct answer, or click the link image(s)
below to further research the concepts in this question (if desired).

Research Concepts:
Ventilation, Assist Control

We update eBooks quarterly and Apps daily based on user feedback.


Please tap flag to report any questions that need improvement.
Question 64: A patient on a mechanical ventilator develops high peak
inspiratory pressures. What does this indicate?

Choices:
1. Improving respiratory mechanics
2. Decreased lung resistance
3. Increased respiratory system compliance
4. Decreased respiratory system compliance
Answer: 4 - Decreased respiratory system compliance
Explanations:
Decreasing lung compliance is manifested by a high peak inspiratory
pressure (PIP).
Other causes of increased PIP include kinked tubing; increased mucus,
indicating a need to suction; and cough.
PIP is the highest level of pressure applied to the lungs during inhalation. In
mechanical ventilation, the PIP number reflects a positive pressure in
cmH2O. In normal breathing, it is sometimes referred to as the maximal
inspiratory pressure, which has a negative value.
PIP increases with airway resistance and may be increased with
bronchospasm, secretions, biting down on ventilation tubing, and decreased
lung compliance. PIP should never be chronically higher than 40 cmH2O
unless the patient has acute respiratory distress syndrome.

Go to the next page if you knew the correct answer, or click the link image(s)
below to further research the concepts in this question (if desired).

Research Concepts:
Ventilation, Mechanical

We update eBooks quarterly and Apps daily based on user feedback.


Please tap flag to report any questions that need improvement.
Question 65: Which is not associated with primary pulmonary
hypertension?

Choices:
1. Primary pulmonary hypertension is seen more in males than females
2. CREST syndrome
3. Cocaine and amphetamine abuse
4. HIV
Answer: 1 - Primary pulmonary hypertension is seen more in males than
females

Explanations:
Primary pulmonary hypertension is seen more in females than males.
It is associated with connective tissue disease, autoimmune disease, cocaine
abuse, HIV, and thyroid disease.
Other causes of pulmonary hypertension include left heart disease and
chronic lung disease.
Treatment of pulmonary hypertension is focused on treating the underlying
disorder in cases of secondary pulmonary hypertension, while specific
treatments such as PDE-5 inhibitors and prostacyclin analogs are reserved
for group I pulmonary hypertension.

Go to the next page if you knew the correct answer, or click the link image(s)
below to further research the concepts in this question (if desired).

Research Concepts:
Pulmonary Hypertension

We update eBooks quarterly and Apps daily based on user feedback.


Please tap flag to report any questions that need improvement.
Section 2

Question 66: Which of the following parameters is not measured in


spirometry?

Choices:
1. Vital capacity (VC)
2. Forced vital capacity (FVC)
3. Tidal volume
4. Carbon dioxide volume
Answer: 4 - Carbon dioxide volume
Explanations:
Spirometry can help diagnose lung diseases such as asthma, chronic
obstructive pulmonary disease, cystic fibrosis, and pulmonary fibrosis.
It can also be used to gauge response to treatment interventions.
VC, FVC and tidal volume, residual volume, total lung capacity, expiratory
reserve volume and peak flow are some of the components measured.
Carbon dioxide volume is not a part of spirometric testing.

Go to the next page if you knew the correct answer, or click the link image(s)
below to further research the concepts in this question (if desired).

Research Concepts:
Pulmonary Function Tests

We update eBooks quarterly and Apps daily based on user feedback.


Please tap flag to report any questions that need improvement.
Question 67: Which of the following findings are commonly seen in
patients with Chlamydophila trachomatis pneumonia?

Choices:
1. Eosinophilia
2. Thrombocytopenia
3. Absolute lymphocytosis
4. Neutropenia
Answer: 1 - Eosinophilia
Explanations:
Eosinophilia is commonly seen in Chlamydophila trachomatis pneumonia.
Absolute lymphocytosis is seen in Epstein-Barr virus infection.
Other changes are not commonly seen in chlamydia infection.

Go to the next page if you knew the correct answer, or click the link image(s)
below to further research the concepts in this question (if desired).

Research Concepts:
Pneumonia, Chlamydia

We update eBooks quarterly and Apps daily based on user feedback.


Please tap flag to report any questions that need improvement.
Question 68: A young male involved in a MVA presents 3 months later with
shortness of breath. Evaluation reveals a long stricture in the right main stem
bronchus. What is the correct treatment?

Choices:
1. Placement of stent
2. Resection and primary anastomosis
3. Pneumonectomy
4. Laser
Answer: 2 - Resection and primary anastomosis
Explanations:
The treatment of delayed stricture is resection and anastomosis.
This is done with a right thoracotomy.
Some form of flap is used to cover the anastomosis to prevent fistulization.

Go to the next page if you knew the correct answer, or click the link image(s)
below to further research the concepts in this question (if desired).

Research Concepts:
Bronchial Stricture

We update eBooks quarterly and Apps daily based on user feedback.


Please tap flag to report any questions that need improvement.
Question 69: A male from the Southeast United States presents with
calcified lymph nodes and a lung mass. He has no other symptoms. Work up
reveals that he has histoplasmosis. What is the best treatment for this patient?

Choices:
1. Observation
2. Nystatin
3. Amphotericin
4. Ketoconazole
Answer: 1 - Observation
Explanations:
With histoplasmosis, if there are no symptoms, one can just observe the
patient.
Symptomatic patients require treatment with amphotericin or the newer
agents like ketoconazole.
In patients with symptoms greater than four weeks, one can initiate
treatment with itraconazole for 8 to 12 weeks. Serial x-rays should be
obtained to assess the response to therapy. Relapse is very common; thus
lifelong monitoring is recommended.
Surgery is only used for diagnosis and treatment of complications. By far
the majority of histoplasmosis lesions can be cured with drug therapy.

Go to the next page if you knew the correct answer, or click the link image(s)
below to further research the concepts in this question (if desired).

Research Concepts:
Histoplasmosis

We update eBooks quarterly and Apps daily based on user feedback.


Please tap flag to report any questions that need improvement.
Question 70: A young male has presented to you for a physical exam.
Routine chest x-ray reveals a possible aspergilloma. Further questioning of the
patient reveals that he did acquire a fungal infection in the remote past but has
never had any symptoms. What is the best next step in management?

Choices:
1. Surgery
2. Observation
3. Prolonged amphotericin administration
4. Cavernostomy
Answer: 2 - Observation
Explanations:
Aspergillus is a filamentous organism, which presents as fragmented
hyphae clustered in a ball (spaghetti and meatballs like). Aspergillus is of
significance only if there are complications.
An aspergilloma on an x-ray will present with a moon shaped, radiolucent
line. The aspergilloma shifts with position. Of all patients who develop an
aspergilloma, about 50 percent will present with hemoptysis. Patients with
hemoptysis and an adequate FEV1 (>1.9) can undergo surgery.
Surgery is usually extensive due to adhesions and requires a lobectomy.
Muscle flaps may be required to fill the residual airspace and stop air leaks.
Those who have no pulmonary reserve should undergo cavernostomy with
a muscle flap. Intracavitary instillation of amphotericin is a last choice
treatment.
Patients with asymptomatic aspergilloma are best left alone. The risk of
complications outweighs its benefit in asymptomatic patients. Hemoptysis
may occur but is rarely life threatening. Aspergillosis occurs almost
exclusively in patients with tuberculosis, sarcoidosis, and
immunosuppressed individuals.

Go to the next page if you knew the correct answer, or click the link image(s)
below to further research the concepts in this question (if desired).

Research Concepts:
Aspergilloma

We update eBooks quarterly and Apps daily based on user feedback.


Please tap flag to report any questions that need improvement.
Question 71: Which of the following is not true about extracorporeal
membrane oxygenation?

Choices:
1. It is used to treat congenital diaphragmatic hernias
2. It can be used to treat meconium aspiration
3. It can be used to treat persistent pulmonary hypertension
4. It can be used to treat congenital pulmonary airway malformation
Answer: 4 - It can be used to treat congenital pulmonary airway
malformation

Explanations:
Extracorporeal membrane oxygenation is not used for congenital
pulmonary airway malformation.
When used in the arterial-venous form, the carotid artery is ligated at the
end of the procedure.
Heparinization is used, and in neonates, central nervous system hemorrhage
is a major complication.

Go to the next page if you knew the correct answer, or click the link image(s)
below to further research the concepts in this question (if desired).

Research Concepts:
Extracorporeal Membrane Oxygenation

We update eBooks quarterly and Apps daily based on user feedback.


Please tap flag to report any questions that need improvement.
Question 72: During the work-up of a patient with suspected psittacosis,
which of the following is almost never done?

Choices:
1. Blood work
2. Culture of organism
3. Serological analysis
4. Imaging studies
Answer: 2 - Culture of organism
Explanations:
The work-up of psittacosis is usually through blood work.
However, psittacosis is never cultured, as this may prove hazardous to lab
personnel.
Diagnosis is most often made by polymerase chain reaction or serology
testing.
A chest x-ray is abnormal in 90% of cases.

Go to the next page if you knew the correct answer, or click the link image(s)
below to further research the concepts in this question (if desired).

Research Concepts:
Psittacosis

We update eBooks quarterly and Apps daily based on user feedback.


Please tap flag to report any questions that need improvement.
Question 73: A child has an arterial blood gas on room air and is treated
with a tonsillectomy. Which of the following blood gas would reflect the initial
situation?

Choices:
1. pH=7.35, PO2=52 mmHg, PCO2=58 mmHg, and bicarbonate= 8 mEq/L
2. pH=7.19, PO2=45 mmHg, PCO2=68 mmHg, and bicarbonate= 3 mEq/L
3. pH=7.60, PO2=95 mmHg, PCO2=20 mmHg, and bicarbonate= 2 mEq/L
4. pH=7.21, PO2=96 mmHg, PCO2=27 mmHg, and bicarbonate=15 mEq/L
Answer: 1 - pH=7.35, PO2=52 mmHg, PCO2=58 mmHg, and bicarbonate= 8
mEq/L

Explanations:
The first option shows respiratory acidosis with compensatory metabolic
alkalosis.
This would be seen in a child with tonsillar hypertrophy leading to chronic
hypoventilation.

Go to the next page if you knew the correct answer, or click the link image(s)
below to further research the concepts in this question (if desired).

Research Concepts:
Tonsilar Hypertrophy

We update eBooks quarterly and Apps daily based on user feedback.


Please tap flag to report any questions that need improvement.
Question 74: You are in the intensive care unit looking after a sick patient
on a ventilator. The patient has developed multiple organ failure, renal
dysfunction, and hepatic encephalopathy. This morning he was found to have
positive blood cultures for enterococcus. The sensitivity tests are not back.
Which single agent would you start this patient for susceptible enterococcus
infection?

Choices:
1. Ceftriaxone
2. Levaquin
3. Nalidixic acid
4. Ampicillin
Answer: 4 - Ampicillin
Explanations:
Enterococcus faecalis is a Gram-positive, commensal organism that is
normally found in the gastrointestinal tract.
Cephalosporins have no activity against Enterococcus.
E. faecalis is resistant to most of the presently available antibiotics like the
aminoglycosides, clindamycin, aztreonam, cephalosporins, sulfonamides
and the semisynthetic penicillins like nafcillin and oxacillin.
Ampicillin remains the drug of choice for susceptible organisms. Another
option is vancomycin.

Go to the next page if you knew the correct answer, or click the link image(s)
below to further research the concepts in this question (if desired).

Research Concepts:
Enterococcus

We update eBooks quarterly and Apps daily based on user feedback.


Please tap flag to report any questions that need improvement.
Question 75: In acute respiratory distress syndrome, there is a picture of
hypoxemia without hypercapnia. Select the most likely explanation of this.

Choices:
1. Albumin solubility
2. Oxygen solubility
3. Carbon dioxide solubility
4. Chloride solubility
Answer: 3 - Carbon dioxide solubility
Explanations:
Carbon dioxide is 20 times more soluble than oxygen.
Gas solubility determines its rate of diffusion across the alveolocapillary
membrane.
Ventilator manipulation to improve oxygenation such as increased PEEP or
inverse I:E ratio may result in decreased ventilation and, consequently,
hypercapnia.
In order to minimize lung injury caused by attempts at improving
ventilation, a mild hypercapnia and subsequent respiratory acidosis is
tolerated.

Go to the next page if you knew the correct answer, or click the link image(s)
below to further research the concepts in this question (if desired).

Research Concepts:
Acute Respiratory Distress Syndrome (ARDS)

We update eBooks quarterly and Apps daily based on user feedback.


Please tap flag to report any questions that need improvement.
Question 76: Which of the following is true regarding end-procedure and
post-procedure care of patients who have undergone a percutaneous lung lesion
biopsy?

Choices:
1. Portable chest x-rays with the patient supine are the most sensitive way to
detect a pneumothorax
2. A drop in oxygen saturation should occur prior to a significant negative
outcome from pneumothorax that would be visible on chest x-ray
3. A CT image after the needle has been withdrawn is not as good as a chest x-
ray for detecting trapped pleural air
4. Chest x-ray immediately after the biopsy is considered the standard of car
Answer: 2 - A drop in oxygen saturation should occur prior to a significant
negative outcome from pneumothorax that would be visible on chest x-ray

Explanations:
No chest radiograph need be obtained immediately after the biopsy unless
symptoms or a dropping oxygen saturation level suggest that a
pneumothorax may be present.
The reason for this is that despite the best intentions of the technologist,
moving the patient to enable performance of a radiograph will probably
entail at least some degree of straining or Valsalva maneuver.
Oxygen saturation provides enough clinical information to permit delayed
acquisition of the first image.
Upright expiratory images in at least two views (e.g. frontal and oblique)
provide the most sensitive method for detecting interpleural air by plain
film radiography.

Go to the next page if you knew the correct answer, or click the link image(s)
below to further research the concepts in this question (if desired).

Research Concepts:
Transthoracic Needle Biopsy

We update eBooks quarterly and Apps daily based on user feedback.


Please tap flag to report any questions that need improvement.
Question 77: During a bronchoscopy, a mass lesion is biopsied and reveals
a papilloma. Which of the following features is not true of a papilloma?

Choices:
1. It is the most common lesion in children and may be single or multiple
2. It is confined to the distal bronchus
3. There is a high rate of recurrence following resection
4. It can sometimes be treated with laser therapy
Answer: 2 - It is confined to the distal bronchus
Explanations:
Papillomas are mostly confined to the larynx and upper trachea. In adults,
they are usually solitary growths.
There can be multiple growths in children. About 50% of solitary growths
are associated with a lung cancer.
Bronchoscopic removal can be done but has a high rate of recurrence if not
adequately excised.
A larger lesion may require an open procedure. There is a risk of malignant
degeneration and surveillance is required.

Go to the next page if you knew the correct answer, or click the link image(s)
below to further research the concepts in this question (if desired).

Research Concepts:
Laryngeal Papillomas

We update eBooks quarterly and Apps daily based on user feedback.


Please tap flag to report any questions that need improvement.
Question 78: A patient is admitted for fever, worsening dyspnea, and a
productive cough. A chest x-ray shows an opacity in the right lung base. Which
of the following disorders often requires chest tube drainage?

Choices:
1. Empyema
2. Emphysema
3. Pneumonia
4. Lung abscess
Answer: 1 - Empyema
Explanations:
Empyema is a common indication for a chest tube.
The most common cause of empyema is parapneumonic effusions, followed
by inadequate evacuation of a hemothorax in trauma patients.
Chest tubes are only effective in the early stages of empyema before the
fibrin peel has developed. The drainage is usually viscous, thick, and
purulent.
In an infant, never be in a rush to insert a chest tube. Congenital lobar
emphysema is a rare disorder that can present like a large pneumothorax. If
a chest tube is inserted in an infant with congenital emphysema, chances are
that the air leak will be unmanageable.

Go to the next page if you knew the correct answer, or click the link image(s)
below to further research the concepts in this question (if desired).

Research Concepts:
Empyema

We update eBooks quarterly and Apps daily based on user feedback.


Please tap flag to report any questions that need improvement.
Question 79: An 18-month-old boy was eating pecans, began choking, and
became cyanotic. The symptoms subsided at the emergency department. The
child was comfortable, but his mother stated the child was "just not himself"
since the choking episode. Chest auscultation demonstrated equal breath sounds
without wheezing. The chest radiograph demonstrated normal results. Which of
the following is the next step in treatment?

Choices:
1. Repeat the chest radiograph in 12 hours
2. Discharge the boy home and tell the mother to return if his condition worsens
3. Schedule a rigid bronchoscopy performed with the child under general
anesthesia after an appropriate time has been allotted for ingesting nothing by
mouth (NPO)
4. Admit the child to the hospital for observation
Answer: 3 - Schedule a rigid bronchoscopy performed with the child under
general anesthesia after an appropriate time has been allotted for ingesting
nothing by mouth (NPO)

Explanations:
Although examination findings were normal, the history strongly indicates
foreign body aspiration.
Findings of the chest radiograph were normal, but radiographs are not
always diagnostic.
This scenario actually occurred, and the child was taken to the operating
room for bronchoscopy.
Pecan pieces were found obstructing the right main bronchus, and the
pecans were removed successfully without complication. A complete
history and physical examination are crucial for proper treatment of patients
in whom foreign body aspiration is suspected.

Go to the next page if you knew the correct answer, or click the link image(s)
below to further research the concepts in this question (if desired).

Research Concepts:
Foreign Body Aspiration

We update eBooks quarterly and Apps daily based on user feedback.


Please tap flag to report any questions that need improvement.
Question 80: Invasive pulmonary aspergillosis is commonly seen in which
type of immunodeficiency?

Choices:
1. Wiskott-Aldrich syndrome
2. Leukocyte adhesion deficiency
3. Common variable immunodeficiency
4. Chronic granulomatous disease
Answer: 4 - Chronic granulomatous disease
Explanations:
Catalase positive organisms are common causes of infection in CGD.
Invasive pulmonary aspergillosis can occur in patents with CGD.
Wiskott Aldrich is an X-linked disorder that involves eczema,
thrombocytopenia, and immunodeficiency.
Giant cytoplasmic granules are commonly seen in peripheral film.

Go to the next page if you knew the correct answer, or click the link image(s)
below to further research the concepts in this question (if desired).

Research Concepts:
Aspergillosis

We update eBooks quarterly and Apps daily based on user feedback.


Please tap flag to report any questions that need improvement.
Question 81: Which of the following is a false claim about extralobar
sequestration?

Choices:
1. May communicate with the gastrointestinal tract
2. May be associated with a diaphragmatic hernia
3. Has major bronchial communication
4. Usually presents in childhood
Answer: 3 - Has major bronchial communication
Explanations:
Although not common, these lesions, which can have a close relationship to
the esophagus, may fistulize with that structure.
This is a distinction from intralobar sequestration. Extralobar sequestration
more commonly is associated with other congenital anomalies.
In both ELS and ILS, bronchial communication is rare.
ILS may not present until late adolescence or adulthood in the form of
recurrent pneumonias. ELS tends to present in infancy from respiratory
compromise.

Go to the next page if you knew the correct answer, or click the link image(s)
below to further research the concepts in this question (if desired).

Research Concepts:
Pulmonary Sequestration

We update eBooks quarterly and Apps daily based on user feedback.


Please tap flag to report any questions that need improvement.
Question 82: Which of the following actions has been shown to decrease
the risk of ventilator acquired pneumonia in an intubated patient?

Choices:
1. Scheduled nasopharyngeal suctioning
2. Breathing treatments with a beta agonist
3. Nasopharyngeal intubation
4. Positioning patient in semirecumbent position
Answer: 4 - Positioning patient in semirecumbent position
Explanations:
Preventative measures for ventilator-acquired pneumonia include
oropharyngeal intubation, semirecumbent positioning, continuous
subglottic suctioning, and daily sedation holidays to assess ability to be
extubated.
Infection with Pseudomonas or Acinetobacter causes increased mortality
rates.
VAP is defined as pneumonia that develops 48 hours after intubation.

Go to the next page if you knew the correct answer, or click the link image(s)
below to further research the concepts in this question (if desired).

Research Concepts:
Pneumonia, Ventilator-associated

We update eBooks quarterly and Apps daily based on user feedback.


Please tap flag to report any questions that need improvement.
Question 83: Single lung transplantation is not recommended for:
Choices:
1. Primary pulmonary hypertension
2. Alpha 1-antitrypsin deficiency
3. Cystic fibrosis
4. Emphysema
Answer: 3 - Cystic fibrosis
Explanations:
Patients with cystic fibrosis must be treated with double lung transplants
due to the nature of their chronic disease.

Go to the next page if you knew the correct answer, or click the link image(s)
below to further research the concepts in this question (if desired).

Research Concepts:
Cystic Fibrosis

We update eBooks quarterly and Apps daily based on user feedback.


Please tap flag to report any questions that need improvement.
Question 84: Hypoplasia of the right lung with anomalous pulmonary
venous blood flow to the inferior vena cava is seen in which of the following
syndromes?

Choices:
1. Shwachman-Diamond syndrome
2. Scimitar syndrome
3. Mounier-Kuhn syndrome
4. Marfan syndrome
Answer: 2 - Scimitar syndrome
Explanations:
Scimitar syndrome derives its name from the scimitar (sword-like) shape of
the vein descending through the diaphragm to the vena cava.
There is the infantile form and the adult form.

Go to the next page if you knew the correct answer, or click the link image(s)
below to further research the concepts in this question (if desired).

Research Concepts:
Lung, Hypoplastic Disease

We update eBooks quarterly and Apps daily based on user feedback.


Please tap flag to report any questions that need improvement.
Question 85: In patients with acute respiratory distress syndrome (ARDS),
which of the following is not an advantage of positive end expiratory pressure
(PEEP)?

Choices:
1. It prevents atelectasis
2. It improves oxygenation.
3. It reduces carbon dioxide
4. It can increase functional residual capacity (FRC)
Answer: 3 - It reduces carbon dioxide
Explanations:
Application of PEEP to ARDS patients has multiple advantages.
By increasing PEEP, atelectasis is prevented and more alveoli are recruited,
increasing FRC.
Increasing PEEP improves oxygenation.
PEEP does not affect carbon dioxide levels in the blood.

Go to the next page if you knew the correct answer, or click the link image(s)
below to further research the concepts in this question (if desired).

Research Concepts:
Positive End-Expiratory Pressure (PEEP)

We update eBooks quarterly and Apps daily based on user feedback.


Please tap flag to report any questions that need improvement.
Question 86: Which of the following triggers the termination of inspiration
in the pressure support mode of ventilatory support?

Choices:
1. Target tidal volume is reached
2. Inspiratory flow decreases below set threshold
3. Plateau pressure decreases below set threshold
4. Target peak pressure is reached
Answer: 2 - Inspiratory flow decreases below set threshold
Explanations:
In pressure support ventilation, inspiration stops when flow falls below a
preset percentage.
Low levels of pressure support less than 5 cmH2O are often utilized to
decrease resistance by overcoming ventilator accessory dead space such as
the circuitry, and its components.
Higher levels of pressure support are introduced to alleviate work of
breathing by introducing positive pressure to compliment the patient’s
spontaneous effort.
If pressure support levels of 10-12 mL/kg are utilized, all of the work of
breathing is being assumed by the ventilator.

Go to the next page if you knew the correct answer, or click the link image(s)
below to further research the concepts in this question (if desired).

Research Concepts:
Pressure Support

We update eBooks quarterly and Apps daily based on user feedback.


Please tap flag to report any questions that need improvement.
Question 87: Mendelson syndrome is best described as which of the
following?

Choices:
1. Hospital-acquired pneumonia
2. Community-acquired pneumonia
3. Aspiration pneumonia
4. Atypical pneumonia
Answer: 3 - Aspiration pneumonia
Explanations:
Mendelson syndrome is characterized by the pulmonary reaction to
aspiration of gastric contents.
Disease severity is correlated with the acidity of gastric contents.

Go to the next page if you knew the correct answer, or click the link image(s)
below to further research the concepts in this question (if desired).

Research Concepts:
Mendelson Syndrome

We update eBooks quarterly and Apps daily based on user feedback.


Please tap flag to report any questions that need improvement.
Question 88: Which of the following is not an indication for flexible
bronchoscopy?

Choices:
1. In patients with pneumonia to help in microbiologic identification of
organisms
2. To obtain lung biopsies to diagnose sarcoidosis
3. For removal of aspirated foreign bodies
4. To place silicone stents in major airways
Answer: 4 - To place silicone stents in major airways
Explanations:
The use of flexible bronchoscopy can be divided into two main categories,
diagnostic and therapeutic purposes. An example of its use as a diagnostic
tool is its use to investigate unexplained symptoms like hemoptysis, chronic
unexplained cough, or stridor.
The diagnosis of sarcoidosis requires tissue biopsy to visualize
noncaseating granulomas in most cases. Bronchoscopy is a valuable tool to
obtain biopsies from the lung or mediastinal and hilar lymph nodes to
diagnose sarcoidosis with a high sensitivity and specificity.
Both flexible bronchoscopy and rigid bronchoscopy can be used to remove
aspirated foreign bodies.
Although many stents can be placed using flexible bronchoscopy, certain
stents (e.g., silicone stents) require rigid bronchoscopy for placement.

Go to the next page if you knew the correct answer, or click the link image(s)
below to further research the concepts in this question (if desired).

Research Concepts:
Bronchoscopy

We update eBooks quarterly and Apps daily based on user feedback.


Please tap flag to report any questions that need improvement.
Question 89: An 18-month-old is brought to the emergency department
with high fever and respiratory distress. He has had all recommended
immunizations. Initially, he had a barking cough, low-grade fevers, and noisy
breathing. Currently, his temperature is 40.1 degrees Celsius, and he has
expiratory and inspiratory stridor. He has not been drooling and is able to drink
fluids. A normal epiglottis is visualized using direct laryngoscopy. Select the
appropriate management.

Choices:
1. Oral antibiotics and discharge to home
2. Intravenous steroids and inhaled racemic epinephrine
3. Intravenous antibiotics and intubation
4. Admission, cool mist, and intravenous steroids
Answer: 3 - Intravenous antibiotics and intubation
Explanations:
The patient initially had viral laryngotracheobronchitis, but the
deterioration makes bacterial tracheitis likely.
This is a rare but life-threatening complication.
The inspiratory and expiratory stridor makes airway obstruction a
possibility.
Even with intubation, the thick secretions can occlude the airway.

Go to the next page if you knew the correct answer, or click the link image(s)
below to further research the concepts in this question (if desired).

Research Concepts:
Tracheitis, Bacterial

We update eBooks quarterly and Apps daily based on user feedback.


Please tap flag to report any questions that need improvement.
Question 90: Which is a common side effect of nebulized N-acetylcysteine?
Choices:
1. Decreased secretions
2. Increase secretions
3. Bronchospasm
4. Altered oxygen delivery
Answer: 3 - Bronchospasm
Explanations:
N-acetylcysteine is administered to thin secretions.
It may induce bronchospasm as a side effect.

Go to the next page if you knew the correct answer, or click the link image(s)
below to further research the concepts in this question (if desired).

Research Concepts:
N Acetylcysteine

We update eBooks quarterly and Apps daily based on user feedback.


Please tap flag to report any questions that need improvement.
Question 91: What anticholinergic medication, when combined with beta-2
adrenergics, can relieve an asthmatic attack?

Choices:
1. Salmeterol
2. Ipratropium
3. Isoproterenol
4. Metaproterenol
Answer: 2 - Ipratropium
Explanations:
Ipratropium bromide is an antimuscarinic bronchodilator.
It is mostly used for a chronic obstructive pulmonary disease but can be
used off-label for acute exacerbations of asthma.
The American College of Chest Physicians recommends a combination of a
short-acting beta-2 agonist and a short-acting anticholinergic for acute
asthmatic attacks.
Both agents can cause tachycardia.

Go to the next page if you knew the correct answer, or click the link image(s)
below to further research the concepts in this question (if desired).

Research Concepts:
Ipratropium

We update eBooks quarterly and Apps daily based on user feedback.


Please tap flag to report any questions that need improvement.
Question 92: A 2-year-old female without significant past medical history
has had a cough for five weeks. She has been treated at a couple of emergency
rooms with different antibiotics for pneumonia. On exam, the right lung is clear
but breath sounds are decreased on the left. Chest X-rays taken after inspiration
and expiration show air trapping on the left only. Select the next step in
management.

Choices:
1. Fluoroscopic exam of the lungs
2. CT of the chest
3. Consult with thoracic surgery for bronchoscopy
4. Sweat chloride test
Answer: 3 - Consult with thoracic surgery for bronchoscopy
Explanations:
Air trapping in the left lung indicates an airway obstruction.
In this age group, the most probable cause is a foreign body.
Rigid bronchoscopy would be diagnostic and therapeutic.
CT or fluoroscopy might assist in the diagnosis.

Go to the next page if you knew the correct answer, or click the link image(s)
below to further research the concepts in this question (if desired).

Research Concepts:
Airway, Foreign Bodies

We update eBooks quarterly and Apps daily based on user feedback.


Please tap flag to report any questions that need improvement.
Question 93: What is the best medication for glottic edema post extubation?
Choices:
1. Racemic epinephrine
2. Albuterol
3. Ipratropium
4. Steroid
Answer: 1 - Racemic epinephrine
Explanations:
Racemic epinephrine has alpha receptor activity.
Can result in vasoconstriction and decrease glottic swelling.

Go to the next page if you knew the correct answer, or click the link image(s)
below to further research the concepts in this question (if desired).

Research Concepts:
Glottic Edema

We update eBooks quarterly and Apps daily based on user feedback.


Please tap flag to report any questions that need improvement.
Question 94: What does The American Thoracic Society recommend a
diagnostic spirometer be calibrated within?

Choices:
1. 1 percent
2. 2 percent
3. 5 percent
4. 10 percent
Answer: 2 - 2 percent
Explanations:
The spirometer must be calibrated within 2 percent.

Go to the next page if you knew the correct answer, or click the link image(s)
below to further research the concepts in this question (if desired).

Research Concepts:
Spirometry

We update eBooks quarterly and Apps daily based on user feedback.


Please tap flag to report any questions that need improvement.
Question 95: Which is the correct initial test in a patient with a productive
cough, night sweats, and weight loss?

Choices:
1. Chest x-ray
2. ABG
3. Spiral CT
4. Mantoux skin test

Photo:Contributed by Wikimedia Commons, CDC, Greg Knobloch (PD-USGov-HHS)


Answer: 4 - Mantoux skin test
Explanations:
A patient presenting with productive cough, night sweats, and weight loss
has to be tested for TB using the mantoux test.

Go to the next page if you knew the correct answer, or click the link image(s)
below to further research the concepts in this question (if desired).

Research Concepts:
Tuberculosis, PPD Skin Test (Tuberculosis Skin Test)

We update eBooks quarterly and Apps daily based on user feedback.


Please tap flag to report any questions that need improvement.
Question 96: Which of the following is not true of acute respiratory distress
syndrome?

Choices:
1. Hypoxia is a common feature
2. Pulmonary wedge pressure is initially high
3. It may result from drowning
4. Fluffy white interstitial infiltrates are seen on X-ray
Answer: 2 - Pulmonary wedge pressure is initially high
Explanations:
Acute respiratory distress syndrome (ARDS) has a sudden onset and is life-
threatening.
There is inflammation of the alveoli, which get filled with exudates and
collapse. There is no gas exchange, leading to hypoxemia.
Ventilator support is often needed. ARDS has multiple etiologies. These can
be separated into inhaled causes, such as drowning, and hematologic
causes, like sepsis.
Initial pulmonary wedge pressure will be less than 18 mm Hg. Radiography
will show fluffy, white, interstitial infiltrates.

Go to the next page if you knew the correct answer, or click the link image(s)
below to further research the concepts in this question (if desired).

Research Concepts:
Acute Respiratory Distress Syndrome (ARDS)

We update eBooks quarterly and Apps daily based on user feedback.


Please tap flag to report any questions that need improvement.
Question 97: Which of the following is a reliable test to discern
hemidiaphragmatic paralysis?

Choices:
1. MRI of the chest
2. Chest radiograph
3. Fluoroscopy of diaphragm
4. None of the above
Answer: 3 - Fluoroscopy of diaphragm
Explanations:
The fluoroscopic "sniff test" is a reliable investigation to diagnose
hemidiaphragmatic paralysis.
The "sniff test" is a real-time assessment of diaphragmatic excursion during
deep inhalation and exhalation.
Chest radiograph and MRI are not valuable in diagnosing
hemidiaphragmatic paralysis.

Go to the next page if you knew the correct answer, or click the link image(s)
below to further research the concepts in this question (if desired).

Research Concepts:
Elevated Hemidiaphragm

We update eBooks quarterly and Apps daily based on user feedback.


Please tap flag to report any questions that need improvement.
Question 98: What should patients with egg allergies be taught about the flu
vaccination?

Choices:
1. The vaccine should be avoided
2. Most egg-allergic persons tolerate the vaccine without any problem
3. A scratch test should be performed before administering the vaccine
4. The normal dose should be cut in half
Answer: 2 - Most egg-allergic persons tolerate the vaccine without any
problem

Explanations:
The vaccine contains only a minute, diluted quantity of egg product.
If a patient can eat moist, scrambled eggs, there should be no problem.
The Centers for Disease Control, American Academy of Pediatrics, and
American Allergy and College of Allergy, Asthma, and Immunology no
longer consider egg allergies a reason to avoid the flu vaccine. Some
providers are still deciding not to give the vaccine to egg-allergic patients,
but the latest literature does not support this school of thought in most
cases.

Go to the next page if you knew the correct answer, or click the link image(s)
below to further research the concepts in this question (if desired).

Research Concepts:
Vaccine, Influenza

We update eBooks quarterly and Apps daily based on user feedback.


Please tap flag to report any questions that need improvement.
Question 99: A 4-year-old female complains of problems breathing. She
likes to eat dirt. She has no past medical history, travel history, or sick contacts.
The patient has bilateral wheezing and hepatomegaly. The family has a dog.
CBC shows leukocytosis with 55 percent eosinophilia. Select the best diagnostic
test.

Choices:
1. Bronchoscopy
2. Toxocara ELISA
3. Gastric aspirate for AFB stain and culture
4. Urine test for legionella antigen
Answer: 2 - Toxocara ELISA
Explanations:
The patient has visceral larva migrans from eating infectious ova.
The parasite commonly infects dogs and the ova are present in the stool.
In humans, the lungs, liver, and brain can be infected.
The diagnostic test is ELISA for Toxocara canis.

Go to the next page if you knew the correct answer, or click the link image(s)
below to further research the concepts in this question (if desired).

Research Concepts:
Toxocara Canis (Visceral Larva Migrans, Toxocariasis)

We update eBooks quarterly and Apps daily based on user feedback.


Please tap flag to report any questions that need improvement.
Question 100: At what point should hospitalization for an asthma
exacerbation occur if there is no significant improvement?

Choices:
1. After 1 dose of inhaled/nebulized short acting bronchodilator
2. After 2 doses of inhaled/nebulized short acting bronchodilator
3. After 3 doses of inhaled/nebulized short acting bronchodilator
4. After 4 doses of inhaled/nebulized short acting bronchodilator
Answer: 3 - After 3 doses of inhaled/nebulized short acting bronchodilator
Explanations:
Decisions on hospitalization for asthma should be carried out after the
patient receives 3 doses of an inhaled/nebulized bronchodilator.
Asthma is the most common chronic childhood illness.
Asthma consists of inflammation, hyperreactivity, and intermittent
obstruction of airways.

Go to the next page if you knew the correct answer, or click the link image(s)
below to further research the concepts in this question (if desired).

Research Concepts:
Asthma

We update eBooks quarterly and Apps daily based on user feedback.


Please tap flag to report any questions that need improvement.
Question 101: A 5 year old boy whom you follow for short bowel
syndrome has a central venous catheter infection. He is admitted to the children's
hospital with fever 104 degrees F. He is also known to be colonized with MRSA.
A central line infection is strongly suspected and vancomycin is started. The
blood culture grows vancomycin-resistant E. faecalis. Appropriate therapy
would now include which of the following?

Choices:
1. Linezolid
2. Meropenem
3. Clindamycin
4. Clofazimine
Answer: 1 - Linezolid
Explanations:
Linezolid represents a new class of antimicrobial agents called the
oxazolidinones that bind to the 50S ribosomal subunit and inhibit protein
synthesis.
Linezolid is active against MRSA, VRE, and penicillin-resistant S.
pneumoniae.

Go to the next page if you knew the correct answer, or click the link image(s)
below to further research the concepts in this question (if desired).

Research Concepts:
Vancomycin-Resistant Enterococci (VRE)

We update eBooks quarterly and Apps daily based on user feedback.


Please tap flag to report any questions that need improvement.
Question 102: What drug is frequently used to treat patients with
granulomatosis with polyangiitis?

Choices:
1. Cyclosporine
2. Dialysis
3. Captopril
4. Cyclophosphamide
Answer: 4 - Cyclophosphamide
Explanations:
Cyclophosphamide treats granulomatosis with polyangiitis.

Go to the next page if you knew the correct answer, or click the link image(s)
below to further research the concepts in this question (if desired).

Research Concepts:
Granulomatosis with Polyangiitis (GPA, Wegener Granulomatosis)

We update eBooks quarterly and Apps daily based on user feedback.


Please tap flag to report any questions that need improvement.
Question 103: You are performing bronchoalveolar lavage in a patient in
the intensive care unit who has had a fever for the past two weeks. All the other
workup has been negative. You introduce the bronchoscope and instill 60 ml of
normal saline into the right lower lobe. Upon suctioning, you only have a return
of 5 ml. You repeat the process with another 60 ml of saline and again only get
back 5 ml. What is the next step?

Choices:
1. Instill another 60 of sterile water
2. Place the patient in Trendelenburg and aspirate again
3. Obtain a chest x-ray
4. Abort the procedure
Answer: 4 - Abort the procedure
Explanations:
Depending on local practice, anywhere from between 20 to 60 ml of room
temperature, sterile normal saline is injected via handheld syringe and then
gradually withdrawn back into the syringe.
This is repeated 3 to 5 times and a total of up to 300 ml is instilled. If only
5% of each aliquot injected returns this indicates that most of the injected
fluid is being retained, the procedure should be aborted.
A return sample yield of 30% or more of the instillate is considered an
adequate return, of which at least 10 ml to 20 ml is required for cellular and
infectious workup.

Go to the next page if you knew the correct answer, or click the link image(s)
below to further research the concepts in this question (if desired).

Research Concepts:
Bronchoalveolar Lavage

We update eBooks quarterly and Apps daily based on user feedback.


Please tap flag to report any questions that need improvement.
Question 104: What aspirated foreign body in children is most likely to be
lethal?

Choices:
1. Popcorn
2. Peanuts
3. Balloons
4. Hot dogs
Answer: 3 - Balloons
Explanations:
Balloons and similar substances are the most common to be lethal if
aspirated by children.

Go to the next page if you knew the correct answer, or click the link image(s)
below to further research the concepts in this question (if desired).

Research Concepts:
Foreign Body Aspiration

We update eBooks quarterly and Apps daily based on user feedback.


Please tap flag to report any questions that need improvement.
Question 105: A neonate undergoes repair of a tracheo-esophageal fistula.
Three months later, he presents with noisy breathing and a barking cough. What
is the most likely diagnosis?

Choices:
1. Post-intubation stenosis
2. Recurrent tracheo-esophageal fistula
3. Tracheomalacia
4. Gastro-esophageal reflux
Answer: 3 - Tracheomalacia
Explanations:
Tracheomalacia is seen in some infants after repair of a tracheo-esophageal
fistula. It presents with noisy breathing and a barking cough. There is also
an increased incidence of pneumonia.
Croup is often mistaken for post-intubation stenosis.
Lateral fluoroscopy reveals collapse of the anterior/posterior trachea.
Some patients may improve with time but others may develop life-
threatening respiratory distress. Others may be unable to come off the
ventilator. These neonates may also develop reflux, which may require a
fundoplication.

Go to the next page if you knew the correct answer, or click the link image(s)
below to further research the concepts in this question (if desired).

Research Concepts:
Fistula, Tracheoesophageal

We update eBooks quarterly and Apps daily based on user feedback.


Please tap flag to report any questions that need improvement.
Question 106: A 17-year-old male hospitalized for treatment of bacterial
endocarditis is found to be cyanotic with bradypnea and pinpoint pupils. What is
the most likely cause of these symptoms?

Choices:
1. Pontine hemorrhage due to embolic event
2. Heroin overdose
3. Clonidine overdose
4. Myocardial abscess with complete heart block
Answer: 2 - Heroin overdose
Explanations:
Pontine hemorrhage due to an embolic event, heroin overdose, and
clonidine overdose can cause pinpoint pupils and respiratory failure.
Myocardial abscess with complete heart block would not cause the
pupillary and respiratory changes though it could cause circulatory
compromise.
In young male patients hospitalized with endocarditis, heroin overdose is
the most likely cause of these symptoms.
Intravenous drug abuse is common in young patients with the diagnosis of
infectious endocarditis.

Go to the next page if you knew the correct answer, or click the link image(s)
below to further research the concepts in this question (if desired).

Research Concepts:
Respiratory Failure

We update eBooks quarterly and Apps daily based on user feedback.


Please tap flag to report any questions that need improvement.
Question 107: A patient sustains a gunshot to the chest and requires
exploration. During the thoracotomy, the lacerated lung is repaired and
immediately the patient becomes hypotensive. A venous air embolism is
suspected. Which of the following is not part of the immediate treatment for this
condition?

Choices:
1. Left lateral decubitus position
2. Aspiration of air from central line
3. Hyperbaric oxygen
4. Trendelenburg
Answer: 4 - Trendelenburg
Explanations:
If venous air embolism is suspected, 100% oxygen should be administered.
Trendelenburg has been used in the past, but studies have shown it to be
non-efficacious and potentially harmful, raising intracranial pressure and
cerebral edema.
Others advocate placing the patient in a left lateral decubitus position to
help prevent the air traveling from the right side of the heart into the lungs.
One can remove air from the venous circulation by aspirating with a central
line catheter. Cardiopulmonary resuscitation may have to be started if the
patient is in shock.
Hyperbaric oxygen sometimes is used for symptomatic cases. It usually is
only necessary if the venous gas shunts to the left causing a cerebral gas
embolic stroke. In animal studies, use of perfluorocarbons has been shown
to enhance reabsorption of air bubbles and decrease neurological deficits.

Go to the next page if you knew the correct answer, or click the link image(s)
below to further research the concepts in this question (if desired).

Research Concepts:
Embolism, Venous Gas

We update eBooks quarterly and Apps daily based on user feedback.


Please tap flag to report any questions that need improvement.
Question 108: In a patient with massive hemoptysis after a pulmonary
artery catheter placement, what is the best initial response?

Choices:
1. Removal of PA catheter
2. Endobronchial intubation
3. Suctioning of oropharynx
4. Check hemoglobin using point of care testing
Answer: 2 - Endobronchial intubation
Explanations:
Massive hemoptysis temporally related to manipulation of a Swan Ganz
(PA) catheter is indicative of pulmonary artery rupture. Endobronchial
intubation is an immediate step in unilateral pulmonary artery rupture to
protect the good lung.

Go to the next page if you knew the correct answer, or click the link image(s)
below to further research the concepts in this question (if desired).

Research Concepts:
Catheterization, Pulmonary Artery

We update eBooks quarterly and Apps daily based on user feedback.


Please tap flag to report any questions that need improvement.
Question 109: Which of the following is the best antibiotic as part of
treatment for empyema?

Choices:
1. Penicillin
2. Ticarcillin
3. Vancomycin
4. Gentamicin
Answer: 3 - Vancomycin
Explanations:
The presence of pus in the chest cavity is termed empyema.
Vancomycin can cover for methicillin resistant strains of Staphylococcus
aureus. This medication is often used in such severe infections.
Multiple antibiotics are generally used including piperacillin/tazobactam or
imipenem, with vancomycin and anaerobic coverage.
Chest tube placement is needed.

Go to the next page if you knew the correct answer, or click the link image(s)
below to further research the concepts in this question (if desired).

Research Concepts:
Empyema

We update eBooks quarterly and Apps daily based on user feedback.


Please tap flag to report any questions that need improvement.
Question 110: Which of the following is the primary concern when a health
professional is impaired due to substance abuse?

Choices:
1. Rehabilitation
2. Patient safety
3. Staffing
4. Absenteeism
Answer: 2 - Patient safety
Explanations:
Rehabilitation of the health professional is the second chief concern in the
area of substance abuse.
Patient safety is the first concern when a health professional is impaired.
Absenteeism is an ongoing issue when a professional has any chronic and
debilitating disease or disorder.
Substance abuse compromises the professional and his or her peers. A
reduced level of function, misleading information, and withheld care has a
widespread effect on all those in the professionals sphere of influence.

Go to the next page if you knew the correct answer, or click the link image(s)
below to further research the concepts in this question (if desired).

Research Concepts:
Medical Errors Prevention

We update eBooks quarterly and Apps daily based on user feedback.


Please tap flag to report any questions that need improvement.
Question 111: Wilson Mikity syndrome is usually seen in which
population?

Choices:
1. Low birth weight infants
2. Infants born to diabetic mothers
3. Infants with HIV
4. Infants born at term
Answer: 1 - Low birth weight infants
Explanations:
Wilson-Mikity syndrome is a rare lung disorder that affects low birth
weight infants. The infants with Wilson Mikity syndrome have no prior
history of mechanical ventilation and yet develop a syndrome that
resembles bronchopulmonary dysplasia.
At autopsy, both the gross and microscopic features of the lungs appear to
be very similar to bronchopulmonary dysplasia.
The treatment is supportive with mechanical ventilation.
Surfactant has been used but only case reports exists on its efficacy. In
infants in whom oxygenation is difficult, other organ systems often fail and
death can result.

Go to the next page if you knew the correct answer, or click the link image(s)
below to further research the concepts in this question (if desired).

Research Concepts:
Wilson-Mikity Syndrome

We update eBooks quarterly and Apps daily based on user feedback.


Please tap flag to report any questions that need improvement.
Question 112: What is the five-year survival rate for Goodpasture
syndrome?

Choices:
1. > 30%
2. > 50%
3. > 80%
4. > 95%
Answer: 3 - > 80%
Explanations:
The five-year survival rate for the syndrome exceeds 80%.
Less than 30 percent of patients require long-term dialysis.
Aggressive treatment of the disease with plasmapheresis, corticosteroids,
and immunosuppressive agents has improved mortality.
The disease affects more whites than blacks.

Go to the next page if you knew the correct answer, or click the link image(s)
below to further research the concepts in this question (if desired).

Research Concepts:
Goodpasture Syndrome (Anti-glomerular Basement Membrane
Antibody Disease)

We update eBooks quarterly and Apps daily based on user feedback.


Please tap flag to report any questions that need improvement.
Question 113: What is the treatment of histoplasmosis in a transplant
recipient?

Choices:
1. Cyclosporine (Neoral)
2. Fluconazole
3. Amphotericin B
4. IVIG
Answer: 3 - Amphotericin B
Explanations:
Amphotericin B is treatment for histoplasmosis.
Patients who are not immunosuppressed and are asymptomatic do not need
treatment for histoplasmosis.
Immunosuppressed patients can develop progressive disseminated
histoplasmosis.

Go to the next page if you knew the correct answer, or click the link image(s)
below to further research the concepts in this question (if desired).

Research Concepts:
Histoplasmosis

We update eBooks quarterly and Apps daily based on user feedback.


Please tap flag to report any questions that need improvement.
Question 114: A young black female has been referred to you because of
bilateral hilar adenopathy due to a suspected granulomatous disease. Which of
the following is not true of this disorder?

Choices:
1. Will have elevated serum angiotensin levels
2. May affect the lung, liver, spleen, and parotid gland
3. Bronchoscopy with lavage is often the initial test
4. Open lung biopsy is generally preferred to mediastinoscopy
Answer: 4 - Open lung biopsy is generally preferred to mediastinoscopy
Explanations:
Sarcoid is a granulomatous disease often seen in black females. It affects
the skin, eyes, parotids, lungs, heart, etc. In the lung, it develops in three
stages.
Stage I is bilateral hilar adenopathy, stage II is bilateral hilar adenopathy
with parenchymal changes, and stage III is involvement of lung only.
CT usually reveals anterior mediastinal and subcarinal lymph nodes.
Calcium and angiotensin levels are usually elevated but correlate poorly
with the disease stage.
A bronchoscopy with lavage is usually the first diagnostic test.
Mediastinoscopy is usually the second test, but is more definitive. Lung
biopsy is generally the last procedure to detect sarcoid.

Go to the next page if you knew the correct answer, or click the link image(s)
below to further research the concepts in this question (if desired).

Research Concepts:
Sarcoidosis

We update eBooks quarterly and Apps daily based on user feedback.


Please tap flag to report any questions that need improvement.
Question 115: Which of the following instruction should be given to a
patient having removal of a chest tube?

Choices:
1. Hold his or her breath
2. Inhale and perform Valsalva
3. Cough during the procedure
4. Exhale before and inhale during removal
Answer: 2 - Inhale and perform Valsalva
Explanations:
Chest tubes are used to treat pneumo or hemothorax.
They provide negative pressure for reexpansion of the lung.
The patient should increase intrathoracic pressure by inhaling and
performing Valsalva.

Go to the next page if you knew the correct answer, or click the link image(s)
below to further research the concepts in this question (if desired).

Research Concepts:
Chest Tube

We update eBooks quarterly and Apps daily based on user feedback.


Please tap flag to report any questions that need improvement.
Question 116: What is livedo reticularis in relation to decompression
illness in aviation?

Choices:
1. Appearance a body takes after death secondary to cardiopulmonary collapse
from decompression sickness
2. Joint pain that occurs secondary to decreased oxygen that occurs during
decompression sickness
3. Neurologic sequelae as a result of tissue damage from the inert gas bubbles
that form in decompression sickness
4. A rash sometimes seen in decompression sickness, caused by damage to the
skin tissues from the inert gas bubbles that form in decompression sickness
Answer: 4 - A rash sometimes seen in decompression sickness, caused by
damage to the skin tissues from the inert gas bubbles that form in decompression
sickness

Explanations:
Livedo reticularis, one possible rash that occurs, is actually due to skin
changes from tissue damage of inert gas bubbles.
There are numerous possible complications and sequelae from
decompression illness; livedo reticularis is the cutaneous manifestation.
Joint pain, neurologic symptoms, and cardiopulmonary symptoms are also
possible.
While individuals may die from decompression illness, the rash that may
occur in living patients is considered livedo reticularis.
Decompression illness occurs secondary to the development of bubbles
outside their normal location and concentration secondary to a decrease in
environmental pressures.

Go to the next page if you knew the correct answer, or click the link image(s)
below to further research the concepts in this question (if desired).

Research Concepts:
Aerospace, Decompression Illness

We update eBooks quarterly and Apps daily based on user feedback.


Please tap flag to report any questions that need improvement.
Question 117: A robust patient with tuberculosis develops a pyogenic
empyema. Nuclear studies reveal that almost the entire lung is non-functioning.
A CT scan reveals that the pleural reaction is intense and that the lung is
compressed. Which of the following is the procedure of choice for this patient?

Choices:
1. Video-assisted thoracoscopic surgery
2. Decortication
3. Pleuropneumonectomy
4. Eloesser flap
Answer: 3 - Pleuropneumonectomy
Explanations:
Indications for surgery in patients with tuberculosis include a persistently
positive sputum culture with cavity after 5 to 6 months of therapy.
Another indication for surgery is localized disease caused by
mycobacterium avium-intracellulare, which is resistant to chemotherapy.
Other indications for surgery include ruling out cancer, massive
hemoptysis, and a bronchopleural fistula secondary to tuberculosis, which
is unresponsive to thoracostomy. In patients with tuberculosis with a
destroyed and non-functioning lung, pleuropneumonectomy is best.
A contraindication to surgery is widespread endobronchial disease as it
leads to poor stump healing. Children with tuberculosis rarely require
resection.

Go to the next page if you knew the correct answer, or click the link image(s)
below to further research the concepts in this question (if desired).

Research Concepts:
Tuberculosis, Surgery

We update eBooks quarterly and Apps daily based on user feedback.


Please tap flag to report any questions that need improvement.
Question 118: A 3 year old child developed brassy cough and high fever.
Chest X-ray shows subglottic narrowing plus ragged tracheal air column. What
is the most likely ethologic agent that caused the underlying problem?

Choices:
1. S. aureus
2. Strep viridans
3. H. influenzae
4. Strep pyrogenes
Answer: 1 - S. aureus
Explanations:
The child most likely has bacterial tracheitis.
This infection usually occur in children less than 3 years of age.
Chest X-ray shows tracheal involvement and narrowing.
Anti staph medications should be started immediately.

Go to the next page if you knew the correct answer, or click the link image(s)
below to further research the concepts in this question (if desired).

Research Concepts:
Tracheitis, Bacterial

We update eBooks quarterly and Apps daily based on user feedback.


Please tap flag to report any questions that need improvement.
Question 119: Which of the following trauma scenarios will not
compromise the airway after endotracheal intubation?

Choices:
1. Unrecognized pneumothorax
2. Massive bronchopleural fistula
3. Placement of endotracheal tube in right mainstem bronchus
4. Gunshot wound to the pelvis
Answer: 4 - Gunshot wound to the pelvis
Explanations:
When there is a pneumothorax that has not been treated and the patient is
ventilated, a tension pneumothorax can result. Thus, prior to intubation, a
chest tube should be inserted.
In cases of bronchial injury, a massive air leak can be precipitated by
intubation. Sometimes the segment of a damaged lung may have to be
occluded with a bronchial blocker to avoid air escape.
In many cases, when blind endotracheal intubation takes place, the tube
may go down the right mainstem bronchus.
Once an x-ray has been done, the condition is recognized and the tube can
be pulled back. It may present with absent breath sounds on the left chest
with a low oxygen saturation.

Go to the next page if you knew the correct answer, or click the link image(s)
below to further research the concepts in this question (if desired).

Research Concepts:
Airway, Management

We update eBooks quarterly and Apps daily based on user feedback.


Please tap flag to report any questions that need improvement.
Question 120: What condition is least likely in subacute blood loss?
Choices:
1. Low hemoglobin
2. Low reticulocyte
3. Microcytic anemia
4. Hypovolemia
Answer: 2 - Low reticulocyte
Explanations:
Subacute blood loss is associated with high a reticulocyte count.
With acute blood loss, the bone marrow has not had enough time to
generate reticulocytes.
Microcytic anemia is seen with chronic blood loss.

Go to the next page if you knew the correct answer, or click the link image(s)
below to further research the concepts in this question (if desired).

Research Concepts:
Anemia, Acute

We update eBooks quarterly and Apps daily based on user feedback.


Please tap flag to report any questions that need improvement.
Question 121: What condition is associated with Hamman sign?
Choices:
1. Pneumomediastinum
2. Pneumonia
3. Pleural effusion
4. Empyema
Answer: 1 - Pneumomediastinum
Explanations:
Hamman sign is a crunching rasping sound heard over the precordium in
patients with mediastinal emphysema.
Pneumothorax can cause subcutaneous emphysema.
Hamman sign is best head over the left lateral position.
The sign may also be heard in patients with Boerhaave syndrome.

Go to the next page if you knew the correct answer, or click the link image(s)
below to further research the concepts in this question (if desired).

Research Concepts:
Pneumothorax, Pneumomediastinum

We update eBooks quarterly and Apps daily based on user feedback.


Please tap flag to report any questions that need improvement.
Question 122: A patient with asthma has been on albuterol but now
presents with shortness of breath and inability to sleep because of a cough.
Which of the following therapies is most appropriate?

Choices:
1. Short course of oral corticosteroids
2. Inhaled corticosteroids
3. Long-acting beta agonist
4. Theophylline
Answer: 2 - Inhaled corticosteroids
Explanations:
Inhaled corticosteroids with a beta-2 adrenergic agonist inhaler are used for
moderate asthma.
If a patient is using a short-acting beta-2 adrenergic agent more than twice a
week, inhaled steroids are added.
Beta-2 agonist use must be monitored. Side effects include tachycardia,
flushing, sweating, and other signs of sympathetic system overdrive.
Iatrogenic hypokalemia is possible.
Weight loss, smoking cessation, occupational change, and self-monitoring
all are important in preventing disease progression and reducing the number
of acute attacks.

Go to the next page if you knew the correct answer, or click the link image(s)
below to further research the concepts in this question (if desired).

Research Concepts:
Asthma Medications

We update eBooks quarterly and Apps daily based on user feedback.


Please tap flag to report any questions that need improvement.
Question 123: A patient with a lung abscess grows anaerobic bacteria.
What is the antibiotic of choice?

Choices:
1. Sulfonamides
2. Cephalexin
3. Clindamycin
4. Ciprofloxacin
Answer: 3 - Clindamycin
Explanations:
Clindamycin is an excellent drug for anaerobic bacteria.
Metronidazole also is effective against anaerobic bacteria.
X-rays and chest CT aid in the diagnosis of a lung abscess. CT will provide
finer detail of the anatomic relations of an abscess. Microbiologic analysis
of sputum can potentially support the diagnosis. If the patient presents with
risk factors for fungi or mycobacteria in their history, these special cultures
should be requested.
Empiric antibiotic therapy should be started upon suspicion of a lung
abscess. Empiric coverage should include those microorganisms likely to
populate the upper airway and oropharynx. Those are gram-positive cocci,
respiratory gram-negative cocci, and aerobic and anaerobic gram-negative
bacilli. If the patient is exposed to a healthcare setting in the weeks before
presentation, coverage for methicillin-resistant Staphylococcus aureus
should be added. Atypical organisms should be considered in the setting of
an abscess that is not improving with broad-spectrum antibiotic therapy.
Abscesses larger than 4 cm are unlikely to resolve with antibiotic therapy
alone and might require surgical or percutaneous intervention.

Go to the next page if you knew the correct answer, or click the link image(s)
below to further research the concepts in this question (if desired).

Research Concepts:
Abscess, Lung

We update eBooks quarterly and Apps daily based on user feedback.


Please tap flag to report any questions that need improvement.
Question 124: Pneumocystis infection often is identified by which of the
following tests?

Choices:
1. India ink staining
2. Silver stain
3. Wood lamp
4. Gram stain
Answer: 2 - Silver stain
Explanations:
A pneumocystis infection is best revealed with methenamine silver stain.
Other stains used include crystal violet, Giemsa, Wright stain, toluidine
blue, and Gram-Weigert.
Some laboratories use direct immunofluorescence using monoclonal
antibodies to detect Pneumocystis organisms.
This method is more sensitive than histologic staining.

Go to the next page if you knew the correct answer, or click the link image(s)
below to further research the concepts in this question (if desired).

Research Concepts:
Pneumonia, Pneumocystis (Carinii) Jiroveci

We update eBooks quarterly and Apps daily based on user feedback.


Please tap flag to report any questions that need improvement.
Question 125: A young patient presents with symptoms of dysphagia. The
CT scan reveals a middle mediastinal mass located just beneath the carina.
Aspiration reveals mucus and respiratory epithelial cells. What is the most likely
diagnosis?

Choices:
1. Bronchogenic cyst
2. Esophageal cysts
3. Leiomyoma
4. Traction diverticulum
Answer: 1 - Bronchogenic cyst
Explanations:
Bronchogenic cysts develop as a result of anomalous development of the
ventral foregut. There is usually a single cyst and it is often filled with
mucus or fluid.
Bronchogenic cysts may be found along the course of the trachea and
esophagus, but the most common location is the hilar or carina region.
Some bronchogenic cysts occur in the mediastinum but rarely connect with
the airways.
Bronchogenic cysts have been reported in the abdomen, neck,
retroperitoneal space, and even the posterior chest wall.

Go to the next page if you knew the correct answer, or click the link image(s)
below to further research the concepts in this question (if desired).

Research Concepts:
Cyst, Bronchogenic

We update eBooks quarterly and Apps daily based on user feedback.


Please tap flag to report any questions that need improvement.
Question 126: How long of breathing 100% oxygen does it take for
pulmonary oxygen toxicity to develop?

Choices:
1. 2 hours
2. 6 hours
3. 8 hours
4. 12 hours
Answer: 4 - 12 hours
Explanations:
Pulmonary oxygen toxicity develops after 12 hours at sea level.
Pulmonary oxygen toxicity develops after 3 hours at 2 to 3 times
atmospheric pressure.
Central nervous system oxygen toxicity only develops under hyperbaric
conditions (160kPa on 100% oxygen or higher).
Retina and lens oxygen toxicity require much longer, cumulative exposure.

Go to the next page if you knew the correct answer, or click the link image(s)
below to further research the concepts in this question (if desired).

Research Concepts:
Toxicity, Oxygen

We update eBooks quarterly and Apps daily based on user feedback.


Please tap flag to report any questions that need improvement.
Question 127: Which of the following is a relatively consistent finding in
moderately anemic patients?

Choices:
1. Bilateral basilar crackles
2. Pale palms
3. Low hematocrit
4. Wide pulse pressure
Answer: 4 - Wide pulse pressure
Explanations:
Pulse pressure is the difference between systolic and diastolic blood
pressure.
The hyperdynamic circulation makes the pulse pressure wider in anemia.
Hematocrit could be falsely elevated in dehydrated patients.
Pale palms are seen in severe anemia.

Go to the next page if you knew the correct answer, or click the link image(s)
below to further research the concepts in this question (if desired).

Research Concepts:
Anemia

We update eBooks quarterly and Apps daily based on user feedback.


Please tap flag to report any questions that need improvement.
Question 128: Which of the following is a common cause of a cough in
children?

Choices:
1. Pneumonia
2. Asthma
3. Rhinitis
4. Sinusitis
Answer: 2 - Asthma
Explanations:
Asthma is one of the most common causes of a cough in children.
Currently, asthma prevalence in the United States is 7.8 %. This is down
from 8.4% in 2010. National surveillance data shows that prevalence varies
by age, gender, race, ethnicity, geographic location, and socioeconomic
status.
Spirometry is a noninvasive and objective measurement of airflow and can
help provide information about airway reversibility after a bronchodilator.
This test is recommended for children ages 5 years and above.
Beta-2 agonists are the preferred medication for quick relief of asthma
symptoms.

Go to the next page if you knew the correct answer, or click the link image(s)
below to further research the concepts in this question (if desired).

Research Concepts:
Asthma, Pediatric

We update eBooks quarterly and Apps daily based on user feedback.


Please tap flag to report any questions that need improvement.
Question 129: A patient received a neuromuscular blocker and had
neuromuscular paralysis for more than 7 hours. Which of the following is false?

Choices:
1. The most likely agent is succinylcholine
2. The most common inherited cause is a mutation of E1 locus on the long arm
of chromosome 3
3. Succinylcholine has a half-life of over an hour.
4. The dibucaine number is high in these patients
Answer: 3 - Succinylcholine has a half-life of over an hour.
Explanations:
An ultra short-acting depolarizing skeletal muscle relaxant, succinylcholine
bonds with motor endplate cholinergic receptors to produce depolarization.
These cause fasciculations.
The neuromuscular block remains as long as sufficient quantities of
succinylcholine remain, and is characterized by a flaccid paralysis. The
elimination half-life of succinylcholine is estimated to be 47 seconds.
Inadvertent overdose, or patients deficient in pseudocholinesterase may
result in prolonged apnea.
Mechanical ventilation with oxygen should be used until recovery.

Go to the next page if you knew the correct answer, or click the link image(s)
below to further research the concepts in this question (if desired).

Research Concepts:
Neuromuscular Blocking Agents

We update eBooks quarterly and Apps daily based on user feedback.


Please tap flag to report any questions that need improvement.
Question 130: A patient with rheumatoid arthritis complains of shortness of
breath. Exam reveals dullness to percussion at the left base. Chest radiograph
shows a left pleural effusion. Left lateral radiograph shows it to be free-flowing.
Thoracentesis is done. Which of the following is least likely to be found in the
pleural fluid?

Choices:
1. Positive rheumatoid factor
2. Yellow color
3. Elevated lactate dehydrogenase (LDH)
4. Low cholesterol
Answer: 4 - Low cholesterol
Explanations:
The pleural effusion secondary to rheumatoid arthritis usually is a yellow
color exudate and contains a low glucose level.
There would be elevated LDH and positive rheumatoid factor.
Cholesterol should be elevated as these lesions are exudative and elevated
cholesterol, as well as LDH and protein, is a characteristic of exudates
Rheumatoid pleural effusions are most common in middle-aged males with
a positive rheumatoid factor.

Go to the next page if you knew the correct answer, or click the link image(s)
below to further research the concepts in this question (if desired).

Research Concepts:
Arthritis, Rheumatoid

We update eBooks quarterly and Apps daily based on user feedback.


Please tap flag to report any questions that need improvement.
Section 3

Question 131: What physical exam finding is consistent with a


diaphragmatic hernia?

Choices:
1. Expiratory wheezing
2. E to A changes
3. Unilateral diminished breath sounds with peristaltic sounds
4. Stridor
Answer: 3 - Unilateral diminished breath sounds with peristaltic sounds
Explanations:
Unilateral diminished breath sounds with peristaltic sounds are indicative of
a diaphragmatic hernia.
Asthma can present with expiratory wheezing.
E to A changes are termed egophony and indicate lung consolidation.
Stridor can represent epiglottitis or croup.

Go to the next page if you knew the correct answer, or click the link image(s)
below to further research the concepts in this question (if desired).

Research Concepts:
Hernia, Diaphragmatic, Congenital

We update eBooks quarterly and Apps daily based on user feedback.


Please tap flag to report any questions that need improvement.
Question 132: A 6-month-old girl presents to the emergency department
with a dry cough and a low-grade fever. Over the last two days, she has become
very irritable and has refused to eat. The mother complains that the girl also has
had a runny nose. On examination, the child has significant wheezing and is also
tachycardic. A test of the nasal discharge shows a respiratory syncytial virus
infection. Following treatment, which condition should the mother be informed
is a risk to the child in the future?

Choices:
1. Pneumonia
2. Lung abscess
3. Cystic fibrosis
4. Asthma
Answer: 4 - Asthma
Explanations:
Bronchiolitis is defined as the first episode of wheezing associated with an
upper respiratory tract infection. The cause is usually a respiratory syncytial
virus. Almost all children are exposed to RSV in the first year of life. Those
that develop bronchiolitis are more likely to have asthma later in life. The
severity of the disease correlates with the likelihood of development of
asthma.
Bronchiolitis infection is common in the winter months and affects 50% of
children in the first two years of life. The white blood cell count is normal,
and a chest x-ray may show air trapping or atelectasis. The treatment
involves supportive care with humidified oxygen and bronchodilators.
Ribavirin may be administered in high-risk immunocompromised children
with severe cases of a respiratory syncytial virus (RSV). The infection is
known to occur frequently in children prone to airway reactivity, and there
is an increased incidence of asthma in later life.
Pneumonia can occur with a respiratory syncytial virus but only in a high-
risk group, such as in immunodeficient patients. The infection is generally a
mild upper respiratory tract infection and resolves with supportive care.
Ribavirin prophylaxis is given to these patients.

Go to the next page if you knew the correct answer, or click the link image(s)
below to further research the concepts in this question (if desired).

Research Concepts:
Respiratory Syncytial Virus Infection (RSV)

We update eBooks quarterly and Apps daily based on user feedback.


Please tap flag to report any questions that need improvement.
Question 133: What is the approximate minimal volume of pleural effusion
that becomes visible on a frontal chest radiograph in adult patients?

Choices:
1. 200 to 500 mL
2. 50 to 100 mL
3. 50 mL
4. 75 mL
Answer: 1 - 200 to 500 mL
Explanations:
Pleural effusion on a frontal chest radiograph demonstrates obliteration of
lateral costophrenic angle with so-called meniscus sign. This usually is seen
with a volume of pleural fluid in a range of 200 to 500 mL.
A lateral decubitus view is capable of demonstrating pleural effusion with
as little as 50 to 75 mL. The suspected side should be placed down.
A lateral standard view, usually as a part of the two-view upright technique,
is capable of demonstrating pleural effusion with as little as 75 to 150 mL
of effusion. This also presents with meniscus sign but at the posterior
sulcus.
If the pleural effusion reaches up to the level of the fourth anterior rib, this
indicates a volume of about 1 L.

Go to the next page if you knew the correct answer, or click the link image(s)
below to further research the concepts in this question (if desired).

Research Concepts:
Effusion, Pleural

We update eBooks quarterly and Apps daily based on user feedback.


Please tap flag to report any questions that need improvement.
Question 134: Which of the following is not true of a Bochdalek hernia?
Choices:
1. It can occur on either side of the chest
2. Intestinal malrotation is present
3. Mortality is attributed to pulmonary hypertension
4. Early surgery is the key to success
Answer: 4 - Early surgery is the key to success
Explanations:
It is better to resuscitate these infants and delay surgery.
Early surgery has a higher mortality.
The hernia occurs more on the left side. Ladd bands are present (but small
bowel is spared), and the overall mortality is from pulmonary hypertension.
Babies with difficult oxygenation do poorest.

Go to the next page if you knew the correct answer, or click the link image(s)
below to further research the concepts in this question (if desired).

Research Concepts:
Hernia, Diaphragmatic, Congenital

We update eBooks quarterly and Apps daily based on user feedback.


Please tap flag to report any questions that need improvement.
Question 135: Which of the following is an indication of acute oxygen
toxicity?

Choices:
1. Alertness
2. Paresthesias
3. Cyanosis
4. Tachycardia
Answer: 2 - Paresthesias
Explanations:
Signs of acute oxygen toxicity include dyspnea, paresthesias, and seizure.
Other symptoms include visual changes, especially tunnel vision, tinnitus,
nausea, twitching, especially of the face, behavioral changes including
irritability, anxiety, and confusion, and dizziness.
Cyanosis is not related to high levels of oxygen.
Alertness is not affected unless a seizure ensues.

Go to the next page if you knew the correct answer, or click the link image(s)
below to further research the concepts in this question (if desired).

Research Concepts:
Toxicity, Oxygen

We update eBooks quarterly and Apps daily based on user feedback.


Please tap flag to report any questions that need improvement.
Question 136: With foreign body aspiration, the likelihood of
complications increases after:

Choices:
1. 12 hours
2. 24-48 hours
3. 48-72 hours
4. >96 hours
Answer: 2 - 24-48 hours
Explanations:
The likelihood of complications increases 24-48 hours after foreign body
aspiration.
Early removal of an aspirated foreign body is imperative.
Complications of an aspirated foreign body includes mediastinitis,
tracheoesophageal fistula, atelectasis, and postobstructive pneumonia.
Other complications may include necrotizing pneumonia or abscess,
suppurative pneumonia, and bronchiectasis.

Go to the next page if you knew the correct answer, or click the link image(s)
below to further research the concepts in this question (if desired).

Research Concepts:
Foreign Body Aspiration

We update eBooks quarterly and Apps daily based on user feedback.


Please tap flag to report any questions that need improvement.
Question 137: Concerning hypersensitivity pneumonitis, which of the
following associations is correct?

Choices:
1. Aspergillus is the causative agent in Bagassosis
2. Botrytis cinerea is the causative agent in sewer workers lung
3. Cladosporium is the causative agent in hot tub lung
4. Rhizopus is the causative agent in wood pulp worker's disease
Answer: 3 - Cladosporium is the causative agent in hot tub lung
Explanations:
Cladosporium is the causative agent in 'Hot tub lung' due to contaminated
mist and mold on ceilings and around hot tub.
Hypersensitivity pneumonitis represents a type IV hypersensitivity reaction.
Fibrosis with bronchiolar obliteration occurs with continued exposure to
antigen.

Go to the next page if you knew the correct answer, or click the link image(s)
below to further research the concepts in this question (if desired).

Research Concepts:
Hypersensitivity Pneumonitis

We update eBooks quarterly and Apps daily based on user feedback.


Please tap flag to report any questions that need improvement.
Question 138: A 6-month old infant is seen in the clinic for work up for
tachypnea. The child has been short of breath and dyspneic for about 2 months.
A chest x-ray reveals that he has a hypoplastic right lung. Which of the
following congenital disorders may have such a radiological feature?

Choices:
1. Congenital lobar emphysema
2. Scimitar syndrome
3. Kartagener syndrome
4. Transposition of the great vessels
Answer: 2 - Scimitar syndrome
Explanations:
Scimitar syndrome is associated with a hypoplastic right lung.
Hypoplastic lung also is associated with markedly decreased ventilation and
no perfusion.
In many cases, there are other associated congenital heart defects present.

Go to the next page if you knew the correct answer, or click the link image(s)
below to further research the concepts in this question (if desired).

Research Concepts:
Scimitar Syndrome

We update eBooks quarterly and Apps daily based on user feedback.


Please tap flag to report any questions that need improvement.
Question 139: Which of the following tests is the most appropriate
screening method for HIV in a 3 month old child?

Choices:
1. Western blot
2. ELISA
3. PCR
4. Viral culture
Answer: 3 - PCR
Explanations:
Maternal antibodies in infants make ELISA an unsuitable screening test.
HIV infection can only be confirmed with use of DNA PCR technique in
infants less than 18 months.
All serology tests including Western blotting are not used in young infants.

Go to the next page if you knew the correct answer, or click the link image(s)
below to further research the concepts in this question (if desired).

Research Concepts:
HIV, Testing

We update eBooks quarterly and Apps daily based on user feedback.


Please tap flag to report any questions that need improvement.
Question 140: Which of the following is not a characteristic of acute
respiratory distress syndrome?

Choices:
1. PA02/FI02 ratio of less than or equal to 300
2. Bilateral lung opacities
3. Mortality of less than 20%
4. Pulmonary capillary wedge pressure less than or equal to 18 mm Hg
Answer: 3 - Mortality of less than 20%
Explanations:
The mortality of ARDS is 35% to 50%.
Acute respiratory distress syndrome (ARDS) is triggered by trauma,
pneumonia, and sepsis. The hallmark is a diffuse injury to alveolar barrier
cells, surfactant dysfunction, activation of the innate immune response, and
abnormal coagulation. ARDS results in impaired gas exchange within the
lungs at the microscopic alveoli level.
The signs and symptoms of ARDS can begin within 2 hours to days of the
inciting event. Signs and symptoms include shortness of breath, tachypnea,
and hypoxia.
ARDS is treated with mechanical ventilation. The possibilities of non-
invasive ventilation are limited to the early period of the disease or to
prevention in individuals who are at risk of developing ARDS with atypical
pneumonia, lung contusion, or after major surgery.

Go to the next page if you knew the correct answer, or click the link image(s)
below to further research the concepts in this question (if desired).

Research Concepts:
Acute Respiratory Distress Syndrome (ARDS)

We update eBooks quarterly and Apps daily based on user feedback.


Please tap flag to report any questions that need improvement.
Question 141: In the treatment of acute respiratory distress syndrome, why
are high levels of positive-end respiratory pressure are needed?

Choices:
1. Improve hypoxemia
2. Facilitate carbon dioxide elimination
3. Increase tidal volume
4. None of the above
Answer: 1 - Improve hypoxemia
Explanations:
Positive end-expiratory pressure helps improve hypoxia in acute respiratory
distress syndrome (ARDS) by opening up collapsed alveoli.
It also helps achieve the end goal of increasing functional residual capacity,
the volume of air left in the lungs at the end of expiration.
Improved off-loading of carbon dioxide and increased ventricular
tachycardia help with respiratory insufficiency but do not reverse ARDS.
High positive end-expiratory pressure keeps open alveoli from collapsing
improving ventilation-perfusion mismatch. Keeping alveoli open also
curtails atelectrauma.

Go to the next page if you knew the correct answer, or click the link image(s)
below to further research the concepts in this question (if desired).

Research Concepts:
Acute Respiratory Distress Syndrome (ARDS)

We update eBooks quarterly and Apps daily based on user feedback.


Please tap flag to report any questions that need improvement.
Question 142: What is a disadvantage of inverse ratio setting during
mechanical ventilation in patients with acute respiratory distress syndrome?

Choices:
1. Atelectasis
2. Increase in lung compliance
3. Air trapping
4. Increase in venous return
Answer: 3 - Air trapping
Explanations:
Inverse ratio ventilation uses the same principles to improve oxygenation as
other ventilatory therapies for acute respiratory distress syndrome.
Air trapping is common with inverse ratio ventilation.
Pressure-controlled inverse ratio ventilation ventilatory mode is when the
majority of time is spent at the higher inspiratory pressure.
The risks of auto-PEEP and hemodynamic deterioration due to the
decreased expiratory time and increased mean airway pressure usually
outweighs the small potential for improved oxygenation.

Go to the next page if you knew the correct answer, or click the link image(s)
below to further research the concepts in this question (if desired).

Research Concepts:
Ventilation, Inverse Ratio

We update eBooks quarterly and Apps daily based on user feedback.


Please tap flag to report any questions that need improvement.
Question 143: A four-month-old infant is brought in with difficulty
breathing. Her illness began three days ago with a cough and runny nose. Her
mother states that she began "breathing heavily" overnight, and her difficulty
breathing has worsened progressively. On exam, there is a respiratory rate of 65
breaths per minute. Auscultation reveals diffuse, bilateral wheezes with scattered
crackles. Mild intercostal retractions are present, and the remainder of her exam
is unremarkable. Which of the following histories would raise concern for this
child having a severe course?

Choices:
1. Unrepaired atrial septal defect
2. Premature birth at 29 weeks E.G.A.
3. Older brother with moderate-persistent asthma
4. All of the above
Answer: 2 - Premature birth at 29 weeks E.G.A.
Explanations:
This child's presentation is consistent with bronchiolitis, a viral infection of
the lower respiratory tract.
Bronchiolitis causes signs of increased work of breathing, such as
tachypnea and retractions, and small airway obstruction, as evidenced by
wheezing and crackles.
Age less than 3 months, a history of prematurity < 30 weeks E.G.A.,
congenital heart disease, neuromuscular disease, immunodeficiency, and
chronic respiratory illness are risk factors for severe disease.
Most cases of bronchiolitis will be mild and self-limited. Children with
moderate-to-severe respiratory distress, signs of dehydration or hypoxia
may require hospitalization for supportive care.

Go to the next page if you knew the correct answer, or click the link image(s)
below to further research the concepts in this question (if desired).

Research Concepts:
Bronchiolitis

We update eBooks quarterly and Apps daily based on user feedback.


Please tap flag to report any questions that need improvement.
Question 144: A hyperacute graft rejection is mediated by which of the
following?

Choices:
1. Preformed antibody
2. T cell
3. Neutrophil
4. Macrophage
Answer: 1 - Preformed antibody
Explanations:
Hyperacute graft rejection due to preformed antibody.
Characterized by fever, rash, and diarrhea.

Go to the next page if you knew the correct answer, or click the link image(s)
below to further research the concepts in this question (if desired).

Research Concepts:
Transplantation, Rejection, Acute

We update eBooks quarterly and Apps daily based on user feedback.


Please tap flag to report any questions that need improvement.
Question 145: A 6-year-old child has had recurrent infections of the upper
and lower respiratory tract. These have included pneumonia, chronic otitis media
requiring tympanostomy tubes, and sinusitis. Chest radiograph shows
dextrocardia. What will biopsy of the nasal mucosa show?

Choices:
1. Monocytic infiltrate
2. Eosinophilic infiltrate
3. Structurally abnormal cilia
4. Thinned nasal mucosa
Answer: 3 - Structurally abnormal cilia
Explanations:
The patient likely has Kartagener syndrome, a defect in the structure and
function of cilia resulting in immotility.
The patients have recurrent otitis media, sinusitis, and pneumonia.
50 percent of patients have Situs inversus.
The basic defect involves defects in the dynein arms and abnormal
cytoskeletal proteins.

Go to the next page if you knew the correct answer, or click the link image(s)
below to further research the concepts in this question (if desired).

Research Concepts:
Ciliary Dysfunction (Kartagener Syndrome, Primary Ciliary
Dyskinesia)

We update eBooks quarterly and Apps daily based on user feedback.


Please tap flag to report any questions that need improvement.
Question 146: What is not a criterion for a diagnosis of acute respiratory
distress syndrome?

Choices:
1. Bilateral fluffy infiltrates on chest x-ray
2. Pa02/Fi02 less than 200
3. Elevated left atrial pressure
4. Absence of chronic lung disease
Answer: 3 - Elevated left atrial pressure
Explanations:
In 2012 the ARDS Definition Task Force announced a new definition and
severity classification system for acute respiratory distress syndrome
(ARDS). This is now known as the "Berlin definition."
In general, patients with ARDS will have bilateral fluffy infiltrates on x-ray,
Pa02/Fi02 less than 300, and normal left atrial pressure.
These criteria are usually applicable in the absence of past lung disease.
Excluding heart failure is no longer required.

Go to the next page if you knew the correct answer, or click the link image(s)
below to further research the concepts in this question (if desired).

Research Concepts:
Acute Respiratory Distress Syndrome (ARDS)

We update eBooks quarterly and Apps daily based on user feedback.


Please tap flag to report any questions that need improvement.
Question 147: Which of the following organisms can be a cause of relative
bradycardia with pneumonia?

Choices:
1. Streptococcus pneumoniae
2. Moraxella catarrhalis
3. Legionella pneumophila
4. Staphylococcus aureus
Answer: 3 - Legionella pneumophila
Explanations:
L. pneumophila is considered an atypical cause of community-acquired
pneumonia (CAP).
It is the only cause of atypical CAP associated with relative bradycardia.
Criteria include being > 12 years old , fever > 102 degrees F, pulse
measured while febrile, normal sinus rhythm, and no beta blockers,
verapamil, or diltiazem.
Definition of relative bradycardia at different levels of febrile illness: 102
degrees F, < 100/min; 103°F, < 110/min; 104 degrees F, 120/min; 105
degrees F, < 130/min; 106 degree F, < 140/min.

Go to the next page if you knew the correct answer, or click the link image(s)
below to further research the concepts in this question (if desired).

Research Concepts:
Legionnaires' Disease (Legionella Infection)

We update eBooks quarterly and Apps daily based on user feedback.


Please tap flag to report any questions that need improvement.
Question 148: Which occurrence at a healthcare facility requires
mandatory notification to official agencies?

Choices:
1. Abuse of a child
2. Patient falls
3. Emergency room statistics
4. Injuries to staff
Answer: 1 - Abuse of a child
Explanations:
Reportable incidents are accidental firearm injuries, the death of a patient
and child abuse.

Go to the next page if you knew the correct answer, or click the link image(s)
below to further research the concepts in this question (if desired).

Research Concepts:
Child Abuse and Neglect

We update eBooks quarterly and Apps daily based on user feedback.


Please tap flag to report any questions that need improvement.
Question 149: Which of the following tests would be essential to diagnose
idiopathic pulmonary hemosiderosis?

Choices:
1. Lung biopsy
2. Chest CT
3. Broncho alveolar lavage
4. Sputum examination
Answer: 3 - Broncho alveolar lavage
Explanations:
BAL would identify hemosiderin laden macrophages and this finding is
essential to make such a diagnosis.
Sputum examination would not reveal anything in this case.

Go to the next page if you knew the correct answer, or click the link image(s)
below to further research the concepts in this question (if desired).

Research Concepts:
Idiopathic Pulmonary Hemosiderosis

We update eBooks quarterly and Apps daily based on user feedback.


Please tap flag to report any questions that need improvement.
Question 150: Which of the following hemodynamic changes is not
associated with positive pressure ventilation?

Choices:
1. Decreased cardiac output
2. Increased venous return
3. Increased right ventricular pressure
4. Increased afterload
Answer: 2 - Increased venous return
Explanations:
Positive pressure ventilation (PPV) decreases venous return and reduces
cardiac output but increases right ventricular afterload.
PPV is practiced when a bag-mask device is utilized. A bag-valve-mask is
utilized until a definitive airway can be placed or condition improves. In the
pediatric population, a bag-valve-mask device can assist with oxygenation
and ventilation with positive results, especially in newborns.
Positive pressure ventilation also can be obtained by the use of a
mechanical ventilator.The ventilator will push the air or oxygen into the
trachea. Pulmonary barotrauma can occur with positive pressure ventilation.
Lung injury associated with positive pressure ventilation can occur and
mimics acute respiratory distress syndrome.

Go to the next page if you knew the correct answer, or click the link image(s)
below to further research the concepts in this question (if desired).

Research Concepts:
Ventilation, Positive Pressure

We update eBooks quarterly and Apps daily based on user feedback.


Please tap flag to report any questions that need improvement.
Question 151: A 4-year-old with pertussis does not tolerate macrolides.
Which antibiotic can be prescribed as an alternative?

Choices:
1. First-generation cephalosporin
2. Trimethoprim-sulfamethoxazole
3. Penicillin
4. Second-generation cephalosporin
Answer: 2 - Trimethoprim-sulfamethoxazole
Explanations:
Trimethoprim-sulfamethoxazole is an alternative antibiotic for the treatment
of pertussis.
Penicillin and first- and second-generation cephalosporins are not effective
against Bordetella pertussis.
Trimethoprim-sulfamethoxazole is also indicated in cases of macrolide
resistance.
Antibiotic treatment against pertussis is most likely to ameliorate symptoms
if given during the catarrhal phase.

Go to the next page if you knew the correct answer, or click the link image(s)
below to further research the concepts in this question (if desired).

Research Concepts:
Pertussis

We update eBooks quarterly and Apps daily based on user feedback.


Please tap flag to report any questions that need improvement.
Question 152: Which of the following pulmonary lymph node drainage
systems is incorrect?

Choices:
1. Right upper lobe drains to right paratracheal and then to neck
2. Right middle lobe drains to subcarinal and then to right paratracheal
3. Right lower lobe drains to subcarinal and then to right paratracheal
4. Left lower lobe drains subcarinal and then to left paratracheal
Answer: 4 - Left lower lobe drains subcarinal and then to left paratracheal
Explanations:
The left lower lobe drains to the subcarinal lymph nodes, then to the right
paratracheal.
When evaluating a patient with a left upper lobe mass, a Chamberlain
procedure is done.
The left upper lobe cancers usually drain to the para-aortic and subaortic
nodes and thus a Chamberlain procedure is done.
When evaluating a patient with a lower lobe lung mass, a cervical
mediastinoscopy is done.

Go to the next page if you knew the correct answer, or click the link image(s)
below to further research the concepts in this question (if desired).

Research Concepts:
Cancer, Lung

We update eBooks quarterly and Apps daily based on user feedback.


Please tap flag to report any questions that need improvement.
Question 153: A 4-year-old has fatigue on exertion. The patient has a
history of recurrent apneic episodes triggered by vomiting or fever. Neurological
examination discloses mild asymmetric ptosis. Test for acetylcholine receptor
antibodies is negative. EMG studies show decremental response at 10 Hz
stimulation but absence of decremental response at 2 Hz in rested muscle. This is
an example of which congenital myasthenic syndrome?

Choices:
1. Congenital myasthenic syndrome with episodic apnea
2. End-plate acetylcholine esterase deficiency
3. Slow-channel congenital myasthenic syndrome
4. Lambert-Eaton myasthenic syndrome
Answer: 1 - Congenital myasthenic syndrome with episodic apnea
Explanations:
This patient has the characteristic features of congenital myasthenic
syndrome with episodic apnea: recurrent apneic episodes triggered by fever
or vomiting, mild ptosis, absence of decremental response on 2 Hz
stimulation and the presence of decremental response on 10 Hz stimulation.
Muscle biopsy confirms the diagnosis by showing end-plate acetylcholine
receptor deficiency without postsynaptic structural abnormalities.
The abnormality involves a presynaptic defect.
There is a marked decrease in the number of acetylcholine quanta released
by the nerve impulse due to a defect in the synthesis or axonal transport of
vesicle precursors from the anterior horn cell to the nerve terminal.

Go to the next page if you knew the correct answer, or click the link image(s)
below to further research the concepts in this question (if desired).

Research Concepts:
Apnea, Congenital Myasthenic Syndrome with Episodic Apnea

We update eBooks quarterly and Apps daily based on user feedback.


Please tap flag to report any questions that need improvement.
Question 154: Which of the following diseases is known to cause repeated
pulmonary hemorrhages in children?

Choices:
1. Idiopathic pulmonary hemosiderosis
2. Granulomatosis with polyangiitis
3. Polyarteritis nodosa
4. Tuberculosis
Answer: 1 - Idiopathic pulmonary hemosiderosis
Explanations:
Idiopathic pulmonary hemosiderosis is a rare disease found mostly in
children that causes recurrent episodes of diffuse alveolar hemorrhage.
Alveolar bleeding may eventually produce fibrosis and pulmonary
hemosiderosis.
Diffuse alveolar hemorrhage is characterized by dyspnea, hemoptysis, and
alveolar opacities on chest radiographs.

Go to the next page if you knew the correct answer, or click the link image(s)
below to further research the concepts in this question (if desired).

Research Concepts:
Idiopathic Pulmonary Hemosiderosis

We update eBooks quarterly and Apps daily based on user feedback.


Please tap flag to report any questions that need improvement.
Question 155: In what way does obstructive sleep apnea (OSA) differ
between children and adults?

Choices:
1. Cardiopulmonary damage only is seen in adults
2. Adults tend to have apnea during non-REM sleep while children tend to have
REM sleep-associated apnea
3. Children with OSA often have failure to thrive
4. There is more daytime drowsiness in children
Answer: 3 - Children with OSA often have failure to thrive
Explanations:
Cardiopulmonary damage from obstructive sleep apnea (OSA) occurs in
both children and adults.
Children with OSA often have a failure to thrive while adults tend to be
obese.
Children with OSA are less likely to have sleep arousals and have less
daytime sleepiness but have more attention and behavior problems.
In children, the most common cause of OSA is enlarged tonsils and
adenoids. In adults, it is most commonly associated with obesity, male
gender, and advancing age.

Go to the next page if you knew the correct answer, or click the link image(s)
below to further research the concepts in this question (if desired).

Research Concepts:
Apnea, Obstructive Sleep Apnea

We update eBooks quarterly and Apps daily based on user feedback.


Please tap flag to report any questions that need improvement.
Question 156: Which of the following is true of high-frequency jet
ventilation?

Choices:
1. Protects surfactant
2. Decreases diffusion of oxygen
3. Decreases diffusion of carbon dioxide
4. All of the above
Answer: 1 - Protects surfactant
Explanations:
High-frequency jet ventilation protects surfactant.
It enhances oxygen diffusion.
It enhances carbon dioxide diffusion.
Respiratory rates in this type of ventilation are 100-200 breaths/minute.

Go to the next page if you knew the correct answer, or click the link image(s)
below to further research the concepts in this question (if desired).

Research Concepts:
Ventilation, High Frequency

We update eBooks quarterly and Apps daily based on user feedback.


Please tap flag to report any questions that need improvement.
Question 157: What does one call cycles of rapid deep breathing followed
by shallow breathing and apnea?

Choices:
1. Bainbridge reflex
2. Cheyne-Stokes respiration
3. Kussmaul breathing
4. Pulsus paradoxus
Answer: 2 - Cheyne-Stokes respiration
Explanations:
Cheyne-Stokes breathing is rapid deep breathing followed by a gradual
decrease and then apnea.
Cheyne-Stokes breathing oscillates from apnea to hyperpnea.
Cheyne-Stokes breathing is associated with changes in blood oxygen and
carbon dioxide levels.

Go to the next page if you knew the correct answer, or click the link image(s)
below to further research the concepts in this question (if desired).

Research Concepts:
Cheyne Stokes Respirations

We update eBooks quarterly and Apps daily based on user feedback.


Please tap flag to report any questions that need improvement.
Question 158: In a patient with Hanta virus pulmonary syndrome, what is
the best treatment besides supportive care?

Choices:
1. Interferon
2. Ribavirin
3. Zidovudine
4. Amantadine
Answer: 2 - Ribavirin
Explanations:
Ribavirin is often used to treat patients with Hanta virus.
However, the dug is only effective in the early stages of the disease.
In the late stages, it appears that no other drug works.

Go to the next page if you knew the correct answer, or click the link image(s)
below to further research the concepts in this question (if desired).

Research Concepts:
Hantavirus Cardiopulmonary Syndrome

We update eBooks quarterly and Apps daily based on user feedback.


Please tap flag to report any questions that need improvement.
Question 159: In a patient on mechanical ventilation who is undergoing a
spontaneous breathing trial, which of the following is most suggestive of
extubation failure, or need for reintubation, should the patient be extubated?

Choices:
1. Follows commands appropriately
2. Sleepy yet easily arousable
3. Respiratory rate stays at the 10 to 16 breaths per minute range
4. Oxygen saturation drops from 100 percent to 89 percent
Answer: 4 - Oxygen saturation drops from 100 percent to 89 percent
Explanations:
Indicators of failure to wean are a rise in heart rate of 24 beats/minute, a
drop in oxygen saturation below 90%, a rise in minute ventilation above 10
L/minute, and a rise in respiratory rate by 14 breaths/minute.
About 15% of patients fail extubation, the optimal rate is unknown, but is
probably 5% to 10%.
Higher rates of failure are likely to lead to prolongation of intubation.
Approach to extubation should consider a resolution of the disease process
and candidates who successfully pass a spontaneous breathing trial.

Go to the next page if you knew the correct answer, or click the link image(s)
below to further research the concepts in this question (if desired).

Research Concepts:
Ventilation, Ventilator Management

We update eBooks quarterly and Apps daily based on user feedback.


Please tap flag to report any questions that need improvement.
Question 160: What bacterial pathogen is a common cause of hospital-
acquired pneumonia?

Choices:
1. Listeria
2. Pseudomonas aeruginosa
3. E. coli
4. Mycoplasma
Answer: 2 - Pseudomonas aeruginosa
Explanations:
Pseudomonas aeruginosa and Streptococcus pneumonia are the most
common pathogens for hospital-acquired pneumonia.
Treatment involves a macrolide plus a third-generation cephalosporin.

Go to the next page if you knew the correct answer, or click the link image(s)
below to further research the concepts in this question (if desired).

Research Concepts:
Pneumonia, Nosocomial

We update eBooks quarterly and Apps daily based on user feedback.


Please tap flag to report any questions that need improvement.
Question 161: A 10 year old boy has had a cough for 8 weeks. He has had
no fever or upper respiratory symptoms. Physical exam is normal. The CBC,
chest x-ray, and sinus CT are normal. Spirometry is normal. A tic disorder is
being considered. Which of the following would be most consistent with this
diagnosis?

Choices:
1. Absence of the cough at night
2. Family history of a tic
3. Improvement with inhaled beta agonists
4. Elevated IgE
Answer: 1 - Absence of the cough at night
Explanations:
The absence of a cough at night makes the diagnosis of psychogenic cough
most likely.
Medications are not effective for this condition.
Possible treatments include speech therapy, self-hypnosis, and suggestion
therapy.
Suggestion therapy involves using a placebo and having a caregiver say that
there are exercises that will help control the cough. Referral to a
psychologist is sometimes necessary.

Go to the next page if you knew the correct answer, or click the link image(s)
below to further research the concepts in this question (if desired).

Research Concepts:
Psychogenic Cough

We update eBooks quarterly and Apps daily based on user feedback.


Please tap flag to report any questions that need improvement.
Question 162: Which of the following conditions can be treated with high-
frequency ventilation?

Choices:
1. Increased potential for infection
2. Bronchopleural fistula with large air leak
3. Ineffective airway clearance
4. Increased intracranial pressure
Answer: 2 - Bronchopleural fistula with large air leak
Explanations:
Insufficient humidification results in thick secretions and possible mucus
plugs.
High-frequency ventilation is used in patients with large air leaks and those
who are difficult to ventilate.
High-frequency ventilation is a type of mechanical ventilation which
utilizes a respiratory rate greater than four times the normal value. High-
frequency ventilation is thought to reduce ventilator-associated lung injury,
especially in the context of acute respiratory syndrome and acute lung
injury. High-frequency ventilation may be used by itself or in combination
with conventional mechanical ventilation. Devices that need conventional
mechanical ventilation typically do not produce the same lung protective
effects as those that can operate without tidal breathing.
With conventional ventilation where tidal volumes normally exceed dead
space, gas exchange is largely related to the bulk flow of gas to the alveoli.
With high-frequency ventilation, the tidal volumes used are smaller than
anatomical and equipment dead space, and therefore alternative
mechanisms of gas exchange occur.

Go to the next page if you knew the correct answer, or click the link image(s)
below to further research the concepts in this question (if desired).

Research Concepts:
Ventilation, High Frequency

We update eBooks quarterly and Apps daily based on user feedback.


Please tap flag to report any questions that need improvement.
Question 163: Which is not a chest finding in patients with a splenic
abscess?

Choices:
1. Elevation of left hemidiaphragm
2. Dullness at left base
3. Hyperresonant chest
4. Left basilar rales
Answer: 3 - Hyperresonant chest
Explanations:
Chest findings are very non-specific.
One-third of patients may have dullness at the left lung base, left basilar
rales, and elevation of the left hemidiaphragm.
Diagnosis of a splenic abscess is a clinical challenge. Plain radiographs of
the chest can reveal many findings indicative of a splenic abscess such as
an elevated left hemidiaphragm and left-sided pleural effusion with or
without left basal atelectasis. An ultrasonogram typically demonstrates an
area of decreased or absent echogenicity and splenomegaly. An
ultrasonogram is quick and can be done at the bedside. A CT scan is the
gold standard for diagnosis. The scan also helps with treatment planning by
delineating the details of the abscess and the topography of the surrounding
structures.
High-dose parenteral, broad-spectrum antibiotics are of paramount
importance while further diagnostic and therapeutic arrangements are made.
The gold standard for treatment of splenic abscess is splenectomy; however,
recent studies have shown success with different approaches based on
abscess characteristics. Percutaneous aspiration may be a less invasive
option in patients who are at high risk for surgery or a temporary solution
used as a bridge to surgery, avoiding the risk of a fulminant and potentially
life-threatening infection. A percutaneous aspiration is a successful
approach when the abscess collection is unilocular or bilocular, with a
complete and thick wall and no internal septations. Aspiration is easier to
achieve when the content is liquid enough to be drained. If there are
multiple collections or associated coagulopathy, either laparoscopic or open
surgical treatment is preferred.

Go to the next page if you knew the correct answer, or click the link image(s)
below to further research the concepts in this question (if desired).

Research Concepts:
Abscess, Splenic

We update eBooks quarterly and Apps daily based on user feedback.


Please tap flag to report any questions that need improvement.
Question 164: Which is not a feature of idiopathic pulmonary arterial
hypertension?

Choices:
1. Recurrent syncope
2. Elevated systemic systolic blood pressure
3. Dyspnea
4. Oxygen desaturation with exertion
Answer: 2 - Elevated systemic systolic blood pressure
Explanations:
Recurrent syncope, dyspnea, and oxygen desaturation are seen in idiopathic
pulmonary arterial hypertension (IPAH).
IPAH is a progressive disease that affects the precapillary pulmonary
vasculature for which an exact underlying risk factor is unknown.
The pulmonary artery pressure is persistently more than 25 mmHg at rest
and more than 30 mmHg during exercise. It is a rare but a fatal disease
which has a high mortality.
If left untreated, it may result in increasing back pressures and ultimately
right heart failure and death. The blood vessels going to or within the lung
become narrow making it difficult for the heart to pump blood due to the
restriction of pulmonary circulation. Pulmonary artery resistance is
increased due to vasoconstriction, vascular remodeling, and thrombosis.

Go to the next page if you knew the correct answer, or click the link image(s)
below to further research the concepts in this question (if desired).

Research Concepts:
Pulmonary Hypertension, Idiopathic

We update eBooks quarterly and Apps daily based on user feedback.


Please tap flag to report any questions that need improvement.
Question 165: After anthrax inhalation, what is the classic radiological
finding?

Choices:
1. Unilateral effusion
2. Bilateral effusion
3. Widened mediastinum
4. Interstitial infiltrate
Answer: 3 - Widened mediastinum
Explanations:
Inhalation anthrax presents with widened mediastinum. It is a sign of
mediastinitis.
Those at risk include wool mill, slaughterhouse, and tannery workers. Such
workers may breath in anthrax spores if working with infected animals as
well as contaminated animal products.
Inhalational anthrax routinely begins in mediastinal/hilar lymph nodes prior
to spreading.
Without treatment, inhalational anthrax has an 85-90 percent mortality.
With aggressive treatment, it has a 45 percent mortality.

Go to the next page if you knew the correct answer, or click the link image(s)
below to further research the concepts in this question (if desired).

Research Concepts:
Anthrax

We update eBooks quarterly and Apps daily based on user feedback.


Please tap flag to report any questions that need improvement.
Question 166: Which of the following is the best diagnostic test for
Rhodococcus equi infection?

Choices:
1. Serology
2. Polymerase chain reaction
3. Culture
4. Direct examination
Answer: 3 - Culture
Explanations:
Serology has little role in the diagnosis of Rhodococcus infection.
While polymerase chain reaction methods, including 16S rRNA techniques,
have a role diagnosis in culture negative cases, it is not the preferred
method of diagnosis.
Culture is the best method to diagnose infection. The causative organism is
an obligate gram-positive coccobacillus that can be easily grown on several
body fluids and tissue samples.
While direct examination may show the causative bacteria in certain cases,
it is inferior to culture as a means of diagnosis.

Go to the next page if you knew the correct answer, or click the link image(s)
below to further research the concepts in this question (if desired).

Research Concepts:
Rhodococcus Equi

We update eBooks quarterly and Apps daily based on user feedback.


Please tap flag to report any questions that need improvement.
Question 167: What statement is false about bronchiolitis obliterans?
Choices:
1. Repeat transplantation can be undertaken for bronchiolitis obliterans
syndrome
2. Augmented immunosuppression can be used for treatment
3. Dense fibrosis obliterates the bronchial wall and lumen
4. Treatment reverses the condition
Answer: 4 - Treatment reverses the condition
Explanations:
Bronchiolitis obliterans (BO) is a disorder of the small airways that affects
both adults and children. About 50% of lung transplant patients have some
evidence of BO after the first year. Patients will present with a dry cough,
dyspnea, and a serial decline in FEV1.
Eventually, the disease progresses leading to dense fibrosis of the distal
airways. The fibrosis is irreversible, and there is no satisfactory treatment to
date.
Bronchiolitis obliterans is a difficult condition to treat. A biopsy is not
always required to make a diagnosis. The therapeutic options for this
disorder are limited and consist of augmented immunosuppression, but is
usually progressive.
Apart from lung transplantation, where it is usually called "bronchiolitis
obliterans syndrome (BOS)", bone marrow transplantation leading to graft
versus host disease can also lead to bronchiolitis obliterans. Other less
common conditions that lead to bronchiolitis obliterans include rheumatoid
arthritis involving the lung.

Go to the next page if you knew the correct answer, or click the link image(s)
below to further research the concepts in this question (if desired).

Research Concepts:
Bronchiolitis Obliterans (Obliterative Bronchiolitis, Constrictive
Bronchiolitis)

We update eBooks quarterly and Apps daily based on user feedback.


Please tap flag to report any questions that need improvement.
Question 168: A patient with a malignant pleural effusion (MPE) has been
admitted for pleurodesis. A thoracostomy tube is placed and 3.3 liters is rapidly
evacuated from the right chest. The patient immediately begins coughing
violently and complaining of pleuritic chest pain. The chest x-ray shows
unilateral haziness and fluffy infiltrates. What is the most likely diagnosis?

Choices:
1. Traumatic chest insertion into the lung
2. Intercostal neuralgia
3. Re-expansion pulmonary edema
4. Congestive heart failure
Answer: 3 - Re-expansion pulmonary edema
Explanations:
In patients with a malignant pleural effusion, it must be determined whether
the lung will expand or not after evacuation of the effusion.
If the lung expands completely, the effusion can be treated by sclerosis to
prevent recurrence. If the lung is trapped and the patient is symptomatic, a
Denver shunt may be an option.
Most of these patients are too ill to undergo pleurectomy/decortication and
the malignant pleural effusion is usually resistant to chemotherapy.
Unilateral re-expansion edema occurs when pleural fluid is rapidly
evacuated in a patient with lung collapse that has persisted for more than 2-
3 days. Patients present with violent coughing and air hunger.

Go to the next page if you knew the correct answer, or click the link image(s)
below to further research the concepts in this question (if desired).

Research Concepts:
Pulmonary Edema, Noncardiogenic

We update eBooks quarterly and Apps daily based on user feedback.


Please tap flag to report any questions that need improvement.
Question 169: Which of the following breathing patterns is characterized
by four to five breaths of equal tidal volume followed by apnea?

Choices:
1. Tachypnea
2. Breath holding
3. Kussmaul
4. Biot
Answer: 4 - Biot
Explanations:
Biot respiration is an abnormal respiratory pattern characterized by apnea
followed by 4 to 5 normal breaths then apnea.
Biot respiration is caused by damage to the pons due to strokes, trauma, or
by pressure on the pons due to or tentorial or uncal herniation.
Biot respirations can be caused by opioid use.
Biot respirations are distinguished from ataxic respiration by having more
regularity and similar-sized inspirations Ataxic respirations are more
completely irregular breaths and pauses. As the breathing pattern
deteriorates, it merges with ataxic respirations. Biot respirations are
equivalent to Cheyne-Stokes respiration, although the two definitions are
separated in some academic settings.

Go to the next page if you knew the correct answer, or click the link image(s)
below to further research the concepts in this question (if desired).

Research Concepts:
Abnormal Respirations

We update eBooks quarterly and Apps daily based on user feedback.


Please tap flag to report any questions that need improvement.
Question 170: Which of the following PaO2/FiO2 ratios defines moderate
acute respiratory distress syndrome?

Choices:
1. Less than 100
2. Less than 150
3. Less than 200
4. Less than 300
Answer: 3 - Less than 200
Explanations:
The Berlin definition of acute respiratory distress syndrome (ARDS)
categorizes ARDS severity on the basis of PaO2/FiO2 ratios when a patient
is on 5 cm of continuous positive airway pressure. A ratio of between 100
and less than 200 defines moderate ARDS. Mild ARDS is between 200 and
300. Severe ARDS id less than or equal to 100.
Bilateral opacities in the absence of heart failure also is a criterion for the
diagnosis.
The signs and symptoms of ARDS can begin within 2 hours to days of the
inciting event. Signs and symptoms include shortness of breath, tachypnea,
and hypoxia.
Radiographic findings of pulmonary edema affecting both lungs and
unrelated to heart failure suggest ARDS. Ultrasound findings include
anterior subpleural consolidations, absence or reduction of lung sliding,
“spared areas” of normal parenchyma, pleural line abnormalities, and
nonhomogeneous distribution of B-lines.

Go to the next page if you knew the correct answer, or click the link image(s)
below to further research the concepts in this question (if desired).

Research Concepts:
Acute Respiratory Distress Syndrome (ARDS)

We update eBooks quarterly and Apps daily based on user feedback.


Please tap flag to report any questions that need improvement.
Question 171: What condition should suspect in children who have nasal
polyps?

Choices:
1. Asthma
2. Cystic fibrosis
3. Colonic polyps
4. Neurofibromas
Answer: 2 - Cystic fibrosis
Explanations:
Nasal polyps in children could be secondary to cystic fibrosis.
Up to 30% of children with cystic fibrosis have nasal polyps.
Sweat testing is indicated in children with nasal polyps.
Other diseases associated with nasal polyps are aspirin sensitivity and
asthma.

Go to the next page if you knew the correct answer, or click the link image(s)
below to further research the concepts in this question (if desired).

Research Concepts:
Nasal Polyps

We update eBooks quarterly and Apps daily based on user feedback.


Please tap flag to report any questions that need improvement.
Question 172: Which of the following does not improve long-term
mortality for acute respiratory distress syndrome?

Choices:
1. Corticosteroids
2. Diuretics
3. Antibiotics
4. Oxygen
Answer: 1 - Corticosteroids
Explanations:
Acute respiratory distress syndrome (ARDS) is associated with pulmonary
edema.
Corticosteroids decrease inflammation, increase ventilatory free days, and
improve oxygenation.
Despite these short-term gains, the 60-day mortality does not improve.
Steroids worsen mortality if administered to late phase ARDS, ie, >13 days
after onset.

Go to the next page if you knew the correct answer, or click the link image(s)
below to further research the concepts in this question (if desired).

Research Concepts:
Acute Respiratory Distress Syndrome (ARDS)

We update eBooks quarterly and Apps daily based on user feedback.


Please tap flag to report any questions that need improvement.
Question 173: What disorder can develop in children who have
bronchiolitis?

Choices:
1. Chronic obstructive pulmonary disease
2. Emphysema
3. Asthma
4. Recurrent pneumonia
Answer: 3 - Asthma
Explanations:
A history of recurrent wheezing episodes that improves with albuterol and a
family history of atopy (asthma, allergic rhinitis or eczema) support a
diagnosis of asthma in children.
Viruses, such as those that cause bronchiolitis, are a common trigger of
exacerbations in children with asthma.
One large study showed that 16 to 23 percent of asthmatics had a history of
bronchiolitis, while only 8 to 12 percent of nonasthmatic patients had such
a history.
It is unclear if there is a causal relationship between bronchiolitis and
asthma.

Go to the next page if you knew the correct answer, or click the link image(s)
below to further research the concepts in this question (if desired).

Research Concepts:
Bronchiolitis

We update eBooks quarterly and Apps daily based on user feedback.


Please tap flag to report any questions that need improvement.
Question 174: A tall male presents to you with complaints of coughing
episodes. On examination, subcutaneous emphysema is observed on the entire
chest wall and neck. He is not in respiratory distress and the chest x-ray reveals
mediastinal air. Which of the following is most appropriate for the management
of this patient?

Choices:
1. Chest tubes on either side of the chest to evacuate the air
2. Small superficial skin incisions to allow for the air to egress
3. Observation and repeat chest x-ray
4. CT scan of the chest
Answer: 3 - Observation and repeat chest x-ray
Explanations:
Observation is appropriate in non-trauma patients.
Some asthmatics will develop episodes of coughing and subcutaneous
emphysema. They can be observed and the emphysema disappears with
time.
If subcutaneous emphysema is associated with a pneumothorax, which is
refractory to chest tube suction, it should raise concern for tracheal
bronchial disruption.
Bronchoscopy is required if injury to the airways is suspected.

Go to the next page if you knew the correct answer, or click the link image(s)
below to further research the concepts in this question (if desired).

Research Concepts:
Subcutaneous Emphysema

We update eBooks quarterly and Apps daily based on user feedback.


Please tap flag to report any questions that need improvement.
Question 175: A 20-month-old boy has a cough, decreased appetite, and
tachypnea for five days, and now has a high fever. The exam shows a fever,
abdominal distention, grunting, intercostal retractions, rhonchi in the right lung,
and dullness to percussion. The chest radiograph shows a left lower lobe
infiltrate. Which of the following is the most likely infectious organism?

Choices:
1. Streptococcus pneumoniae
2. Mycoplasma pneumoniae
3. Moraxella catarrhalis
4. Staphylococcus aureus
Answer: 4 - Staphylococcus aureus
Explanations:
Staphylococcal pneumonia is characterized by initial upper respiratory
symptoms followed by rapid progression to respiratory distress and high
fever.
Staphylococcal pneumonia is on the right side in 65 percent of cases.
Pneumococcus would be a reasonable second choice as it is also associated
with pleural effusions and high fever.
Abdominal pain is common in lower lobe pneumonia in the pediatric
population and can lead to abdominal distention.

Go to the next page if you knew the correct answer, or click the link image(s)
below to further research the concepts in this question (if desired).

Research Concepts:
Pneumonia, Bacterial

We update eBooks quarterly and Apps daily based on user feedback.


Please tap flag to report any questions that need improvement.
Question 176: Which of the following is a characteristic of inverse-ratio
ventilation, in comparison to standard mechanical ventilation?

Choices:
1. Decreased weaning times
2. Improved carbon dioxide elimination
3. Improved patient comfort
4. Longer inspiratory phase
Answer: 4 - Longer inspiratory phase
Explanations:
Inverse ratio ventilation is a method of mechanical ventilation where more
time being spent in inspiration than expiration.
It may cause intrinsic PEEP.
The inability to exhale completely can cause patient discomfort and anxiety.
It improves oxygenation but has not yet been shown to improve outcomes.

Go to the next page if you knew the correct answer, or click the link image(s)
below to further research the concepts in this question (if desired).

Research Concepts:
Ventilation, Inverse Ratio

We update eBooks quarterly and Apps daily based on user feedback.


Please tap flag to report any questions that need improvement.
Question 177: When is a thoracotomy usually indicated?
Choices:
1. Pulmonary contusion
2. Flail chest
3. Hemothorax with initial blood loss of 700 mL
4. Lung collapse with an air leak that fails to seal after 2 weeks
Answer: 4 - Lung collapse with an air leak that fails to seal after 2 weeks
Explanations:
Thoracotomy is used to treat a variety of problems in the chest.
In trauma, the use of thoracotomy has declined. The majority of
hemothoraces can be treated with a chest tube.
A thoracotomy is indicated if the patient has an uncontrolled air leak with
the failure of the lung to expand.
A bronchoscopy may be required prior to surgery. However, if the lung
remains upright even with an air leak, it may be worth observing the patient
for a few days.

Go to the next page if you knew the correct answer, or click the link image(s)
below to further research the concepts in this question (if desired).

Research Concepts:
Air Leak

We update eBooks quarterly and Apps daily based on user feedback.


Please tap flag to report any questions that need improvement.
Question 178: What is the approximate mortality for patients with acute
respiratory distress syndrome?

Choices:
1. 100 percent
2. 4 percent
3. 40 percent
4. 80 percent

Photo:Contributed by Wikimedia Commons (Public Domain)


Answer: 3 - 40 percent
Explanations:
The mortality for patients with acute respiratory distress syndrome (ARDS)
averages about 40 percent.
Younger patients tend to have better survival rates.
Most patients who survive ARDS have some impairment of pulmonary
function at 12 months.
Extracorporeal Membrane Oxygenation (ECMO), which became popular
after the H1N1 outbreak in 2009, is a possible treatment for patients with
ARDS. Because ECMO can increase the chance of survival, it is advisable
to transfer a patient with ARDS to a facility trained in the use of ECMO.
Indications for its use include persistent low oxygen saturation and elevated
carbon dioxide levels, despite adequate ventilatory support. Young patients
with viral pneumonias such as the ones that accompany Influenza A have
the best chances of survival. Studies are still underway on ECMO's benefits
versus its risks.

Go to the next page if you knew the correct answer, or click the link image(s)
below to further research the concepts in this question (if desired).

Research Concepts:
Acute Respiratory Distress Syndrome (ARDS)

We update eBooks quarterly and Apps daily based on user feedback.


Please tap flag to report any questions that need improvement.
Question 179: What is the advantage of pressure controlled ventilation
(PCV) over volume-cycled ventilation (VCV)?

Choices:
1. Lowers work of breathing
2. Decreases risk of respiratory acidosis
3. Does not require sedation
4. Better assists spontaneous breathing
Answer: 1 - Lowers work of breathing
Explanations:
During VCV, there is an increase in airway pressure in response to
decreased compliance and elevated resistance. These factors can increase
barotrauma.
With PCV, the maximum airway pressure delivered to the lung is limited by
design, but this also results in variable minute and tidal volume. PCV also
can cause barotrauma.
With today's machines, there is little advanatge of PCV over VCV in
patients who are not breathing spontaneously.
PCV, in general, decreases the work of breathing and enhances patient
comfort because of its ability to adjust for variable respiratory demands.

Go to the next page if you knew the correct answer, or click the link image(s)
below to further research the concepts in this question (if desired).

Research Concepts:
Ventilation, Pressure Controlled

We update eBooks quarterly and Apps daily based on user feedback.


Please tap flag to report any questions that need improvement.
Question 180: A patient has asthma and a 90% of personal best peak
expiratory flow?What is the zone?

Choices:
1. Blue
2. Yellow
3. Red
4. Green
Answer: 4 - Green
Explanations:
The green zone is 80% to100% of personal best.
The yellow zone is 50% to 80% of personal best.
The red zone is below 50% of personal best.
There is no blue zone.

Go to the next page if you knew the correct answer, or click the link image(s)
below to further research the concepts in this question (if desired).

Research Concepts:
Asthma

We update eBooks quarterly and Apps daily based on user feedback.


Please tap flag to report any questions that need improvement.
Question 181: In a patient with a pulmonary embolus, what is the duration
of treatment?

Choices:
1. 1 month
2. 1 week
3. 3-6 months
4. For life
Answer: 3 - 3-6 months
Explanations:
Anticoagulation therapy is generally continued for 3-6 months.
Patients with factor C or S deficiency may need long-term anticoagulation.

Go to the next page if you knew the correct answer, or click the link image(s)
below to further research the concepts in this question (if desired).

Research Concepts:
Embolism, Pulmonary

We update eBooks quarterly and Apps daily based on user feedback.


Please tap flag to report any questions that need improvement.
Question 182: When does chronic transplant rejection occur?
Choices:
1. Minutes
2. Days
3. Months
4. Seconds
Answer: 3 - Months
Explanations:
Hyperacute transplant rejection occurs within minutes.
Acute rejection occurs within days.
Chronic rejection can occur in months to years.

Go to the next page if you knew the correct answer, or click the link image(s)
below to further research the concepts in this question (if desired).

Research Concepts:
Transplantation, Rejection, Chronic

We update eBooks quarterly and Apps daily based on user feedback.


Please tap flag to report any questions that need improvement.
Question 183: When is the best time for patients with asthma to measure
peak flow rates?

Choices:
1. In the afternoon
2. In the evening
3. Before bedtime
4. In the morning
Answer: 4 - In the morning
Explanations:
The guidelines for peak flow monitoring require assessment each morning.

Go to the next page if you knew the correct answer, or click the link image(s)
below to further research the concepts in this question (if desired).

Research Concepts:
Peak Flow Rate Measurement

We update eBooks quarterly and Apps daily based on user feedback.


Please tap flag to report any questions that need improvement.
Question 184: Which of the following medications is primarily used to
manage airway reactivity- induced asthma in singers?

Choices:
1. Corticosteroid inhalers
2. Bronchodilator inhalers
3. Oral medications alone
4. Voice rest
Answer: 3 - Oral medications alone
Explanations:
Because oral inhalants, even without propellants, may cause laryngitis,
management with oral medications is preferred for managing airway
reactivity- induced asthma in singers.
Singers with asthma often are treated prophylactically as to avoid
precipitation of an attack during an event.
Some practitioners recommend the use of theophylline as it is an effective
oral agent.
The downside to theophylline use is the potential for adverse effects like
tremor, anxiety, and tachycardia.

Go to the next page if you knew the correct answer, or click the link image(s)
below to further research the concepts in this question (if desired).

Research Concepts:
Asthma

We update eBooks quarterly and Apps daily based on user feedback.


Please tap flag to report any questions that need improvement.
Question 185: What is the most common type of tracheoesophageal fistula
(TEF)?

Choices:
1. Blind proximal pouch with a distal TEF
2. Proximal TEF
3. H- type fistula
4. Blind proximal esophageal pouch with proximal fistula
Answer: 1 - Blind proximal pouch with a distal TEF
Explanations:
There are five described types of tracheoesophageal fistulae with the most
common being a blind pouch of the upper esophagus with the lower
esophagus connecting with the trachea.
This is seen in 84% of cases. Eight percent of patients have esophageal
atresia without fistula.
Four percent have fistula without esophageal atresia (H-shaped), and 3%
have esophageal atresia with both distal and proximal fistulas.
Only 1% of patients have esophageal atresia with proximal fistula.

Go to the next page if you knew the correct answer, or click the link image(s)
below to further research the concepts in this question (if desired).

Research Concepts:
Fistula, Tracheoesophageal

We update eBooks quarterly and Apps daily based on user feedback.


Please tap flag to report any questions that need improvement.
Question 186: How is Bacillus anthracis contracted?
Choices:
1. Colon
2. Animal skins
3. Brain
4. Blood
Answer: 2 - Animal skins
Explanations:
Bacillus anthracis can be contracted from handling hides.
B. anthracis has an antiphagocytic capsule needed for full virulence. The
organism produces three plasmid-coded exotoxins; edema factor (a
calmodulin-dependent adenylate cyclase, causes elevation of intracellular
cAMP, responsible for the severe edema); lethal toxin (responsible for
tissue necrosis); and protective antigen (mediates cell entry of edema factor
and lethal toxin).
The symptoms of anthrax infection are dependent on the type of infection
and can take anywhere from 1 day to 2 months to appear. All types of
anthrax have the potential to spread throughout the body and cause severe
illness.
Cutaneous anthrax, the most common form, causes a localized,
inflammatory, black, necrotic lesion. Inhalation is a rare but highly fatal
form, is characterized by flu-like symptoms. Gastrointestinal, also rare but
also potentially fatal results from ingestion of spores. Symptoms include
fever and chills, swelling of the neck, painful swallowing, hoarseness,
nausea and vomiting, diarrhea, flushing and red eyes, and swelling of the
abdomen. Injection, is similar to cutaneous anthrax, but spreads throughout
the body faster and can be harder to recognize and treat compared to
cutaneous anthrax.

Go to the next page if you knew the correct answer, or click the link image(s)
below to further research the concepts in this question (if desired).

Research Concepts:
Anthrax

We update eBooks quarterly and Apps daily based on user feedback.


Please tap flag to report any questions that need improvement.
Question 187: To relieve a right tension pneumothorax, where should the
needle be placed?

Choices:
1. Fourth intercostal space, left of the mediastinum
2. Second intercostal space, left of the mediastinum
3. Fourth intercostal space, right of the mediastinum
4. Second intercostal space, right of the mediastinum
Answer: 4 - Second intercostal space, right of the mediastinum
Explanations:
Needle decompression is done only if tube thoracostomy is not available,
such as in the field. In the emergency department, an arrow catheter that is
9 to 12 french can be converted to a tube thoracostomy later. Proper needle
placement to treat tension pneumothorax is in the second intercostal space,
mid-clavicular, over the third rib on the affected side with a 14 gauge, 3.25-
inch needle in adults and 16- to 18-gauge, 2-inch needle in pediatric
patients. Prepare site if possible with an antiseptic solution, then insert the
needle at a 90-degree angle. A pop will be heard when the pleural space has
been entered, and a hiss of air will be heard as the pneumothorax is
decompressed if a tension pneumothorax was present. The catheter is then
advanced into the skin, and the needle is removed. Secure the catheter in
place without covering the end so air can escape.
A chest tube should be placed after needle decompression to the right fourth
or fifth intercostal space, anterior or mid-axillary line.
A tension pneumothorax requires immediate intervention due to mediastinal
shift towards the unaffected side causing compression on the heart, great
vessels, and trachea. Venous return is significantly lessened, and the patient
may die if untreated.
A tension pneumothorax should be suspected if a patient in severe
respiratory distress does not improve with other interventions, if the trachea
deviates, or there is decreased or absent breath sounds on the affected side.
Tachycardia will be present but may be masked in the presence of beta-
blocker use.

Go to the next page if you knew the correct answer, or click the link image(s)
below to further research the concepts in this question (if desired).

Research Concepts:
Pneumothorax, Tension And Traumatic


We update eBooks quarterly and Apps daily based on user feedback.
Please tap flag to report any questions that need improvement.
Question 188: After a serious inhalation burn injury, most patients develop
features of ARDS within how many days?

Choices:
1. One
2. Two
3. Four
4. Six
Answer: 2 - Two
Explanations:
ARDS normally presents by day 2 with pulmonary injury.

Go to the next page if you knew the correct answer, or click the link image(s)
below to further research the concepts in this question (if desired).

Research Concepts:
Inhalation Injury

We update eBooks quarterly and Apps daily based on user feedback.


Please tap flag to report any questions that need improvement.
Question 189: Besides Pneumocystis jiroveci, which other lung pathology
in HIV patients requires a lung biopsy?

Choices:
1. CMV infection
2. Kaposi sarcoma
3. Tuberculosis infection
4. Aspergillosis
Answer: 2 - Kaposi sarcoma
Explanations:
Kaposi sarcoma occurs in 10-30 percent of HIV patients.
Besides the infiltrates, patients may have a pleural effusion. Associated skin
lesions may be present.
Thoracentesis usually does not reveal the diagnosis.
The yield via bronchoscopy is low and a lung biopsy is required.

Go to the next page if you knew the correct answer, or click the link image(s)
below to further research the concepts in this question (if desired).

Research Concepts:
Cancer, Sarcoma, Kaposi

We update eBooks quarterly and Apps daily based on user feedback.


Please tap flag to report any questions that need improvement.
Question 190: Which of the following is a common complication of a
congenital pulmonary airway malformations?

Choices:
1. Cancer
2. Pulmonary hypoplasia
3. Infection
4. Bronchitis
Answer: 3 - Infection
Explanations:
Recurrent infection is the most common presentation of this disorder.
Pulmonary blastomas, rhabdomyosarcoma, adenocarcinoma in situ, and
minute squamous cell carcinoma can all develop.
Congenital cystic adenomatoid malformations are now known as congenital
pulmonary airway malformations (CPAMs).
The type 4 CPAM should be considered a malignant lesion.
Large masses can cause pulmonary hypoplasia and respiratory failure.

Go to the next page if you knew the correct answer, or click the link image(s)
below to further research the concepts in this question (if desired).

Research Concepts:
Congenital Pulmonary Airway Malformation

We update eBooks quarterly and Apps daily based on user feedback.


Please tap flag to report any questions that need improvement.
Question 191: What percentage of sudden infant death syndrome (SIDS)
deaths occurs before 6 months of age?

Choices:
1. 40 percent
2. 60 percent
3. 85 percent
4. 95 percent
Answer: 4 - 95 percent
Explanations:
Ninety-five percent of all sudden infant death syndrome (SIDS) deaths
occur by 6 months of age.
SIDS is the most common cause of unexpected death in infants.
The peak incidence of SIDS is between 2 and 4 months.
SIDS is responsible for 40 to 50 percent of postneonatal infant mortality.

Go to the next page if you knew the correct answer, or click the link image(s)
below to further research the concepts in this question (if desired).

Research Concepts:
Sudden Infant Death Syndrome

We update eBooks quarterly and Apps daily based on user feedback.


Please tap flag to report any questions that need improvement.
Question 192: A patient presents with moderate wheezing, eosinophilia,
and elevated IgE levels but does not respond to traditional antiasthmatic drugs.
What is the most likely diagnosis?

Choices:
1. Histoplasmosis
2. Allergic bronchopulmonary aspergillosis
3. Alpha1-antitrypsin deficiency
4. DiGeorge syndrome
Answer: 2 - Allergic bronchopulmonary aspergillosis
Explanations:
Allergic bronchopulmonary aspergillosis is a hypersensitivity reaction to
the aspergillus antigen.
It usually presents with bronchospasm, wheezing, and bronchiectasis.
Blood work may reveal eosinophilia and elevated IgE.
These people do not respond to conventional antiasthmatic drugs but do
respond to corticosteroids.

Go to the next page if you knew the correct answer, or click the link image(s)
below to further research the concepts in this question (if desired).

Research Concepts:
Aspergillosis

We update eBooks quarterly and Apps daily based on user feedback.


Please tap flag to report any questions that need improvement.
Question 193: When should antibiotics be started in acute chest syndrome?
Choices:
1. If fever is present
2. If white blood count is elevated
3. If there is a productive cough
4. In every case
Answer: 4 - In every case
Explanations:
Diagnostically, pneumonia and acute chest syndrome are very difficult to
differentiate.
Clinicians should have an extremely low threshold to start antibiotics on
suspected acute chest syndrome patients.
Even though a pulmonary embolism is a primary cause in adults, antibiotics
should be started in both adults and children.
Patients should be started on a third generation cephalosporin (cefotaxime
or ceftriaxone) for bacterial coverage and a macrolide (azithromycin or
erythromycin) for atypical coverage. If there is a concern for methicillin-
resistant Staphylococcus aureus, vancomycin should be added. Treatment
should last 7 to 10 days.

Go to the next page if you knew the correct answer, or click the link image(s)
below to further research the concepts in this question (if desired).

Research Concepts:
Chest Syndrome, Acute

We update eBooks quarterly and Apps daily based on user feedback.


Please tap flag to report any questions that need improvement.
Question 194: Which of the following about mucoepidermoid tumors is
correct?

Choices:
1. The majority are located in the periphery
2. They are very responsive to radiation
3. Surgery is always undertaken after chemotherapy
4. The prime factor that determines their behavior is the histology grade
Answer: 4 - The prime factor that determines their behavior is the histology
grade

Explanations:
Mucoepidermoid cancers occur in the fifth or sixth decade with equal
occurrence in males and females.
Mucoepidermoid is a rare tumor that arises in the central tracheo-bronchial
tree. There is a low grade and high grade tumor based on histology.
The low-grade variety may rarely infiltrate local organs but is not
metastatic. The high grade is an aggressive tumor and highly metastatic.
They may present with bronchial irritation, wheezing cough, and
hemoptysis. Biopsy is safe under bronchoscopy.
Surgery is complete resection as these cancers are resistant to both
chemotherapy and radiation.

Go to the next page if you knew the correct answer, or click the link image(s)
below to further research the concepts in this question (if desired).

Research Concepts:
Cancer, Lung, Mucoepidermoid Tumor

We update eBooks quarterly and Apps daily based on user feedback.


Please tap flag to report any questions that need improvement.
Question 195: Which of the following parameters suggests that successful
extubation from mechanical ventilation is unlikely?

Choices:
1. PaO2 of 60 torr on FIO2 < 0.40
2. PEEP < 7.5 cm H2O
3. Respiratory rate < 25
4. Negative inspiratory force of -10 cmH2O
Answer: 4 - Negative inspiratory force of -10 cmH2O
Explanations:
Patients with a negative inspiratory force (NIF) of 15 cm H2O are not ready
for extubation. The NIF has to be around 25 cm H20.
Being mentally alert is a very important criterion for extubation. Most
studies show that the rate of reintubation is highest in patients with a
neurological disease or brain injury.
Before extubation, one must assess the cough strength while the patient is
only on a T-piece. The ability to cough up secretions is vital after
extubation. Otherwise, the airway will be congested.
Many risk factors can predict failed extubation, but ultimately one must
have some clinical judgment. Common risk factors include advanced age,
left ventricular dysfunction, anemia, renal dysfunction, massive blood
transfusion, and altered mentation.

Go to the next page if you knew the correct answer, or click the link image(s)
below to further research the concepts in this question (if desired).

Research Concepts:
Ventilation, Ventilator Management

We update eBooks quarterly and Apps daily based on user feedback.


Please tap flag to report any questions that need improvement.
Section 4

Question 196: Which childhood illness has been identified as a risk factor
for asthma?

Choices:
1. H influenzae
2. Epiglottitis
3. Croup
4. Acute bronchiolitis
Answer: 4 - Acute bronchiolitis
Explanations:
Bronchiolitis has been linked as a risk factor for asthma, but there is no
linear association.
Respiratory syncytial virus (RSV) may predispose children to asthma later
in life by altering a specific subset of T cells.
However, the majority of infants with bronchiolitis do not develop asthma.
Almost 50 percent of asthmatic children had bronchiolitis before 1 year of
age, but virtually 100 percent of children have RSV infection by 2 years of
age.

Go to the next page if you knew the correct answer, or click the link image(s)
below to further research the concepts in this question (if desired).

Research Concepts:
Asthma

We update eBooks quarterly and Apps daily based on user feedback.


Please tap flag to report any questions that need improvement.
Question 197: Which of the following is the optimal order in which
mechanical ventilation parameters should be decreased as a patient's condition
improves?

Choices:
1. FIO2, PEEP, rate, CPAP
2. PEEP, rate, FIO2, CPAP
3. CPAP, PEEP, rate, FIO2
4. FIO2, rate, PEEP, CPAP
Answer: 1 - FIO2, PEEP, rate, CPAP
Explanations:
Weaning requires that the patient is ready, able to cough up secretions,
easily arousable, and no longer on IV sedation.
The arterial blood gas should be within normal limits, the FIO2 should be
no more than .50, and the Pa02/FiO2 should be > 200 mmHg.
The patient should tolerate a trial of spontaneous breathing with normal
vital signs.
Continuous positive airway pressure (CPAP) delivers positive pressure
throughout the respiratory cycle and keeps alveoli open. Patients who can
tolerate CPAP for 30 minutes may be ready for extubation.

Go to the next page if you knew the correct answer, or click the link image(s)
below to further research the concepts in this question (if desired).

Research Concepts:
Ventilation, Ventilator Management

We update eBooks quarterly and Apps daily based on user feedback.


Please tap flag to report any questions that need improvement.
Question 198: Silo-filler disease is caused by inhaling which of the
following substances?

Choices:
1. Nitrogen dioxide
2. Nitrous oxide
3. Nitric acid
4. Nitrous acid
Answer: 1 - Nitrogen dioxide
Explanations:
The nitrogen dioxide (NO2) that causes silo-filler disease is released by
fermentation in the silos.
Pathologically, the lesion varies from pulmonary edema, chemical
bronchopneumonia, severe congestion, and bronchiolitis obliterans.
NO2 is a reddish-brown gas with a harsh smell at high concentrations. It is
colorless and odorless at low concentration. NO2 is an irritant of the
mucous membranes that causes pulmonary diseases such as occupational
lung disease, asthma, and chronic obstructive pulmonary disease.
Occupational exposures are the highest risk of toxicity. It is one of the
major air pollutants capable of causing severe health hazards such as
coronary artery disease and stroke. Known sources of nitrogen gas
poisoning include automobile exhaust, power stations, and noncombustible
sources such as anaerobic fermentation of food grains and anaerobic
digestion of biodegradable waste.

Go to the next page if you knew the correct answer, or click the link image(s)
below to further research the concepts in this question (if desired).

Research Concepts:
Silo Filler's Disease

We update eBooks quarterly and Apps daily based on user feedback.


Please tap flag to report any questions that need improvement.
Question 199: The most frequent imaging finding in patients with cystic
fibrosis is which of the following?

Choices:
1. Lower lung predominant bronchiectases
2. Upper lung predominant bronchiectases
3. Bronchiectases with tracheal stenosis
4. Pneumonia
Answer: 2 - Upper lung predominant bronchiectases
Explanations:
Upper lung predominant bronchiectases are the most common imaging
finding in patients with cystic fibrosis (CF).
In chronic cases, the hila are displaced cephalad due to recurrent bouts of
infection and inflammation resulting in post-inflammatory volume loss and
fibrosis.
Lower lung predominant bronchiectases are seen in primary ciliary
dyskinesia, hypoglobulinemia, post-aspiration and post lung transplantation
Tracheal stenosis can occur in patients with CF as a result of combined
chronic airway infection and respiratory failure requiring prolonged
intubation. However, it is less frequent than an increased diameter of the
trachea due to fixed air-trapping.

Go to the next page if you knew the correct answer, or click the link image(s)
below to further research the concepts in this question (if desired).

Research Concepts:
Cystic Fibrosis

We update eBooks quarterly and Apps daily based on user feedback.


Please tap flag to report any questions that need improvement.
Question 200: What is a common EKG finding in patients with a
pulmonary embolism?

Choices:
1. Left bundle branch block
2. Supraventricular tachycardia
3. Sinus tachycardia
4. Accelerated junctional rhythm
Answer: 3 - Sinus tachycardia
Explanations:
A common EKG finding in patients diagnosed with pulmonary embolism
(PE) is sinus tachycardia.
The classic EKG findings are S wave in Lead I, Q wave in Lead III, and T
wave Inversion in Lead III. S1, Q3, T3 is seen in under 20% of PE cases.
CT pulmonary angiography is the first-line imaging study used for
diagnosis.
The D-dimer blood test is highly sensitive but not specific. It can be used to
rule out pulmonary embolism when clinical suspicion is low to moderate.

Go to the next page if you knew the correct answer, or click the link image(s)
below to further research the concepts in this question (if desired).

Research Concepts:
Embolism, Pulmonary

We update eBooks quarterly and Apps daily based on user feedback.


Please tap flag to report any questions that need improvement.
Question 201: Which of the following statements about chronic silicosis is
true?

Choices:
1. Symptoms usually begin 8 to 12 weeks after exposure
2. Lifelong treatment with systemic glucocorticoids reduces mortality
3. The risk for hematologic malignancies such as lymphoma is increased
4. Fibrosis tends to preferentially affect the upper lobes
Answer: 4 - Fibrosis tends to preferentially affect the upper lobes
Explanations:
Silicosis is an occupational lung disease caused by inhalation of silicon
dioxide particles.
Diagnosis can usually be made based on history and imaging alone and
lung biopsy is only necessary in atypical cases. Imaging is characterized by
upper lobe predominant nodules and fibrosis.
Acute and chronic forms of silicosis are distinct in clinical presentation and
histopathology.
No specific treatment exists aside from supportive care. Glucocorticoids
may be effective during periods of acute decompensation, but do not have a
role in long-term management.

Go to the next page if you knew the correct answer, or click the link image(s)
below to further research the concepts in this question (if desired).

Research Concepts:
Silicosis (Coal Worker Pneumoconiosis)

We update eBooks quarterly and Apps daily based on user feedback.


Please tap flag to report any questions that need improvement.
Question 202: Which of the following is the best test to diagnose
sequestration in an infant?

Choices:
1. MRI
2. CT scan
3. Fluoroscopy
4. Doppler ultrasound
Answer: 4 - Doppler ultrasound
Explanations:
Real-time ultrasonography and Doppler imaging are reliable methods of
determining the systemic origin of blood supply.
Ultrasonography will show a sequestration usually as homogeneous, well-
defined echo dense mass.
Color flow using Doppler will show the anomalous vessel arising from the
aorta.
CT scan or MRA can be used but this requires having to sedate the infant.

Go to the next page if you knew the correct answer, or click the link image(s)
below to further research the concepts in this question (if desired).

Research Concepts:
Pulmonary Sequestration

We update eBooks quarterly and Apps daily based on user feedback.


Please tap flag to report any questions that need improvement.
Question 203: What is the best method to evacuate a 4-week old traumatic
hemothorax?

Choices:
1. Chest tube
2. Lytic therapy
3. Video-assisted thoracic surgery
4. Thoracotomy
Answer: 4 - Thoracotomy
Explanations:
An old hemothorax will often become solidified and need open
thoracotomy to remove the peel and debris.

Go to the next page if you knew the correct answer, or click the link image(s)
below to further research the concepts in this question (if desired).

Research Concepts:
Hemothorax

We update eBooks quarterly and Apps daily based on user feedback.


Please tap flag to report any questions that need improvement.
Question 204: Which of the following claims about bronchiectasis is
FALSE?

Choices:
1. The disease is more pronounced in the lower lobes
2. Massive hemoptysis is due to erosion of branches of the bronchial artery
3. Contrast bronchography is done prior to surgery
4. Most cases are acquired
Answer: 3 - Contrast bronchography is done prior to surgery
Explanations:
Hemoptysis in bronchiectasis is from enlarged bronchial arteries. This can
be treated percutaneously with catheter embolization.
Most cases of bronchiectasis are acquired.
Bronchograms are of only historic interest, the bronchial tree is now studied
with high resolution chest CT.
Acquired etiologies of bronchiectasis include infectious etiologies (both
bacterial and TB), aspiration, foreign bodies, alcoholism, and allergies. The
most common non-acquired cause is cystic fibrosis.

Go to the next page if you knew the correct answer, or click the link image(s)
below to further research the concepts in this question (if desired).

Research Concepts:
Bronchiectasis

We update eBooks quarterly and Apps daily based on user feedback.


Please tap flag to report any questions that need improvement.
Question 205: Which one of the following treatment options is
recommended for respiratory syncytial virus infection?

Choices:
1. Supportive care
2. Corticosteroids
3. Albuterol
4. Chest physical therapy
Answer: 1 - Supportive care
Explanations:
Supportive care is the mainstay of treatment for bronchiolitis caused by
respiratory syncytial virus (RSV). Supportive care may consist of nasal
suction and lubrication, antipyretics for fever, and oxygen or ventilatory
support in the setting of hypoxia or acute respiratory failure, and assurance
of adequate hydration.
Many therapeutic options have been tried historically to treat RSV and
bronchiolitis, but have proven in trials to be of no benefit. These include
beta-agonists such as albuterol, corticosteroids, antibiotics, chest physical
therapy, and inhaled racemic epinephrine.
Ribavirin is approved in the United States for the treatment of RSV
infection, but is expensive, has safety concerns including occupational
administration hazards, and has not been shown to decrease hospitalization
or mortality.
Palivizumab is a form of passive immunization available for prophylaxis of
RSV but is not effective in the treatment of RSV.

Go to the next page if you knew the correct answer, or click the link image(s)
below to further research the concepts in this question (if desired).

Research Concepts:
Respiratory Syncytial Virus Infection (RSV)

We update eBooks quarterly and Apps daily based on user feedback.


Please tap flag to report any questions that need improvement.
Question 206: Of the following types of infants, whom does Wilson-Mikity
syndrome commonly affect?

Choices:
1. Low birth weight infants
2. Infants of diabetic mothers
3. Large for gestational age infants
4. Only female infants
Answer: 1 - Low birth weight infants
Explanations:
Wilson-Mikity syndrome is a rare lung disorder that affects low birth
weight infants.
The condition is similar to bronchopulmonary dysplasia.
It does present with severe respiratory distress.

Go to the next page if you knew the correct answer, or click the link image(s)
below to further research the concepts in this question (if desired).

Research Concepts:
Wilson-Mikity Syndrome

We update eBooks quarterly and Apps daily based on user feedback.


Please tap flag to report any questions that need improvement.
Question 207: Which of the following is not true about an atraumatic
pneumothorax in a tall, thin male who smokes heavily?

Choices:
1. It occurs commonly in smokers
2. It is common in males who are tall and thin
3. It may be associated with chylothorax
4. Pilots are best treated with surgery after the first episode
Answer: 3 - It may be associated with chylothorax
Explanations:
Spontaneous pneumothorax occurs due to rupture of apical blebs or bulla.
These individuals typically have no underlying lung disease.
The main physiological consequence of pneumothorax in young people is
loss of vital capacity and a decrease in the partial pressure of oxygen.
A pneumomediastinum may occur from excess intra-alveolar pressures. Air
escapes and will dissect into the mediastinum.
Less than 5% will have blood in the chest from breakdown of adhesions.
The bleeding is usually greater and stops spontaneously.

Go to the next page if you knew the correct answer, or click the link image(s)
below to further research the concepts in this question (if desired).

Research Concepts:
Pneumothorax, Spontaneous

We update eBooks quarterly and Apps daily based on user feedback.


Please tap flag to report any questions that need improvement.
Question 208: Which of the following complications is specifically
associated with insertion of a central venous femoral catheter?

Choices:
1. Central line-associated bloodstream infection
2. Bladder puncture
3. Guidewire embolism
4. Cardiac dysrhythmia
Answer: 2 - Bladder puncture
Explanations:
In general, complications from central venous access can be classified into
early and late complications.
A complication specific to the femoral site is bladder puncture.
Some complications that can occur with insertion at all central venous
access sites are central line-associated bloodstream infections (CLABSIs),
catheter fragmentation resulting in a guidewire embolism, and cardiac
dysrhythmias.
CLABSIs are the most common complication of central venous catheter
placement, resulting in significant morbidity and mortality, as well as,
increased healthcare costs.

Go to the next page if you knew the correct answer, or click the link image(s)
below to further research the concepts in this question (if desired).

Research Concepts:
Central Venous Access, Femoral Vein

We update eBooks quarterly and Apps daily based on user feedback.


Please tap flag to report any questions that need improvement.
Question 209: What is the most likely diagnosis in an intubated patient
with no left chest wall rise?

Choices:
1. Left mainstem intubation
2. Pneumothorax
3. Right mainstem intubation
4. Lobar pneumonia

Photo:Contributed by Scott Dulebohn, MD


Answer: 3 - Right mainstem intubation
Explanations:
Right mainstem intubation leads to right lung inflation only.
Thus, it is very important to listen for breath sounds bilaterally.
In addition, one should always obtain a chest x-ray after an intubation.
The endotracheal tube should be about 2 to 2.5 centimeters above the
carina.

Go to the next page if you knew the correct answer, or click the link image(s)
below to further research the concepts in this question (if desired).

Research Concepts:
Intubation, Endotracheal Tube, Medications

We update eBooks quarterly and Apps daily based on user feedback.


Please tap flag to report any questions that need improvement.
Question 210: Which vasculitic syndrome has been associated with
zafirlukast?

Choices:
1. Kawasaki syndrome
2. Granulomatosis with polyangiitis
3. Eosinophilic granulomatosis with polyangiitis
4. Goodpasture syndrome
Answer: 3 - Eosinophilic granulomatosis with polyangiitis
Explanations:
Zafirlukast can cause elevations in liver enzymes. It is contraindicated in
patients with hepatic impairment, including cirrhosis.
Rarely, EGPA (Eosinophilic granulomatosis with polyangiitis) has been
reported in patients who take zafirlukast. EGPA is also known as Churg
Strauss syndrome.
Common side effects of zafirlukast include stomach upset and headache.
Zafirlukast is not indicated for the treatment of acute asthma attacks,
including status asthmaticus.

Go to the next page if you knew the correct answer, or click the link image(s)
below to further research the concepts in this question (if desired).

Research Concepts:
Churg Strauss Syndrome (Allergic Granulomatosis)

We update eBooks quarterly and Apps daily based on user feedback.


Please tap flag to report any questions that need improvement.
Question 211: Which of the following factors is not used for croup severity
scoring?

Choices:
1. Level of consciousness
2. Cyanosis
3. Stridor
4. Oxygen saturation
Answer: 4 - Oxygen saturation
Explanations:
The Westley croup score outlines elements of the physical examination in
croup. The level of consciousness is given a score of 0 to 5.
Cyanosis is a worrisome finding often associated with impending airway
failure.
Stridor is common and is associated with moderate or more severe croup.
Oxygen saturation is not used to assess croup scoring.

Go to the next page if you knew the correct answer, or click the link image(s)
below to further research the concepts in this question (if desired).

Research Concepts:
Croup

We update eBooks quarterly and Apps daily based on user feedback.


Please tap flag to report any questions that need improvement.
Question 212: A 16-yr-old girl with pectus excavatum is brought for
evaluation. She has a history of exercise-induced breathlessness. Which of the
following investigation will help decide if she needs surgical repair?

Choices:
1. Chest radiograph
2. Echocardiogram
3. CT of the chest
4. Pulmonary function testing

Photo:Contributed Girish Sharma, MD


Answer: 3 - CT of the chest
Explanations:
CT of the chest is used to calculate Haller index. This is used to decide
surgical repair of pectus excavatum.
Haller index is the ratio of lateral chest diameter to the sternum-spine
distance on CT chest. A value of Haller index up to 2.5 is normal, and a
value of greater than 3.5 is an indication for surgical correction.
Patients with a mild to moderate deformity are usually asymptomatic;
however, patients with marked pectus excavatum deformity may present
with exercise intolerance.
Patients with severe distortion of the rib cage (figure 1) may have a
mechanical disadvantage in term of lung compression, cardiac
displacement, and atrial compression demonstrated by echocardiogram.

Go to the next page if you knew the correct answer, or click the link image(s)
below to further research the concepts in this question (if desired).

Research Concepts:
Pectus Excavatum

We update eBooks quarterly and Apps daily based on user feedback.


Please tap flag to report any questions that need improvement.
Question 213: Which of the following can help prevent atelectasis after
transsphenoidal hypophysectomy?

Choices:
1. Cough
2. Deep breathing
3. Sneezing
4. Oxygen
Answer: 2 - Deep breathing
Explanations:
Deep breathing is the best method to prevent atelectasis formation.
Coughing and sneezing are contraindicated after transsphenoidal
hypophysectomy.
Atelectasis may be avoided by stopping smoking at least 6 to 8 weeks
before a procedure.
Taking adequate precautions to prevent atelectasis is important before and
after any surgical procedure.

Go to the next page if you knew the correct answer, or click the link image(s)
below to further research the concepts in this question (if desired).

Research Concepts:
Transsphenoidal Hypophysectomy

We update eBooks quarterly and Apps daily based on user feedback.


Please tap flag to report any questions that need improvement.
Question 214: Which of the following is the correct depiction of Enghoff's
equation?

Choices:
1. VT/VDphys = (PaCO2 - PECO2)/PaCO2
2. VDphys/VT = (PaO2 - PEO2)/PaO2
3. VDphys/VT = (PaCO2 - PECO2)/PaCO2
4. VDphys/VT = (PaCO2 - PECO2)/PaO2
Answer: 3 - VDphys/VT = (PaCO2 - PECO2)/PaCO2
Explanations:
Enghoff's equation which can be used to determine dead space in
capnography.
V/Q changes, not necessarily that of increased V/Q (i.e., dead space), but
decreased V/Q (i.e., pulmonary shunting) can change VDphys/VT
measurements and make it less reliable. However, it still is useful for acute
respiratory distress mortality prediction.
A normal physiological dead space should be one-third of a healthy
patient's tidal volume and is estimated around 2 mL/kg in a healthy adult.
Dead space ventilation can be derived from this equation, as VDphys x
Respiratory Rate (RR) = VD in L/min. This would represent the amount of
air moved through the lungs per minute that do not contribute to gas
exchange.

Go to the next page if you knew the correct answer, or click the link image(s)
below to further research the concepts in this question (if desired).

Research Concepts:
Anatomy, Airway, Anatomic Dead Space

We update eBooks quarterly and Apps daily based on user feedback.


Please tap flag to report any questions that need improvement.
Question 215: Which of the following causes bronchodilatation?
Choices:
1. Beta 1 agonists
2. Beta 2 agonists
3. Alpha 1 agonists
4. Alpha 2 agonists
Answer: 2 - Beta 2 agonists
Explanations:
Beta 2 agonists induce bronchodilatation.

Go to the next page if you knew the correct answer, or click the link image(s)
below to further research the concepts in this question (if desired).

Research Concepts:
Bronchodilators

We update eBooks quarterly and Apps daily based on user feedback.


Please tap flag to report any questions that need improvement.
Question 216: What is the gold standard to diagnose obstructive sleep
apnea in children?

Choices:
1. Polysomnography
2. Nocturnal pulse oximetry
3. Sleep questionnaire
4. Imaging of the upper airway
Answer: 1 - Polysomnography
Explanations:
The gold standard to diagnose obstructive sleep apnea (OSA) is
polysomnography performed overnight in a sleep lab. Polysomnography
can differentiate OSA from snoring and also assess the severity.
When the performance of polysomnography is impractical or unavailable,
sleep questionnaires and for hospitalized patients, nocturnal pulse oximetry
can be used to diagnose OSA.
Imaging of the upper airway may identify upper airway abnormalities but
does not correlate well with OSA.
Sleep questionnaires and in home sleep pulse oximetry have shown good
positive predictive value to diagnose OSA, but are not the gold standard.

Go to the next page if you knew the correct answer, or click the link image(s)
below to further research the concepts in this question (if desired).

Research Concepts:
Apnea, Children

We update eBooks quarterly and Apps daily based on user feedback.


Please tap flag to report any questions that need improvement.
Question 217: When an infant is born with dextrocardia, what thoracic
condition should be ruled out?

Choices:
1. Sequestration
2. Congenital lobar emphysema
3. Bronchiectasis
4. Congenital diaphragmatic hernia
Answer: 3 - Bronchiectasis
Explanations:
Once dextrocardia is diagnosed, one should always think about Kartagener
syndrome and rule out bronchiectasis.
Infants with dextrocardia are prone to several esophageal, bowel, and
cardiac problems, which if left uncorrected can prove to be fatal.
Dextrocardia may be due to embryonic arrest where the heart is simply
farther right in the chest than normal.
In dextrocardia with situs inversus, the heart is a mirror image situated in
the right chest cavity.

Go to the next page if you knew the correct answer, or click the link image(s)
below to further research the concepts in this question (if desired).

Research Concepts:
Dextrocardia

We update eBooks quarterly and Apps daily based on user feedback.


Please tap flag to report any questions that need improvement.
Question 218: What percentage of bacterial pneumonias have
parapneumonic effusions?

Choices:
1. 5 to 20 percent
2. 25 to 30 percent
3. 35 to 45 percent
4. >45 percent
Answer: 3 - 35 to 45 percent
Explanations:
More than 40 percent of patients with bacterial pneumonia develop
parapneumonic effusions.
This number rises to 60 percent in patients with pneumococcal infections.
Pleural effusions thicker than 1 cm should be aspirated.
All empyemas should be drained.

Go to the next page if you knew the correct answer, or click the link image(s)
below to further research the concepts in this question (if desired).

Research Concepts:
Pneumonia

We update eBooks quarterly and Apps daily based on user feedback.


Please tap flag to report any questions that need improvement.
Question 219: What is the single best predictor of lung function in healthy
children?

Choices:
1. Weight
2. Chest circumference
3. Height
4. Abdominal girth
Answer: 3 - Height
Explanations:
The single best predictor of lung function in children is height.
Reference (predicted) values are based on healthy subjects with the same
anthropometric (sex, age and height), and when relevant, ethnic
characteristics of the test subject.
Height should be measured with a stadiometer with shoes off.
When height cannot be measured arm span can be used as an estimate.

Go to the next page if you knew the correct answer, or click the link image(s)
below to further research the concepts in this question (if desired).

Research Concepts:
Pulmonary Function Tests

We update eBooks quarterly and Apps daily based on user feedback.


Please tap flag to report any questions that need improvement.
Question 220: Effective treatment of a stricture in the main bronchus after
a lung transplant includes which of the following?

Choices:
1. Pneumonectomy
2. Laser
3. Dilatation
4. Stenting
Answer: 4 - Stenting
Explanations:
Stenting is presently the best treatment for strictures after lung transplant.
Flexible bronchoscopy can help when stenting airway strictures.
Sometimes bronchial strictures go undiagnosed for many years before
symptoms develop.
Stenting also can help with proximal lesions in the airways.

Go to the next page if you knew the correct answer, or click the link image(s)
below to further research the concepts in this question (if desired).

Research Concepts:
Bronchial Stricture

We update eBooks quarterly and Apps daily based on user feedback.


Please tap flag to report any questions that need improvement.
Question 221: A male presents with fever, headache, and myalgias. Exam
and x-rays confirm pneumonia and hepatitis. Which of the following is the most
likely cause?

Choices:
1. Borrelia burgdorferi
2. Rickettsia quintana
3. Rickettsia rickettsii
4. Coxiella burnetii
Answer: 4 - Coxiella burnetii
Explanations:
Coxiella burnetii causes Q fever.
Q fever usually starts with flu-like symptoms, and in 1/2 of patients,
pneumonia develops.
In many cases, hepatic is also associated.
Borrelia burgdorferi causes Lyme disease, Rickettsia rickettsii causes
Rocky Mountain spotted fever, and Rickettsia quintana causes trench fever.

Go to the next page if you knew the correct answer, or click the link image(s)
below to further research the concepts in this question (if desired).

Research Concepts:
Q Fever

We update eBooks quarterly and Apps daily based on user feedback.


Please tap flag to report any questions that need improvement.
Question 222: A male working as a glass blower presents with an
asymptomatic bulging mass on the upper chest. It becomes more obvious with
cough. What is the best radiographic technique for evaluation?

Choices:
1. Posteroanterior chest radiograph
2. Lateral chest radiograph
3. Oblique radiograph tangential to the defect
4. Rib views
Answer: 3 - Oblique radiograph tangential to the defect
Explanations:
The history of glassblowing suggests frequently increased intrathoracic
pressure. This, in addition to the physical exam findings, suggests a lung
hernia.
The majority of lung hernias are acquired, most frequently seen in chronic
obstructive pulmonary disease patients or those with prior thoracotomy, but
also can be an occupational hazard as in this case. The hernia may present
as a non-tender crepitant swelling which enlarges on forced expiration.
Lung hernias could be cervical, intercostal, or diaphragmatic. The
intercostal variety is the most common at 60 to 80 percent.
Pneumonocele, or hernia of the lung, is best diagnosed on an oblique
radiograph tangential to the defect, especially during a Valsalva maneuver.

Go to the next page if you knew the correct answer, or click the link image(s)
below to further research the concepts in this question (if desired).

Research Concepts:
Hernia, Lung

We update eBooks quarterly and Apps daily based on user feedback.


Please tap flag to report any questions that need improvement.
Question 223: The prolonged partial pressure of oxygen exceeding what
concentration can cause central nervous system oxygen toxicity and seizures?

Choices:
1. 1 ATA (14.7psi, 101 kPa, 1.01 bar)
2. 1.6 ATA (23.5psi, 162 kPa, 1.62 bar)
3. 2 ATA (29.4 psi, 202 kPa, 2.02 bar)
4. 2.4 ATA (35.3 psi, 243 kPa, 2.43 bar)
Answer: 2 - 1.6 ATA (23.5psi, 162 kPa, 1.62 bar)
Explanations:
Prolonged breathing of a gas with a FiO2 greater than 60 kPa can lead to
pulmonary toxicity and eventually irreversible pulmonary fibrosis, but this
takes many hours or days.
At a FiO2 greater than 160 kPa, acute oxygen toxicity can occur causing
convulsions with little or no warning.
Breathing air containing 21% oxygen risks acute oxygen toxicity at depths
greater than 66 meters of seawater.
When breathing 100% oxygen there is a risk of convulsion at only 6 meters
of seawater.

Go to the next page if you knew the correct answer, or click the link image(s)
below to further research the concepts in this question (if desired).

Research Concepts:
Toxicity, Oxygen

We update eBooks quarterly and Apps daily based on user feedback.


Please tap flag to report any questions that need improvement.
Question 224: A 1-month old infant undergoes a chest x-ray during the
workup of an illness. The radiologist states that the infant may have a
bronchogenic cyst. In general, which symptom is a bronchogenic cyst of the
mediastinum in an infant most likely to produce?

Choices:
1. Cyanosis
2. Dysphagia
3. Cough
4. Facial swelling
Answer: 3 - Cough
Explanations:
In general, it is the location of the cyst and size that determines the
symptoms.
In most cases, the cyst compresses adjacent structures. In small children,
cough is a very common symptom.
Other symptoms may include wheezing and respiratory distress.
The cyst may also get infected and present with fever and pneumonia-like
symptoms.

Go to the next page if you knew the correct answer, or click the link image(s)
below to further research the concepts in this question (if desired).

Research Concepts:
Cyst, Bronchogenic

We update eBooks quarterly and Apps daily based on user feedback.


Please tap flag to report any questions that need improvement.
Question 225: What is the sensitivity of a spiral CT in the diagnosis of
pulmonary embolism?

Choices:
1. 50 percent
2. 70 percent
3. 80 percent
4. >95 percent
Answer: 4 - >95 percent
Explanations:
Spiral CT offers a fast diagnosis for PE.
Spiral CT scan has sensitivity of >95 percent for PE.
It has a negative predictive value of 99%.

Go to the next page if you knew the correct answer, or click the link image(s)
below to further research the concepts in this question (if desired).

Research Concepts:
Embolism, Pulmonary

We update eBooks quarterly and Apps daily based on user feedback.


Please tap flag to report any questions that need improvement.
Question 226: Which of the following is not correct about the repair of
tracheoesophageal fistulas?

Choices:
1. The procedure is best performed after weaning from ventilation
2. A gastrostomy and a feeding tube may be necessary
3. Stenting of the trachea and primary repair of the esophagus provides optimum
results
4. Esophageal diversion is rarely required
Answer: 3 - Stenting of the trachea and primary repair of the esophagus
provides optimum results

Explanations:
A tracheoesophageal fistula in a patient on mechanical ventilation is best
repaired after weaning.
If the patient cannot be weaned off ventilation, the placement of the
endotracheal tube below the fistula will prevent more soilage. In addition a
gastrostomy and a feeding jejunostomy may be helpful.
Diversion of the esophagus is not done.
Stenting of the trachea is not helpful but with the availability of new wire
mesh stents, an esophageal stent may be helpful in some critically ill
patients.

Go to the next page if you knew the correct answer, or click the link image(s)
below to further research the concepts in this question (if desired).

Research Concepts:
Fistula, Tracheoesophageal

We update eBooks quarterly and Apps daily based on user feedback.


Please tap flag to report any questions that need improvement.
Question 227: Which of the following is not an indication for surgery in
tuberculosis?

Choices:
1. Bronchopleural fistula
2. Destroyed lung
3. Suspected carcinoma
4. A large pleural effusion
Answer: 4 - A large pleural effusion
Explanations:
Pleural fluid in tuberculosis is an exudate with a pH < 7.3, decreased
glucose, and elevated lymphocytes (possibly lymphoma). Acid-fast bacilli
may be present in the pleural fluid. Pleural biopsies can be performed.
Chest tubes are not recommended as they may promote an empyema.
Tuberculosis pleural effusions are best treated with chemotherapy and
thoracentesis. However, mixed empyemas do occur and are difficult to
treat.
Despite chest tube drainage, the lung usually fails to expand. Decortication
will not help re-expand the destroyed lung. Before undertaking
thoracotomy, lung function should be determined.
An extrapleural pneumonectomy is usually done. If decortication is done in
tuberculosis, both the parietal and visceral pleura must be decorticated.
Open thoracotomy is indicated when the empyema cannot be managed by a
closed procedure, and the patient is a candidate for surgery.

Go to the next page if you knew the correct answer, or click the link image(s)
below to further research the concepts in this question (if desired).

Research Concepts:
Tuberculosis

We update eBooks quarterly and Apps daily based on user feedback.


Please tap flag to report any questions that need improvement.
Question 228: Which of the following statements about ribavirin therapy
for respiratory syncytial virus (RSV) is correct?

Choices:
1. It is not safe to use by nebulizer around pregnant women
2. It has been shown reproducibly to decrease the length of ventilation and the
length of hospital stay
3. It can be given orally to treat RSV
4. It is an antiviral medication that functions by inhibiting the F glycoprotein
associated with membrane fusion of RSV to host cells
Answer: 1 - It is not safe to use by nebulizer around pregnant women
Explanations:
Ribavirin for respiratory syncytial virus (RSV) must be given by nebulizer.
It can be used for the most severely affected individuals but is of marginal
benefit.
It can cause fetal malformations.
Current evidenced-based practice does not support the use of ribavirin.
Ribavirin is an antiviral nucleoside analog that has been linked to adverse
effects for patients with limited positive effects. The risk of adverse effects
to the patient and provider likely outweigh benefits of the drug; however, in
the case of immunocompromised patients with severe RSV, it may be
considered. Palivizumab is an immunization recommended for RSV in
high-risk infants and children no older than 24 months. The efficacy of
palivizumab has been established, and it functions by binding the F
glycoprotein used by RSV for fusion with host cells.

Go to the next page if you knew the correct answer, or click the link image(s)
below to further research the concepts in this question (if desired).

Research Concepts:
Respiratory Syncytial Virus Infection (RSV)

We update eBooks quarterly and Apps daily based on user feedback.


Please tap flag to report any questions that need improvement.
Question 229: An 8-week-old child is brought to the emergency
department with a cough and rapid respirations. The infant is afebrile but has a
respiratory rate of 60 breaths per minute and oxygen saturation of 93 percent.
There are bilateral rhonchi and rales. The child was treated at two weeks of age
for conjunctivitis with a topical antibiotic. Select the most likely infectious
organism.

Choices:
1. Group B Streptococcus
2. Gram-negative rods
3. Chlamydia trachomatis
4. Streptococcus pneumoniae
Answer: 3 - Chlamydia trachomatis
Explanations:
Chlamydia trachomatis is acquired during birth from the mother.
Conjunctivitis and pneumonia can occur from two to twelve weeks of age.
Culture of secretions and antibody titers will confirm the diagnosis.
Macrolide antibiotics are used for treatment but increase the risk of pyloric
stenosis.

Go to the next page if you knew the correct answer, or click the link image(s)
below to further research the concepts in this question (if desired).

Research Concepts:
Pneumonia, Chlamydia

We update eBooks quarterly and Apps daily based on user feedback.


Please tap flag to report any questions that need improvement.
Question 230: A 2-year-old child is suspected of having aspirated a
radiolucent foreign body but cannot cooperate with inspiratory and expiratory
chest radiographs. What is the next option?

Choices:
1. Ventilation/perfusion scan
2. Airway fluoroscopy
3. CT of the chest with contrast
4. CT of the chest without contrast
Answer: 2 - Airway fluoroscopy
Explanations:
Airway fluoroscopy would identify air trapping in many children with
foreign body aspiration.

Go to the next page if you knew the correct answer, or click the link image(s)
below to further research the concepts in this question (if desired).

Research Concepts:
Foreign Body Aspiration

We update eBooks quarterly and Apps daily based on user feedback.


Please tap flag to report any questions that need improvement.
Question 231: A 4-week old male child is brought in for stridor that the
mother has noticed since birth. She notes that the stridor is most marked when
the child is supine and disappears completely when the baby is placed prone.
The diagnosis is consistent with which of the following conditions?

Choices:
1. Laryngomalacia
2. Gastroesophageal reflux disease
3. Croup
4. Foreign body
Answer: 1 - Laryngomalacia
Explanations:
Laryngomalacia is a common cause of stridor in babies and neonates.
Wheezing, respiratory distress, or cyanosis may also be seen.
Stridor occurs most commonly when the baby is placed supine and
decreases/ or disappears on prone position. This is due to increased
floppiness and worse obstruction in the supine position.
It results from congenital softening of the laryngeal tissues that cause it to
be soft and malformed. This results in flopping of the tissues over the
airways in supine position creating obstruction.
Most cases of laryngomalacia resolve spontaneously by about 2 years of
age. Severe obstruction, distress, or failure to improve may require surgical
correction. Close follow up is required.

Go to the next page if you knew the correct answer, or click the link image(s)
below to further research the concepts in this question (if desired).

Research Concepts:
Laryngomalacia

We update eBooks quarterly and Apps daily based on user feedback.


Please tap flag to report any questions that need improvement.
Question 232: Which of the following are sequelae associated with
inhalational exposure to fluoropolymers?

Choices:
1. Respiratory failure
2. Pulmonary fibrosis
3. Death
4. All of the above have been associated with fluorpolymer exposure
Answer: 4 - All of the above have been associated with fluorpolymer
exposure

Explanations:
Most patients recover within 24 hours, although recovery has taken days to
weeks.
Pulmonary symptoms may last longer than other symptoms, such as nausea
and fever.
The illness develops within minutes to hours of exposure. Although those
most affected are those using the products, symptoms have been seen
following second-hand exposure.
While most patients recover fully, persistent symptoms are reported,
including lung function abnormalities and pulmonary fibrosis.

Go to the next page if you knew the correct answer, or click the link image(s)
below to further research the concepts in this question (if desired).

Research Concepts:
Toxicity, Fluorides and Fluorocarbons

We update eBooks quarterly and Apps daily based on user feedback.


Please tap flag to report any questions that need improvement.
Question 233: What anthropomorphic measurement is the most important
in predicting the presence of obstructive sleep apnea syndrome?

Choices:
1. Tongue bulk
2. Neck circumference
3. Body mass index
4. Chest circumference
Answer: 2 - Neck circumference
Explanations:
Neck circumference is the most important correlate with obstructive sleep
apnea (OSA).
However, a 1:1 correlation does not exist, and the only true diagnostic
maneuver that confirms OSA is polysomnography.
In the United States, approximately 4% of men and 2% of women meet
criteria for OSA. Prevalence is higher in Hispanic, African American, and
Asian populations.
In children, the most common cause of OSA is enlarged tonsils and
adenoids. In adults, it is most commonly associated with obesity, male
gender, and advancing age.

Go to the next page if you knew the correct answer, or click the link image(s)
below to further research the concepts in this question (if desired).

Research Concepts:
Apnea, Obstructive Sleep Apnea

We update eBooks quarterly and Apps daily based on user feedback.


Please tap flag to report any questions that need improvement.
Question 234: The effects of succinylcholine are prolonged when there is
an absence of which enzyme?

Choices:
1. Glucose-6-phosphate dehydrogenase
2. Plasma cholinesterase
3. Dopamine b hydroxylase
4. Cytochrome P450 oxidase
Answer: 2 - Plasma cholinesterase
Explanations:
Decreased levels of plasma cholinesterase are seen in patients with genetic
defects, pregnancy, malignancy, severe liver disease, chronic renal failure,
burns, infections, anemia, myxedema, and peptic ulcer disease.
This can lead to prolonged effect of succinylcholine.
The activity of plasma cholinesterase can be decreased by use of oral
contraceptives, corticosteroids, MAO inhibitors, or organophosphate
insecticides.

Go to the next page if you knew the correct answer, or click the link image(s)
below to further research the concepts in this question (if desired).

Research Concepts:
Succinylcholine

We update eBooks quarterly and Apps daily based on user feedback.


Please tap flag to report any questions that need improvement.
Question 235: Which of the following is the most likely mechanism of
massive hemoptysis?

Choices:
1. Bleeding from a pulmonary artery
2. Bleeding from a bronchial artery
3. Bleeding from lung parenchymal capillaries
4. Tear of a major bronchus.
Answer: 2 - Bleeding from a bronchial artery
Explanations:
Massive hemoptysis most likely originating from a bronchial artery.
Pulmonary artery bleeding is rare as these vessels are under low pressure.
Tuberculosis (TB) is a major cause of massive hemoptysis world wide. In
developed countries TB is uncommon and malignancy is the major cause of
massive hemoptysis in these countries.
Massive hemoptysis can be thought of as enough blood to either cause
airway obstruction or to cause hemodynamic instability. The definition in
the literature based on amount varies.

Go to the next page if you knew the correct answer, or click the link image(s)
below to further research the concepts in this question (if desired).

Research Concepts:
Hemoptysis

We update eBooks quarterly and Apps daily based on user feedback.


Please tap flag to report any questions that need improvement.
Question 236: A 14-year old boy has just been diagnosed with asthma. The
mother wants to know if there is a time period during the day when the asthmatic
crises are serious. Based on current evidence, you tell her that the highest
morbidity and mortality from asthma often occurs during which of the following
times?

Choices:
1. After breakfast
2. Between 2:00 pm and 4:00 pm
3. After dusk
4. Between 4:00 am and 6:00 am
Answer: 4 - Between 4:00 am and 6:00 am
Explanations:
Bronchoconstriction is highest between the hours of 4:00 am and 6:00 am.
The highest morbidity and mortality from asthma is observed during this
time.
These patients may have a more significant decrease in cortisol levels or
increased vagal tone at night.
Studies also show an increase in inflammation compared with controls and
with patients with daytime asthma.

Go to the next page if you knew the correct answer, or click the link image(s)
below to further research the concepts in this question (if desired).

Research Concepts:
Asthma

We update eBooks quarterly and Apps daily based on user feedback.


Please tap flag to report any questions that need improvement.
Question 237: A patient is admitted to the hospital following a near-
drowning event. Which of the following complications is most likely to arise
from such an event?

Choices:
1. Tension pneumothorax
2. Pulmonary embolism
3. Epiglottitis
4. Acute respiratory distress syndrome
Answer: 4 - Acute respiratory distress syndrome
Explanations:
Near drowning commonly results in acute respiratory distress syndrome
(ARDS).
Fluid aspiration may wash out surfactant, leading to pulmonary edema and
ARDS.
ARDS from nonfatal drowning may result in decreased lung compliance,
ventilation-perfusion mismatch, and intrapulmonary shunting, leading to
hypoxemia causing diffuse organ dysfunction.

Go to the next page if you knew the correct answer, or click the link image(s)
below to further research the concepts in this question (if desired).

Research Concepts:
Drowning

We update eBooks quarterly and Apps daily based on user feedback.


Please tap flag to report any questions that need improvement.
Question 238: A patient with symptomatic pulmonary sarcoidosis is
referred to discuss treatment options. Which of the following is the most
appropriate initial option to prevent pulmonary fibrosis?

Choices:
1. Cyclophosphamide
2. Systemic corticosteroids
3. Chest irradiation
4. Immunoglobulins
Answer: 2 - Systemic corticosteroids
Explanations:
In symptomatic patients with sarcoidosis, glucocorticoids administration is
aimed at the prevention of pulmonary fibrosis.
The course of treatment is usually 3 to 6 months, but many patients do not
tolerate the side effects.
Inhaled corticosteroids have been tried but results have been variable.
Other drugs to treat sarcoid include methotrexate, chloroquine, and
cyclophosphamide.

Go to the next page if you knew the correct answer, or click the link image(s)
below to further research the concepts in this question (if desired).

Research Concepts:
Sarcoidosis

We update eBooks quarterly and Apps daily based on user feedback.


Please tap flag to report any questions that need improvement.
Question 239: What is the recommended regimen for Mycobacterium
avium intracellulare (MAC) infection?

Choices:
1. Clarithromycin, rifampin, and ethambutol
2. Clarithromycin alone
3. Clarithromycin, ciprofloxacin, doxycycline, and sulfamethoxazole
4. Rifampin and ethambutol
Answer: 1 - Clarithromycin, rifampin, and ethambutol
Explanations:
Clarithromycin is the mainstay of therapy for Mycobacterium avium
intracellulare (MAC). Clarithromycin concentrations decrease in the
presence of rifampin.
Clarithromycin is the only drug in the MAC regimens for which there is a
good correlation between in vitro susceptibility and clinical response.
With this regimen, one study showed 74% of patients to have a favorable
microbiologic response; favorable is defined as three consecutive negative
sputum cultures and at least 12 months of therapy.
Furthermore, these favorable outcomes were unrelated to plasma
clarithromycin levels.

Go to the next page if you knew the correct answer, or click the link image(s)
below to further research the concepts in this question (if desired).

Research Concepts:
Mycobacterium Avium Intracellulare

We update eBooks quarterly and Apps daily based on user feedback.


Please tap flag to report any questions that need improvement.
Question 240: What is the most common presenting feature of
Pneumocystis jiroveci pneumonia?

Choices:
1. Fever
2. Exertional dyspnea
3. Chest discomfort
4. Weight loss
Answer: 2 - Exertional dyspnea
Explanations:
Progressive exertional dyspnea is the most common presenting feature of
Pneumocystis jiroveci pneumonia.
Other features include fever, non-productive cough, chest discomfort,
weight loss, chills, and hemoptysis.
The physical exam may reveal tachycardia, fever, tachypnea, and mild
crackles.
In some patients, extrapulmonary symptoms may include confusion, retinal
cotton wool spots, enlarged thyroid, and lymphadenopathy.

Go to the next page if you knew the correct answer, or click the link image(s)
below to further research the concepts in this question (if desired).

Research Concepts:
Pneumonia, Pneumocystis (Carinii) Jiroveci

We update eBooks quarterly and Apps daily based on user feedback.


Please tap flag to report any questions that need improvement.
Question 241: A child with obstructive sleep apnea (OSA) undergoes
tonsillectomy and adenoidectomy. Which of the following is true?

Choices:
1. There is almost always an immediate improvement in airway symptoms
2. There is a higher risk of postoperative respiratory failure than in children
undergoing the same procedure without OSA
3. There is more postoperative bleeding in patients with OSA than without
4. If there is no bleeding, overnight observation is not needed
Answer: 2 - There is a higher risk of postoperative respiratory failure than in
children undergoing the same procedure without OSA

Explanations:
Children with severe OSA, neuromuscular disease, craniofacial syndromes,
medical comorbidities, or less than three years of age are at higher risk for
complications and should be kept overnight.
Improvement after surgery usually occurs after edema resolves and residual
anesthetic agents are eliminated.
Bleeding complications are similar between the two groups.
In children, the most common cause of OSA is enlarged tonsils and
adenoids.

Go to the next page if you knew the correct answer, or click the link image(s)
below to further research the concepts in this question (if desired).

Research Concepts:
Apnea, Obstructive Sleep Apnea

We update eBooks quarterly and Apps daily based on user feedback.


Please tap flag to report any questions that need improvement.
Question 242: Which of the following statements about procalcitonin is
FALSE?

Choices:
1. Procalcitonin is a precursor to the hormone calcitonin
2. Procalcitonin has greater specificity for serious bacterial infections than CRP
3. Procalcitonin cannot be used to distinguish bacterial from viral infections
4. Procalcitonin levels rise within 3-6 hours of infection, peak at 12-48 hours and
rapidly decline during recovery
Answer: 3 - Procalcitonin cannot be used to distinguish bacterial from viral
infections

Explanations:
Procalcitonin is a serum biomarker of inflammation specifically related to
bacterial infections.
Its up-regulation is blocked by interferon-gamma, a cytokine released in
viral infections. Therefor, procalcitonin can differentiate bacterial from viral
infections. Procalcitonin cannot be used in patients who are
immunocompromised.
Antimicrobial therapy can usually be stopped when serum procalcitonin
levels have dropped >90% from the levels at initial diagnosis of severe
sepsis but not prior to 5 days of therapy.
Procalcitonin levels should be rechecked 48-72 hours after beginning
antibiotic therapy. They should also be checked after stopping antibiotic
treatment if the patient is not improving clinically.

Go to the next page if you knew the correct answer, or click the link image(s)
below to further research the concepts in this question (if desired).

Research Concepts:
Procalcitonin

We update eBooks quarterly and Apps daily based on user feedback.


Please tap flag to report any questions that need improvement.
Question 243: An adolescent has a summer job at a pet shop where he is
responsible for cleaning bird cages. He develops fever, chills and a non
productive cough. On physical examination he is noted to have a red throat and
right basilar crackles. Which of the following is the most likely diagnosis?

Choices:
1. Sarcoidosis
2. Lung cancer
3. Psittacosis
4. Histoplasmosis
Answer: 3 - Psittacosis
Explanations:
Psittacosis is caused by Chlamydia psittaci.
C. psittaci can infect parrots. canaries and other birds.
Psittacosis can be an occupational disease in zoo and pet shop and other
workers involved with birds.

Go to the next page if you knew the correct answer, or click the link image(s)
below to further research the concepts in this question (if desired).

Research Concepts:
Psittacosis

We update eBooks quarterly and Apps daily based on user feedback.


Please tap flag to report any questions that need improvement.
Question 244: Which of the following fungal infections are frequently
mistaken for lung cancer on imaging?

Choices:
1. Blastomycosis
2. Histoplasmosis
3. Coccidioidomycosis
4. Candidiasis
Answer: 1 - Blastomycosis
Explanations:
Blastomycosis is seen more commonly in the Southeast United States.
Radiologically, it most commonly presents as an air-space disease, but focal
masslike consolidation is the second most common radiologic finding,
occurring in 17 to 31 percent. In the chronic form, it can be cavitary.
Entry of the infection is via inhalation of the conidia into the lungs.
Dissemination can then occur to the skin, bone, prostate gland, and central
nervous system.
A culture of the organism is done from sputum or tissue biopsies. The latter
is quite easy and effective when skin lesions are identified.
Infection can be chronic or acute in immunocompetent individuals. All
patients should be treated to prevent dissemination and recurrence.

Go to the next page if you knew the correct answer, or click the link image(s)
below to further research the concepts in this question (if desired).

Research Concepts:
Blastomycosis

We update eBooks quarterly and Apps daily based on user feedback.


Please tap flag to report any questions that need improvement.
Question 245: The maximal number of alveoli in both lungs is reached by
what age?

Choices:
1. 5 years
2. 8 years
3. 10 years
4. 20 years
Answer: 2 - 8 years
Explanations:
A full term infant normally reaches the maximal number of alveoli at age
eight.

Go to the next page if you knew the correct answer, or click the link image(s)
below to further research the concepts in this question (if desired).

Research Concepts:
Anatomy, Thorax, Lungs

We update eBooks quarterly and Apps daily based on user feedback.


Please tap flag to report any questions that need improvement.
Question 246: In a patient that is not paralyzed but has central nervous
system dysfunction what is the preferred ventilation pattern?

Choices:
1. Assist Control Ventilation (AC)
2. Synchronous Intermittent Mandatory Ventilation (SIMV)
3. Continuous Mandatory Ventilation (CMV)
4. Intermittent Mandatory Ventilation (IMV)
Answer: 2 - Synchronous Intermittent Mandatory Ventilation (SIMV)
Explanations:
Volume-cycled breaths that coincide with spontaneous lung function is
termed Synchronized Intermittent Mandatory Ventilation (SIMV).
SIMV is most appropriate for patients with CNS dysfunction who tend to
exhibit variations in tidal volumes and respiratory drive.
Between mechanical breaths the patient can breathe unassisted and
spontaneously.
If the patient's respiratory drive is insufficient to prevent hypercarbia a set
Respiratory Rate can be used.

Go to the next page if you knew the correct answer, or click the link image(s)
below to further research the concepts in this question (if desired).

Research Concepts:
Ventilation, Mechanical

We update eBooks quarterly and Apps daily based on user feedback.


Please tap flag to report any questions that need improvement.
Question 247: A patient with HIV requires prevention for Mycobacterium
avium intracellulare. What is the drug of choice?

Choices:
1. Clarithromycin
2. Oxytetracycline
3. Penicillin G
4. Aztreonam
Answer: 1 - Clarithromycin
Explanations:
MAC prophylaxis is recommended for adult patients with CD4 counts less
than 50.
Daily clarithromycin or weekly azithromycin can be used.
Treatment of disseminated MAC is treated with 2 or 3 agents including
macrolides, ethambutol, and rifamycins.

Go to the next page if you knew the correct answer, or click the link image(s)
below to further research the concepts in this question (if desired).

Research Concepts:
Mycobacterium Avium Intracellulare

We update eBooks quarterly and Apps daily based on user feedback.


Please tap flag to report any questions that need improvement.
Question 248: A 6-month-old child is admitted with an apparent life-
threatening event, but evaluation including history, physical, CT of the head,
electrolytes, EEG, serum glucose are all normal. Select appropriate management.

Choices:
1. Home apnea monitor for 1 month
2. Routine health care
3. Home apnea monitor for 2 months and home nursing visits
4. Home apnea monitoring, home nursing visits, and visits to the provider
weekly
Answer: 2 - Routine health care
Explanations:
Apnea monitors are appropriate for premature infants until 43 weeks of
gestational age.
There is no evidence that monitoring will be of benefit to prevent death at a
more advanced age.
Home monitoring is more likely to disrupt the family and has no proven
benefit in this situation.
More frequent visits to the primary care provider may be reassuring. As of
2016, the American Academy of Pediatrics recommends ALTE no longer
be used. Brief resolved unexplained event (BRUE) will now be the
replacement.

Go to the next page if you knew the correct answer, or click the link image(s)
below to further research the concepts in this question (if desired).

Research Concepts:
Brief Resolved Unexplained Event (BRUE)

We update eBooks quarterly and Apps daily based on user feedback.


Please tap flag to report any questions that need improvement.
Question 249: Which of the following gas factors does not drop with high
altitude?

Choices:
1. Total pressure
2. Partial pressure of oxygen
3. Fraction of oxygen
4. Oxygen Saturation
Answer: 3 - Fraction of oxygen
Explanations:
With the drop in total gas pressure and partial pressure of oxygen, the
fraction of each component gas remains the same.
At altitude, the percentage of each gas, otherwise known as fraction, will
stay the same but the overall content pressure times fraction of inspired
oxygen will decrease proportionally to the ambient pressure.
Lower levels of oxygen delivery can decrease oxygen saturation in the
blood.
As the ambient pressure decreases at altitude, the density of gas also
decreases.

Go to the next page if you knew the correct answer, or click the link image(s)
below to further research the concepts in this question (if desired).

Research Concepts:
Fraction of Inspired Oxygen (FiO2)

We update eBooks quarterly and Apps daily based on user feedback.


Please tap flag to report any questions that need improvement.
Question 250: Which of the following is a true statement about sudden
infant death syndrome (SIDS)?

Choices:
1. 99% of cases occur in infants less than 6 months
2. There is a genetic predisposition
3. The sibling of a child who died of SIDS is 5 times more likely to also die of
SIDS
4. Mothers less than 20 years of age have decreased risk of having a child with
SIDS
Answer: 3 - The sibling of a child who died of SIDS is 5 times more likely to
also die of SIDS

Explanations:
There does not seem to be a genetic predisposition for sudden infant death
syndrome (SIDS).
Ninety percent of cases occur in infants less than 6 months.
Mothers less than 20 years have increased risk of having children with
SIDS.
Maternal smoking is a risk factor for SIDS.

Go to the next page if you knew the correct answer, or click the link image(s)
below to further research the concepts in this question (if desired).

Research Concepts:
Sudden Infant Death Syndrome

We update eBooks quarterly and Apps daily based on user feedback.


Please tap flag to report any questions that need improvement.
Question 251: Performed a low-lying tracheostomy on a baby can result in
damage to which of the following?

Choices:
1. Left carotid artery
2. Left brachiocephalic vein
3. Jugular vein
4. Superior vena cava
Answer: 2 - Left brachiocephalic vein
Explanations:
One should never dissect laterally when performing a tracheostomy because
damage to the recurrent laryngeal nerve may occur.
When the tracheostomy is placed too low or the neck is hyperextended, one
can easily damage the left brachiocephalic vein or the right brachiocephalic
trunk.
Other injuries may include a pneumomediastinum or pneumothorax.
If there is an injury to the posterior tracheal wall, a tracheoesophageal
fistula can occur.

Go to the next page if you knew the correct answer, or click the link image(s)
below to further research the concepts in this question (if desired).

Research Concepts:
Tracheostomy, Pediatric

We update eBooks quarterly and Apps daily based on user feedback.


Please tap flag to report any questions that need improvement.
Question 252: Which of the following is the correct order when weaning a
patient from mechanical ventilation?

Choices:
1. Decrease FIO2, then positive end-expiratory pressure (PEEP), then rate, then
arterial blood gas (ABG)
2. Decrease PEEP, then FIO2, then rate, then ABG
3. Decrease FIO2, then PEEP, then ABG, then rate
4. Decrease rate, then FIO2, then PEEP, then ABG
Answer: 1 - Decrease FIO2, then positive end-expiratory pressure (PEEP),
then rate, then arterial blood gas (ABG)

Explanations:
Always decrease FI02 first. Then decrease positive end-expiratory pressure
(PEEP) and ventilator rate followed by a repeat arterial blood gas.
To successfully wean, the lung disorder must have stabilized.
The patient must have a Fi02 of less than 0.5 and a PEEP of less than 7
cmH2O.
The patient also must be hemodynamically stable and able to initiate
spontaneous breaths and follow commands.

Go to the next page if you knew the correct answer, or click the link image(s)
below to further research the concepts in this question (if desired).

Research Concepts:
Ventilation, Ventilator Management

We update eBooks quarterly and Apps daily based on user feedback.


Please tap flag to report any questions that need improvement.
Question 253: In which of the following patient populations would you be
most likely see Nocardiosis?

Choices:
1. Chronic granulomatous disease
2. Hyper IgE syndrome
3. IgA deficiency
4. Severe combined immunodeficiency
Answer: 1 - Chronic granulomatous disease
Explanations:
Chronic Granulomatous Disease is the most common neutrophilic disorder.
The most common organisms causing pulmonary infections in these
patients are Aspergillus sp, S aureus. B cepacia , Nocardia, and S
marcescens.
In patients with hyper IgE syndrome, S aureus is the most common cause of
pneumonia.
Pneumocystis jiroveci is the most common cause of pneumonia in patients
with severe combined immunodeficiency; bacterial and viral etiologies can
also occur but less frequently.

Go to the next page if you knew the correct answer, or click the link image(s)
below to further research the concepts in this question (if desired).

Research Concepts:
Nocardiosis

We update eBooks quarterly and Apps daily based on user feedback.


Please tap flag to report any questions that need improvement.
Question 254: Which is true about Hamman sign noted on palpation of the
chest?

Choices:
1. Holosystolic murmur
2. A sound heard after systole
3. Presence of air in the mediastinum
4. Crepitus at the lower lung bases
Answer: 3 - Presence of air in the mediastinum
Explanations:
Hamman sign is heard over the mediastinum due to mediastinal emphysema
or pneumomediastinum.
It is a rasping, crunching sound occurring with each heartbeat.
It is best heard with the patient in the left lateral decubitus position.

Go to the next page if you knew the correct answer, or click the link image(s)
below to further research the concepts in this question (if desired).

Research Concepts:
Pneumomediastinum

We update eBooks quarterly and Apps daily based on user feedback.


Please tap flag to report any questions that need improvement.
Question 255: Which of the following treatments of nasal obstruction can
decrease the amount of continuous positive airway pressure required to treat
obstructive sleep apnea and improve patient compliance?

Choices:
1. Pharmacotherapy
2. Immunotherapy
3. Surgery
4. All of the above
Answer: 4 - All of the above
Explanations:
The use of pharmacotherapy, immunotherapy, and surgery decrease the
amount of pressure required and therefore improve compliance.
Obstructive sleep apnea (OSA) is characterized by episodes of complete
collapse of the airway upon entering deep sleep. This leads to oxygen
desaturation and arousal, resulting in nonrestorative sleep.
Weight loss and the use of continuous positive airway pressure (CPAP) are
the most effective treatments. The diligent adherence to nightly CPAP use
can result in near complete resolution of symptoms.
The primary treatment for OSA in a child is tonsillectomy and
adenoidectomy.

Go to the next page if you knew the correct answer, or click the link image(s)
below to further research the concepts in this question (if desired).

Research Concepts:
Apnea, Obstructive Sleep Apnea

We update eBooks quarterly and Apps daily based on user feedback.


Please tap flag to report any questions that need improvement.
Question 256: When 100 percent oxygen is used in adults at sea level how
soon can oxygen toxicity develop?

Choices:
1. 4 hours
2. 10 hours
3. 16 hours
4. 24 hours
Answer: 2 - 10 hours
Explanations:
A patient can develop oxygen toxicity after breathing greater than 95%
percent oxygen for 10 hours.
Pulmonary toxicity is the first to develop as the lungs are exposed to the
highest concentration of oxygen in the human body.
Central nervous system toxicity can only occur under hyperbaric conditions
Prolonged exposure to high inspired fractions of oxygen can cause damage
to the retina. This is different than hyperoxic myopia which is a transient
condition affecting the lens.

Go to the next page if you knew the correct answer, or click the link image(s)
below to further research the concepts in this question (if desired).

Research Concepts:
Toxicity, Oxygen

We update eBooks quarterly and Apps daily based on user feedback.


Please tap flag to report any questions that need improvement.
Question 257: A 2 year old is suspected of having aspirated a radiolucent
foreign body. Select the most appropriate radiographs in a cooperative child.

Choices:
1. PA and lateral chest radiographs
2. Bilateral decubitus views
3. CT of the chest with contrast
4. AP and lateral chest radiographs
Answer: 2 - Bilateral decubitus views
Explanations:
Hyperinflation of the lung on expiration would be suspicious for a foreign
body.
Decubitus films show dependent volume loss but this would not occur if
there in the presence of obstructive foreign body.
A two year old usually will not co-operate with inspiratory-expiratory films.
CT has the risk of increased radiation and may require sedation in a child of
this age.

Go to the next page if you knew the correct answer, or click the link image(s)
below to further research the concepts in this question (if desired).

Research Concepts:
Airway, Foreign Bodies

We update eBooks quarterly and Apps daily based on user feedback.


Please tap flag to report any questions that need improvement.
Question 258: Which of the following medications potentiate reactive
airway disease (RAD)/asthma?

Choices:
1. Beta blockers
2. Aspirin
3. NSAIDs
4. All of the above
Answer: 4 - All of the above
Explanations:
Beta blockers, aspirin, and NSAIDs can potentiate reactive airway disease
(RAD).
Triggers for RAD may include dust, mildew, pets, smoke, perfume, and
pollen.
Smoke from cigarettes or other sources is a direct lung irritant.

Go to the next page if you knew the correct answer, or click the link image(s)
below to further research the concepts in this question (if desired).

Research Concepts:
Asthma

We update eBooks quarterly and Apps daily based on user feedback.


Please tap flag to report any questions that need improvement.
Question 259: A 10-year-old patient with cystic fibrosis needs to undergo
postural drainage. Which of the following medications is indicated to loosen and
thin mucous secretions in his airways?

Choices:
1. Guaifenesin
2. Levofloxacin
3. Dornase alfa
4. Ephedrine
Answer: 3 - Dornase alfa
Explanations:
Dornase alfa is indicated for this procedure.
It is a recombinant deoxyribonuclease enzyme.
It hydrolyzes mucous DNA and reduces secretion viscosity.
It is administered via a nebulizer.

Go to the next page if you knew the correct answer, or click the link image(s)
below to further research the concepts in this question (if desired).

Research Concepts:
Dornase Alfa

We update eBooks quarterly and Apps daily based on user feedback.


Please tap flag to report any questions that need improvement.
Question 260: Which of the following is a frequent finding among patients
on synchronized intermittent mandatory ventilation?

Choices:
1. Pneumothorax
2. Hypercarbia
3. Increased work when breathing
4. Hypoxia
Answer: 3 - Increased work when breathing
Explanations:
Patients on synchronized intermittent mandatory ventilation often attempt
spontaneous breathing despite a high resistance circuit that increases the
work of breathing.
This can be prevented by providing pressure support to decrease the
patient's work of breathing.
Synchronized intermittent mandatory ventilation (SIMV) guarantees a
specific number of breaths, but unlike assist-control ventilation, patient
breaths are partially their own which reduces the risk of hyperinflation or
alkalosis.
Pressure support in addition to spontaneous breaths may reduce the work of
breathing. SIMV decreases cardiac output in patients with left-ventricular
dysfunction.

Go to the next page if you knew the correct answer, or click the link image(s)
below to further research the concepts in this question (if desired).

Research Concepts:
Ventilation, Synchronized Intermittent Mandatory

We update eBooks quarterly and Apps daily based on user feedback.


Please tap flag to report any questions that need improvement.
Section 5

Question 261: Which of the following statements about bronchial atresia is


true?

Choices:
1. A congenital lesion, it often presents in the late teenage years
2. Most common location: RLL
3. Segmental lung collapse is associated
4. Major differential consideration is a tumor
Answer: 1 - A congenital lesion, it often presents in the late teenage years
Explanations:
Bronchial atresia is a congenital interruption of a proximal bronchus with
associated mucus impaction (bronchocele) just distal.
Most commonly involved segment: apicoposterior segment, LUL.
Collateral air-flow through pores of Kohn cause hyperlucency of involved
lung segment secondary to air trapping.
Differential DX includes ABPA and CF, other causes of mucus impaction.

Go to the next page if you knew the correct answer, or click the link image(s)
below to further research the concepts in this question (if desired).

Research Concepts:
Bronchial Atresia

We update eBooks quarterly and Apps daily based on user feedback.


Please tap flag to report any questions that need improvement.
Question 262: Which of the following is not a potential complication of
mechanical ventilation?

Choices:
1. Barotrauma
2. Decreased venous return
3. Increased afterload
4. Reduced cardiac output
Answer: 3 - Increased afterload
Explanations:
Ventilation reduces venous return and cardiac output.
It also reduces renal blood flow.
While life-saving, mechanical ventilation often has potential complications,
including airway injury, pneumothorax, alveolar damage, and ventilator-
associated pneumonia.
Other complications include diaphragm atrophy and oxygen toxicity.

Go to the next page if you knew the correct answer, or click the link image(s)
below to further research the concepts in this question (if desired).

Research Concepts:
Ventilation, Ventilator Complications

We update eBooks quarterly and Apps daily based on user feedback.


Please tap flag to report any questions that need improvement.
Question 263: Which of the following fungal infection is known to cause
severe fibrosis of the mediastinum?

Choices:
1. Histoplasmosis
2. Blastomycosis
3. Sporotrichosis
4. Candida
Answer: 1 - Histoplasmosis
Explanations:
Progressive fibrosis of the mediastinum can occur with histoplasmosis.
The fibrosis often leads to distortion of the anatomy.
Esophageal fistulas and tracheal adhesions have been reported following
histoplasmosis infection.

Go to the next page if you knew the correct answer, or click the link image(s)
below to further research the concepts in this question (if desired).

Research Concepts:
Histoplasmosis

We update eBooks quarterly and Apps daily based on user feedback.


Please tap flag to report any questions that need improvement.
Question 264: Imaging findings in congenital proximal interruption of the
right pulmonary artery include all except:

Choices:
1. Hypoplasia of ipsilateral lung and hyperexpansion of contralateral lung
2. Scimitar sign of anomalous pulmonary venous drainage
3. No evidence of air trapping on expiratory films
4. Fine peripheral reticular pattern from hypertrophy of bronchial and other
transpleural vasculature
Answer: 2 - Scimitar sign of anomalous pulmonary venous drainage
Explanations:
The scimitar sign is associated with hypogenetic lung syndrome. Pulmonary
venous drainage is normal with proximal PA interruption.
Since the lung development is normal, there is no air-trapping.
Distal pulmonary arteries are normal, but supplied by collateral systemic
arteries.
Left sided pulmonary artery interruption is often associated with CHD,
particularly Tetralogy of Fallot.

Go to the next page if you knew the correct answer, or click the link image(s)
below to further research the concepts in this question (if desired).

Research Concepts:
Pulmonary Artery Malformation

We update eBooks quarterly and Apps daily based on user feedback.


Please tap flag to report any questions that need improvement.
Question 265: Which of the following is not an advantage of using positive
end expiratory pressure (PEEP) for the treatment of acute respiratory distress
syndrome (ARDS)?

Choices:
1. Prevention of atelectasis
2. Permits reduction of FIO2
3. Decreases pulmonary edema
4. Decreases shunting
Answer: 3 - Decreases pulmonary edema
Explanations:
Using PEEP to treat ARDS may actually increase pulmonary edema by
decreasing pulmonary lymphatic drainage.
It may prevent atelectasis, decrease shunting, and allow reduction of FIO2.

Go to the next page if you knew the correct answer, or click the link image(s)
below to further research the concepts in this question (if desired).

Research Concepts:
Acute Respiratory Distress Syndrome (ARDS)

We update eBooks quarterly and Apps daily based on user feedback.


Please tap flag to report any questions that need improvement.
Question 266: In order to avoid false positives from the BCG skin test, it is
recommended that one should also test for which of the following?

Choices:
1. CD 8 cells
2. Interferon gamma
3. Interleukin 2
4. Presence of granulomas in the lung
Answer: 2 - Interferon gamma
Explanations:
There is a lot of controversy on the use of the Mantoux test in people who
have already been immunized with BCG.
The interferon test relies on the fact that T cells release interferon when
exposed to specific antigens.
This may help avoid false positives and unnecessary drug therapy.

Go to the next page if you knew the correct answer, or click the link image(s)
below to further research the concepts in this question (if desired).

Research Concepts:
Bacillus Calmette Guerin (BCG), Skin Test

We update eBooks quarterly and Apps daily based on user feedback.


Please tap flag to report any questions that need improvement.
Question 267: A young African-American female is sent for evaluation to
rule out sarcoid. Her chest x-ray reveals bilateral hilar adenopathy with upper-
lobe predominant parenchymal disease. What is the diagnostic procedure that
has the highest diagnostic yield for this condition?

Choices:
1. Bronchoscopy with lavage/biopsy
2. Mediastinoscopy
3. CT scan
4. Chest radiograph
Answer: 2 - Mediastinoscopy
Explanations:
The definitive diagnostic procedure of choice is mediastinoscopy, which
has the highest yield.
Bronchoscopy is usually the first procedure but is only positive in around
60% of patients.
Serum amyloid A, soluble interleukin-2 receptor, lysozyme, angiotensin-
converting enzyme, and glycoprotein KL-6 are serum markers.
Routine CT is unlikely to provide additional information, but CT guided
biopsy may have a higher yield.

Go to the next page if you knew the correct answer, or click the link image(s)
below to further research the concepts in this question (if desired).

Research Concepts:
Sarcoidosis

We update eBooks quarterly and Apps daily based on user feedback.


Please tap flag to report any questions that need improvement.
Question 268: The vasculitic phase of which syndrome is marked by
purpura and papules in the presence of chest pain, malaise, numbness,
hemoptysis, and arthralgia?

Choices:
1. Eosinophilic granulomatosis with polyangiitis (EGPA)
2. Sweet
3. Ehlers-Danlos
4. Muir-Torre
Answer: 1 - Eosinophilic granulomatosis with polyangiitis (EGPA)
Explanations:
Cutaneous presentations include subcutaneous nodules, reddish macules,
and various hemorrhagic lesions.
EPGA is a rare condition.

Go to the next page if you knew the correct answer, or click the link image(s)
below to further research the concepts in this question (if desired).

Research Concepts:
Churg Strauss Syndrome (Allergic Granulomatosis)

We update eBooks quarterly and Apps daily based on user feedback.


Please tap flag to report any questions that need improvement.
Question 269: Which of the following is least likely to result in venous air
embolism?

Choices:
1. Hemodialysis
2. Diving
3. Spontaneous pneumothorax
4. Endoscopy
Answer: 3 - Spontaneous pneumothorax
Explanations:
Venous air embolism can be associated with hemodialysis,
cardiopulmonary bypass, and extracorporeal membrane oxygenation. Cases
of venous gas embolism, especially portal gas, have occurred with upper
endoscopic procedures where gas is used under pressure to expand
compartments.
Other scenarios where venous gas embolisms have been reported include
central venous catheterization, penetrating or blunt trauma, high-pressure
mechanical ventilation, pulmonary biopsy, anesthesia, and thoracocentesis.
Scuba diving is mostly associated with an asymptomatic venous gas
embolism. When these bubbles overwhelm the pulmonary filtration system,
decompression sickness can develop.
Spontaneous pneumothorax can be associated with smoking, especially in
young, thin males. It typically is not associated with gas embolism except in
the setting of pulmonary over-expansion> In this case, it usually results in
arterial gas embolism rather than venous.

Go to the next page if you knew the correct answer, or click the link image(s)
below to further research the concepts in this question (if desired).

Research Concepts:
Embolism, Venous Gas

We update eBooks quarterly and Apps daily based on user feedback.


Please tap flag to report any questions that need improvement.
Question 270: Which of the following tests are not recommended during
evaluation of a child presenting with low-risk brief resolved unexplained event
(BRUE)?

Choices:
1. White blood cell count
2. 12-lead EKG
3. Pertussis testing
4. Rapid Respiratory syncytial virus antigen test
Answer: 1 - White blood cell count
Explanations:
Low-risk brief resolved unexplained event (BRUE) is characterized by age
less than 60 days, the first occurrence of BRUE, event lasting < 1 minute,
no resuscitation by provider and absence of concerning history or
examination findings. Children in this category need a brief period of
observation and generally do not need any lab tests or imaging.
The American Academy of Pediatrics (AAP) guideline for management
recommends considering the performance of a 12-lead electrocardiogram to
detect prolonged QT interval, testing for pertussis in the setting of an
outbreak, and rapid respiratory syncytial virus (RSV) antigenic testing if
available. This is a weak recommendation.
The AAP guideline considers the performance of WBC count, blood gas
measurement, blood cultures, cerebrospinal fluid cultures, and tests for
inborn errors of metabolism as unnecessary tests. This is a strong
recommendation.
The AAP guideline does not make any recommendations for testing for
infants in with higher-risk BRUE category.Performance of appropriate lab
tests should be based on concerns identified in the history and physical
examination

Go to the next page if you knew the correct answer, or click the link image(s)
below to further research the concepts in this question (if desired).

Research Concepts:
Brief Resolved Unexplained Event (BRUE)

We update eBooks quarterly and Apps daily based on user feedback.


Please tap flag to report any questions that need improvement.
Question 271: What intervention first should be performed for a
mechanically ventilated patient who abruptly develops respiratory distress and
oxygen desaturation?

Choices:
1. Change the endotracheal tube
2. Order arterial blood gas
3. Manually ventilate the patient
4. Order chest x-ray
Answer: 3 - Manually ventilate the patient
Explanations:
Utilize the mnemonic "DOPE."
D: Displacement of endotracheal tube (ETT) O: Obstruction P:
Pneumothorax E: Equipment
The pneumonic DOPE is a reminder of the most common causes of post-
intubation hypoxia or deterioration.
Displacement - check the ETT for displacement or dislodgement.
Obstruction - check the ETT for obstructions, such as a mucous plug or
ventilator tubing kink. Pneumothorax - get an x-ray. Equipment failure -
disconnect from the ventilator and bag manually.

Go to the next page if you knew the correct answer, or click the link image(s)
below to further research the concepts in this question (if desired).

Research Concepts:
Ventilation, Mechanical

We update eBooks quarterly and Apps daily based on user feedback.


Please tap flag to report any questions that need improvement.
Question 272: Which of the following is used for the treatment of post-
lung transplant malignancy?

Choices:
1. Additional chemotherapy
2. Radiation therapy
3. Increased dose of present chemotherapy
4. Reduced dose of present chemotherapy
Answer: 4 - Reduced dose of present chemotherapy
Explanations:
Post-transplant malignancies are associated with Epstein-Barr infections.
The malignancies are treated by decreasing the dose of chemotherapy and
giving acyclovir.
In the last decade, rapamycin inhibitors have been advocated in transplant
patients, both to suppress the immune system and decrease the risk of
cancer.
Retrospective studies have shown a decreased incidence of de novo cancers
in patients treated with rapamycin inhibitors.

Go to the next page if you knew the correct answer, or click the link image(s)
below to further research the concepts in this question (if desired).

Research Concepts:
Cancer, Lung

We update eBooks quarterly and Apps daily based on user feedback.


Please tap flag to report any questions that need improvement.
Question 273: What is the correct treatment of fibrothorax?
Choices:
1. Lytic therapy
2. VATS
3. Thoracotomy
4. Thoracostomy
Answer: 3 - Thoracotomy
Explanations:
Causes of fibrothorax include traumatic hemothorax, empyema,
chylothorax, tuberculosis, and talc pleurodesis.
The best treatment of fibrothorax is to remove the thick peel. This is more
easily done with thoracotomy (decortication). Lytic therapy and VATS are
used in early onset empyema. By removing the thick peel, the compressed
lung is re-expanded and the space obliterated.
Contraindications to decortication include extensive disease in collapsed
lung, bronchial stenosis, debilitated patient, and contralateral parenchymal
disease.
One has to be careful during decortication because injury can occur to the
lower brachial plexus, phrenic nerve, subclavian artery, and sympathetic
chain. If after decortication residual air space remains, this can be
obliterated with either a thoracoplasty (last choice) or muscle flap.

Go to the next page if you knew the correct answer, or click the link image(s)
below to further research the concepts in this question (if desired).

Research Concepts:
Fibrothorax

We update eBooks quarterly and Apps daily based on user feedback.


Please tap flag to report any questions that need improvement.
Question 274: Which of the following is not an indication for use of
continuous positive airway pressure (CPAP)?

Choices:
1. A 13-year old boy with Down syndrome, hypotonia, and obstructive sleep
apnea (OSA)
2. A 13-hour old preterm neonate with respiratory distress syndrome from
surfactant deficiency
3. A 13-month old infant with Respiratory syncytial virus (RSV) bronchiolitis
being weaned from mechanical ventilation
4. 13-year old girl with an intentional drug overdose, bradypnea, and impending
respiratory arrest
Answer: 4 - 13-year old girl with an intentional drug overdose, bradypnea,
and impending respiratory arrest

Explanations:
Obstructive sleep apnea (OSA) is a cessation or pause in breathing while
asleep, caused by airway collapse from a variety of causes such as obesity,
hypotonia, and adenotonsillar hypertrophy. CPAP can be used to maintain
airway patency in these instances.
Neonatal literature supports the use of CPAP to treat preterm infants whose
lungs have not yet fully developed and who may have respiratory distress
syndrome from surfactant deficiency. CPAP is also used to treat hypoxia
and decrease the work of breathing in infants with acute infectious
processes such as bronchiolitis and pneumonia or for those with collapsible
airways such as in tracheomalacia.
CPAP can be used to deliver PEEP via a face mask with a tight seal prior to
intubation. It can also be used as a mode on the ventilator prior to
extubation or to support oxygenation through non-invasive interfaces after
extubation.
CPAP should not be used in place of mechanical ventilation for patients that
are apneic, cannot protect their airway or need full ventilator support.

Go to the next page if you knew the correct answer, or click the link image(s)
below to further research the concepts in this question (if desired).

Research Concepts:
Continuous Positive Airway Pressure (CPAP)

We update eBooks quarterly and Apps daily based on user feedback.


Please tap flag to report any questions that need improvement.
Question 275: Which of the following is not true of a mucormycosis
infection?

Choices:
1. Extensive infection occurs in diabetics
2. Usually the organism invades blood vessels, causing thrombosis and
infarction
3. Cultures are usually negative
4. Immediate treatment with amphotericin prevents the high morbidity
Answer: 4 - Immediate treatment with amphotericin prevents the high
morbidity

Explanations:
Mucormycosis affects rhinocerebral sinuses and pulmonary tissues.
Mucormycosis is seen in diabetics, the immunosuppressed, leukemics, and
neutropenic patients. The fungi appear as right angle-shaped hyphae and
cause thrombosis of blood, leading to infarction and gangrene.
The best treatment for mucormycosis is radical surgical debridement of
infected tissue.
Amphotericin coverage must be used at all times. It carries a high mortality
despite surgery.

Go to the next page if you knew the correct answer, or click the link image(s)
below to further research the concepts in this question (if desired).

Research Concepts:
Mucormycosis

We update eBooks quarterly and Apps daily based on user feedback.


Please tap flag to report any questions that need improvement.
Question 276: Which of the following exanthems can be associated with
mycoplasmal pneumonia?

Choices:
1. Erythema migrans
2. Erythema multiforme
3. Pityriasis rosea
4. None of the above
Answer: 2 - Erythema multiforme
Explanations:
Erythema multiforme can be associated with mycoplasmal pneumonia.
The rash is typically secondary to an allergic reaction or infection.
Erythema migrans is associated with Lyme disease.
Pityriasis rosea has an associated herald patch and usually manifests in a
Christmas tree pattern.

Go to the next page if you knew the correct answer, or click the link image(s)
below to further research the concepts in this question (if desired).

Research Concepts:
Erythema, Multiforme

We update eBooks quarterly and Apps daily based on user feedback.


Please tap flag to report any questions that need improvement.
Question 277: According to the Apnea-hypopnea index, how many
episodes per hour is indicative of mild obstructive sleep apnea?

Choices:
1. 5 to 15
2. 15 to 30
3. More than 30
4. None of the above
Answer: 1 - 5 to 15
Explanations:
The Apnea-hypopnea index (AHI) is derived from the total number of
apneas and hypopneas divided by total sleep time.
Mild obstructive sleep apnea (OSA) is considered an AHI of 5 to 15
episodes per hour.
Moderate OSA is considered 15 to 30 episodes per hour. Severe OSA is
considered to be more than 30 episodes per hour.

Go to the next page if you knew the correct answer, or click the link image(s)
below to further research the concepts in this question (if desired).

Research Concepts:
Apnea, Obstructive Sleep Apnea

We update eBooks quarterly and Apps daily based on user feedback.


Please tap flag to report any questions that need improvement.
Question 278: A patient with a lung abscess is treated with intravenous
antibiotics. He is clinically improving but his chest x-ray still shows a persistent
air-fluid level. What is the next step in the management of this patient?

Choices:
1. Surgery
2. Change antibiotics
3. Continue present management
4. Percutaneous drainage
Answer: 3 - Continue present management
Explanations:
Most causes of lung abscess respond to antibiotics and resolve in 3-4
months. In addition to chest physiotherapy, a therapeutic bronchoscopy may
be required to establish drainage.
Surgical intervention in uncomplicated cases is rare. Reasons for surgery
include a very large abscess, massive hemoptysis, empyema, bronchial
obstruction, and suspicion of cancer.
Generally, the chest x-ray lags behind clinical improvement. If the patient is
improving clinically, the presence of an air-fluid level, with or without
surrounding infiltrate, is not an indication for surgery.
Percutaneous drainage of a lung abscess is done if the patient remains septic
despite 2-3 weeks of antibiotic therapy, if the abscess cavity is increasing in
size, or if the abscess is under tension.

Go to the next page if you knew the correct answer, or click the link image(s)
below to further research the concepts in this question (if desired).

Research Concepts:
Abscess, Lung

We update eBooks quarterly and Apps daily based on user feedback.


Please tap flag to report any questions that need improvement.
Question 279: In an individual with draining sinus tracts on the chest wall,
what is the most likely causative agent?

Choices:
1. S. aureus
2. Pseudomonas
3. Actinomycosis
4. PCP
Answer: 3 - Actinomycosis
Explanations:
Actinomycosis is notorious affecting the cervical region.
The organism usually produces sinus tracts that contain sulfur.
Actinomycosis is highly sensitive to penicillin.
The sinus tracts are often mistaken for tuberculosis.

Go to the next page if you knew the correct answer, or click the link image(s)
below to further research the concepts in this question (if desired).

Research Concepts:
Actinomycosis

We update eBooks quarterly and Apps daily based on user feedback.


Please tap flag to report any questions that need improvement.
Question 280: Which of the following is most likely to improve
oxygenation in a patient with acute respiratory distress syndrome?

Choices:
1. Increase expiratory time
2. Increase tidal volume
3. Increase positive end-expiratory pressure
4. Increase inspiratory flow rate
Answer: 3 - Increase positive end-expiratory pressure
Explanations:
There is no specific treatment for acute respiratory distress syndrome
(ARDS). Care is supportive and the goal is to provide adequate oxygenation
without causing further injury to the lungs.
Increasing positive end-expiratory pressure (PEEP) results in increased
mean airway pressure, which is generally associated with improved
oxygenation.
The use of low tidal volumes for ventilation of patients with ARDS is
associated with reduced mortality. Low tidal volumes are appropriate even
when their use is associated with hypercarbia.
There is some evidence that pressure-controlled ventilation may be
beneficial in patients with ARDS. Few studies show decreased organ
system failures in patients with this form of mechanical ventilation.

Go to the next page if you knew the correct answer, or click the link image(s)
below to further research the concepts in this question (if desired).

Research Concepts:
Acute Respiratory Distress Syndrome (ARDS)

We update eBooks quarterly and Apps daily based on user feedback.


Please tap flag to report any questions that need improvement.
Question 281: What is most common cause of unilateral diaphragmatic
paralysis excluding postoperative causes?

Choices:
1. Idiopathic
2. Malignancy
3. Infection
4. Cervical spondylosis
Answer: 1 - Idiopathic
Explanations:
Up to two-thirds of unilateral diaphragmatic paralysis cases are idiopathic.
Unilateral diaphragmatic paralysis is more common on the left side.
Thirty percent of symptomatic unilateral diaphragmatic paralysis is due to
malignancy with phrenic nerve involvement. Other causes include surgical
or penetrating trauma and neurologic disease. Regional anesthesia blocks of
the brachial plexus (interscalene and supraclavicular) can block the phrenic
nerve.
Bilateral diaphragmatic paralysis is symptomatic more often than unilateral
diaphragmatic paralysis.

Go to the next page if you knew the correct answer, or click the link image(s)
below to further research the concepts in this question (if desired).

Research Concepts:
Unilateral Diaphragmatic Paralysis

We update eBooks quarterly and Apps daily based on user feedback.


Please tap flag to report any questions that need improvement.
Question 282: What is the estimated time a person's brain can be anoxic
from cardiopulmonary failure and not develop permanent brain damage?

Choices:
1. 10 minutes
2. 5 minutes
3. 2 minutes
4. 20 minutes
Answer: 2 - 5 minutes
Explanations:
After a person is without cardiopulmonary function for 4 to 6 minutes,
permanent brain damage is almost certain.

Go to the next page if you knew the correct answer, or click the link image(s)
below to further research the concepts in this question (if desired).

Research Concepts:
Anoxic Encephalopathy

We update eBooks quarterly and Apps daily based on user feedback.


Please tap flag to report any questions that need improvement.
Question 283: A 17-year-old female who is in her second trimester of
pregnancy presents for pulmonary functioning testing to compare with her
results prior to pregnancy. What would be found on pulmonary function testing
and arterial blood gas measurement?

Choices:
1. Forced expiratory volume per second (FEV1) increased, tidal volume
increased, PO2 decreased, PCO2 decreased
2. FEV1 unchanged, tidal volume decreased, PO2 decreased, PCO2 increased
3. FEV1 decreased, tidal volume increased, PO2 decreased, PCO2 decreased
4. FEV 1 unchanged, tidal volume increased, PO2 increased, PCO2 decreased
Answer: 4 - FEV 1 unchanged, tidal volume increased, PO2 increased, PCO2
decreased

Explanations:
FEV1 is not changed by pregnancy.
Physiologic changes in pregnancy lead to an increased tidal volume with
slightly increased P02 and decreased PCO2.
Pulse oximetry can be useful in assessing the severity of an asthma attack
or monitoring for deterioration.
Peak flow measures also can be used to assess asthma. They always should
be checked against a nomogram and the individual patient's normal baseline
function.

Go to the next page if you knew the correct answer, or click the link image(s)
below to further research the concepts in this question (if desired).

Research Concepts:
Asthma

We update eBooks quarterly and Apps daily based on user feedback.


Please tap flag to report any questions that need improvement.
Question 284: A patient experiencing seasonal allergy symptoms is
receiving cetirizine. To identify the offending allergens, the patient is scheduled
for diagnostic skin tests (intradermal testing). How long prior to testing should
cetirizine be withheld?

Choices:
1. 24 hours before the tests
2. 48 hours before the tests
3. 4 days before the tests
4. 1 week before the tests
Answer: 3 - 4 days before the tests
Explanations:
Cetirizine should be withheld starting 4 days before skin tests (intradermal
testing) are scheduled.
Antihistamines can prevent, reduce, or mask positive skin test results.
Waiting as late as 24 to 48 hours before the skin tests to discontinue the
cetirizine would most likely invalidate the skin test results.
It would not be necessary to discontinue the cetirizine as much as 1 week
before the skin tests.

Go to the next page if you knew the correct answer, or click the link image(s)
below to further research the concepts in this question (if desired).

Research Concepts:
Allergy, Testing

We update eBooks quarterly and Apps daily based on user feedback.


Please tap flag to report any questions that need improvement.
Question 285: At what angle of scoliosis are restrictive lung disease and
cardiac complications common?

Choices:
1. 60 degrees
2. 10 degrees
3. 30 degrees
4. 20 degrees
Answer: 1 - 60 degrees
Explanations:
Severe scoliosis can lead to restrictive lung disease and cor pulmonale.
Surgical treatment is required if there are greater than 45 degrees of
curvature.
Braces may suffice for lesser degrees of scoliosis.

Go to the next page if you knew the correct answer, or click the link image(s)
below to further research the concepts in this question (if desired).

Research Concepts:
Lung, Restrictive Disease

We update eBooks quarterly and Apps daily based on user feedback.


Please tap flag to report any questions that need improvement.
Question 286: Which of the following statements best describes the role of
the rapid shallow breathing index (RSBI) in assessing extubation readiness
among mechanically ventilated patients?

Choices:
1. Patients with an RSBI greater than 105 are more likely to fail extubation than
patients with RSBI less than 105
2. RSBI can only be measured when the patient is alert and cooperative
3. RSBI is only a useful predictor of extubation success among patients with
high minute ventilation
4. The extubation failure rate among patients with RSBI less than 105 is less
than 10%
Answer: 1 - Patients with an RSBI greater than 105 are more likely to fail
extubation than patients with RSBI less than 105

Explanations:
The rapid shallow breathing index (RSBI) is calculated by dividing
respiratory rate by tidal volume. Its measurement does not require the
patient's cooperation.
In general, the lower the RSBI, the better.
The likelihood of extubation failure is notably higher among patients with
an RSBI greater than 105.
RSBI dependant weaning does not improve clinical outcomes.

Go to the next page if you knew the correct answer, or click the link image(s)
below to further research the concepts in this question (if desired).

Research Concepts:
Ventilator Weaning

We update eBooks quarterly and Apps daily based on user feedback.


Please tap flag to report any questions that need improvement.
Question 287: Which of the following is not a HIPAA violation?
Choices:
1. Giving patient information to a concerned relative
2. Sharing the patient's gynecologic history with a plastic surgeon who is
performing an elective facial procedure
3. Showing the record to a curious colleague
4. Providing select and applicable documentation to the appropriate physician
Answer: 4 - Providing select and applicable documentation to the appropriate
physician

Explanations:
Giving confidential information to concerned and curious relatives is a
violation of HIPAA laws.
Sharing any information that is not relevant to the procedure or treatment
being performed is prohibited.
Divulging any information or part of the record to anyone who is not
directly involved in the care of the patient is a HIPAA violation.
Any information needed to provide care and treatment, in a current matter,
is in keeping with the information that may be released under HIPAA law.

Go to the next page if you knew the correct answer, or click the link image(s)
below to further research the concepts in this question (if desired).

Research Concepts:
Health Insurance Portability and Accountability Act (HIPAA)

We update eBooks quarterly and Apps daily based on user feedback.


Please tap flag to report any questions that need improvement.
Question 288: Which of the following is a symptom of oxygen toxicity in a
hyperbaric setting?

Choices:
1. Diarrhea
2. Sneezing
3. Seizures
4. Abdominal pain
Answer: 3 - Seizures
Explanations:
Symptoms of central nervous system oxygen toxicity develop more quickly
than pulmonary oxygen toxicity under hyperbaric conditions.
Warning signs may include twitching, tunnel vision, anxiety, nausea,
vertigo, hallucinations, irritability, and tinnitus.
However, only 50% of seizures in a hyperbaric, hyperemic environment are
preceded by these warning signs. A seizure can be a symptom of oxygen
toxicity but occurs almost exclusively in the hyperbaric setting.
This can occur during deep sea diving or with medical hyperbaric oxygen
treatment.

Go to the next page if you knew the correct answer, or click the link image(s)
below to further research the concepts in this question (if desired).

Research Concepts:
Toxicity, Oxygen

We update eBooks quarterly and Apps daily based on user feedback.


Please tap flag to report any questions that need improvement.
Question 289: Which of the following is an advantage of skin testing over
serum assays such as RAST to determine specific IgE?

Choices:
1. Skin testing is not affected by administration of antihistamines
2. Skin testing has greater sensitivity than RAST
3. Skin testing is associated with less risk of allergic reaction
4. Skin testing is not confounded by dermographism
Answer: 2 - Skin testing has greater sensitivity than RAST
Explanations:
The primary advantages of the serum assays (as-IgE and RAST) in
comparison with allergen skin testing are their safety and that the results are
not influenced by skin disease or medications.
Overall, the results of these tests correlate well with those obtained by skin
testing and provocation challenges. The as-IgE assays are not as sensitive as
the skin tests.
In patients with histories of life-threatening reactions to foods, insect stings,
drugs, or latex, skin testing is still required because of its higher sensitivity
even if the as-IgE assay result is negative.
Skin testing is not confounded by dermographism.

Go to the next page if you knew the correct answer, or click the link image(s)
below to further research the concepts in this question (if desired).

Research Concepts:
Allergy, Testing

We update eBooks quarterly and Apps daily based on user feedback.


Please tap flag to report any questions that need improvement.
Question 290: A patient has granulomatosis with polyangiitis and has
glomerulonephritis, cavitary lung disease, and tracheal stenosis. She is on
cyclophosphamide 2 mg/kg daily. Which of the following is NOT a potential
side effect at this dose?

Choices:
1. Infertility
2. Alopecia
3. Hemorrhagic cystitis
4. Bone marrow suppression
Answer: 2 - Alopecia
Explanations:
Granulomatosis with polyangiitis can be treated with glucocorticoids with
cyclophosphamide, increasing the five-year survival from 5 to greater than
70 percent.
Cyclophosphamide is a cytotoxic alkylating agent.
At 2 mg/kg, the incidence of cystitis is 30 percent and of bladder cancer is 6
percent.
Alopecia is rare at this dose.

Go to the next page if you knew the correct answer, or click the link image(s)
below to further research the concepts in this question (if desired).

Research Concepts:
Granulomatosis with Polyangiitis (GPA, Wegener Granulomatosis)

We update eBooks quarterly and Apps daily based on user feedback.


Please tap flag to report any questions that need improvement.
Question 291: A patient that would not be placed on an organ donation
wait list for a lung transplant would be which of the following?

Choices:
1. Patient with emphysema
2. Patient with repeated interstitial lung disease
3. Patient with current pneumocystis jiroveci infection
4. Patient with cystic fibrosis
Answer: 3 - Patient with current pneumocystis jiroveci infection
Explanations:
Any patient with an active infection is not a candidate for a lung transplant.
The infection can spread during surgery and use of postoperative
chemotherapy can cause sepsis.
Patients with malignancy in the past 2 years also are not candidates for the
procedure unless it is a localized skin cancer.
Acute medical instability is a contraindication to the procedure.

Go to the next page if you knew the correct answer, or click the link image(s)
below to further research the concepts in this question (if desired).

Research Concepts:
Transplantation, Lung

We update eBooks quarterly and Apps daily based on user feedback.


Please tap flag to report any questions that need improvement.
Question 292: A young male has a collapsed airway due to a defect in the
cartilage. Which of the following syndromes is the most likely diagnosis?

Choices:
1. Williams-Campbell syndrome
2. Turner syndrome
3. Marfan syndrome
4. Ehlers-Danlos syndrome
Answer: 1 - Williams-Campbell syndrome
Explanations:
Williams-Campbell syndrome is associated with defective cartilage in the
bronchi.
The disorder leads to airway collapse and bronchiectasis.
Williams-Campbell syndrome is a form of congenital cystic bronchiectasis
that results in the airway collapse and bronchiectasis. Patients present with
a persistent cough, wheezing, and impaired lung function.
Williams-Campbell syndrome is thought to occur from a deficiency of
cartilage formation in the fourth- to sixth-order segmental bronchi.
Radiologically, the lungs are overinflated, and bronchoscopy will reveal
bronchomalacia.

Go to the next page if you knew the correct answer, or click the link image(s)
below to further research the concepts in this question (if desired).

Research Concepts:
Williams-Campbell Syndrome

We update eBooks quarterly and Apps daily based on user feedback.


Please tap flag to report any questions that need improvement.
Question 293: Which of the following will trigger the low-pressure alarm
in a ventilated patient?

Choices:
1. Increased secretions
2. Endotracheal tube cuff leak
3. Uncontrolled pain
4. Coughing
Answer: 2 - Endotracheal tube cuff leak
Explanations:
The low-pressure alarm is an indicator that the patient is disconnected from
the ventilator.
The patient may not be receiving the set tidal volume.
Low-pressure alarms are used to detect the drop in peak inspiratory
pressure that occurs when a patient becomes disconnected from a ventilator.
Alarms are ineffective if the low-pressure alarm point is adjusted too low or
preset too low by the manufacturer. Failure to detect such a disconnect is
potentially fatal.

Go to the next page if you knew the correct answer, or click the link image(s)
below to further research the concepts in this question (if desired).

Research Concepts:
Ventilation, Mechanical

We update eBooks quarterly and Apps daily based on user feedback.


Please tap flag to report any questions that need improvement.
Question 294: What is the survival for idiopathic pulmonary arterial
hypertension?

Choices:
1. 3 years
2. 10 years
3. 30 years
4. 50 years
Answer: 1 - 3 years
Explanations:
Primary pulmonary hypertension has an unknown etiology.
Associated with right ventricular hypertrophy.
The mean survival is less than four years from the time of diagnosis in
adults and one to two years in children.
Treatment is with vasodilators, but definitive treatment requires lung
transplantation.

Go to the next page if you knew the correct answer, or click the link image(s)
below to further research the concepts in this question (if desired).

Research Concepts:
Pulmonary Hypertension, Idiopathic

We update eBooks quarterly and Apps daily based on user feedback.


Please tap flag to report any questions that need improvement.
Question 295: What type of inheritance does cystic fibrosis exhibit?
Choices:
1. Autosomal dominant
2. Autosomal recessive
3. X-link dominant
4. X-link recessive
Answer: 2 - Autosomal recessive
Explanations:
Cystic fibrosis (CF) is an autosomal recessive disorder.
This means that for each pregnancy resulting from two healthy parents who
are CF carriers, a 25% chance of an infant with CF is expected. Also, a 50%
chance of an infant who is a carrier is expected, and a 25% chance of the
infant neither carrying nor being affected by the disease is expected.
Those of Northern European ancestry are the most commonly affected by
CF. In this population, one per 3,000 newborns is diagnosed annually,
typically before six months of age. CF occurs equally in females and males.
CF leads to acute and chronic lung infections, sinus infections, steatorrhea,
clubbed fingers and toes, and poor growth with signs and symptoms
typically appearing during infancy or early childhood. It is one of the most
lethal genetic diseases in Caucasians.

Go to the next page if you knew the correct answer, or click the link image(s)
below to further research the concepts in this question (if desired).

Research Concepts:
Cystic Fibrosis

We update eBooks quarterly and Apps daily based on user feedback.


Please tap flag to report any questions that need improvement.
Question 296: Which of the following statements about trapped lung is
true?

Choices:
1. It is best managed with pleurodesis
2. Inspiratory pleural pressure is excessively negative and fluid is usually
transudative
3. The pleura is inflamed and pleural fluid usually has a very low glucose level
4. Inspiratory pleural pressure is usually positive and fluid is usually exudative
Answer: 2 - Inspiratory pleural pressure is excessively negative and fluid is
usually transudative

Explanations:
Trapped lung is the term used for a lung that is unable to expand, generally
as a result of pleural thickening from a remote inflammatory process. The
trapped lung has a reduced compliance, resulting in the generation of a
markedly negative intrapleural pressure as the chest wall expands during
inspiration but the lung is unable to do so. Hydrostatic forces result in the
formation of pleural fluid in the affected pleural space, and that fluid is
usually transudative. Because the inflammatory process leading to pleural
scarring and thickening is usually a remote process the pleura is generally
fibrotic and not inflamed at the time trapped lung is diagnosed.
The pleural effusion that occurs in the setting of trapped lung essentially
fills the space that the underexpanded lung does not. A mediastinal shift is
not common.
Making the diagnosis of trapped lung is important chiefly to spare the
patient unnecessary diagnostic or therapeutic procedures. Many patients
with trapped lung are asymptomatic, and no intervention is needed. If
trapped lung causes sufficient restriction to impair the patient’s breathing
surgery should be considered. The appropriate surgery is decortication of
the lung, stripping away the thickened pleura that is impairing lung
expansion. There is no role for pleurodesis in the treatment of trapped lung.

Go to the next page if you knew the correct answer, or click the link image(s)
below to further research the concepts in this question (if desired).

Research Concepts:
Lung, Trapped

We update eBooks quarterly and Apps daily based on user feedback.


Please tap flag to report any questions that need improvement.
Question 297: Which of the following complications is not associated with
recurrent acute chest syndrome?

Choices:
1. Chronic hypoxia
2. Pulmonary hypertension
3. Emphysema
4. Cor pulmonale
Answer: 3 - Emphysema
Explanations:
Recurrent acute chest syndrome results in pulmonary injury with intimal
hyperplasia and fibrosis, with the development of chronic pulmonary
infiltrates, chest pain, and hypoxia, and the eventual development of
pulmonary hypertension and cor pulmonale.
Emphysema does not occur as a result of recurrent acute chest syndrome.
Patients can develop pulmonary fibrosis. There is an association between
acute chest syndrome and asthma.
Prevention of infections with immunizations and prophylactic antibiotics,
transfusion therapy, and hydroxyurea therapy have been shown to decrease
the frequency of recurrent acute chest syndrome.
Pulmonary function testing is recommended every 2 to 3 years for patients
with a history of acute chest syndrome to monitor for both restrictive and
obstructive lung disease.

Go to the next page if you knew the correct answer, or click the link image(s)
below to further research the concepts in this question (if desired).

Research Concepts:
Chest Syndrome, Acute

We update eBooks quarterly and Apps daily based on user feedback.


Please tap flag to report any questions that need improvement.
Question 298: Which of the following are features of high-altitude
pulmonary edema (HAPE)?

Choices:
1. HAPE typically occurs 1 to 4 days after rapid ascent to altitudes greater than
8000 feet
2. HAPE is more commonly seen in first-time climbers who are 30 to 40 years of
age
3. Warm weather and dehydration at high altitude are predisposing factors in
HAPE
4. Symptoms remain consistent upon returning to low altitude
Answer: 1 - HAPE typically occurs 1 to 4 days after rapid ascent to altitudes
greater than 8000 feet

Explanations:
High-altitude pulmonary edema (HAPE) is an acute process, which tends to
affect young people or previously acclimatized people ascending to high
altitudes after a short stay at low altitudes.
Additional predisposing factors include physical exertion in cold weather at
high altitude.
Symptoms include cyanosis, rales, cough with productive frothy sputum,
and dyspnea.
HAPE is the most fatal form of severe high-altitude illness. HAPE is a form
of noncardiogenic pulmonary edema that occurs secondary to hypoxia. If
left untreated, it can progress to dyspnea at rest, rales, cyanosis, and a
mortality rate of up to 50%. SpO2 is often 10% less than expected for
altitude, and the patient will often appear better than expected given their
level of hypoxemia and SpO2 value, which typically resides around 40 to
70%. If available, a chest x-ray may show patchy alveolar infiltrates with
normal sized mediastinum and heart. Ultrasound may show B-lines
consistent with pulmonary edema. ECG may show signs of right axis
deviation and ischemia. Rapid correction of clinical status and SpO2 with
supplemental oxygen, in a patient with infiltrates on chest x-ray, is
pathognomonic of HAPE. Even if available, labs are of limited utility, and
the clinician should always consider concomitant acute mountain sickness
and high-altitude cerebral edema (HACE).

Go to the next page if you knew the correct answer, or click the link image(s)
below to further research the concepts in this question (if desired).

Research Concepts:
Altitude Illness, Pulmonary Syndromes, High Altitude Pulmonary
Edema (HAPE)
We update eBooks quarterly and Apps daily based on user feedback.
Please tap flag to report any questions that need improvement.
Question 299: Which of the following is NOT true about Nocardiosis?
Choices:
1. Is an opportunistic infection
2. Mildly acid fast and confused with tuberculosis
3. Has subcutaneous abscess and draining sinuses that exude sulfur granules
4. Need high doses of penicillin for treatment
Answer: 4 - Need high doses of penicillin for treatment
Explanations:
Nocardiosis is resistant to all penicillin derivatives and requires
sulfonamides.

Go to the next page if you knew the correct answer, or click the link image(s)
below to further research the concepts in this question (if desired).

Research Concepts:
Nocardiosis

We update eBooks quarterly and Apps daily based on user feedback.


Please tap flag to report any questions that need improvement.
Question 300: Immunoprophylaxis against respiratory syncytial virus
infection is recommended for all except which of the following children?

Choices:
1. Premature infants
2. Children with hemodynamically significant congenital heart disease
3. Infants with bronchopulmonary dysplasia
4. HIV infected children
Answer: 4 - HIV infected children
Explanations:
The American Academy of Pediatrics (AAP) recommends that
immunoprophylaxis be considered for certain groups of children who are at
risk for severe respiratory syncytial virus infection.
These include infants and young children with bronchopulmonary
dysplasia, prematurity, and hemodynamically significant congenital heart
disease.
The recommendation does not specifically recommend immunoprophylaxis
for HIV infected children.
Palivizumab is dosed monthly for prophylaxis.

Go to the next page if you knew the correct answer, or click the link image(s)
below to further research the concepts in this question (if desired).

Research Concepts:
Palivizumab

We update eBooks quarterly and Apps daily based on user feedback.


Please tap flag to report any questions that need improvement.
Question 301: A patient has been in the intensive care unit for over a
month. He has had multiple problems and now has a fever and a diffuse
infiltrative process in both lungs. Sputum, urine, and blood cultures are all
negative and it is decided to perform a bronchoalveolar lavage. From which lobe
of the lung will you try and perform the lavage in such a case?

Choices:
1. Right upper
2. Right lower
3. Left upper
4. Right middle
Answer: 4 - Right middle
Explanations:
If a localized disease process predominates, usually identified on
radiological imaging such as a chest x-ray or computed tomography (CT) of
the chest, then the bronchoscope is guided to that specific region and
wedged into the subsegment with visualization of the distal airway ideally
in the center of the image.
If diffuse, the heterogeneous disease is present on imaging, the most
commonly preferred locations for lavage would be the right middle lobe or
lingula.
As bronchoscopy is usually performed with the patient lying supine, the
anteriorly projected location of these segments allows gravity to assist with
maximal bronchoalveolar lavage (BAL) return, hence we prefer these sites.
If these segments are not accessible, the superior or anterior segment of
either lower lobe may also be used. In theory, this process enables lavage of
up to 1 million alveoli.

Go to the next page if you knew the correct answer, or click the link image(s)
below to further research the concepts in this question (if desired).

Research Concepts:
Bronchoalveolar Lavage

We update eBooks quarterly and Apps daily based on user feedback.


Please tap flag to report any questions that need improvement.
Question 302: Select the correct statement about apnea of prematurity
(AOP).

Choices:
1. It resolves by 40 weeks postconceptual age in all preterms
2. Events terminate without intervention
3. It is due to immaturity of central respiratory control
4. Methyxanthines are routinely recommended
Answer: 3 - It is due to immaturity of central respiratory control
Explanations:
Immaturity of both the neurologic and respiratory systems are key factors in
the pathogenesis of apnea of prematurity. Events may be associated with
hypoxemia and reflex bradycardia and require resuscitation.
It generally resolves by about 36 -40 weeks postconceptual age.
It may persist beyond 40 weeks postconceptual age in the most premature
infants born between 24-28 weeks gestational age.
Apnea of prematurity is defined as periodic breathing with pathologic apnea
in a premature infant.

Go to the next page if you knew the correct answer, or click the link image(s)
below to further research the concepts in this question (if desired).

Research Concepts:
Apnea, Infant

We update eBooks quarterly and Apps daily based on user feedback.


Please tap flag to report any questions that need improvement.
Question 303: Which of the following statements is true of pediatric airway
management?

Choices:
1. The tracheal opening is more posterior and inferior than in adults
2. Immediate cricothyroidotomy is indicated in the management of epiglottitis
3. Cuffed tubes should be smaller than the uncuffed tubes
4. Suction the nose of the infant before the mouth
Answer: 3 - Cuffed tubes should be smaller than the uncuffed tubes
Explanations:
Bag-mask ventilation is appropriate while preparing to intubate. The
tracheal opening is located more anteriorly and superiorly than in adults.
Cuffed endotracheal tubes are preferable to decrease the air leak in
ventilated patients. Bedside ultrasound can be used to assess the appropriate
size of the endotracheal tube. Glideslope is a recent adjuvant in airway
management.
Needle cricothyroidotomy is preferable to surgical cricothyroidotomy in
children under 12 years of age. They have a smaller cricothyroid membrane
and the larynx is compliant, funnel-shaped, and rostral in position.
Hyperextension of the neck can kink the soft tracheal cartilage, so the
provider should mildly hyper-extend the child's head. A folded towel is
often required as a shoulder roll to achieve a neutral position of the neck
and open up the airway. A straight laryngoscope blade is preferable.
When using a bulb syringe to suction an infant, it is important to suction the
mouth before the nose to avoid aspiration. The first step is to depress the
bulb syringe and then place it in the mouth and nose. Infants are prone to
vagal stimulation, and suctioning can lead to bradycardia. Suction must not
last for more than 10 seconds.

Go to the next page if you knew the correct answer, or click the link image(s)
below to further research the concepts in this question (if desired).

Research Concepts:
Airway, Management

We update eBooks quarterly and Apps daily based on user feedback.


Please tap flag to report any questions that need improvement.
Question 304: What are the most common causes of bacterial pneumonia
in neonates?

Choices:
1. Group B Streptococcus and Listeria monocytogenes
2. Group B Streptococcus and Escherichia coli
3. Staphylococcus aureus and Escherichia coli
4. Staphylococcus aureus and Listeria monocytogenes
Answer: 2 - Group B Streptococcus and Escherichia coli
Explanations:
The incidence of group B streptococcus pneumonia has been dropping with
screening of mothers.
The most common organisms are still group B streptococcus and
Escherichia coli.
The next most common causes are coagulase negative Staph and Listeria in
the early neonatal period.
After four days of age, the differential needs to be expanded to include S.
aureus, Enterobacter, Pseudomonas, Serratia, Candida, Acinetobacter, and
anaerobes.

Go to the next page if you knew the correct answer, or click the link image(s)
below to further research the concepts in this question (if desired).

Research Concepts:
Pneumonia, Pediatric

We update eBooks quarterly and Apps daily based on user feedback.


Please tap flag to report any questions that need improvement.
Question 305: A patient has daily symptoms of asthma and needs to use a
short-acting beta-2 agonist inhaler daily. Now he has been experiencing twice
weekly exacerbations once during the day and once during the night. How is the
patient classified?

Choices:
1. Step 1: mild intermittent
2. Step 2: mild persistent
3. Step 3: moderate persistent
4. Step 4: severe persistent
Answer: 3 - Step 3: moderate persistent
Explanations:
Daily symptoms along with exacerbations affecting activity and nocturnal
symptoms that occur more than once per week are classified as step 3,
moderate persistent asthma.
Asthma affects approximately 15% to 20% of people in developed
countries and about 2% to 4% in less developed countries. It is significantly
more common in children.
Spirometry is the diagnostic method of choice and will show an obstructive
pattern that is partially or completely resolved by salbutamol. The Forced
Expiratory Volume per second (FEV1) and FEV1/ Forced Vital Capacity
(FVC) ratio is decreased in asthma.
Bronchodilators such as beta-2 agonists and muscarinic antagonists are the
mainstay of treatment. Inhaled and oral steroids are used adjunctively.

Go to the next page if you knew the correct answer, or click the link image(s)
below to further research the concepts in this question (if desired).

Research Concepts:
Asthma

We update eBooks quarterly and Apps daily based on user feedback.


Please tap flag to report any questions that need improvement.
Question 306: A homeless man receives a PPD test at a local free clinic
and returns 48hrs for have his skin test read. Which of the following would be
indicative of a positive PPD test?

Choices:
1. Induration of 5mm
2. Induration of 10 mm
3. Induration of 15 mm
4. Erythema is a stronger diagnostic measurement than the diameter of the
induration
Answer: 2 - Induration of 10 mm
Explanations:
Induration of 5mm or more is positive in a person who has deficient cell
mediated immunity.
Induration of 10 mm or more is positive in a person with high risk factors,
such as homeless persons, IV drug users or nursing home residents.
Induration of 15 mm or more is positive in persons who have no known risk
factors.
Induration, not simply erythema, must be measured in order to confidently
translate PPD results.

Go to the next page if you knew the correct answer, or click the link image(s)
below to further research the concepts in this question (if desired).

Research Concepts:
Tuberculosis, PPD Skin Test (Tuberculosis Skin Test)

We update eBooks quarterly and Apps daily based on user feedback.


Please tap flag to report any questions that need improvement.
Question 307: A 5-year-old female had an adenotonsillectomy for
obstructive sleep apnea. What condition places her at increased risk for residual
apnea and should be considered for a follow-up polysomnogram?

Choices:
1. Age less than 3 years
2. Marfan syndrome
3. Developmental delay
4. Down syndrome
Answer: 4 - Down syndrome
Explanations:
Children at risk for residual apnea following adenotonsillectomy should
have a follow-up evaluation with overnight polysomnogram.
Children with moderate to severe apnea and those with craniofacial
dysmorphology are examples of high-risk children that should be evaluated
postoperatively for residual apnea.
Severely obese children and those with syndromes associated with obesity
and hypotonia also are high-risk and should be evaluated postoperatively.
Prader Willi and Down syndromes are both characterized by obesity and
hypotonia.

Go to the next page if you knew the correct answer, or click the link image(s)
below to further research the concepts in this question (if desired).

Research Concepts:
Apnea, Snoring, Pediatric

We update eBooks quarterly and Apps daily based on user feedback.


Please tap flag to report any questions that need improvement.
Question 308: In which of the following West zones must the pulmonary
artery catheter be placed for optimum readings?

Choices:
1. Zone 1
2. Zone 2
3. Zone 3
4. Zone 4
Answer: 3 - Zone 3
Explanations:
In Zone III, the pulmonary artery pressure is greater than the pulmonary
venous pressure which is greater than the pulmonary alveolar pressure.

Go to the next page if you knew the correct answer, or click the link image(s)
below to further research the concepts in this question (if desired).

Research Concepts:
Catheterization, Pulmonary Artery

We update eBooks quarterly and Apps daily based on user feedback.


Please tap flag to report any questions that need improvement.
Question 309: Which of the following is associated with a conservative
fluid management strategy among patients with acute respiratory distress
syndrome?

Choices:
1. Lower mortality
2. Increased need for dialysis
3. Increase in ventilator-free days
4. Higher mortality
Answer: 3 - Increase in ventilator-free days
Explanations:
Patients with acute respiratory distress syndrome (ARDS) treated with a
conservative approach to fluid management receive a lower volume of
fluids and have less positive fluid balance than those who are not treated
conservatively.
This conservative approach to fluid management has been associated with
lower lung injury scores and more ventilator-free days when compared to a
more liberal approach to fluid management.
The conservative approach does not result in more organ failures, more
vasopressor use, or any difference in mortality.
The syndrome is associated with a high mortality rate between 20% and
50%. The mortality rate with ARDS varies based on severity, age, and the
presence of underlying medical conditions.

Go to the next page if you knew the correct answer, or click the link image(s)
below to further research the concepts in this question (if desired).

Research Concepts:
Acute Respiratory Distress Syndrome (ARDS)

We update eBooks quarterly and Apps daily based on user feedback.


Please tap flag to report any questions that need improvement.
Question 310: Which of the following findings would help differentiate
acute respiratory distress syndrome from cardiogenic pulmonary edema?

Choices:
1. Hypoxemia
2. Bilateral infiltrates
3. Decreased lung compliance
4. Pulmonary capillary wedge pressure of 16 mmHg
Answer: 4 - Pulmonary capillary wedge pressure of 16 mmHg
Explanations:
Cardiogenic pulmonary edema will have pulmonary capillary wedge
pressures greater than 18 mmHg.
Both conditions may have hypoxemia, bilateral infiltrates, and decreased
lung compliance.
Generally, radiographic findings of pulmonary edema affecting both lungs
and unrelated to heart failure suggest acute respiratory distress syndrome.
Ultrasound findings include anterior subpleural consolidations, absence or
reduction of lung sliding, “spared areas” of normal parenchyma, pleural line
abnormalities, and nonhomogeneous distribution of B-lines.

Go to the next page if you knew the correct answer, or click the link image(s)
below to further research the concepts in this question (if desired).

Research Concepts:
Acute Respiratory Distress Syndrome (ARDS)

We update eBooks quarterly and Apps daily based on user feedback.


Please tap flag to report any questions that need improvement.
Question 311: In what instance would a "high-lying" position be ideal for a
central venous femoral catheter?

Choices:
1. Central venous pressure monitoring
2. Emergency resuscitation
3. Administration of caustic medications
4. Frequent blood sampling
Answer: 1 - Central venous pressure monitoring
Explanations:
The ideal position for a "high-lying" central venous femoral catheter is
when the catheter tip is located at the junction of the inferior vena cava
(IVC) and right atrium of the heart.
The ideal position for a "low-lying" central venous femoral catheter is the
catheter tip being located within the IVC but below the renal veins.
The indication for line placement prompts the proceduralist to consider the
need for a "high-lying" and "low-lying" central venous femoral catheter.
When monitoring of central venous oxygen saturation and central venous
pressure is indicated, a "high-lying" position would be ideal as it would
provide more accurate data.

Go to the next page if you knew the correct answer, or click the link image(s)
below to further research the concepts in this question (if desired).

Research Concepts:
Central Venous Access, Femoral Vein

We update eBooks quarterly and Apps daily based on user feedback.


Please tap flag to report any questions that need improvement.
Question 312: What is the treatment for patients with complete bilateral
diaphragmatic paralysis?

Choices:
1. Theophylline
2. Nasal biPAP
3. Mechanical ventilation
4. 100% FIO2
Answer: 3 - Mechanical ventilation
Explanations:
Mechanical ventilation is the only way to treat bilateral diaphragmatic
paralysis.
Most patients retain carbon dioxide and require ventilation.
Tracheostomy with positive pressure ventilation is necessary for many
patients.
Often times, bilateral diaphragmatic paralysis is the result of progressing
amyotrophic lateral sclerosis (ALS). Patients can be treated initially with
BiPAP but may be offed intubation when that is no longer effective.

Go to the next page if you knew the correct answer, or click the link image(s)
below to further research the concepts in this question (if desired).

Research Concepts:
Diaphragmatic Paralysis

We update eBooks quarterly and Apps daily based on user feedback.


Please tap flag to report any questions that need improvement.
Question 313: Which of the following statements is false about lung
transplants?

Choices:
1. It is done for cystic fibrosis
2. Single lung transplant is required for patients with cystic fibrosis
3. Contraindicated in pulmonary hypertension
4. More economical than heart transplant
Answer: 2 - Single lung transplant is required for patients with cystic fibrosis
Explanations:
Patients with cystic fibrosis (CF) patients require a double lung transplant.
A single lung transplant in these patients will quickly lead to contamination
of the transplanted lung.
Patients with CF have a very high mortality when the FEV1 is 30 percent or
less. Other factors that are indicators of poor prognosis include weight loss,
hemoptysis, and frequent admissions.
Double lung transplantation is also reserved for patients with generalized
bronchiectasis, pulmonary artery hypertension and young patients with
chronic obstructive pulmonary disease.

Go to the next page if you knew the correct answer, or click the link image(s)
below to further research the concepts in this question (if desired).

Research Concepts:
Transplantation

We update eBooks quarterly and Apps daily based on user feedback.


Please tap flag to report any questions that need improvement.
Question 314: What test should be ordered to diagnose an African
American male who presents complaining of joint pain and cough who is found
to have a violet rash on his cheeks and nose and hilar adenopathy on chest x-ray?

Choices:
1. Transbronchial lung biopsy
2. Skin biopsy
3. Chest CT scan
4. Serum and urine calcium level
Answer: 1 - Transbronchial lung biopsy
Explanations:
Sarcoidosis is a multisystem inflammatory disease of unknown etiology
that usually presents with cough, fever, anorexia, arthralgias, and dyspnea
on exertion.
A biopsy is typically required for diagnosis and a transbronchial lung
biopsy has a high yield.
Hilar lymphadenopathy on chest x-ray is classic for sarcoidosis while other
physical manifestations include hepatosplenomegaly, erythema nodosum,
and granulomatous uveitis.
ACE levels are elevated in 60% at the time of diagnosis and hypercalcemia
and hypercalciuria may occur but are not diagnostic.

Go to the next page if you knew the correct answer, or click the link image(s)
below to further research the concepts in this question (if desired).

Research Concepts:
Sarcoidosis

We update eBooks quarterly and Apps daily based on user feedback.


Please tap flag to report any questions that need improvement.
Question 315: A patient with severe hypoxemic respiratory failure is on
pressure-controlled inverse ratio ventilation and develops diffuse subcutaneous
emphysema. Which of the following should be performed?

Choices:
1. Bronchoscopy to rule out endotracheal tube obstruction
2. Bilateral chest tubes
3. Cervical skin incision to release subcutaneous air
4. Increase inspiratory time on ventilator
Answer: 2 - Bilateral chest tubes
Explanations:
Inverse ratio ventilation can result in auto-positive end-expiratory pressure
and lead to tension pneumothorax.
One can decrease inspiratory time or increase expiratory time to allow the
excess air to escape.
If subcutaneous emphysema is present, it is safe to place bilateral chest
tubes.

Go to the next page if you knew the correct answer, or click the link image(s)
below to further research the concepts in this question (if desired).

Research Concepts:
Ventilation, Inverse Ratio

We update eBooks quarterly and Apps daily based on user feedback.


Please tap flag to report any questions that need improvement.
Question 316: How many apneic or hypopneic episodes per hour occur
during moderate sleep apnea?

Choices:
1. 1 to 5
2. 5 to 10
3. 10 to 15
4. 15 to 30
Answer: 4 - 15 to 30
Explanations:
The apnea-hypopnea index is the number of apneas and hypopneas divided
by the total sleep time.
Mild obstructive sleep apnea (OSA) has 5 to 15 episodes per hour,
moderate OSA has 15 to 30 episodes, and severe OSA has more than 30.
Weight loss and the use of continuous positive airway pressure (CPAP) are
the most effective treatments. The diligent adherence to nightly CPAP use
can result in near complete resolution of symptoms.
Patients should be counseled to avoid alcohol, benzodiazepines, opiates,
and some antidepressants which may worsen their condition.

Go to the next page if you knew the correct answer, or click the link image(s)
below to further research the concepts in this question (if desired).

Research Concepts:
Apnea, Obstructive Sleep Apnea

We update eBooks quarterly and Apps daily based on user feedback.


Please tap flag to report any questions that need improvement.
Question 317: Which of the following is not a criterion for extubation of
patients?

Choices:
1. Tidal volume greater than 5 to 7 ml/kg
2. Oxygen saturation greater than 92% on FIO2 of 40%
3. PCO2 less than 55
4. Respiration rate less than 16 to 18 breaths per minute
Answer: 3 - PCO2 less than 55
Explanations:
There are many criteria that must be met prior to the extubation of a patient.
Besides the blood gas results, the patient must be alert and cooperative.
The ideal parameters of extubation include a tidal volume of 5 to 7 ml/kg,
good oxygenation on 40% FIO2, and low respiration rate.
If the PCO2 is elevated, then the patient is not yet breathing well. Anyone
with a PCO2 over 45 must not be extubated. The rapid shallow breathing
index can be followed to decide who is ready for extubation. An index less
than 105 should suffice in a patient that satisfy all other criteria.

Go to the next page if you knew the correct answer, or click the link image(s)
below to further research the concepts in this question (if desired).

Research Concepts:
Ventilation, Ventilator Management

We update eBooks quarterly and Apps daily based on user feedback.


Please tap flag to report any questions that need improvement.
Question 318: A young afebrile child with a remote history of
staphylococcal pneumonia in the past presents to you with an x-ray revealing a
cluster of air spaces in the previous area of consolidation. Which is appropriate?

Choices:
1. Observation
2. CT guided drainage
3. Antibiotics
4. Chest tube
Answer: 1 - Observation
Explanations:
Pneumatoceles can develop after Staphylococcal infections.
They develop rapidly and fluctuate in size.
Occasionally, they can get infected, rupture, and produce an empyema.
Spontaneous remissions occur in most children. Surgical intervention or
chest tube is not required.

Go to the next page if you knew the correct answer, or click the link image(s)
below to further research the concepts in this question (if desired).

Research Concepts:
Pneumatocele

We update eBooks quarterly and Apps daily based on user feedback.


Please tap flag to report any questions that need improvement.
Question 319: Which disorder is associated with recurrent pulmonary
hemorrhages in children?

Choices:
1. Idiopathic pulmonary hemosiderosis
2. Granulomatosis with polyangiitis
3. Lung abscess
4. Chronic bronchitis
Answer: 1 - Idiopathic pulmonary hemosiderosis
Explanations:
Idiopathic pulmonary hemosiderosis is mostly seen in children aged 1 to 7
years.
It is characterized by recurrent, diffuse alveolar hemorrhage that can lead to
pulmonary hemosiderosis and fibrosis.
It only affects about 1 out every million.
Findings include dyspnea, hemoptysis, and perihilar or basilar alveolar
infiltrates on plain radiographs.

Go to the next page if you knew the correct answer, or click the link image(s)
below to further research the concepts in this question (if desired).

Research Concepts:
Idiopathic Pulmonary Hemosiderosis

We update eBooks quarterly and Apps daily based on user feedback.


Please tap flag to report any questions that need improvement.
Question 320: Bronchogenic cysts are usually located in what part of the
mediastinum?

Choices:
1. Anterior
2. Middle
3. Posterior
4. Inferior
Answer: 2 - Middle
Explanations:
Mediastinal bronchogenic cysts are located in the middle mediastinum.
They arise from the bronchial tree, which is in the middle mediastinum.
The posterior mediastinum contains neurogenic tumors. Lymphoma is often
found in the anterior mediastinum.
Bronchogenic cysts are congenital. They arise in the bronchial tree and
represent a form of bronchopulmonary foregut malformation. If they
enlarge, they can cause local compression. Bronchogenic cysts are the most
common of the foregut duplication cysts.
Bronchogenic cysts are rare. They can be found either in the mediastinum
or the lungs. Mediastinal bronchogenic cysts have an equal gender
incidence, but pulmonary bronchogenic cysts have a male predominance.

Go to the next page if you knew the correct answer, or click the link image(s)
below to further research the concepts in this question (if desired).

Research Concepts:
Cyst, Bronchogenic

We update eBooks quarterly and Apps daily based on user feedback.


Please tap flag to report any questions that need improvement.
Question 321: A previously healthy 6-year-old boy is brought to the
emergency department 12 hours after nearly drowning. He is moderately
dyspneic but not cyanotic. His blood pressure is 108/70 mmHg, and his pulse
rate is 112 beats per minute. There is no visible injury, and auscultation of the
chest reveals no abnormalities. What is the most important initial intervention?

Choices:
1. Chest x-ray
2. Arterial blood gas analysis
3. Bronchoscopy examination
4. Provide supplemental oxygen
Answer: 4 - Provide supplemental oxygen
Explanations:
Initial management of near-drowning should place emphasis on respiratory
status. Oxygen should be provided during the evaluation.
Intubation may be required if the blood gas reveals high carbon dioxide
(CO2), low pH, or low pO2. In alert patients, bi-level positive airway
pressure can be tried.
Intubated patients may require positive end-expiratory pressure to maintain
oxygenation. It may also help to open up the small airways.
A chest x-ray may be obtained after initial stabilization of the patient.

Go to the next page if you knew the correct answer, or click the link image(s)
below to further research the concepts in this question (if desired).

Research Concepts:
Drowning

We update eBooks quarterly and Apps daily based on user feedback.


Please tap flag to report any questions that need improvement.
Question 322: In an otherwise healthy individual who is suspected of
having tuberculosis, what induration of the tuberculin skin test is considered
significant?

Choices:
1. 5 millimeters
2. 10 millimeters
3. 15 millimeters
4. Any size

Photo:Contributed by Wikimedia Commons (PD-US HHS-CDC)


Answer: 3 - 15 millimeters
Explanations:
In an otherwise healthy individual with no risk factors for tuberculosis, an
induration of 15 millimeters is considered significant.
Lesions 5 millimeters or greater are significant in individuals with HIV,
organ transplant patients, and those who have a fibrotic lesion on an x-ray.
Lesions larger than 10 millimeters are significant in recent immigrants,
children younger than 4 years of age, residents of long-term facilities, and
those with medical disorders such as diabetes mellitus or cancer.
Causes of a false positive test include prior BCG vaccine administration or
infection by nontuberculous mycobacteria. False positives can also be
caused by poor technique and scratches of the injected area.

Go to the next page if you knew the correct answer, or click the link image(s)
below to further research the concepts in this question (if desired).

Research Concepts:
Tuberculosis

We update eBooks quarterly and Apps daily based on user feedback.


Please tap flag to report any questions that need improvement.
Question 323: In which of the following fungal infections is surgery
preferred over medical management?

Choices:
1. Coccidioidomycosis
2. Blastomycosis
3. Mucormycosis
4. Histoplasmosis
Answer: 3 - Mucormycosis
Explanations:
Mucormycosis requires extended surgical removal.
Mucormycosis is rapidly invasive in diabetics and immunosuppressed
patients.
Aggressive management with amphotericin coverage is required.
It carries a high mortality despite prompt surgical intervention.

Go to the next page if you knew the correct answer, or click the link image(s)
below to further research the concepts in this question (if desired).

Research Concepts:
Mucormycosis

We update eBooks quarterly and Apps daily based on user feedback.


Please tap flag to report any questions that need improvement.
Question 324: For which of the following is inhaled flunisolide FDA
approved to treat?

Choices:
1. Chronic obstructive pulmonary disease
2. Status asthmaticus
3. Prophylaxis and maintenance treatment of asthma
4. Nicotine withdrawal symptoms
Answer: 3 - Prophylaxis and maintenance treatment of asthma
Explanations:
Inhaled flunisolide is FDA approved for the prophylaxis and maintenance
of asthma.
The use of corticosteroid inhalers helps to reduce or eliminate the need for
oral corticosteroids in steroid-dependent asthma patients.
Inhaled corticosteroids are not indicated for the treatment of an acute
asthma exacerbation.
Supplemental corticosteroids may be needed during stress or severe asthma
attacks.

Go to the next page if you knew the correct answer, or click the link image(s)
below to further research the concepts in this question (if desired).

Research Concepts:
Corticosteroids, Inhaled

We update eBooks quarterly and Apps daily based on user feedback.


Please tap flag to report any questions that need improvement.
Question 325: For diaphragmatic pacing, which of the following
statements is not true?

Choices:
1. Patients with hiccups may need diaphragmatic pacing
2. Central alveolar hypoventilation may require pacing
3. Quadriplegia above C5 may require pacing
4. Children do better with bilateral pacing
Answer: 3 - Quadriplegia above C5 may require pacing
Explanations:
Diaphragmatic pacing only works if the quadriplegia is above C2. The
phrenic nerve has to be intact for pacing.
The origin of the phrenic nerve is at C3, 4, and 5.
Current indications for diaphragmatic pacing are central alveolar
hypoventilation and high quadriplegia. Contraindications to pacing include
injury to C3-C5 motor neurons, phrenic nerve injury, myopathies, severe
parenchymal injury, and thoracic cage deformity.
The pacing electrodes are placed in the thorax (because of accessory
phrenic nerve in the neck) and pacing is started 2 weeks later.

Go to the next page if you knew the correct answer, or click the link image(s)
below to further research the concepts in this question (if desired).

Research Concepts:
Diaphragmatic Pacing

We update eBooks quarterly and Apps daily based on user feedback.


Please tap flag to report any questions that need improvement.
Section 6

Question 326: For a patient who has anaphylaxis from penicillin, what is
the best choice to cover gram-positive organisms?

Choices:
1. Sulfonamide
2. Cephalexin
3. Erythromycin
4. Gentamycin
Answer: 3 - Erythromycin
Explanations:
Macrolide antibiotics are good substitutes for patients who are allergic to
penicillin.

Go to the next page if you knew the correct answer, or click the link image(s)
below to further research the concepts in this question (if desired).

Research Concepts:
Bacteria, Gram Positive

We update eBooks quarterly and Apps daily based on user feedback.


Please tap flag to report any questions that need improvement.
Question 327: What is the most important marker for prognosis in patients
with sarcoidosis?

Choices:
1. Levels of angiotensin converting enzyme
2. Levels of calcium
3. Response to steroids
4. The initial chest x-ray
Answer: 4 - The initial chest x-ray
Explanations:
The initial chest x-ray is probably the most important marker for prognosis
in patients with sarcoidosis.
Even though corticosteroids are widely used to treat sarcoidosis, their
effectiveness is not clear.

Go to the next page if you knew the correct answer, or click the link image(s)
below to further research the concepts in this question (if desired).

Research Concepts:
Sarcoidosis

We update eBooks quarterly and Apps daily based on user feedback.


Please tap flag to report any questions that need improvement.
Question 328: A full-term newborn develops respiratory distress
immediately after birth. Prenatal sonogram was unremarkable. There are
substernal retractions and a scaphoid abdomen. Intubation is needed. Arterial
blood gas results are pH 7.23, P02 60 and PCO2 52. Radiograph shows air fluid
levels in the thoracic cavity. What is the correct statement?

Choices:
1. ARDS due to traumatic delivery
2. Immediate exploration and repair of the defect is required
3. Size of defect correlates with severity of symptoms
4. Any abdominal organ may be involved in the pathology
Answer: 4 - Any abdominal organ may be involved in the pathology
Explanations:
This neonate has a congenital diaphragmatic hernia.
Embryologic diaphragm fusion usually occurs between the eighth and
twelfth week of development.
Sonogram may not show the condition if the abdominal contents had
returned during the study. Any abdominal organ can be involved.
The condition can cause pulmonary hypoplasia, pulmonary hypertension,
and neonatal respiratory distress. Initial treatment is medical with
extracorporeal membrane oxygenation (ECMO).

Go to the next page if you knew the correct answer, or click the link image(s)
below to further research the concepts in this question (if desired).

Research Concepts:
Hernia, Diaphragmatic, Congenital

We update eBooks quarterly and Apps daily based on user feedback.


Please tap flag to report any questions that need improvement.
Question 329: A 2-year-old child is diagnosed with pertussis. Which of the
following about prophylaxis of this disease is INCORRECT?

Choices:
1. It should be given to the patient's entire family
2. It should be given to all members of the family who have not received DTaP
or DTaP within the last 2 years
3. It can be with erythromycin, clarithromycin, or azithromycin
4. It should be given to the caretaker at daycare
Answer: 2 - It should be given to all members of the family who have not
received DTaP or DTaP within the last 2 years

Explanations:
Regardless of immunization status, all close contacts should be treated.
Choices include erythromycin for 14 days, azithromycin for 5 days, or
clarithromycin for 7 days.
Only azithromycin should be used for children less than 1 month.
Very rare complications of DTaP vaccination are long term seizures, coma,
and permanent brain damage.

Go to the next page if you knew the correct answer, or click the link image(s)
below to further research the concepts in this question (if desired).

Research Concepts:
Pertussis, Prophylaxis

We update eBooks quarterly and Apps daily based on user feedback.


Please tap flag to report any questions that need improvement.
Question 330: Which is the least likely cause of lung atelectasis?
Choices:
1. Tumors
2. Aspiration of a foreign body
3. Thick secretions
4. CPAP
Answer: 4 - CPAP
Explanations:
Atelectasis is collapse of a portion or all of a lung.
Tumors, foreign bodies and thick mucus can cause complete obstruction
leading to atelectasis distally.
Bronchoscopy, CXR and CT scan can delineate causes of atelectasis.
Pneumonia is a frequent complication if obstruction is not relieved in a
timely manner.

Go to the next page if you knew the correct answer, or click the link image(s)
below to further research the concepts in this question (if desired).

Research Concepts:
Atelectasis

We update eBooks quarterly and Apps daily based on user feedback.


Please tap flag to report any questions that need improvement.
Question 331: An African American female comes to the emergency room
with facial weakness. The right side started yesterday and the left side today. She
has had no constitutional symptoms or rashes. She lives in Chicago and has no
pets. She is employed in an office and has no hobbies with exposures to
chemicals. She has hypertension and is on hydrochlorothiazide. Blood pressure
is 130/85 mm Hg, pulse is 85, respirations 16, and she is afebrile. The right side
of the face is paralyzed, but she can raise her eyebrow on the left. Chest
radiograph has bilateral hilar lymphadenopathy. MRI with gadolinium show
enhancement of the seventh cranial nerve and meninges bilaterally. Lumbar
puncture yields CSF with an opening pressure of 12 cm H2O, with no red cells,
21 white cells with 82% lymphocytes and 28 percent neutrophils, protein 72
mg/dL, and glucose 62 mg/dL. Gram stain is negative. Select the most likely
diagnosis.

Choices:
1. Multiple sclerosis
2. Sarcoidosis
3. Lyme disease
4. Tuberculosis
Answer: 2 - Sarcoidosis
Explanations:
5 to 10 percent of patients with sarcoidosis have neurologic manifestations.
The presenting complaint of sarcoidosis is neurologic symptoms half the
time.
The most common neurologic findings are cranial nerve involvement,
basilar meningitis, myelopathy, and anterior hypothalamic disease.
Treatment initially would be prednisone 0.5 mg/kg daily.

Go to the next page if you knew the correct answer, or click the link image(s)
below to further research the concepts in this question (if desired).

Research Concepts:
Sarcoidosis

We update eBooks quarterly and Apps daily based on user feedback.


Please tap flag to report any questions that need improvement.
Question 332: A 5-year-old presents with mild persistent asthma.
According to the most recent the Expert Panel Report 3 guidelines, which of the
following therapies is preferred?

Choices:
1. Theophylline
2. Low dose inhaled corticosteroid
3. Leukotriene receptor antagonist
4. Sodium cromolyn
Answer: 2 - Low dose inhaled corticosteroid
Explanations:
Theophylline is considered an alternate option for children with mild
persistent asthma according to the Expert Panel Report 3 guidelines.
Low dose inhaled corticosteroids are preferred for children with mild
persistent asthma in this age group.
A leukotriene receptor antagonist such as montelukast is considered an
alternative option for children with mild persistent asthma.
Sodium cromolyn continues to be listed as an alternate option for mild
persistent asthma in this age group, although it is no longer used in this
setting.

Go to the next page if you knew the correct answer, or click the link image(s)
below to further research the concepts in this question (if desired).

Research Concepts:
Asthma, Pediatric

We update eBooks quarterly and Apps daily based on user feedback.


Please tap flag to report any questions that need improvement.
Question 333: A 3-year-old boy is brought to the emergency department
with a near drowning episode. He was revived at the scene and has stable vital
signs. He does appear dyspneic and has a respiration rate of 28 breaths per
minute. Which of the following tests should be obtained next?

Choices:
1. Chest x-ray
2. Arterial blood gas
3. Bronchoscopy
4. Electrocardiogram
Answer: 2 - Arterial blood gas
Explanations:
Near drowning patients may develop signs of acute respiratory distress
syndrome very rapidly.
If the patient is stable, an arterial blood gas should be obtained.
If the patient is unstable, he should be intubated first. A chest x-ray and a
blood gas should follow.
A bronchoscopic exam is not part of the routine management of these
patients.

Go to the next page if you knew the correct answer, or click the link image(s)
below to further research the concepts in this question (if desired).

Research Concepts:
Drowning

We update eBooks quarterly and Apps daily based on user feedback.


Please tap flag to report any questions that need improvement.
Question 334: What is the major long-term complication after the
performance of mediastinal tracheostomy?

Choices:
1. Hemorrhage
2. Injury to recurrent nerve
3. Pneumothorax
4. Mediastinitis
Answer: 1 - Hemorrhage
Explanations:
Bleeding from the innominate artery is a major concern after mediastinal
tracheostomy.
Mediastinal tracheostomies should be deemed to be permanent.
These tracheostomies are also difficult to maintain.
The quality of life after a mediastinal tracheostomy is poor.

Go to the next page if you knew the correct answer, or click the link image(s)
below to further research the concepts in this question (if desired).

Research Concepts:
Mediastinal Tracheostomy

We update eBooks quarterly and Apps daily based on user feedback.


Please tap flag to report any questions that need improvement.
Question 335: Which of the following can cause atelectasis?
Choices:
1. Tumor
2. Mucus plug
3. Foreign body
4. All of the above
Answer: 4 - All of the above
Explanations:
Obstructive atelectasis can occur at any level of the trachea or bronchus.
Typical causes of atelectasis include tumor, foreign body, or mucus plug.
Direct signs of atelectasis include opacification of a collapsed lobe and
fissural displacement.
Indirect signs of atelectasis include hilar displacement, ipsilateral
mediastinal shift, ipsilateral volume loss, ipsilateral diaphragm elevation,
adjacent rib crowding, and compensatory hyperlucency of the
nonatelectatic lung.

Go to the next page if you knew the correct answer, or click the link image(s)
below to further research the concepts in this question (if desired).

Research Concepts:
Atelectasis

We update eBooks quarterly and Apps daily based on user feedback.


Please tap flag to report any questions that need improvement.
Question 336: What percent of sarcoidosis is associated with bilateral hilar
adenopathy?

Choices:
1. 10 percent
2. 40 percent
3. 70 percent
4. 90 percent
Answer: 4 - 90 percent
Explanations:
The hallmark of sarcoidosis is noncaseating granulomas.
Bilateral hilar adenopathy is present in 90 percent of patients.

Go to the next page if you knew the correct answer, or click the link image(s)
below to further research the concepts in this question (if desired).

Research Concepts:
Sarcoidosis

We update eBooks quarterly and Apps daily based on user feedback.


Please tap flag to report any questions that need improvement.
Question 337: Which of the following can be used for a simple, quick, and
reliable antigen test of bodily fluids that will detect Legionella pneumophila
serogroup 1?

Choices:
1. Serum
2. Urine
3. Sputum
4. Saliva
Answer: 2 - Urine
Explanations:
Legionella antigen of serogroup 1 may be detectable in the urine early in
the course of the clinical illness.
Serology may take as many as 8 weeks to demonstrate a diagnostic increase
in antibody titre.

Go to the next page if you knew the correct answer, or click the link image(s)
below to further research the concepts in this question (if desired).

Research Concepts:
Legionnaires' Disease (Legionella Infection)

We update eBooks quarterly and Apps daily based on user feedback.


Please tap flag to report any questions that need improvement.
Question 338: Which of the following is the most common extrapulmonary
site of involvement of Rhodococcus equi?

Choices:
1. Bone
2. Brain
3. Liver
4. Kidney
Answer: 2 - Brain
Explanations:
After the lungs, the brain and skin are the most common sites that are
involved in Rhodococcus equi infection.
Extrapulmonary involvement without lung disease can occur in up to 25%
of cases.
Bone and renal involvement have been reported.
Rhodococcus equi causes zoonotic infections. It is found in grazing
animals. Most humans are infected from horses.

Go to the next page if you knew the correct answer, or click the link image(s)
below to further research the concepts in this question (if desired).

Research Concepts:
Rhodococcus Equi

We update eBooks quarterly and Apps daily based on user feedback.


Please tap flag to report any questions that need improvement.
Question 339: Which of the following can cause tension pneumothorax?
Choices:
1. Flail chest
2. Cardiac tamponade
3. Clamping of a chest tube
4. All of the above
Answer: 3 - Clamping of a chest tube
Explanations:
Clamping of a chest tube can lead to tension pneumothorax.
It is caused by the buildup of air or fluid in the pleural space leading.
A chest tube should never be clamped more than a few seconds.
Flail chest and cardiac tamponade do not cause tension pneumothorax.

Go to the next page if you knew the correct answer, or click the link image(s)
below to further research the concepts in this question (if desired).

Research Concepts:
Pneumothorax, Tension And Traumatic

We update eBooks quarterly and Apps daily based on user feedback.


Please tap flag to report any questions that need improvement.
Question 340: A patient is being started on an inhaled corticosteroid to
improve management of her asthma. Which counseling point should she be
given regarding her new medication?

Choices:
1. She can expect some fluid retention and weight gain with the use of the
steroid
2. Expect some diarrhea as she starts the drug, but this will normalize in about a
week
3. She should expect it to take a couple of weeks for the steroid to make a
noticeable difference
4. Take the drug at night because it will make her very drowsy
Answer: 3 - She should expect it to take a couple of weeks for the steroid to
make a noticeable difference

Explanations:
It is important to let patients know that inhaled corticosteroids will not have
the immediate effect that they might be used to with their beta-adrenergic
inhaler.
If this expectation is not set, the patient may mistakenly feel the medication
is not working which could lead to an adherence issue.
Given adequate time, inhaled corticosteroids are very helpful in long-term
asthma control.
Inhaled corticosteroids exhibit very few of the systemic side-effects of oral
corticosteroids such as fluid retention, weight gain, and hyperglycemia
because they have minimal systemic absorption.

Go to the next page if you knew the correct answer, or click the link image(s)
below to further research the concepts in this question (if desired).

Research Concepts:
Corticosteroids, Inhaled

We update eBooks quarterly and Apps daily based on user feedback.


Please tap flag to report any questions that need improvement.
Question 341: In comparison to the vocal cords, where is the laryngeal
mask airway usually placed?

Choices:
1. Superior
2. Inferior
3. Adjacent
4. Flush
Answer: 1 - Superior
Explanations:
The laryngeal mask airway (LMA) is secured superior to the vocal cords.
An LMA or laryngeal mask is a medical device that keeps the airway open
during anesthesia or unconsciousness. It is a supraglottic airway.
An LMA is composed of an airway tube that connects to an elliptical mask.
A cuff is inserted through the mouth and down the trachea where it forms
an airtight seal on top of the glottis. This differs from tracheal tubes which
pass through the glottis.
An LMA is commonly used to channel oxygen or anesthesia gas to a
patient's lungs during surgery and in the pre-hospital setting for
unconscious patients.

Go to the next page if you knew the correct answer, or click the link image(s)
below to further research the concepts in this question (if desired).

Research Concepts:
Airway, Laryngeal Mask

We update eBooks quarterly and Apps daily based on user feedback.


Please tap flag to report any questions that need improvement.
Question 342: Which of the following is not a long-term consequence of
obstructive sleep apnea in children?

Choices:
1. Cor pulmonale
2. Obesity
3. Growth retardation
4. Neurocognitive deficits
Answer: 2 - Obesity
Explanations:
Obstructive sleep apnea (OSA) commonly is associated with obesity but
does not cause weight gain. In fact, chronic hypoxemia caused by OSA
leads to increased work of breathing that increases the metabolic needs in
the face of lower caloric intake. This results in failure to thrive and grow.
OSA produces intermittent nocturnal hypoxia to the growing brain which
causes sleep fragmentation. This is attributed to the causation of
neurocognitive deficits and behavior problems including ADHD in
children.
OSA also causes pulmonary hypertension that initially results in
compensatory right ventricular hypertrophy, subsequently ventricular strain,
and eventually cor pulmonale.
Early treatment of OSA can reduce several of the complications, especially
those related to behavior and performance improvement in neurocognitive
skills.

Go to the next page if you knew the correct answer, or click the link image(s)
below to further research the concepts in this question (if desired).

Research Concepts:
Apnea, Children

We update eBooks quarterly and Apps daily based on user feedback.


Please tap flag to report any questions that need improvement.
Question 343: Which of the following mycobacteria has the highest
incidence of resistance in the United States?

Choices:
1. Mycobacterium tuberculosis
2. Mycobacterium kansasii
3. Mycobacterium avium intracellular (MAI)
4. Mycobacterium bovis
Answer: 3 - Mycobacterium avium intracellular (MAI)
Explanations:
Mycobacterium avium intracellular (MAI) has the highest incidence of
resistance.
Mycobacterium avium intracellular has a lipid-rich, hydrophobic cell wall
which is substantially thicker than most other bacteria.
The thickness and composition of the cell wall render mycobacteria
impermeable to hydrophilic nutrients and resistant to heavy metals,
disinfectants, and antibiotics.
Pulmonary disease due to MAI is becoming more common in the clinical
setting.

Go to the next page if you knew the correct answer, or click the link image(s)
below to further research the concepts in this question (if desired).

Research Concepts:
Mycobacterium Avium Intracellulare

We update eBooks quarterly and Apps daily based on user feedback.


Please tap flag to report any questions that need improvement.
Question 344: Which of the following is not true of tropical pulmonary
eosinophilia?

Choices:
1. It can present with wheezing and cough
2. The spleen is usually enlarged
3. The disorder is often caused by Mycobacterium tuberculosis
4. The disorder may present with enlarged neck nodes
Answer: 3 - The disorder is often caused by Mycobacterium tuberculosis
Explanations:
Tropical pulmonary eosinophilia is characterized by cough, asthma, and an
enlarged spleen.
The disorder presents with severe cough and weight loss.
Adenopathy is common and eosinophils are increased in the blood.

Go to the next page if you knew the correct answer, or click the link image(s)
below to further research the concepts in this question (if desired).

Research Concepts:
Tropical Pulmonary Eosinophilia

We update eBooks quarterly and Apps daily based on user feedback.


Please tap flag to report any questions that need improvement.
Question 345: In distinguishing benign and malignant pleural thickening,
which of the following findings does not favor malignancy?

Choices:
1. Circumferential thickening
2. Thickening greater than 1 cm
3. Involvement of the costophrenic sulcus
4. Nodularity
Answer: 3 - Involvement of the costophrenic sulcus
Explanations:
Pleural thickening involving the costophrenic sulcus is not suggestive of
malignancy, but pleural thickening of the mediastinal pleura is suggestive of
malignancy.
Nonmalignant causes of pleural thickening include prior infection like
empyema or tuberculosis, prior hemothorax, pleurodesis, asbestos
exposure, and prior radiation therapy.
Malignant causes of pleural thickening include metastases to pleura from
pulmonary and other cancers and primary pleural tumors like malignant
mesothelioma.
Pleural plaques are seen most commonly with a prior history of asbestos
exposure. These are focal areas of thickening and may appear calcified.
Asbestos-related pleural plaques are benign.

Go to the next page if you knew the correct answer, or click the link image(s)
below to further research the concepts in this question (if desired).

Research Concepts:
Effusion, Pleural

We update eBooks quarterly and Apps daily based on user feedback.


Please tap flag to report any questions that need improvement.
Question 346: A newborn who develops aspiration pneumonia should be
evaluated for which of the following conditions?

Choices:
1. Tracheoesophageal fistula
2. Hypothyroidism
3. Cystic fibrosis
4. Human immunodeficiency virus
Answer: 1 - Tracheoesophageal fistula
Explanations:
Tracheoesophageal fistula is a congenital defect seen in newborns. The
incidence is 1 in 1,500 to 3,000 live births. Boys and girls are equally
affected. The condition is commonly associated with esophageal atresia.
Cases have been associated with Down syndrome and trisomy 18.
Symptoms include excessive secretions with coughing and aspiration after
feedings. Complications include the development of cyanosis and aspiration
pneumonia. Diagnosis can be established by the inability to pass a red
rubber catheter or nasogastric tube into the stomach.
Frontal and lateral radiographs confirm the level of obstruction. If gas is
noted below the diaphragm, then an associated fistula is present. If gas is
not noted, the patient most likely is affected by esophageal atresia alone.
Care must be taken to avoid aspiration of dye during diagnostic tests.
Treatment involves withholding oral feedings and providing surgical
correction. Prognosis is generally good; however, some patients may
develop incompetence of the lower esophageal sphincter with chronic
reflux symptoms.

Go to the next page if you knew the correct answer, or click the link image(s)
below to further research the concepts in this question (if desired).

Research Concepts:
Fistula, Tracheoesophageal

We update eBooks quarterly and Apps daily based on user feedback.


Please tap flag to report any questions that need improvement.
Question 347: When is a double lumen tube ideally used?
Choices:
1. Pneumonia
2. Foreign object ingestion
3. Lung carcinoma
4. Aspiration
Answer: 4 - Aspiration
Explanations:
In order to segregate the lungs, double lumen tubes are used when there is
aspiration.

Go to the next page if you knew the correct answer, or click the link image(s)
below to further research the concepts in this question (if desired).

Research Concepts:
Double Lumen Endobronchial Tubes

We update eBooks quarterly and Apps daily based on user feedback.


Please tap flag to report any questions that need improvement.
Question 348: A patient who suffered a multiorgan injury is intubated in
the intensive care unit. His ventilator settings rate is 14, FiO2 60%, total volume
450ml, and positive end-expiratory pressure of 12. At night, the nurse notices a
significant amount of subcutaneous emphysema and elevated peak pressures.
What should be the next step in the management of this patient?

Choices:
1. Blood gas
2. CT scan
3. Place chest tubes
4. Bronchoscopy
Answer: 3 - Place chest tubes
Explanations:
In any ventilated patient with a high positive end-expiratory pressure
(PEEP), barotrauma can often present with a pneumothorax. Sometimes it
may present as subcutaneous emphysema.
Many patients on the ventilator with high PEEP have prophylactic chest
tubes inserted. PEEP can also cause a decline in cardiac output.
In such a scenario the placement of chest tubes can be life saving, as
hemodynamic compromise can occur anytime. No time should be wasted
obtaining chest x-rays.
In rare cases, the endotracheal tube can cause a posterior tear in the trachea
and cause subcutaneous emphysema.

Go to the next page if you knew the correct answer, or click the link image(s)
below to further research the concepts in this question (if desired).

Research Concepts:
Pneumothorax, Iatrogenic, Spontaneous

We update eBooks quarterly and Apps daily based on user feedback.


Please tap flag to report any questions that need improvement.
Question 349: Which is NOT a common cause of superior vena cava
syndrome?

Choices:
1. Bronchogenic cancer
2. Cervical rib
3. Mediastinal fibrosis
4. Histoplasmosis
Answer: 2 - Cervical rib
Explanations:
Malignant mediastinal tumors cause more than 80% of cases of superior
vena cava syndrome.
75 to 80% are bronchogenic carcinomas, most commonly small cell. 10 to
15% are due to non-Hodgkin lymphoma.
Cervical rib is not a known cause.
Benign causes of SVC are mediastinal fibrosis, benign tumors such as a
retrosternal thyroid, and infections such as histoplasmosis.

Go to the next page if you knew the correct answer, or click the link image(s)
below to further research the concepts in this question (if desired).

Research Concepts:
Superior Vena Cava Syndrome

We update eBooks quarterly and Apps daily based on user feedback.


Please tap flag to report any questions that need improvement.
Question 350: Which of the following is not a criterion used to assess
readiness for extubation among patients on mechanical ventilation?

Choices:
1. Mean inspiratory pressure greater than 20
2. Tidal volume greater than 5 cc/kg
3. Respiratory rate greater than 6 and less than 22
4. Forced expiratory volume (FEV1) greater than 1 liter
Answer: 4 - Forced expiratory volume (FEV1) greater than 1 liter
Explanations:
FEV is not a criterion for extubation.
Ventilator weaning and extubating are two distinct processes. Predicting
whether a patient is ready for extubations is based upon many different
physiologic variables. No single parameter can accurately predict which
patients will resume spontaneous breathing.
About 10% to 15% of patients will fail extubation in intensive care unit.
The approach to extubation is first to determine if the disease process has
resolved to the point that the patient can survive extubated. Candidates for
extubation are identified by a spontaneous breathing trial.
A spontaneous breathing trial involves the following steps.(1) It should be
conducted while the patient is still connected to the ventilator circuit. (2)
When using the ventilator a PS of 5 to 7 cm H2O and 1 to 5 cm H20 PEEP
will overcome increased work of breathing through the circuit. (3) If still on
the ventilator the patient should have "minimal ventilator settings.” (4) The
initial trial should last 30 to 120 minutes. (5) If it is not clear the patient has
not passed 120 minutes, the spontaneous breathing trial should be
abandoned. The shorter the period the patient was intubated, the shorter the
spontaneous breathing trial. Eighty percent of patients who tolerate this
time may be removed from the ventilator.

Go to the next page if you knew the correct answer, or click the link image(s)
below to further research the concepts in this question (if desired).

Research Concepts:
Ventilation, Ventilator Management

We update eBooks quarterly and Apps daily based on user feedback.


Please tap flag to report any questions that need improvement.
Question 351: Which of the following is not a use for fiberoptic
bronchoscopy?

Choices:
1. Unexplained hemoptysis
2. Positive sputum cytology
3. Assessment of airway patency
4. Removal of foreign bodies
Answer: 4 - Removal of foreign bodies
Explanations:
Removal of foreign bodies is done with rigid bronchoscopy.
A positive sputum cytology is an indication for the diagnostic use of
bronchoscopy.
Unexplained hemoptysis and assessment of airway patency are also
diagnostic uses of bronchoscopy.

Go to the next page if you knew the correct answer, or click the link image(s)
below to further research the concepts in this question (if desired).

Research Concepts:
Airway, Foreign Bodies

We update eBooks quarterly and Apps daily based on user feedback.


Please tap flag to report any questions that need improvement.
Question 352: A 6-week-old infant is brought to the emergency department
by a distraught mother after she noticed that her child stopped breathing and
turned blue. The infant resumed breathing after she started shaking him which
resulted in a big cry. What is the most likely cause of this event?

Choices:
1. Idiopathic
2. Cyanotic heart disease
3. Gastroesophageal reflux
4. Breath-holding spell
Answer: 1 - Idiopathic
Explanations:
The infant in this vignette developed an episode of apnea. No underlying
cause is identified in up to 50% of children presenting with apnea.
Gastroesophageal reflux (GER) is commonly implicated with apnea in
infants, but causation has never been established. A 2014 systematic review
could identify only one study that showed a temporal relationship between
reflux and apnea; it concluded that there is insufficient evidence to attribute
GER as a cause.
Some infants may have poor coordination of sucking and breathing, and the
aspiration of milk induces an exaggerated laryngeal chemical reflex that
depresses the respiratory center and causes apnea. Aspiration occurs more
frequently among children with neurological problems that cause
swallowing dysfunction
Duct dependent congenital heart lesions can present with apneic events with
associated cyanosis around this age but are far less common than
gastroesophageal reflux. Their cyanosis is exacerbated upon crying rather
than improved. Breath-holding spells occur in older children as part of their
temper-tantrums and methemoglobinemia produces sustained bluish
discoloration of the infant.
A thorough history and complete physical examination are needed to
exclude underlying causes of apnea before attributing an idiopathic cause.
History should particularly focus on the characteristics of the event and
must include pre-event, event, and post-event details in addition to
perinatal, family and social history, and the relationship of the event to
feeding.

Go to the next page if you knew the correct answer, or click the link image(s)
below to further research the concepts in this question (if desired).

Research Concepts:
Apnea, Infant
We update eBooks quarterly and Apps daily based on user feedback.
Please tap flag to report any questions that need improvement.
Question 353: Which of the following is NOT a feature of an extralobar
sequestration?

Choices:
1. May communicate with the gastrointestinal tract
2. May be associated with a diaphragmatic hernia
3. Has major bronchial communication
4. Usually presents in childhood
Answer: 3 - Has major bronchial communication
Explanations:
The majority of sequestrations have no connections with the bronchus.

Go to the next page if you knew the correct answer, or click the link image(s)
below to further research the concepts in this question (if desired).

Research Concepts:
Pulmonary Sequestration

We update eBooks quarterly and Apps daily based on user feedback.


Please tap flag to report any questions that need improvement.
Question 354: Which of the following is not a part of Wells Criteria for
predicting a pulmonary embolism?

Choices:
1. Heart rate >100bpm
2. Recent surgery
3. Hemoptysis
4. Tachypnea
Answer: 4 - Tachypnea
Explanations:
The Wells Criteria was introduced to help identify patients at risk for a
pulmonary embolus.
The Wells Criteria include a prior history of PE, HR > 100, recent surgery,
clinical signs of DVT, hemoptysis, and a recent cancer.
Wells Criteria generally apply to younger patients without co-morbidity.

Go to the next page if you knew the correct answer, or click the link image(s)
below to further research the concepts in this question (if desired).

Research Concepts:
Embolism, Pulmonary

We update eBooks quarterly and Apps daily based on user feedback.


Please tap flag to report any questions that need improvement.
Question 355: When child abuse is suspected, which of the following
statements is false?

Choices:
1. A practitioner is required by law to report evidence of child abuse even if it is
just mere suspicion
2. Practitioners are at great legal risk when they report suspected abuse
3. All states provide complete legal immunity to practitioners who report
suspected child abuse
4. Most parents threaten lawsuits
Answer: 2 - Practitioners are at great legal risk when they report suspected
abuse

Explanations:
An intentionally false accusation of child abuse can be legally prosecuted.
As long as the history and physical is well documented and there is a
logical reason for suspicion, there is minimal risk when a potential child
case is suspected and reported.
In general, a practitioner is required by law to report suspected child abuse
even if it is a mere suspicion.
Patients may threaten lawsuits but practitioners who document accurately
are generally completed protected from liability.

Go to the next page if you knew the correct answer, or click the link image(s)
below to further research the concepts in this question (if desired).

Research Concepts:
Child Abuse and Neglect

We update eBooks quarterly and Apps daily based on user feedback.


Please tap flag to report any questions that need improvement.
Question 356: Which is not true of congenital diaphragmatic hernias?
Choices:
1. Patients who never have postductal PO2 greater than 100 torr are at a high risk
of complication
2. Antenatal diagnosis provides a favorable prognosis
3. ECMO has not really improved survival
4. Pulmonary hypoplasia occurs in both lungs
Answer: 2 - Antenatal diagnosis provides a favorable prognosis
Explanations:
Antenatal diagnosis provides a poor prognosis.
The earlier the symptoms or diagnosis the worse is the prognosis.
Significant clinical judgment is required in managing these infants.
Despite the availability of ECMO, results are not perfect and complications
continue to occur.

Go to the next page if you knew the correct answer, or click the link image(s)
below to further research the concepts in this question (if desired).

Research Concepts:
Hernia, Diaphragmatic, Congenital

We update eBooks quarterly and Apps daily based on user feedback.


Please tap flag to report any questions that need improvement.
Question 357: Pickwickian syndrome is usually not associated with which
of the following?

Choices:
1. Obstructive sleep apnea
2. Increased inspiratory work
3. Higher intra abdominal pressure
4. Hyperventilation
Answer: 4 - Hyperventilation
Explanations:
Pickwickian syndrome generally involves hypoventilation due to obesity.
Charles Dickens described a boy so affected in the novel "The Pickwick
Papers".
There is obstructive sleep apnea with increased inspiratory work and intra-
abdominal pressure.

Go to the next page if you knew the correct answer, or click the link image(s)
below to further research the concepts in this question (if desired).

Research Concepts:
Pickwickian Syndrome

We update eBooks quarterly and Apps daily based on user feedback.


Please tap flag to report any questions that need improvement.
Question 358: Which of the following is true of acute hyperventilation
syndrome?

Choices:
1. Female preponderance
2. 1% of all cases of hyperventilation syndrome
3. Precipitating event is usually emotional stressor
4. All of the above
Answer: 4 - All of the above
Explanations:
Acute hyperventilation syndrome (HVS) is seen more in women than men,
some estimate a rate of 7:1.
It represents 1 percent of all cases of HVS.
An emotionally precipitating event can often be identified.

Go to the next page if you knew the correct answer, or click the link image(s)
below to further research the concepts in this question (if desired).

Research Concepts:
Ventilation, Hyperventilation

We update eBooks quarterly and Apps daily based on user feedback.


Please tap flag to report any questions that need improvement.
Question 359: Which component of pulmonary function testing correlates
with the cough reflex?

Choices:
1. Residual volume
2. Total lung capacity
3. Vital capacity
4. Maximum inspiratory pressure
Answer: 4 - Maximum inspiratory pressure
Explanations:
Maximum inspiratory pressure (MIP) is defined as the maximum negative
pressure during inspiration.
A normal MIP is 50 to 100 cmH2O.
A MIP less than 20 cmH2O indicates an inadequate cough reflex.
Both the rapid shallow breathing index and MIP act as a good predictor of
successful weaning from a ventilator.

Go to the next page if you knew the correct answer, or click the link image(s)
below to further research the concepts in this question (if desired).

Research Concepts:
Pulmonary Function Tests

We update eBooks quarterly and Apps daily based on user feedback.


Please tap flag to report any questions that need improvement.
Question 360: What is the best prophylaxis for acute mountain sickness?
Choices:
1. Acetazolamide
2. Doxycycline
3. Furosemide
4. Oxygen
Answer: 1 - Acetazolamide
Explanations:
Acetazolamide started 24 hours prior to ascent and for two days at altitude
is recommended prophylaxis.
Dexamethasone is reserved for those allergic to acetazolamide.
Furosemide has no place in the prophylaxis or treatment of acute mountain
sickness.
Oxygen may be used as support but not prophylaxis.

Go to the next page if you knew the correct answer, or click the link image(s)
below to further research the concepts in this question (if desired).

Research Concepts:
Mountain Sickness, Acute

We update eBooks quarterly and Apps daily based on user feedback.


Please tap flag to report any questions that need improvement.
Question 361: What is the most common malignancy in post-transplant
patients?

Choices:
1. B-cell lymphomas
2. Skin cancers
3. Cervical cancers
4. T-cell lymphomas
Answer: 2 - Skin cancers
Explanations:
The incidence of malignancy increases after transplantation.
Skin cancer is the most frequent cancer and occurs in 3% to 10% of
individuals.
Skin cancers in organ transplant recipients include squamous cell, basal
cell, and Kaposi sarcoma.
Many patients have multiple skin cancers. These have a higher risk of
recurrence than in non-immunosuppressed individuals.

Go to the next page if you knew the correct answer, or click the link image(s)
below to further research the concepts in this question (if desired).

Research Concepts:
Cancer, Skin (Integument)

We update eBooks quarterly and Apps daily based on user feedback.


Please tap flag to report any questions that need improvement.
Question 362: Which of the following is consistent with a true apneic
episode in infancy?

Choices:
1. Cyclical respiratory pauses of 5 to 10 seconds
2. Cessation of breathing for 18 seconds
3. Cessation of breathing for 25 seconds with pallor
4. Frustrated 9-month old who turns blue after prolonged crying
Answer: 3 - Cessation of breathing for 25 seconds with pallor
Explanations:
Although apnea means cessation of breathing, it should be differentiated
from other common benign conditions such as periodic breathing and
breath-holding spells. Periodic breathing is the occurrence of brief periods
of apnea that occur in short cycles of 5 to 10 seconds.
Breath-holding spells occur predominantly in the 6 to18 month age group
when a prolonged cry is triggered by anger or frustration and followed by a
brief period of apnea that may include a color change to cyanosis or pallor.
According to the American Academy of Pediatric, apnea of infancy is
defined as an unexplained episode of cessation of breathing for 20 seconds
or longer and a shorter respiratory pause when associated with bradycardia,
cyanosis, pallor, or marked hypotonia.
It is critical to recognize true apnea to initiate appropriate interventions. In
some cases, this may include resuscitation in addition to a thorough
evaluation to determine the underlying cause.

Go to the next page if you knew the correct answer, or click the link image(s)
below to further research the concepts in this question (if desired).

Research Concepts:
Apnea, Infant

We update eBooks quarterly and Apps daily based on user feedback.


Please tap flag to report any questions that need improvement.
Question 363: Which treatment often is a complete cure for obstructive
sleep apnea?

Choices:
1. Continuous positive airway pressure
2. Significant weight loss
3. Uvulopalatopharyngoplasty
4. Oral appliance therapy
Answer: 2 - Significant weight loss
Explanations:
Weight loss may be the only treatment needed.
Uvulopalatopharyngoplasty can be effective.
Continuous positive airway pressure (CPAP) treats the condition but may
be poorly tolerated.
Oral appliance therapy is sometimes used for those who cannot tolerate
CPAP.

Go to the next page if you knew the correct answer, or click the link image(s)
below to further research the concepts in this question (if desired).

Research Concepts:
Apnea, Obstructive Sleep Apnea

We update eBooks quarterly and Apps daily based on user feedback.


Please tap flag to report any questions that need improvement.
Question 364: Which of the following is unlikely to be the etiology of
chronic cough in children?

Choices:
1. Asthma
2. Allergic rhinitis
3. Postinfection
4. Foreign body
Answer: 4 - Foreign body
Explanations:
The most common etiology of chronic cough in children is postinfectious.
Other common causes include asthma, postnasal drip, and irritants.
Less common causes are foreign bodies, cystic fibrosis, congenital
abnormalities, and psychogenic.
Nasal foreign bodies cause unilateral purulent nasal discharge.

Go to the next page if you knew the correct answer, or click the link image(s)
below to further research the concepts in this question (if desired).

Research Concepts:
Cough, Chronic

We update eBooks quarterly and Apps daily based on user feedback.


Please tap flag to report any questions that need improvement.
Question 365: Which of the following is false about congenital lobar
emphysema?

Choices:
1. Associated with congenital heart disease
2. Endobronchial obstruction is seen in the majority of children
3. Generally, upper lobes are involved
4. May mimic a pneumothorax
Answer: 2 - Endobronchial obstruction is seen in the majority of children
Explanations:
Generally, these infants are asymptomatic at birth and then develop
respiratory distress.
Extrinsic bronchial obstruction may play a role in some cases but overall it
appears that obstruction is not present in all the cases. A ball valve like
effect in the bronchi results in air trapping. Some have shown defective
cartilage and that could account for collapse of the bronchial wall.
If bronchial obstruction is suspected, bronchoscopy should be done. Care
must be taken into not interpreting the emphysematous lobe as a
pneumothorax. The disease process usually affects the upper lobes and may
be bilateral. Males are affected more than females.
Cardiac anomalies may be present and if suspected an ECHO is needed.
Lobectomy is curative and most children have a normal life span.

Go to the next page if you knew the correct answer, or click the link image(s)
below to further research the concepts in this question (if desired).

Research Concepts:
Emphysema, Lobar, Congenital

We update eBooks quarterly and Apps daily based on user feedback.


Please tap flag to report any questions that need improvement.
Question 366: Which of the following is not diagnosed using an ultrasound
in utero?

Choices:
1. Congenital diaphragmatic hernia
2. Extralobar sequestration
3. Congenital adenomatoid malformation
4. Congenital bronchiectasis
Answer: 4 - Congenital bronchiectasis
Explanations:
The diagnosis of bronchiectasis is done with a high-resolution CT scan.
The key feature on the scan will be dilated bronchiole diameter, which is
much bigger than the bronchial artery. This is often referred to as the signet
sign.
Other features on the scan will reveal tapering of larger airways and
multiple air-fluid levels in the dilated bronchioles.
CT scan also can assess the lung periphery for damage. Bronchography
rarely is used for diagnosis because of improvement in CT imaging.

Go to the next page if you knew the correct answer, or click the link image(s)
below to further research the concepts in this question (if desired).

Research Concepts:
Bronchiectasis

We update eBooks quarterly and Apps daily based on user feedback.


Please tap flag to report any questions that need improvement.
Question 367: What is the best way to avoid barotrauma during mechanical
ventilation?

Choices:
1. Increasing positive end-expiratory pressure (PEEP)
2. Limiting tidal volume and plateau pressure to less than 30 to 50 cm water
column
3. Giving nebulizers
4. Giving antibiotics
Answer: 2 - Limiting tidal volume and plateau pressure to less than 30 to 50
cm water column

Explanations:
In those receiving mechanical ventilation, barotrauma is most commonly
associated with acute respiratory distress syndrome.
While there is no accepted safe pressure at which there is no risk the most
effective predictor of risk is plateau pressure or peak airway pressure (PAP),
which indicates alveolar distension.
Other risk factors are the aspiration of stomach contents, as well as pre-
existing diseases such as chronic lung disease and pneumonia.
The incidence of barotrauma from mechanical ventilation has decreased
substantially due to the recognition that this is a complication.

Go to the next page if you knew the correct answer, or click the link image(s)
below to further research the concepts in this question (if desired).

Research Concepts:
Ventilation, Barotrauma And Mechanical

We update eBooks quarterly and Apps daily based on user feedback.


Please tap flag to report any questions that need improvement.
Question 368: Which of the following statements regarding extralobar
sequestration is false?

Choices:
1. Venous return is via azygous vein or inferior vena cava
2. Resection is required
3. It is sometimes discovered during a Bochdalek hernia repair
4. It is typically diagnosed in adulthood
Answer: 4 - It is typically diagnosed in adulthood
Explanations:
Extralobar sequestrations are usually diagnosed in childhood due to mass-
like appearance, respiratory distress, and frequent seeding with infection.
Ten percent are incidentally discovered in adults, including those
undergoing repair of congenital Bochdalek hernias.
Extralobar sequestrations make up only 25% of all bronchopulmonary
sequestrations and are more common in males. They have a greater
likelihood of being associated with other congenital anomalies.
Eighty percent of extralobar sequestrations are found between the
diaphragm and lower lobes in the posterior basal segment of the lower lobe
region. They are more common on the left side.

Go to the next page if you knew the correct answer, or click the link image(s)
below to further research the concepts in this question (if desired).

Research Concepts:
Pulmonary Sequestration

We update eBooks quarterly and Apps daily based on user feedback.


Please tap flag to report any questions that need improvement.
Question 369: Which of the following pathogens is associated with the
development of post-lung-transplant lymphoproliferative disorder (PTLD)?

Choices:
1. Cytomegalovirus
2. Epstein-Barr virus
3. Herpes simplex virus
4. Flavivirus
Answer: 2 - Epstein-Barr virus
Explanations:
It is felt that Epstein-Barr virus is associated with post-lung-transplant
lymphomas.
These lymphomas are treated by decreasing the intensity of
immunosuppressive therapy.
Post-lung-transplant lymphoproliferative disorder (PTLD) is one of the
most frequent post-transplant malignancies.
The risk of PTLD is directly proportional to the intensity and duration of
the immunosuppression used for the transplant.

Go to the next page if you knew the correct answer, or click the link image(s)
below to further research the concepts in this question (if desired).

Research Concepts:
Lymphoproliferative Disorders

We update eBooks quarterly and Apps daily based on user feedback.


Please tap flag to report any questions that need improvement.
Question 370: Which of the following is true about interpleural analgesia?
Choices:
1. Always requires instillation of a local anesthetic into visceral pleura.
2. Analgesia is produced by diffusion of local anesthetic into regional spinal
nerves
3. Is less effective in the presence of pleural adhesions
4. Should not be performed in the presence of chest tubes
Answer: 3 - Is less effective in the presence of pleural adhesions
Explanations:
Interpleural analgesia involves instillation of local anesthetic into the
interpleural space located between the visceral and parietal pleura.
It produces analgesia primarily by diffusion of local anesthetic into the
intercostal nerves.
Adhesions may limit the flow of local anesthetic in the pleural space, thus
limiting the block's efficacy.
Interpleural analgesia is not contraindicated in the presence of chest tubes,
although it may be less effective due to loss of local anesthetic through the
tubes.

Go to the next page if you knew the correct answer, or click the link image(s)
below to further research the concepts in this question (if desired).

Research Concepts:
Interpleural Analgesia

We update eBooks quarterly and Apps daily based on user feedback.


Please tap flag to report any questions that need improvement.
Question 371: Which diuretic is used in the treatment of mountain
sickness?

Choices:
1. Acetazolamide
2. Furosemide
3. Thiazide
4. Spironolactone
Answer: 1 - Acetazolamide
Explanations:
Acetazolamide is a carbonic anhydrase inhibitor.
Acetazolamide is a mild diuretic occasionally used to treat glaucoma,
benign intracranial hypertension and/or congestive heart failure.
Acetazolamide causes a mild metabolic acidosis through the increased
excretion of bicarbonate, so it can aid in the treatment of metabolic
alkalosis. Thus, there is a suggested role in prevention or treatment of acute
mountain sickness.
The mechanism by which it works is not fully understood.

Go to the next page if you knew the correct answer, or click the link image(s)
below to further research the concepts in this question (if desired).

Research Concepts:
Altitude Illness, Pulmonary Syndromes, High Altitude Pulmonary
Edema (HAPE)

We update eBooks quarterly and Apps daily based on user feedback.


Please tap flag to report any questions that need improvement.
Question 372: Which of the following is incorrect regarding proper
placement of a laryngeal mask airway (LMA)?

Choices:
1. LMA size 3 for a small woman
2. LMA size 4 for a large woman
3. LMA size 5 for a large man
4. For LMA insertion, the ideal position is the head neutral

Photo:Contributed by Scott H. Plantz, MD, FAAEM


Answer: 4 - For LMA insertion, the ideal position is the head neutral
Explanations:
The ideal positioning for insertion of a laryngeal tube airway is the
"sniffing" position.
A laryngeal mask airway is a good choice for securing the airway of a
crashing patient, especially for a pediatric patient if intubation is difficult.
A laryngeal mask is an airway tube that connects to an elliptical mask with
a cuff that can be either inflating or self-sealing. Once inserted, the mask
conforms to the anatomy with the bowl of the mask facing the space
between the vocal cords.
After insertion, the tip of the laryngeal mask sits in the oropharynx against
the muscular valve that is located at the upper portion of the esophagus.

Go to the next page if you knew the correct answer, or click the link image(s)
below to further research the concepts in this question (if desired).

Research Concepts:
Airway, Laryngeal Mask

We update eBooks quarterly and Apps daily based on user feedback.


Please tap flag to report any questions that need improvement.
Question 373: A 4-year-old child with cerebral palsy and cognitive
development of a 9-month-old is brought in with a fever and a cough. According
to the parents, she drools all the time, has chronic hoarseness, and has had 3
episodes of pneumonia. What is the significance of chronic hoarseness in the
child?

Choices:
1. Neurologic impairment
2. Gastroesophageal reflux disease and aspiration
3. Vocal cord paralysis
4. Mycoplasma pneumonia
Answer: 2 - Gastroesophageal reflux disease and aspiration
Explanations:
This neurologically impaired patient is at increased risk for aspiration.
Vocal cord paralysis or other pathology is possible but less likely.
Hoarseness is not a common finding in Mycoplasma pneumonia.

Go to the next page if you knew the correct answer, or click the link image(s)
below to further research the concepts in this question (if desired).

Research Concepts:
Hoarseness

We update eBooks quarterly and Apps daily based on user feedback.


Please tap flag to report any questions that need improvement.
Question 374: Which electrolyte disturbance is most commonly found with
theophylline overdose?

Choices:
1. Hyponatremia
2. Hypokalemia
3. Hypermagnesemia
4. Hypophosphatemia
Answer: 2 - Hypokalemia
Explanations:
Hypokalemia is common in patients with a theophylline overdose, but they
should be monitored for hyperglycemia and hypercalcemia as well.

Go to the next page if you knew the correct answer, or click the link image(s)
below to further research the concepts in this question (if desired).

Research Concepts:
Toxicity, Theophylline

We update eBooks quarterly and Apps daily based on user feedback.


Please tap flag to report any questions that need improvement.
Question 375: In HIV patients who have pulmonary tuberculosis and are
treated with HAART (highly active anti-retroviral therapy), which anti-
tuberculosis drug is usually avoided?

Choices:
1. Rifampin
2. Ethambutol
3. Pyrazinamide
4. Isoniazid
Answer: 1 - Rifampin
Explanations:
Treatment regimens for active or latent tuberculosis in patients with HIV
infection are similar to those used in HIV-negative patients, but dose
adjustments may be necessary.
The most significant differences involve the avoidance of rifampin in
patients who are on protease inhibitors. Rifabutin may be used in place of
rifampin in such patients.
Rifampin induces the liver microsomal enzymes and lowers the effective
levels of HAART in the systemic circulation.

Go to the next page if you knew the correct answer, or click the link image(s)
below to further research the concepts in this question (if desired).

Research Concepts:
Rifampin

We update eBooks quarterly and Apps daily based on user feedback.


Please tap flag to report any questions that need improvement.
Question 376: Which of the following statements about congenital
pulmonary airway malformation is FALSE?

Choices:
1. It arises from an embryonic disruption before the thirty-fifth day that causes
improper development of bronchioles
2. It is usually identified on prenatal ultrasound around the 20th week
3. There is increased risk of pulmonary hypoplasia
4. Infection is uncommon
Answer: 4 - Infection is uncommon
Explanations:
Congenital pulmonary airway malformation is an uncommon lung anomaly
where cystic cavities occur in the lung tissues.
Recurrent infection is of the most common presentation of this disorder.
Large cystic masses can cause pulmonary hypoplasia and respiratory
failure.

Go to the next page if you knew the correct answer, or click the link image(s)
below to further research the concepts in this question (if desired).

Research Concepts:
Congenital Pulmonary Airway Malformation

We update eBooks quarterly and Apps daily based on user feedback.


Please tap flag to report any questions that need improvement.
Question 377: An infant born with a tracheoesophageal fistula returns at 5
months with expiratory stridor and wheezing. What is the most probable
diagnosis?

Choices:
1. Tracheomalacia
2. Recurrence of the tracheoesophageal fistula
3. Gastroesophageal reflux
4. Bronchiolitis
Answer: 1 - Tracheomalacia
Explanations:
After surgical repair of a TE fistula patients can develop tracheomalacia, or
collapse of the trachea.
Recurrence of the fistula would have symptoms of choking and aspiration
during feeding.
GERD would be evidenced by symptoms after feeding.

Go to the next page if you knew the correct answer, or click the link image(s)
below to further research the concepts in this question (if desired).

Research Concepts:
Tracheomalacia

We update eBooks quarterly and Apps daily based on user feedback.


Please tap flag to report any questions that need improvement.
Question 378: What is NOT a common cause of bilateral diaphragmatic
paralysis?

Choices:
1. Amyotrophic lateral sclerosis
2. Multiple sclerosis
3. Post-polio syndrome
4. Open heart surgery
Answer: 4 - Open heart surgery
Explanations:
The most common causes of bilateral diaphragmatic paralysis include
amyotrophic lateral scleroses, post-polio syndrome, and multiple sclerosis.

Go to the next page if you knew the correct answer, or click the link image(s)
below to further research the concepts in this question (if desired).

Research Concepts:
Diaphragmatic Paralysis

We update eBooks quarterly and Apps daily based on user feedback.


Please tap flag to report any questions that need improvement.
Question 379: In which of the following conditions is high frequency
ventilation (HFV) not useful?

Choices:
1. Tracheal surgery
2. Bronchopleural fistula
3. Adult respiratory distress syndrome
4. Carinal surgery
Answer: 3 - Adult respiratory distress syndrome
Explanations:
HFV is an alternative to conventional mechanical ventilation. It requires
small tidal volumes at higher frequency. It can be used as an open or closed
technique.
The open system has no endotracheal tube and the gas outflow pathway is
not mechanically established. In the closed system, there is an endotracheal
tube. This method is safer, with better performance and PEEP can be added.
The advantages are that there is only minimal lung movement, lower
pressures result in less air leak, and it can be used with small catheters for
distal airway ventilation.
HFV can maintain mean airway pressure but the peak airway pressure is
decreased. Gas exchange is by diffusion and humidification is required to
prevent drying of the airways.
Adult respiratory distress syndrome is not treated with HFV. The results
have not shown any difference between HFV and routine mechanical
ventilation.

Go to the next page if you knew the correct answer, or click the link image(s)
below to further research the concepts in this question (if desired).

Research Concepts:
Ventilation, High Frequency

We update eBooks quarterly and Apps daily based on user feedback.


Please tap flag to report any questions that need improvement.
Question 380: Which of the following is indicated in the treatment of
hyperventilation syndrome?

Choices:
1. Antibiotics
2. Atropine
3. Benzodiazepines
4. None of the above
Answer: 3 - Benzodiazepines
Explanations:
Hyperventilation syndrome is most often caused by anxiety.
Benzodiazepines are indicated in the treatment.
Selective serotonin reuptake inhibitors are also helpful.
Atropine is used in the treatment of organophosphate toxicity.

Go to the next page if you knew the correct answer, or click the link image(s)
below to further research the concepts in this question (if desired).

Research Concepts:
Ventilation, Hyperventilation

We update eBooks quarterly and Apps daily based on user feedback.


Please tap flag to report any questions that need improvement.
Question 381: What is the best test to image a pulmonary arteriovenous
malformation?

Choices:
1. CT pulmonary angiogram
2. Aortogram
3. Conventional pulmonary angiography
4. Lung perfusion imaging with MAA
Answer: 1 - CT pulmonary angiogram
Explanations:
Pulmonary AV malformations may be acquired or congenital. 40 percent
are associated with the Osler Weber Rendu syndrome.
AVMs may be multiple, bilateral, and commonly occur in the lower lobes.
There may be a right to left shunt with paradoxical embolization, stroke,
TIA, or brain abscess as an initial manifestation of the disorder. For this
reason, choice 4 is not a recommended imaging method.
With large shunts, cyanosis and clubbing may be present. Usually the
cardiac parameters are normal.
Historically, conventional pulmonary angiography was the best test to
diagnose AVMs, but today CT pulmonary angiography is preferred as it is
safer, less costly, less invasive, and more anatomically informative. Repeat
subselective injections are not required to search for multiple lesions. To
perform therapeutic coiling or embolization, conventional pulmonary
angiography is still required.

Go to the next page if you knew the correct answer, or click the link image(s)
below to further research the concepts in this question (if desired).

Research Concepts:
Lung, Arteriovenous Malformation

We update eBooks quarterly and Apps daily based on user feedback.


Please tap flag to report any questions that need improvement.
Question 382: Which congenital anomaly in newborns requires urgent
surgical repair?

Choices:
1. Pyloric stenosis
2. Epiphrenic diverticulum
3. Esophageal diverticulum
4. Diaphragmatic hernia
Answer: 4 - Diaphragmatic hernia
Explanations:
Infants with diaphragmatic hernias need orogastric decompression to
prevent bowel distension and lung compression at birth. One should avoid
high peak inspiratory pressures and be alert for pneumothorax.
Infants with diaphragmatic hernias have immature lung and may require
surfactant and extracorporeal membrane oxygenation.
Fetal surgery for a diaphragmatic hernia has not been shown to improve
survival and is rarely done. Today, infants are first stabilized before
undertaking surgery.
Surgery may be delayed until the infant is optimized. Some surgeons prefer
to operate after the pulmonary artery pressures are normal for at least 24 to
48 hours.

Go to the next page if you knew the correct answer, or click the link image(s)
below to further research the concepts in this question (if desired).

Research Concepts:
Hernia, Diaphragmatic, Congenital

We update eBooks quarterly and Apps daily based on user feedback.


Please tap flag to report any questions that need improvement.
Question 383: Which of the following statements about congenital
pulmonary airway malformations is true?

Choices:
1. Result of toxic insult to pregnant patient
2. Histologically 4 types
3. Can be detected on prenatal ultrasound
4. Type III is most common
Answer: 3 - Can be detected on prenatal ultrasound
Explanations:
Congenital pulmonary airway malformation (CPAM) is a congenital lesion,
often see on fetal ultrasound. There are 3 histologic types: Type I, few >2
cm cysts; type II, multiple >1 cm cysts; and type III numerous < 5 mm
microcysts that may appear solid.
They differ from sequestration in that they are supplied by pulmonary
arteries. Sequestration is supplied by systemic arteries.
Type II often is associated congenital renal and CNS anomalies. With Type
III, prenatal US often shows hydrops (polyhydramnios).
Over half of cases are type I; type II is approximately 40 percent; and type
III is < 5 percent.

Go to the next page if you knew the correct answer, or click the link image(s)
below to further research the concepts in this question (if desired).

Research Concepts:
Congenital Pulmonary Airway Malformation

We update eBooks quarterly and Apps daily based on user feedback.


Please tap flag to report any questions that need improvement.
Question 384: Which is true of dysphagia lusoria?
Choices:
1. Seen in patients with gonorrhea
2. Seen in patients with achalasia
3. Seen in patients with anomalous subclavian artery
4. Seen in patients with double esophagus
Answer: 3 - Seen in patients with anomalous subclavian artery
Explanations:
Dysphagia lusoria is a rare disorder caused by an aberrant right subclavian
artery.
The right subclavian artery forms a ring around the trachea or esophagus
and leads to extrinsic compression of the esophagus.
The diagnosis is made by CT angiogram. Surgery is only indicated in
patients who are symptomatic.
Surgery is performed by a supraclavicular incision. Care must be taken not
to injure the brachial plexus.

Go to the next page if you knew the correct answer, or click the link image(s)
below to further research the concepts in this question (if desired).

Research Concepts:
Dysphagia

We update eBooks quarterly and Apps daily based on user feedback.


Please tap flag to report any questions that need improvement.
Question 385: What can be used to reverse the neuropathic side effects of
isoniazid?

Choices:
1. Vitamin C
2. Niacin
3. Pyridoxine
4. Thiamine
Answer: 3 - Pyridoxine
Explanations:
Peripheral neuritis is the most common adverse effect with the use of
isoniazid. It is associated with a relative pyridoxine deficiency.
Most of the toxic reactions are corrected by pyridoxine (vitamin B6)
supplementation.
In fact, formulas of isoniazid come together with pyridoxine.
The peripheral neuropathy is not completely reversed even after
discontinuation of the drug. Liver function also needs to be monitored in
patients who take isoniazid.

Go to the next page if you knew the correct answer, or click the link image(s)
below to further research the concepts in this question (if desired).

Research Concepts:
Toxicity, Isoniazid

We update eBooks quarterly and Apps daily based on user feedback.


Please tap flag to report any questions that need improvement.
Question 386: When is a diagnosis of tuberculosis most certain?
Choices:
1. Mantoux test 8 mm with history of Bacille Calmette-Guerin vaccine
2. Redness and itchiness at skin test site
3. Normal x-ray with positive skin test
4. Mantoux test 15 mm with night sweats
Answer: 4 - Mantoux test 15 mm with night sweats
Explanations:
Tuberculosis is likely if the Mantoux test is 15 mm.
The acid-fast bacteria stain is fast and sensitive, but one requires a very
high load of bacteria to confirm positivity.
If the chest x-ray shows a lesion, the sputum is positive for acid fast
bacteria, and the purified protein derivative (PPD) skin test is positive, start
therapy.
For otherwise healthy adults, the PPD skin test cut off is 15 mm or more.

Go to the next page if you knew the correct answer, or click the link image(s)
below to further research the concepts in this question (if desired).

Research Concepts:
Tuberculosis

We update eBooks quarterly and Apps daily based on user feedback.


Please tap flag to report any questions that need improvement.
Question 387: In children with AIDS, what is the most common cause of
diffuse lymphoid pneumonitis?

Choices:
1. Cytomegalovirus
2. Epstein-Barr virus
3. Pneumocystic carinii pneumonia
4. Varicella virus
Answer: 2 - Epstein-Barr virus
Explanations:
Epstein-Barr virus causes a diffuse lymphoid pneumonitis in children with
HIV.

Go to the next page if you knew the correct answer, or click the link image(s)
below to further research the concepts in this question (if desired).

Research Concepts:
Pneumonia, Lymphocytic Interstitial

We update eBooks quarterly and Apps daily based on user feedback.


Please tap flag to report any questions that need improvement.
Question 388: Which of the following is false regarding blastomycosis
infection in immunocompromised patients?

Choices:
1. Acute respiratory distress syndrome can be a complication, particularly in the
elderly or immunocompromised patient
2. Infection begins in the lungs, then disseminates to the skin, bone, and brain
3. Despite antifungal treatment, mortality is high in immunocompromised
patients
4. It is endemic to the southwest United States
Answer: 4 - It is endemic to the southwest United States
Explanations:
Blastomycosis is a serious illness with high mortality despite antifungal
treatment in the immunocompromised patient, especially patients with
AIDS.
Blastomycosis is endemic to the central and southeastern United States.
Unlike other systemic and deep fungal infections, blastomycosis can occur
in immunocompetent and immunocompromised patients.
Because of the severity of the disease, initial therapy with amphotericin B is
advised for most immunocompromised patients with blastomycosis.

Go to the next page if you knew the correct answer, or click the link image(s)
below to further research the concepts in this question (if desired).

Research Concepts:
Blastomycosis

We update eBooks quarterly and Apps daily based on user feedback.


Please tap flag to report any questions that need improvement.
Question 389: Bosentan is a drug that is approved for use in pulmonary
artery hypertension. What is the mechanism of action of this drug?

Choices:
1. It is a calcium channel blocker
2. It is a cholinergic antagonist
3. It stimulates beta-2 receptors on bronchial smooth muscles
4. It blocks endothelin receptors on vascular endothelium and smooth muscle
Answer: 4 - It blocks endothelin receptors on vascular endothelium and
smooth muscle

Explanations:
Endothelin receptors are found in vascular endothelium and smooth muscle.
Stimulation of these receptors is associated with vasoconstriction.
Bosentan acts by blocking endothelin receptors on vascular endothelium
and smooth muscle.
Bosentan is administered orally.

Go to the next page if you knew the correct answer, or click the link image(s)
below to further research the concepts in this question (if desired).

Research Concepts:
Pulmonary Hypertension

We update eBooks quarterly and Apps daily based on user feedback.


Please tap flag to report any questions that need improvement.
Question 390: A neonate undergoes repair of a tracheo-esophageal fistula.
Three months later, he presents with noisy breathing and a barking cough. The
diagnosis is most easily confirmed with which of the following?

Choices:
1. Bronchoscopy
2. Cine CT
3. Chest x-ray
4. Fluoroscopy
Answer: 4 - Fluoroscopy
Explanations:
Generally, fluoroscopy is available and is easier than the other studies.

Go to the next page if you knew the correct answer, or click the link image(s)
below to further research the concepts in this question (if desired).

Research Concepts:
Tracheomalacia

We update eBooks quarterly and Apps daily based on user feedback.


Please tap flag to report any questions that need improvement.
Section 7

Question 391: Which monitoring device is most suitable for the monitoring
of a patient with carbon monoxide (CO) poisoning?

Choices:
1. Capnograph
2. CO oximeter
3. ABG
4. Pulse oximeter
Answer: 2 - CO oximeter
Explanations:
Carboxyhemoglobin levels must be monitored in a patient with carbon
monoxide poisoning.
CO-oximetry measures CO levels accurately.
Pulse Oximetry can not distinguish between carboxyhemoglobin and
oxyhemoglobin so it is not a reliable indicator in carbon monoxide
poisoning of the patients overall condition.

Go to the next page if you knew the correct answer, or click the link image(s)
below to further research the concepts in this question (if desired).

Research Concepts:
Toxicity, Carbon Monoxide

We update eBooks quarterly and Apps daily based on user feedback.


Please tap flag to report any questions that need improvement.
Question 392: A child is started on ribavirin for respiratory syncytial virus.
4 days later, the child is admitted because of a major side effect that led to severe
fatigue. Which of the following is a major side effect of ribavirin?

Choices:
1. Liver failure
2. Renal failure
3. Hemolytic anemia
4. Allergy
Answer: 3 - Hemolytic anemia
Explanations:
Ribavirin tends to accumulate in red blood cells in some patients.
Ribavirin can induce a hemolytic anemia due to inhibition of IMP
dehydrogenase.
This side effect is most common when it is combined with zidovudine.

Go to the next page if you knew the correct answer, or click the link image(s)
below to further research the concepts in this question (if desired).

Research Concepts:
Ribavirin

We update eBooks quarterly and Apps daily based on user feedback.


Please tap flag to report any questions that need improvement.
Question 393: Which of the following is not used to treat post intubation
laryngeal edema?

Choices:
1. Racemic epinephrine
2. Oxygen
3. Steroids
4. Albuterol
Answer: 4 - Albuterol
Explanations:
Post intubation laryngeal edema usually requires the treatment of oxygen,
steroids, and racemic epinephrine.
The mixture of helium and oxygen is potentially effective.
Positive pressure ventilation should not be used as it may delay
reintubation.
Albuterol would be of no benefit.

Go to the next page if you knew the correct answer, or click the link image(s)
below to further research the concepts in this question (if desired).

Research Concepts:
Post Intubation Laryngeal Edema

We update eBooks quarterly and Apps daily based on user feedback.


Please tap flag to report any questions that need improvement.
Question 394: A 4-year-old child with cerebral palsy and cognitive
development of a 9-month-old is brought in with a fever and a cough. According
to the parents, she drools all the time, has chronic hoarseness, and has had 3
episodes of pneumonia. The child recovers after treatment of radiographically
proven pneumonia. What test should be done?

Choices:
1. Fluoroscopic video swallow study
2. Esophageal pH monitoring
3. CT of the chest
4. Upper endoscopy
Answer: 2 - Esophageal pH monitoring
Explanations:
This neurologically impaired patient is at increased risk for aspiration.
Gastroesophageal reflux is best diagnosed with esophageal pH monitoring.

Go to the next page if you knew the correct answer, or click the link image(s)
below to further research the concepts in this question (if desired).

Research Concepts:
Gastroesophageal Reflux Disease

We update eBooks quarterly and Apps daily based on user feedback.


Please tap flag to report any questions that need improvement.
Question 395: A patient was started on an antibiotic for a urinary tract
infection. She continued taking the antibiotic for 4 months because of recurrent
dysuria. Now she presents with dyspnea and a dry cough. She denies any weight
loss or smoking. What is the most likely etiology of these new symptoms?

Choices:
1. Sulfamethoxazole
2. Nitrofurantoin
3. Congestive heart failure
4. Goodpasture syndrome
Answer: 2 - Nitrofurantoin
Explanations:
Nitrofurantoin has been associated with lung fibrosis.
Nitrofurantoin-related lung injury is associated with a gradual onset of
dyspnea and decreased exercise tolerance.
These effects on the lung are usually only seen with long-term use, usually
greater than 4 months.
Other features of nitrofurantoin-induced lung injury may be a dry cough
and a low-grade fever.

Go to the next page if you knew the correct answer, or click the link image(s)
below to further research the concepts in this question (if desired).

Research Concepts:
Nitrofurantoin

We update eBooks quarterly and Apps daily based on user feedback.


Please tap flag to report any questions that need improvement.
Question 396: Pneumonia found in New Mexico became known as Navajo
pneumonia. What is the causative organism?

Choices:
1. Legionella
2. Coccidiomycosis
3. Mycoplasma
4. Hanta virus
Answer: 4 - Hanta virus
Explanations:
Hantavirus cardiopulmonary syndrome was first recognized in May of
1993.
It occurred in Southwestern U.S. in the Four Corners region. Initially, the
infection was thought to be 100% fatal but milder cases were identified.
Patients start with fever and chills, progress to cough and dyspnea, and may
have cardiovascular collapse.
Since this initial outbreak, it has been identified as a virus with a reservoir
in rodents and has been seen in Asia.

Go to the next page if you knew the correct answer, or click the link image(s)
below to further research the concepts in this question (if desired).

Research Concepts:
Hantavirus Cardiopulmonary Syndrome

We update eBooks quarterly and Apps daily based on user feedback.


Please tap flag to report any questions that need improvement.
Question 397: A 2-week-old female is brought to the emergency
department with a cough, conjunctivitis, nasal congestion, and an infiltrate on
chest x-ray. What is the most likely etiologic agent?

Choices:
1. Group B streptococcus
2. Respiratory syncytial virus
3. Chlamydia
4. Escherichia coli
Answer: 3 - Chlamydia
Explanations:
Chlamydia infection in the newborn usually presents between 5 and 14 days
after delivery.
The preferred method of diagnosis is with a swab under the upper eyelid.
Systemic antibiotics are recommended, even for conjunctivitis alone.
Erythromycin, not azithromycin, is the drug of choice.

Go to the next page if you knew the correct answer, or click the link image(s)
below to further research the concepts in this question (if desired).

Research Concepts:
Chlamydia

We update eBooks quarterly and Apps daily based on user feedback.


Please tap flag to report any questions that need improvement.
Question 398: What test is the gold standard for diagnosing pulmonary
hypertension?

Choices:
1. Chest X-ray
2. Cardiac catheterization
3. Pulmonary angiography
4. Ventilation-perfusion lung scanning
Answer: 2 - Cardiac catheterization
Explanations:
Cardiac catheterization is the gold standard for the diagnosis of pulmonary
hypertension.
Catheterization typically reveals increased right ventricular/pulmonary
artery pressures and pulmonary vascular resistance.
Ventilation-perfusion lung scanning is typically utilized to rule out chronic
thromboembolic disease.
Pulmonary angiography can also be used to exclude thromboembolic
disease.

Go to the next page if you knew the correct answer, or click the link image(s)
below to further research the concepts in this question (if desired).

Research Concepts:
Pulmonary Hypertension

We update eBooks quarterly and Apps daily based on user feedback.


Please tap flag to report any questions that need improvement.
Question 399: Which of the following does not cause a posterior
indentation of the esophagus on a barium esophagography?

Choices:
1. Pulmonary artery sling
2. Aberrant subclavian artery
3. Double aortic arch
4. Aberrant left carotid artery
Answer: 1 - Pulmonary artery sling
Explanations:
In patients with a pulmonary artery sling, the left pulmonary artery
aberrantly arises from the right pulmonary artery and courses to the left
hilum between the esophagus and trachea.
Pulmonary artery sling presents only with respiratory findings. Esophageal
symptoms are not observed.
Pulmonary artery sling causes an anterior indention of the esophagus, and is
the only vascular congenital anomaly to do so.
All vascular rings encircle the esophagus and trachea, extending posteriorly
to the esophagus.

Go to the next page if you knew the correct answer, or click the link image(s)
below to further research the concepts in this question (if desired).

Research Concepts:
Pulmonary Artery Sling

We update eBooks quarterly and Apps daily based on user feedback.


Please tap flag to report any questions that need improvement.
Question 400: What is the initial treatment for most children with
obstructive sleep apnea (OSA)?

Choices:
1. Continuous positive airway pressure (CPAP)
2. Adenoidectomy and tonsillectomy
3. Supplemental oxygen at night
4. Weight loss
Answer: 2 - Adenoidectomy and tonsillectomy
Explanations:
Most children with OSA documented by polysomnography are treated with
a tonsillectomy and adenoidectomy.
Children with OSA are generally not obese.
Some may require uvulopalatoplasty if T and A is not effective.
CPAP is used for treatment of adults with OSA, which usually is associated
with obesity.

Go to the next page if you knew the correct answer, or click the link image(s)
below to further research the concepts in this question (if desired).

Research Concepts:
Apnea, Obstructive Sleep Apnea

We update eBooks quarterly and Apps daily based on user feedback.


Please tap flag to report any questions that need improvement.
Question 401: Which of the following medications is best used for
immunoprophylaxis of respiratory syncytial virus infection in highly susceptible
children?

Choices:
1. Steroids
2. Ribavirin
3. Respiratory syncytial virus immunoglobulin
4. Palivizumab
Answer: 4 - Palivizumab
Explanations:
Palivizumab is a humanized monoclonal antibody against the respiratory
syncytial virus (RSV) F glycoprotein.
It is approved for use in infants and children less than 24 months old with
preterm birth (less than 35 weeks), bronchopulmonary dysplasia, or
hemodynamically significant congenital heart disease.
Palivizumab does not interfere with immune response to MMR and
varicella vaccines.
It is administered intramuscularly monthly during RSV season.

Go to the next page if you knew the correct answer, or click the link image(s)
below to further research the concepts in this question (if desired).

Research Concepts:
Palivizumab

We update eBooks quarterly and Apps daily based on user feedback.


Please tap flag to report any questions that need improvement.
Question 402: Which is NOT true regarding tracheoesophageal fistula?
Choices:
1. Preterm birth is not a risk factor
2. The incidence of TEF is about 1 in 3000
3. Symptoms decrease with head tilt position
4. Symptoms increase with lateral position
Answer: 3 - Symptoms decrease with head tilt position
Explanations:
An associated risk for TEF is preterm births.
Symptoms increase in patients placed in the lateral or head tilt position.

Go to the next page if you knew the correct answer, or click the link image(s)
below to further research the concepts in this question (if desired).

Research Concepts:
Fistula, Tracheoesophageal

We update eBooks quarterly and Apps daily based on user feedback.


Please tap flag to report any questions that need improvement.
Question 403: How much should inspiratory flow increase after a pre and
post bronchodilator spirometry study to determine a positive response in a
patient with reversible airway disease?

Choices:
1. 10 percent
2. 15 percent
3. 25 percent
4. 30 percent
Answer: 2 - 15 percent
Explanations:
The inspiratory Flow should increase a minimum of 15 percent to
determine a positive response to bronchodilators in a patient with reversible
airway disease.

Go to the next page if you knew the correct answer, or click the link image(s)
below to further research the concepts in this question (if desired).

Research Concepts:
Bronchodilators

We update eBooks quarterly and Apps daily based on user feedback.


Please tap flag to report any questions that need improvement.
Question 404: A 3-year-old girl with known past medical history of sickle
cell disease (HbSS) presents to the emergency department with a cough. She has
never had an episode of acute chest syndrome and has no other significant
history. The patient appears comfortable. Her chest x-ray shows suspicion of left
lower lobe early infiltrate. The patient is afebrile. What is the disposition from
the emergency department?

Choices:
1. Albuterol nebulizer and discharge home
2. Albuterol, supplemental oxygen, intravenous fluids, and discharge home
3. Broad spectrum antibiotics, pain control, supplemental oxygen, intravenous
fluids, and admit
4. Broad spectrum antibiotics, pain control, supplemental oxygen,
corticosteroids, albuterol, intravenous fluids, and admit
Answer: 3 - Broad spectrum antibiotics, pain control, supplemental oxygen,
intravenous fluids, and admit

Explanations:
Broad-spectrum antibiotics should to given to every patient with acute chest
syndrome. Infection is more likely in the pediatric population than in adult
patients.
Bronchodilators are indicated when there is underlying asthma. However,
they can be administered in any case in which bronchospasm is identified or
suspected.
Corticosteroids have been shown to shorten the length of stay but carry the
risk of a higher rate of rebound vaso-occlusive crisis, increased risk of
readmission, and increased risk of fat emboli.
According to the diagnostic criteria of an acute chest, this patient has acute
chest syndrome with infiltrate plus a cough. An acute chest should be
treated early and aggressively, usually requiring admission to the hospital.

Go to the next page if you knew the correct answer, or click the link image(s)
below to further research the concepts in this question (if desired).

Research Concepts:
Chest Syndrome, Acute

We update eBooks quarterly and Apps daily based on user feedback.


Please tap flag to report any questions that need improvement.
Question 405: Which of the following is not a lung complication of
systemic lupus erythematosus?

Choices:
1. Acute interstitial pneumonitis
2. Pleuritis
3. Bronchiolitis obliterans organizing pneumonia
4. Progressive pulmonary fibrosis
Answer: 4 - Progressive pulmonary fibrosis
Explanations:
The most common pulmonary complication of SLE is pleuritis.
Progressive pulmonary fibrosis is not seen.
Other pulmonary complications include hemorrhage, pulmonary vascular
disease, loss of lung volumes, and uremic pulmonary edema,.
Patients with SLE are also more prone to infections.

Go to the next page if you knew the correct answer, or click the link image(s)
below to further research the concepts in this question (if desired).

Research Concepts:
Systemic Lupus Erythematosus (SLE)

We update eBooks quarterly and Apps daily based on user feedback.


Please tap flag to report any questions that need improvement.
Question 406: At what level of positive end expiratory pressure (PEEP) is
monitoring of cardiac output required?

Choices:
1. 5 cm water
2. 7.5 cm water
3. 10 cm water
4. 15 cm water
Answer: 3 - 10 cm water
Explanations:
Once a PEEP of 10 is reached, it is recommended that cardiac output is
monitored.
If the patient continues to maintain a stable blood pressure with good urine
output, the monitoring may be delayed.
When the blood pressure falls with increased heart rate, fluids may be
required.
At any level of PEEP, if hypotension ensues, careful consideration of the
ventilation strategies should be put in place.

Go to the next page if you knew the correct answer, or click the link image(s)
below to further research the concepts in this question (if desired).

Research Concepts:
Positive End-Expiratory Pressure (PEEP)

We update eBooks quarterly and Apps daily based on user feedback.


Please tap flag to report any questions that need improvement.
Question 407: A young female has had 3 bouts of hemoptysis. Work up
reveals an isolated AV malformation in the lung. Which of these statements is
incorrect?

Choices:
1. Can be associated with Osler Weber Rendu syndrome
2. Can be multiple
3. Surgery involves a lobectomy
4. Embolization can occasionally be used
Answer: 3 - Surgery involves a lobectomy
Explanations:
These lesions can be multiple, bilateral, and subpleural.
Treatment requires a conservative surgical resection with preservation of
lung tissue.

Go to the next page if you knew the correct answer, or click the link image(s)
below to further research the concepts in this question (if desired).

Research Concepts:
Lung, Arteriovenous Malformation

We update eBooks quarterly and Apps daily based on user feedback.


Please tap flag to report any questions that need improvement.
Question 408: A 5-year-old female is admitted with pneumonia. It is her
5th admission over the past year. Each episode is characterized by dyspnea,
wheezing, hemoptysis, and cyanosis and resolves after three days of antibiotics.
Follow chest X-rays are always normal. The current exam shows findings at the
left lung base and digital clubbing. Select the best diagnostic test.

Choices:
1. Sweat chloride
2. Bronchoalveolar lavage
3. CT of the chest
4. Pulmonary function tests
Answer: 2 - Bronchoalveolar lavage
Explanations:
The most probable diagnosis is idiopathic pulmonary hemosiderosis.
Recurrent hemoptysis makes pulmonary hemorrhage likely.
Clubbing indicates a chronic process.
Hemosiderin-laden macrophages would be found on bronchoalveolar
lavage.

Go to the next page if you knew the correct answer, or click the link image(s)
below to further research the concepts in this question (if desired).

Research Concepts:
Idiopathic Pulmonary Hemosiderosis

We update eBooks quarterly and Apps daily based on user feedback.


Please tap flag to report any questions that need improvement.
Question 409: After an unsuccessful intubation attempt, a laryngeal mask
airway was placed. A laryngeal mask airway is contraindicated in patients with
what condition?

Choices:
1. Spine injury
2. Head trauma
3. Giving birth
4. Propensity to aspirate
Answer: 4 - Propensity to aspirate
Explanations:
Laryngeal mask airways (LMAs) may be used in patients with spinal
injuries. They do not require neck movement to place.
LMAs may be used in patients with head injuries.
LMAs may be used in patients who are giving birth.
The laryngeal mask does not protect the lungs from aspiration. Thus,
laryngeal masks are unsuitable for people who are prone to aspiration.

Go to the next page if you knew the correct answer, or click the link image(s)
below to further research the concepts in this question (if desired).

Research Concepts:
Airway, Management

We update eBooks quarterly and Apps daily based on user feedback.


Please tap flag to report any questions that need improvement.
Question 410: What percentage of sudden infant death syndrome is most
likely due to non-accidental trauma?

Choices:
1. 1 to 5 percent
2. 6 to 10 percent
3. 11 to 15 percent
4. Greater than 20 percent
Answer: 1 - 1 to 5 percent
Explanations:
It can only be estimated, but one to five percent of sudden infant death is
probably due to child abuse.

Go to the next page if you knew the correct answer, or click the link image(s)
below to further research the concepts in this question (if desired).

Research Concepts:
Sudden Infant Death Syndrome

We update eBooks quarterly and Apps daily based on user feedback.


Please tap flag to report any questions that need improvement.
Question 411: Which of the following is not a criterion in the systemic
inflammatory response syndrome?

Choices:
1. Temperature
2. Pulse
3. Respiratory rate
4. PaO2
Answer: 4 - PaO2
Explanations:
PaO2 is not part of the SIRS criteria.
SIRS is determined by temperature, pulse, respiratory rate, WBC count, and
PCO2.

Go to the next page if you knew the correct answer, or click the link image(s)
below to further research the concepts in this question (if desired).

Research Concepts:
Systemic Inflammatory Response Syndrome

We update eBooks quarterly and Apps daily based on user feedback.


Please tap flag to report any questions that need improvement.
Question 412: How many hours after admission is nosocomial pneumonia
defined according to the Center for Disease Control?

Choices:
1. 24
2. 48
3. 72
4. None of the above
Answer: 3 - 72
Explanations:
A nosocomial pneumonia is one defined as occurring more than 72 hours
post-admission.
Other names include hospital-acquired pneumonia or healthcare-associated
pneumonia.
It is the second most common nosocomial infection.
It is typically bacterial in origin.

Go to the next page if you knew the correct answer, or click the link image(s)
below to further research the concepts in this question (if desired).

Research Concepts:
Pneumonia, Nosocomial

We update eBooks quarterly and Apps daily based on user feedback.


Please tap flag to report any questions that need improvement.
Question 413: Which of the following ECG signs is not seen in patients
with a pulmonary embolism?

Choices:
1. Right bundle branch block
2. Right shift of the QRS
3. Peaked P waves
4. Prolonged PR interval
Answer: 4 - Prolonged PR interval
Explanations:
ECG findings in a patient with a pulmonary embolism (PE) are non-
specific.
Possible ECG findings include a right bundle branch block, a right shift of
the QRS, and peaked P waves.
PR interval prolongation or first-degree heart block is not seen in cases of
PE.
The most common ECG abnormalities are tachycardia and nonspecific ST-
T wave abnormalities.

Go to the next page if you knew the correct answer, or click the link image(s)
below to further research the concepts in this question (if desired).

Research Concepts:
Embolism, Pulmonary

We update eBooks quarterly and Apps daily based on user feedback.


Please tap flag to report any questions that need improvement.
Question 414: Which is false regarding extralobar and intralobar
sequestration?

Choices:
1. Both have an anomalous arterial supply and venous drainage
2. Usually located at the base of the lung
3. Intralobar sequestrations communicate with major bronchi
4. Diagnosis requires arteriography
Answer: 3 - Intralobar sequestrations communicate with major bronchi
Explanations:
It is rare for sequestrations to communicate with major bronchi.
Small perforations may be present in smaller bronchial segments.
Even though CT can diagnose sequestrations, the course of the anomalous
vessels is best depicted on angiogram. Resection can be performed without
major morbidity and is curative.
Intralobar lesions require lobectomy, whereas extralobar lesions that are
invested with a separate layer of visceral pleura are simply resected.

Go to the next page if you knew the correct answer, or click the link image(s)
below to further research the concepts in this question (if desired).

Research Concepts:
Pulmonary Sequestration

We update eBooks quarterly and Apps daily based on user feedback.


Please tap flag to report any questions that need improvement.
Question 415: Which of the following statements about ultrasound of the
chest is true?

Choices:
1. Ultrasound-guided marking of a site for later aspiration of pleural fluid has
been shown to reduce the risk of procedure-related pneumothorax
2. The presence of B-lines and lung sliding rules out the presence of
pneumothorax at the location being visualized
3. B-lines become less prominent in the presence of increased lung water
4. Ultrasound is less sensitive than lateral decubitus chest radiograph for the
detection of small pleural effusions.
Answer: 2 - The presence of B-lines and lung sliding rules out the presence
of pneumothorax at the location being visualized

Explanations:
Ultrasound (US) has become a valuable tool in the evaluation of a variety
of thoracic conditions. It is portable, does not involve radiation, and can be
used in real time. One of the first uses of US to be broadly adopted was
pleural space assessment. US can detect small quantities of pleural fluid,
and is more sensitive than lateral decubitus radiographs, though less
sensitive than computed tomography (CT) of the chest. The use of
ultrasound in pulmonary medicine has since expanded considerably.
US is also very useful in localizing a site for thoracentesis. The likelihood
of a successful thoracentesis is increased when US guidance is used
compared to thoracentesis done using only physical examination. The use
of real-time US for localization of a puncture site is associated with a lower
rate of complications including pneumothorax when compared to
thoracentesis done without ultrasound guidance. However, when the
marking of a site for thoracentesis is separated in time from the
performance of the procedure the incidence of pneumothorax is not
reduced.
Ultrasound has been shown to be an excellent technique for the detection of
pneumothorax, with a sensitivity exceeding that of chest radiography. The
lack of lung sliding and B-lines on US is highly predictive of
pneumothorax. The presence of these same findings effectively excludes
pneumothorax at the site of imaging; multiple sites on the chest should be
examined to exclude pneumothorax anywhere in the hemithorax.
B-lines, also known as lung rockets or lung comets, are markers of
interstitial lung edema, the US equivalent of Kerley-B lines on the chest
radiograph.

Go to the next page if you knew the correct answer, or click the link image(s)
below to further research the concepts in this question (if desired).

Research Concepts:
Lung Ultrasound

We update eBooks quarterly and Apps daily based on user feedback.


Please tap flag to report any questions that need improvement.
Question 416: Which of the following is false about the prenatal
presentation of congenital pulmonary airway malformations (CPAMs) of the
lung?

Choices:
1. The appearance of CPAMs on prenatal ultrasound ranges from incidental
findings of cystic-appearing lesions to massive pulmonary involvement
2. Fetal hydrops is common
3. Lesions regress in more than 55 percent of cases
4. Majority disappear by the time of delivery
Answer: 4 - Majority disappear by the time of delivery
Explanations:
The appearance of CPAMs on prenatal ultrasound ranges from incidental
findings of cystic-appearing lesions to massive pulmonary involvement.
Fetal hydrops develops in as many as 40 percent of cases.
Lesions regress during the course of gestation in as much as 59 percent of
cases.
Complete spontaneous resolution in the postnatal period has been reported
in a small number of cases.

Go to the next page if you knew the correct answer, or click the link image(s)
below to further research the concepts in this question (if desired).

Research Concepts:
Congenital Pulmonary Airway Malformation

We update eBooks quarterly and Apps daily based on user feedback.


Please tap flag to report any questions that need improvement.
Question 417: In a 4-month-old with rectal prolapse, the sweat chloride
reveals a concentration of 57 meq/L (normal is less than 60 mEq/L). What test
should be performed next?

Choices:
1. Barium enema
2. DNA testing
3. Colonoscopy
4. Repeat sweat chloride test
Answer: 4 - Repeat sweat chloride test
Explanations:
In order to diagnose cystic fibrosis, one has to perform two separate
chloride tests at different times. The level should be more than 60 mEq/L.
The correct approach in the above case would be to repeat the test.
Pulmonary function tests can be done after the age of 5-6 years.
The genetic defect is located on chromosome 7, but genetic testing is better
delayed until after making the diagnosis.

Go to the next page if you knew the correct answer, or click the link image(s)
below to further research the concepts in this question (if desired).

Research Concepts:
Cystic Fibrosis

We update eBooks quarterly and Apps daily based on user feedback.


Please tap flag to report any questions that need improvement.
Question 418: An adolescent with confirmed perennial aeroallergen
sensitivity and difficult to control severe persistent asthma has been referred for
treatment with omalizumab. Which of the following will be necessary to
calculate her dose and dosing frequency?

Choices:
1. Body weight and height
2. Serum total IgE and weight
3. Body weight and gender
4. Chronological age and race
Answer: 2 - Serum total IgE and weight
Explanations:
Omalizumab is a monoclonal anti IgE preparation.
It is indicated for adults and children 12 years of age and older with
difficult to control moderate to severe persistent asthma and confirmed
sensitivity to aeroallergens.
Dose and dosing frequency is calculated based on pretreatment serum total
IgE and age.
Omalizumab should only be administered in a healthcare setting equipped
to handle anaphylaxis.

Go to the next page if you knew the correct answer, or click the link image(s)
below to further research the concepts in this question (if desired).

Research Concepts:
Omalizumab

We update eBooks quarterly and Apps daily based on user feedback.


Please tap flag to report any questions that need improvement.
Question 419: An 18-month-old female is admitted with a left upper lobe
pneumonia and the blood culture is positive for Staphylococcus aureus. After 24-
hours the child becomes acutely hypoxic, tachypneic, and hypotensive. On the
way to see the patient, what equipment should be requested to be at the bedside?

Choices:
1. Thoracentesis kit to drain a pleural effusion
2. Chest tube set up
3. Repeat chest radiograph
4. Fluid bolus and new antibiotics
Answer: 2 - Chest tube set up
Explanations:
Acute decompensation in a patient with pneumonia can be secondary to
tension pneumothorax.
Chest tube insertion should be done urgently.
Other complications do not require as rapid of an intervention.

Go to the next page if you knew the correct answer, or click the link image(s)
below to further research the concepts in this question (if desired).

Research Concepts:
Pneumothorax, Tension And Traumatic

We update eBooks quarterly and Apps daily based on user feedback.


Please tap flag to report any questions that need improvement.
Question 420: An individual with the diagnosis of cystic fibrosis presents
with rapid onset of severe respiratory distress. On examination, he has decreased
air entry with decreased breath sounds and tactile fremitus of the left chest. What
is the most likely diagnosis?

Choices:
1. Pneumonia
2. Acute bronchospasm
3. Acute pneumothorax
4. Asthma exacerbation
Answer: 3 - Acute pneumothorax
Explanations:
The presentation is indicative of acute pneumothorax, which often presents
with tachycardia, dyspnea, and ipsilateral, pleuritic chest pain.
Other findings include hyperresonance on percussion of the lung and
decreased tactile fremitus with decreased air entry.
Due to the patient's history of cystic fibrosis and no other obvious cause
such as trauma or iatrogenic, this is considered secondary spontaneous
pneumothorax (SSP).
Depending on the patient's presentation and stability, immediate
intervention with needle decompression may be indicated. All cases of SSP
should be considered for admission for further evaluation and management.

Go to the next page if you knew the correct answer, or click the link image(s)
below to further research the concepts in this question (if desired).

Research Concepts:
Pneumothorax, Spontaneous

We update eBooks quarterly and Apps daily based on user feedback.


Please tap flag to report any questions that need improvement.
Question 421: Chronic poisoning with which of the following heavy metals
causes pulmonary fibrosis?

Choices:
1. Cadmium
2. Chromium
3. Arsenic
4. Mercury
Answer: 2 - Chromium
Explanations:
Chromium inhalation can cause pulmonary fibrosis and lung cancer.
Cadmium inhalation can cause pneumonitis.
Mercury can cause nausea, metallic taste, gingival-stomatitis, tremor,
neurasthenia, nephrotic syndrome, and hypersensitivity.

Go to the next page if you knew the correct answer, or click the link image(s)
below to further research the concepts in this question (if desired).

Research Concepts:
Pulmonary Fibrosis, Interstitial (Nonidiopathic)

We update eBooks quarterly and Apps daily based on user feedback.


Please tap flag to report any questions that need improvement.
Question 422: A young child develops staphylococcal pneumonia. A chest
tube is placed for a large pleural collection. The fluid continues to accumulate in
the chest. What is the next step?

Choices:
1. Thoracotomy
2. Thoracentesis
3. Video assisted thoracoscopy
4. Place a second chest tube
Answer: 4 - Place a second chest tube
Explanations:
Staphylococcal pneumonia occurs in children and presents as a rapid toxic
illness. Diagnosis is easily confirmed by a consolidation on x-ray.
Empyema results in 50% of children with staphylococcus pneumonia. In
children, thoracotomy is rarely needed and chest tube drainage will suffice.
Occasionally a second chest tube will be required. If sufficient drainage is
provided, an empyema does not result.
The most frequent complication is a pneumatocele, which a cluster of air
spaces in the area of consolidation. These may get infected or rupture but
they generally resolve on their own.

Go to the next page if you knew the correct answer, or click the link image(s)
below to further research the concepts in this question (if desired).

Research Concepts:
Pneumonia, Empyema

We update eBooks quarterly and Apps daily based on user feedback.


Please tap flag to report any questions that need improvement.
Question 423: What therapy is most effective in patients with
bronchiolitis?

Choices:
1. Corticosteroids
2. Bronchodilators
3. Ribavirin
4. Oxygen
Answer: 4 - Oxygen
Explanations:
Typically, supplemental oxygen is required in treating bronchiolitis.
Hypoxia is a common complication of bronchiolitis, and supplemental
oxygen should be provided for SpO2 of less than 90%.
Consider maintaining higher SpO2 for those with fever, acidosis, or
hemoglobinopathies due to oxyhemoglobin desaturation curve, or for
increased work of breathing.
The mainstay of treatment is supportive care including oxygen, hydration,
and antipyretics.

Go to the next page if you knew the correct answer, or click the link image(s)
below to further research the concepts in this question (if desired).

Research Concepts:
Bronchiolitis

We update eBooks quarterly and Apps daily based on user feedback.


Please tap flag to report any questions that need improvement.
Question 424: Which of the following physiologic parameters is not a
criterion suggesting extubation failure?

Choices:
1. Respiratory rate > 25 breaths/minute for 2 hours
2. Pulse > 140 bpm
3. Oxygen saturation < 90 percent
4. Normal pH
Answer: 4 - Normal pH
Explanations:
pH <7.33 is a criterion for extubation failure.
Also, a sustained pulse increase or decrease of > 20 percent pre-extubation
is a criterion for extubation failure.
Other signs are a PO2 < 80 mmHg or a fraction of inspired oxygen greater
than 50 percent.
Lastly, hypercapnia with a partial pressure of carbon dioxide greater than 45
mmHg or greater than 20 percent pre-extubation.

Go to the next page if you knew the correct answer, or click the link image(s)
below to further research the concepts in this question (if desired).

Research Concepts:
Extubation

We update eBooks quarterly and Apps daily based on user feedback.


Please tap flag to report any questions that need improvement.
Question 425: Which of the following about bronchiectasis is false?
Choices:
1. Cylindrical bronchiectasis is an early stage of the disease
2. Bronchoscopy is required for diagnosis
3. The affected lobe is shrunken, scarred, and airless
4. Middle lobe syndrome can be due to either intrinsic or extrinsic causes
Answer: 2 - Bronchoscopy is required for diagnosis
Explanations:
Because bronchiectasis involves second- to fourth- order bronchi, it is
difficult to assess the lung with bronchoscopy.
Bronchoscopy is done to aspirate secretions and rule out a foreign body.
The affected segment is usually airless, scarred, and shrunken.
High-resolution CT is used for diagnosis. The middle lobe syndrome occurs
from compression of the long bronchus by lymph nodes.
The lobe becomes atelectatic and may become infected. Atelectasis is rare
if collateral ventilation occurs from the incomplete fissure.

Go to the next page if you knew the correct answer, or click the link image(s)
below to further research the concepts in this question (if desired).

Research Concepts:
Bronchiectasis

We update eBooks quarterly and Apps daily based on user feedback.


Please tap flag to report any questions that need improvement.
Question 426: What is the mechanism of action of zileuton?
Choices:
1. Antagonizes LTD4 receptors
2. Inhibits 5-lipoxygenase
3. Inhibits phospholipase
4. Stimulates beta-receptors
Answer: 2 - Inhibits 5-lipoxygenase
Explanations:
Leukotrienes, particularly LTC4 and LTD4 are thought to play a role in
many of the symptoms of asthma, including bronchoconstriction.
Zileuton is an FDA-approved oral inhibitor of 5-lipoxygenase, the first
enzyme in the pathway from arachidonic acid to leukotrienes.
Zileuton is contraindicated in active liver disease or significantly elevated
transaminase levels.
Zafirlukast is another drug used to interrupt the leukotriene pathway, but it
works as a leukotriene D4 antagonist.

Go to the next page if you knew the correct answer, or click the link image(s)
below to further research the concepts in this question (if desired).

Research Concepts:
Zileuton

We update eBooks quarterly and Apps daily based on user feedback.


Please tap flag to report any questions that need improvement.
Question 427: Which of the following factors is associated with an
increased risk of extubation failure among patients who have passed a
spontaneous breathing trial?

Choices:
1. Use of pressure support as weaning method
2. Glasgow coma scale less than 10
3. Age greater than 75 years
4. Female gender
Answer: 2 - Glasgow coma scale less than 10
Explanations:
Approximately 15% of patients who have been extubated after mechanical
ventilation will require reintubation.
Among those patients who have passed a spontaneous breathing trial,
reintubation has been associated with a number of variables including
decreased Glasgow coma scale, increased secretion volume, poor cough,
and hypercarbia.
Age, gender, and weaning method have not been found to be predictors of
extubation failure.
Use of non-invasive ventilation in the immediate post-extubation period
may prevent re-intubation in patients with hypercarbic respiratory failure
such as chronic obstructive pulmonary disease.

Go to the next page if you knew the correct answer, or click the link image(s)
below to further research the concepts in this question (if desired).

Research Concepts:
Ventilator Weaning

We update eBooks quarterly and Apps daily based on user feedback.


Please tap flag to report any questions that need improvement.
Question 428: Which medication is used for the treatment of Pneumocystis
jiroveci pneumonia in a patient with severe HIV?

Choices:
1. Streptomycin
2. Cephalosporin
3. Corticosteroids
4. Albuterol
Answer: 3 - Corticosteroids
Explanations:
Trimethoprim/sulfamethoxazole is the antibiotic of choice for treating
Pneumocystis jiroveci pneumonia (PJP).
Corticosteroids can be used adjunctively to treat severe PJP in patients who
are HIV infected.
Typically it is recommended to add prednisone when a patient's arterial
blood gas shows a partial pressure of oxygen less than or equal to 70
mmHg, an alveolar-arterial gradient greater than or equal to 35 mmHg, or
hypoxia on pulse oximetry.
Adjunctive corticosteroids are not recommended in patients who do not
have HIV infection.

Go to the next page if you knew the correct answer, or click the link image(s)
below to further research the concepts in this question (if desired).

Research Concepts:
Pneumonia, Pneumocystis (Carinii) Jiroveci

We update eBooks quarterly and Apps daily based on user feedback.


Please tap flag to report any questions that need improvement.
Question 429: A febrile patient has a pulse of 120 bpm and blood pressure
of 65/40 mmHg. The patient has been given 3 liters of crystalloids with minimal
response. What is the next treatment for this patient?

Choices:
1. Norepinephrine
2. Dopamine
3. Digoxin
4. Vasopressin
Answer: 1 - Norepinephrine
Explanations:
Proper use of vasopressors in septic shock currently is debated.
Norepinephrine is more effective than dopamine in treating hypotension
associated with sepsis. Urine output is greater and lactic acid levels are
lower. There is also better perfusion of the splanchnic tissues. Dopamine
results in more tachycardia and arrhythmias.
Care should be given to identify the right sympathomimetic agent by
assessing perfusion before initiation. In early septic shock, the patient may
be in a warm, vasodilatory shock state wherein norepinephrine is
appropriate; however, patients who present in later stages of shock may be
in a cool, vasoconstrictive shock state wherein epinephrine is appropriate.
Most of the time, in fluid-refractory shock states, such as severe sepsis or
septic shock, a combination of norepinephrine and epinephrine are used.
Vasopressin can be further added but should not be used alone.
Individuals with sepsis, who do not respond to early resuscitative fluid
therapy in the emergency department, show evidence of end-organ
hypoperfusion and hemodynamic monitoring and should be admitted to an
intensive care unit.

Go to the next page if you knew the correct answer, or click the link image(s)
below to further research the concepts in this question (if desired).

Research Concepts:
Shock, Septic (Sepsis)

We update eBooks quarterly and Apps daily based on user feedback.


Please tap flag to report any questions that need improvement.
Question 430: Which of the following antimicrobials in not an option for
treating extensive lung abscesses due to Rhodococcus?

Choices:
1. Rifampin
2. Ceftriaxone
3. Levofloxacin
4. Linezolid
Answer: 2 - Ceftriaxone
Explanations:
Rifampin has good activity against macrophage invasion by Rhodococcus,
and it is often employed in treatment
Beta-lactams, except carbapenems, have poor activity against Rhodococcus.
Even when there is in-vitro susceptibility, resistance on therapy is common,
and beta-lactams should not be used.
Quinolones have a role in treatment both for definitive treatment and
suppressive therapy.
Linezolid also has good activity against Rhodococcus, and it is commonly
employed for initial treatment and during relapse.

Go to the next page if you knew the correct answer, or click the link image(s)
below to further research the concepts in this question (if desired).

Research Concepts:
Rhodococcus Equi

We update eBooks quarterly and Apps daily based on user feedback.


Please tap flag to report any questions that need improvement.
Question 431: An infant is born small for gestational age with renal
agenesis, club feet, and a flattened face. Which of the following lung
malformations is most likely to be found in infants with this syndrome?

Choices:
1. Congenital lobar emphysema
2. Pulmonary hypoplasia
3. Congenital pulmonary airway malformation
4. Sequestration
Answer: 2 - Pulmonary hypoplasia
Explanations:
Potter syndrome results from fetal renal disease leading to
oligohydramnios.
Patients have atypical morphologic and pathologic features including a
flattened nose, recessed chin, skeletal malformations, pulmonary
hypoplasia, and ophthalmologic malformations.
It can be caused by obstructive uropathy, bilateral renal agenesis, polycystic
kidney disease, and medullary dysplastic kidney.
Death is usually secondary to renal failure or severe lung hypoplasia.

Go to the next page if you knew the correct answer, or click the link image(s)
below to further research the concepts in this question (if desired).

Research Concepts:
Potter Syndrome

We update eBooks quarterly and Apps daily based on user feedback.


Please tap flag to report any questions that need improvement.
Question 432: A 17-year-old describes a sore throat, fever, and general
malaise for five days. Blood work is unremarkable but a sputum smear reveals
smudge cells. What is the most likely diagnosis?

Choices:
1. Influenza
2. Mycoplasma
3. Adenovirus
4. HIV
Answer: 3 - Adenovirus
Explanations:
Adenovirus causes both upper and lower respiratory tract infections.
Most infections are subclinical or have mild upper respiratory infection
symptoms.
Transplant patients can have lung involvement with bronchiolitis and
necrotizing bronchitis.
Smudge cells have rounded or ovoid large nuclei with a deeply basophilic
to granular amphophilic mass.

Go to the next page if you knew the correct answer, or click the link image(s)
below to further research the concepts in this question (if desired).

Research Concepts:
Adenoviruses

We update eBooks quarterly and Apps daily based on user feedback.


Please tap flag to report any questions that need improvement.
Question 433: A 4-year-old patient presents with symptoms of dysphagia.
The CT scan reveals a middle mediastinal mass located just beneath the carina.
Aspiration reveals mucus and respiratory epithelial cells. What is the most likely
diagnosis?

Choices:
1. Bronchogenic cyst
2. Esophageal cyst
3. Lipoma
4. Traction diverticulum
Answer: 1 - Bronchogenic cyst
Explanations:
Bronchogenic cysts are rare congenital lesions. They account for 5% to10%
of pediatric mediastinal masses.
The most common location of a bronchogenic cyst is the middle
mediastinum.
Symptoms in infants typically are the result of an airway or esophageal
compression.
Symptomatic bronchogenic cysts should be resected.

Go to the next page if you knew the correct answer, or click the link image(s)
below to further research the concepts in this question (if desired).

Research Concepts:
Cyst, Bronchogenic

We update eBooks quarterly and Apps daily based on user feedback.


Please tap flag to report any questions that need improvement.
Question 434: A 17-year-old presents with a chronic cough with purulent
sputum for 3 months. Prior history reveals numerous episodes of pneumonia and
hemoptysis. On examination, he has expiratory and inspiratory wheezes, coarse
rales, and clubbing. There is a question of bronchiectasis. Which of the
following is the best diagnostic test for confirming this diagnosis?

Choices:
1. Chest radiograph
2. Bronchoscopy
3. Pulmonary function tests
4. High-resolution CT
Answer: 4 - High-resolution CT
Explanations:
High-resolution CT is the current standard for diagnosis of bronchiectasis.
It assesses the morphology of airways as well as the degree of air-trapping.
CT also can detect any other concomitant pathology such as pneumonia.
Pulmonary function tests are useful to assess the patient and can be used to
follow the progression of the disease.
Patients with bronchiectasis usually have an obstructive airway disease with
a decline in FEV1.

Go to the next page if you knew the correct answer, or click the link image(s)
below to further research the concepts in this question (if desired).

Research Concepts:
Bronchiectasis

We update eBooks quarterly and Apps daily based on user feedback.


Please tap flag to report any questions that need improvement.
Question 435: Many factors influence the successful use of an oral
appliance for obstructive sleep apnea. Which of the following is not one of those
factors?

Choices:
1. Ability to retain it in place all night
2. Age
3. Adaptability with dentition changes
4. Ability to adjust the anterior-posterior position
Answer: 2 - Age
Explanations:
Oral appliances are equally effective in older and younger individuals.
This generally is true of all treatments of obstructive sleep apnea (OSA).
Weight loss and the use of continuous positive airway pressure (CPAP) are
the most effective treatments. The diligent adherence to nightly CPAP use
can result in near complete resolution of symptoms. For patients unable to
tolerate CPAP or those who will be unable to access electricity reliably,
there are oral appliances to bring the lower jaw forward that can help with
symptoms of mild to moderate sleep apnea.
Severe OSA can be treated with bilevel positive airway pressure as well as
surgical options which are typically less effective.

Go to the next page if you knew the correct answer, or click the link image(s)
below to further research the concepts in this question (if desired).

Research Concepts:
Apnea, Obstructive Sleep Apnea

We update eBooks quarterly and Apps daily based on user feedback.


Please tap flag to report any questions that need improvement.
Question 436: What is the most common liability in the medical care of
patients?

Choices:
1. Negligence
2. Insufficient informed consent
3. Breach of contract
4. Defamation
Answer: 1 - Negligence
Explanations:
Of all of the potential liabilities in medical malpractice, medical negligence
is the most common.
Medical negligence requires the plaintiff to establish four elements.
The four elements include: 1. There must be the existence of the provider's
duty to the plaintive, usually based on the existence of a physician-patient
relationship; 2. There must be an applicable standard of care and its
violation; 3. There must be damages, a compensable injury; and 4. There
must be a causal connection between the violation of the standard of care in
the injury.
Medical malpractice is a legal cause of action that occurs when a health
care professional deviates from standards their profession, thereby causing
injury to a patient.

Go to the next page if you knew the correct answer, or click the link image(s)
below to further research the concepts in this question (if desired).

Research Concepts:
Negligence

We update eBooks quarterly and Apps daily based on user feedback.


Please tap flag to report any questions that need improvement.
Question 437: In a ventilated patient, what does pressure support
ventilation at low settings help to do?

Choices:
1. Improve oxygenation
2. Expand the lungs
3. Reduce workload of breathing
4. Decrease atelectasis
Answer: 3 - Reduce workload of breathing
Explanations:
Low-pressure support ventilation reduces the workload of breathing.
It does this by overcoming resistance caused by small endotracheal tubes or
ventilator tubing.
Pressure support ventilation enables the patient to determine the inflation
volume and respiratory frequency but not pressure. It is used to augment
spontaneous breathing.
It also can be used to overcome the resistance of ventilator tubing in
another cycle. Five to 10 cmH2O generally is used, especially during
weaning, or to augment spontaneous breathing. Pressure support ventilation
is delivered through a specialized face mask.

Go to the next page if you knew the correct answer, or click the link image(s)
below to further research the concepts in this question (if desired).

Research Concepts:
Ventilation, Pressure Support

We update eBooks quarterly and Apps daily based on user feedback.


Please tap flag to report any questions that need improvement.
Question 438: Bronchial cartilage is derived from which of the following
embryonic layers?

Choices:
1. Ectoderm
2. Endoderm
3. Mesoderm
4. Neuroectoderm
Answer: 3 - Mesoderm
Explanations:
Mesoderm differentiates to become multiple tissues and structures
including muscle, bone, the middle layer of the skin, and connective tissue.
Most of the circulatory system is derived from mesoderm.
Bronchial cartilage is derived from mesoderm.

Go to the next page if you knew the correct answer, or click the link image(s)
below to further research the concepts in this question (if desired).

Research Concepts:
Embryology, Pulmonary

We update eBooks quarterly and Apps daily based on user feedback.


Please tap flag to report any questions that need improvement.
Question 439: A patient with status asthmaticus is admitted to the ER.
Despite all medical interventions, the child still continues to wheeze. You decide
to intubate the child and ventilate him. Which anesthetic should be used?

Choices:
1. IV diazepam
2. Inhalational sevoflurane
3. IV morphine
4. Inhalation nitrous oxide
Answer: 2 - Inhalational sevoflurane
Explanations:
Occasionally, asthmatic episodes do not reverse with medical treatment and
require urgent mechanical ventilation.
Inhalational anesthetics, e.g., sevoflurane, are best used to decrease the
bronchoconstriction in asthma.

Go to the next page if you knew the correct answer, or click the link image(s)
below to further research the concepts in this question (if desired).

Research Concepts:
Inhalational Anesthetic

We update eBooks quarterly and Apps daily based on user feedback.


Please tap flag to report any questions that need improvement.
Question 440: A patient with a well-established tracheostomy has a sudden
onset of large amounts of bright red blood coming from within the tracheal
collar. What is the most likely vascular structure where an erosion may have
occurred?

Choices:
1. Innominate artery
2. Pulmonary artery
3. Subclavian artery
4. Carotid artery
Answer: 1 - Innominate artery
Explanations:
Hemorrhage resulting from a tracheostomy may be due to an erosion of the
innominate artery resulting in a tracheo-innominate fistula (TIF).
Sudden, large amounts of bleeding in this situation should always be
considered a TIF until proven otherwise. The event may be preceded by a
smaller herald or sentinel bleed.
TIF can be a highly lethal complication and requires prompt diagnosis and
treatment. Overinflation of the tracheal cuff may stop the bleeding and
assist in confirming the diagnosis.
Rapid surgical control of bleeding is required. Median sternotomy is the
preferred surgical approach. Endovascular stenting may be utilized in select
patient populations.

Go to the next page if you knew the correct answer, or click the link image(s)
below to further research the concepts in this question (if desired).

Research Concepts:
Fistula, Tracheo Innominate Artery

We update eBooks quarterly and Apps daily based on user feedback.


Please tap flag to report any questions that need improvement.
Question 441: Which of the following pressures is often an indication that
barotraumas may be imminent?

Choices:
1. High peak inspiratory pressure
2. Plateau pressure
3. PEEP level
4. Tidal volume
Answer: 2 - Plateau pressure
Explanations:
Airway pressures have been used to identify patients at risk for barotrauma
in the past.
Plateau pressures provide the best estimate of trans-alveolar gradient and
risk for pneumothorax.
Other measures include high peak inspiratory pressure and levels of PEEP.
Adjustments to the ventilator settings are needed in an attempt to keep
plateau pressure < 30 cmH2O.

Go to the next page if you knew the correct answer, or click the link image(s)
below to further research the concepts in this question (if desired).

Research Concepts:
Ventilation, Barotrauma And Mechanical

We update eBooks quarterly and Apps daily based on user feedback.


Please tap flag to report any questions that need improvement.
Question 442: Which of the following is not true about the indications for
tracheostomy?

Choices:
1. May be indicated for management of secretions
2. May allow for faster weaning off the ventilator
3. May be indicated in patients with severe obstructive sleep apnea intolerant of
continuous positive airway pressure
4. Is always done when the patient has been on the ventilator for 2 weeks
Answer: 4 - Is always done when the patient has been on the ventilator for 2
weeks

Explanations:
Tracheostomy is a commonly performed procedure in hospitals. It is often
used to help make weaning faster and help individuals clear their oral
secretions.
Tracheostomy can help make clearing the excessive mucus easy and also
decrease the resistance to airflow.
Tracheostomy sometimes is done when an individual has epiglottis.
Tracheostomy is generally not performed for at least three to four weeks
when a patient is on a ventilator. Several attempts are made to wean the
patient off the ventilator before one schedules a patient for tracheostomy.

Go to the next page if you knew the correct answer, or click the link image(s)
below to further research the concepts in this question (if desired).

Research Concepts:
Tracheostomy

We update eBooks quarterly and Apps daily based on user feedback.


Please tap flag to report any questions that need improvement.
Question 443: Which of the following is not a sign of pneumonitis?
Choices:
1. Cough
2. Fever
3. Myalgia
4. Hematuria
Answer: 4 - Hematuria
Explanations:
Pneumonitis is characterized by dyspnea, fever, chills, and myalgia.

Go to the next page if you knew the correct answer, or click the link image(s)
below to further research the concepts in this question (if desired).

Research Concepts:
Pneumonitis

We update eBooks quarterly and Apps daily based on user feedback.


Please tap flag to report any questions that need improvement.
Question 444: A patient has acute respiratory distress syndrome. When the
alarm for high peak inspiratory pressure comes on a volume-cycled ventilator,
what is the risk to the patient?

Choices:
1. Bronchospasm
2. Barotrauma
3. Increased venous return
4. Increased mucus
Answer: 2 - Barotrauma
Explanations:
If the high peak inspiratory pressure alarm is left unnoticed in a patient with
decreased lung compliance, there is an increased risk of barotrauma.
Barotrauma is a known complication of mechanical ventilation. It is
frequently encountered in patients with the acute respiratory distress
syndrome.
Barotrauma describes the manifestations of extra-alveolar air during
mechanical ventilation. Although patients receiving nonmechanical
ventilation may have barotrauma, most occur in patients receiving
mechanical ventilation.
The clinical presentation varies from absent symptoms with the subtle
radiographic findings of pulmonary interstitial emphysema to respiratory or
cardiac distress due to a tension pneumothorax. Other manifestations
include pneumopericardium, pneumomediastinum, pneumoperitoneum, and
subcutaneous emphysema. Lung injury associated with barotrauma is
mediated by an increase in alveolar pressure.

Go to the next page if you knew the correct answer, or click the link image(s)
below to further research the concepts in this question (if desired).

Research Concepts:
Ventilation, Barotrauma And Mechanical

We update eBooks quarterly and Apps daily based on user feedback.


Please tap flag to report any questions that need improvement.
Question 445: Which of the following medications is commonly implicated
in malignant hyperthermia?

Choices:
1. Dantrolene
2. Baclofen
3. Succinylcholine
4. Atracurium
Answer: 3 - Succinylcholine
Explanations:
Malignant hyperthermia is characterized by hyperthermia and muscular
rigidity.
It is caused by excessive release of calcium from sarcoplasmic reticulum.
The drug commonly implicated in this situation is succinylcholine.
The treatment for this condition is dantrolene.

Go to the next page if you knew the correct answer, or click the link image(s)
below to further research the concepts in this question (if desired).

Research Concepts:
Succinylcholine

We update eBooks quarterly and Apps daily based on user feedback.


Please tap flag to report any questions that need improvement.
Question 446: Which of the following management strategies is not
appropriate for infants presenting with apnea?

Choices:
1. Reassurance
2. Tactile stimulation
3. Bag-mask ventilation
4. Pulse oximetry
Answer: 1 - Reassurance
Explanations:
The first step in evaluating infants who present with apnea is to determine
the need for resuscitation and stabilization. This may include tactile
stimulation, bag-mask ventilation, and even initiation of CPR.
Reassurance and parent education are indicated if thorough evaluation
characterizes the event into the "low risk" BRUE (brief resolved
unresponsive event) category.
If the patient is stable at the time of evaluation, a thorough history and
physical examination should be performed to determine the underlying
cause. It is particularly important to differentiate the event from benign
causes, such as periodic breathing or breath-holding spells, before
embarking on any work up.
The performance of labs and imaging is not routinely indicated and is
guided by the clinical picture. Cardiorespiratory monitoring and pulse
oximetry is recommended during the period of observation to identify life-
threatening causes of apnea and to reassure the family and the health care
team.

Go to the next page if you knew the correct answer, or click the link image(s)
below to further research the concepts in this question (if desired).

Research Concepts:
Apnea, Infant

We update eBooks quarterly and Apps daily based on user feedback.


Please tap flag to report any questions that need improvement.
Question 447: What is the most appropriate evaluation in a patient with a
newly diagnosed pleural effusion?

Choices:
1. Fiberoptic bronchoscopy
2. Diagnostic thoracentesis
3. Positron emission tomography scan of lungs
4. Pulmonary function testing
Answer: 2 - Diagnostic thoracentesis
Explanations:
With few exceptions, patients who present with a new pleural effusion
should undergo diagnostic thoracentesis.
Thoracentesis will allow determination of whether the pleural fluid is an
exudate or transudate.
A pleural effusion is the most common manifestation of pleural lung
disease.
Determining whether pleural fluid is exudate or transudate aids in leading
to the etiology of the effusion. Draining of pleural fluid by thoracentesis
also may relieve dyspnea.

Go to the next page if you knew the correct answer, or click the link image(s)
below to further research the concepts in this question (if desired).

Research Concepts:
Effusion, Pleural

We update eBooks quarterly and Apps daily based on user feedback.


Please tap flag to report any questions that need improvement.
Question 448: A patient from Miami, Florida presents for care regarding
bilateral fingertip tingling to a physician in Denver, Colorado, elevation 5280
feet. The patient has no past medical history, takes no medication, and is a
vegetarian. She denies nausea, vomiting, chest discomfort or shortness of breath.
Neurologic exam shows intact sensation to pin-prick, temperature, 2-point
discrimination, position sense, and vibratory sense. Gait, deep tendon reflexes,
finger-to-nose, tandem gait, and heal-to-toe walking are all within normal limits.
Select the appropriate next step.

Choices:
1. Electromyography
2. B12 level
3. Acetazolamide in treatment of altitude sickness
4. Reassurance
Answer: 4 - Reassurance
Explanations:
The patient is hyperventilating in response to the decreased atmospheric
pressure.
The patient increases her respiratory rate but decreases her PCO2, creating
a respiratory alkalosis with subsequent dissociation of protein-bound
calcium to ionized calcium. Increased ionized calcium then causes the
sensation of tingling in the fingertips.
Altitude sickness is characterized by headache, nausea with vomiting, and,
in severe cases, pulmonary edema.
She has none of these; therefore, reassurance is the answer.

Go to the next page if you knew the correct answer, or click the link image(s)
below to further research the concepts in this question (if desired).

Research Concepts:
Altitude Illness, Pulmonary Syndromes, High Altitude Pulmonary
Edema (HAPE)

We update eBooks quarterly and Apps daily based on user feedback.


Please tap flag to report any questions that need improvement.
Question 449: Which is NOT a common finding in a patient with
Eosinophilic granulomatosis with polyangiitis (EGPA)?

Choices:
1. Vasculitis
2. Eosinophilia
3. Asthma
4. Appendicitis
Answer: 4 - Appendicitis
Explanations:
EGPA can involve any organ in the body.
Common presentations are vasculitis, eosinophilia, and asthma.
P-ANCA and myeloperoxidase may be positive.

Go to the next page if you knew the correct answer, or click the link image(s)
below to further research the concepts in this question (if desired).

Research Concepts:
Churg Strauss Syndrome (Allergic Granulomatosis)

We update eBooks quarterly and Apps daily based on user feedback.


Please tap flag to report any questions that need improvement.
Question 450: Which is NOT a feature of a diaphragmatic hernia in a
newborn?

Choices:
1. Scaphoid abdomen
2. Hemothorax
3. Midline shift
4. Normal NG tube placement
Answer: 4 - Normal NG tube placement
Explanations:
The NG tube will sometimes be seen above the diaphragm in the CXR.

Go to the next page if you knew the correct answer, or click the link image(s)
below to further research the concepts in this question (if desired).

Research Concepts:
Hernia, Diaphragmatic, Congenital

We update eBooks quarterly and Apps daily based on user feedback.


Please tap flag to report any questions that need improvement.
Question 451: In a young black female with suspected sarcoidosis, which
of the following tests should be used first?

Choices:
1. Mediastinoscopy
2. Lung biopsy
3. Bronchoscopy with lavage
4. Mediastinotomy
Answer: 3 - Bronchoscopy with lavage
Explanations:
Bronchoscopy is the first test in all patients with sarcoidosis.
However, mediastinoscopy is the most definitive study for confirming the
diagnosis.
Sarcoidosis occurs in 3 stages: stage 1 with hilar adenopathy, stage 2 with
parenchymal changes plus adenopathy, stage 3 with just parenchymal
changes.
Mediastinoscopy also obtains enough tissue to rule out lymphoma.

Go to the next page if you knew the correct answer, or click the link image(s)
below to further research the concepts in this question (if desired).

Research Concepts:
Sarcoidosis

We update eBooks quarterly and Apps daily based on user feedback.


Please tap flag to report any questions that need improvement.
Question 452: Which of the following accounts for more deaths than any
other vaccine-preventable disease?

Choices:
1. Human papillomavirus (HPV)
2. Polio
3. Streptococcus pneumoniae
4. Varicella
Answer: 3 - Streptococcus pneumoniae
Explanations:
S. pneumoniae, otherwise known as pneumococci, accounts for more deaths
than any other vaccine-preventable disease.
Ninety percent of deaths attributed to pneumococcal disease could be
prevented by appropriate vaccination.
The pneumococcal vaccine is administered to only 30 percent of individuals
who are eligible to receive it.
Individuals 65 years and older should receive a one-time dose of the
pneumococcal polysaccharide vaccine.

Go to the next page if you knew the correct answer, or click the link image(s)
below to further research the concepts in this question (if desired).

Research Concepts:
Pneumonia, Streptococcus Pneumoniae

We update eBooks quarterly and Apps daily based on user feedback.


Please tap flag to report any questions that need improvement.
Question 453: Select the structure that is subsequently septated to form the
pulmonary artery and aorta.

Choices:
1. Bulboventricular flange
2. Right sinal horn
3. Bulbus cordis
4. Aortic sac
Answer: 3 - Bulbus cordis
Explanations:
The truncus arteriosus and the conus arteriosus are the two parts of the
bulbus cordis.
Ridges appear as bulges in the truncus arteriosus on the left inferior walls
and right superior wall and fuse to be the septum.
The septum spirals as it separates the distal pulmonary artery from the
aorta.

Go to the next page if you knew the correct answer, or click the link image(s)
below to further research the concepts in this question (if desired).

Research Concepts:
Embryology, Bulbus Cordis (Ventricle Bulb)

We update eBooks quarterly and Apps daily based on user feedback.


Please tap flag to report any questions that need improvement.
Question 454: Which physical sign is a clue to the diagnosis in a patient
with bilateral diaphragmatic paralysis?

Choices:
1. Signs of cor pulmonale
2. Limited diaphragmatic excursion
3. Paradoxical inward movement of the abdomen
4. Use of accessory respiratory muscles
Answer: 3 - Paradoxical inward movement of the abdomen
Explanations:
Bilateral diaphragmatic paralysis is serious and most individuals have some
signs and symptoms.
The diagnostic feature is a paradoxical inward movement of the abdomen
during inspiration.
Other features include limited diaphragmatic excursion and lower chest
dullness.
Most patients are tachypneic and use accessory respiratory muscles.

Go to the next page if you knew the correct answer, or click the link image(s)
below to further research the concepts in this question (if desired).

Research Concepts:
Diaphragmatic Paralysis

We update eBooks quarterly and Apps daily based on user feedback.


Please tap flag to report any questions that need improvement.
Question 455: In a symptomatic patient with Q fever, what is the treatment
of choice?

Choices:
1. Erythromycin
2. Sulfasalazine
3. Doxycycline
4. Rifampicin
Answer: 3 - Doxycycline
Explanations:
Most cases of Q fever do not require treatment and resolve with supportive
care.
Doxycycline has been recommended for symptomatic patients.
Other drugs include quinolones, macrolides, and sulfonamides.

Go to the next page if you knew the correct answer, or click the link image(s)
below to further research the concepts in this question (if desired).

Research Concepts:
Q Fever

We update eBooks quarterly and Apps daily based on user feedback.


Please tap flag to report any questions that need improvement.
Section 8

Question 456: Which of the following is the most likely radiologic finding
of Rhodococcus equi infection?

Choices:
1. Lung cavitation
2. Pleural effusion
3. Ground glass opacities
4. Halo and air crescent
Answer: 1 - Lung cavitation
Explanations:
While not unique to Rhodococcus, lung cavitation and subsequent abscess
formation are the hallmarks of Rhodococcus infection. Cavities are seen in
over 50% of lung infections.
Pleural effusion sometimes is seen with pulmonary disease but is not the
most common finding.
Ground glass opacities are seen in a wide range of pulmonary diseases like
cytomegalovirus and pneumocystis but are not the most common findings
in Rhodococcus lung infection.
Halo and air crescent sign are the hallmark of Aspergillosis rather than
Rhodococcus.

Go to the next page if you knew the correct answer, or click the link image(s)
below to further research the concepts in this question (if desired).

Research Concepts:
Rhodococcus Equi

We update eBooks quarterly and Apps daily based on user feedback.


Please tap flag to report any questions that need improvement.
Question 457: A 4-year-old HIV infected child was diagnosed to have
pulmonary tuberculosis. Which of the following statements is true regarding the
management of this child?

Choices:
1. He should be started on HAART first and then the TB will be treated later
2. He should be treated for TB now and start HAART once he finishes the
treatment
3. TB treatment should be started right away and HAART can be initiated after
making sure that he tolerates the antituberculosis medication
4. Both Anti-TB and HAART should be started right away
Answer: 3 - TB treatment should be started right away and HAART can be
initiated after making sure that he tolerates the antituberculosis medication

Explanations:
The wisest thing to do would be to start the TB medication right away.
After few weeks of therapy liver function can be checked and HAART can
be started.
Starting both medication together can result in liver toxicity.
Waiting 6 months until the Anti-TB medication is finished is not
recommended because the child might succumb due to other opportunistic
infections.

Go to the next page if you knew the correct answer, or click the link image(s)
below to further research the concepts in this question (if desired).

Research Concepts:
Tuberculosis, Lung, Primary

We update eBooks quarterly and Apps daily based on user feedback.


Please tap flag to report any questions that need improvement.
Question 458: Which of the following is not a common causes of chronic
cough?

Choices:
1. Asthma
2. Upper airway cough syndrome
3. Laryngeal cancer
4. Gastroesophageal reflux disease (GERD)
Answer: 3 - Laryngeal cancer
Explanations:
Upper airway cough syndrome, GERD, and asthma account for more than
90% of cases of chronic cough.
Laryngeal cancer does not commonly cause a chronic cough.

Go to the next page if you knew the correct answer, or click the link image(s)
below to further research the concepts in this question (if desired).

Research Concepts:
Cough, Chronic

We update eBooks quarterly and Apps daily based on user feedback.


Please tap flag to report any questions that need improvement.
Question 459: A young woman has sinusitis treated with
amoxicillin/clavulanate for 2 weeks. For the previous 2 months, she had fevers,
fatigue, and malaise. Laboratories show creatinine of 2.7, urinalysis shows 1+
protein with 30 red blood cells per high power field and red cell casts. CT of the
chest shows cavitary lesions. Which of the following is the best diagnostic test
for this patient?

Choices:
1. Perinuclear antineutrophil cytoplasmic antibodies
2. Antibodies to antistreptolysin 0
3. Antiproteinase-3 antibodies
4. Antiglomerular basement membrane antibodies
Answer: 3 - Antiproteinase-3 antibodies
Explanations:
Granulomatosis with polyangiitis affects the small vessels of the airways
and kidneys.
The lungs are affected in 85 percent of patients, and in 77 percent there is
rapidly progressive glomerulonephritis.
The upper airways manifest a chronic sinusitis not responsive to antibiotics.
Cytoplasmic antineutrophil cytoplasmic antibodies (c-ANCA) with the
target being proteinase-3.

Go to the next page if you knew the correct answer, or click the link image(s)
below to further research the concepts in this question (if desired).

Research Concepts:
Granulomatosis with Polyangiitis (GPA, Wegener Granulomatosis)

We update eBooks quarterly and Apps daily based on user feedback.


Please tap flag to report any questions that need improvement.
Question 460: What is the most common cause of mortality following a
blood transfusion based on data released by the Center for Disease Control and
Prevention?

Choices:
1. Circulatory overload
2. Sepsis
3. Transfusion-related acute lung injury
4. Non-antibody mediated acute hemolytic reaction
Answer: 3 - Transfusion-related acute lung injury
Explanations:
Transfusion-related acute lung injury (TRALI) is a rare complication of a
blood transfusion but accounts for nearly 37% of all deaths after blood
transfusion.
To prevent poor outcome, aggressive intensive care support is necessary
Acute hemolytic reactions can be fatal but are rare. Death is often due to
multiple organ failure such as renal failure and disseminated intravascular
coagulation. Mortality increases with the volume of incompatible blood
transfused.
Fortunately, the majority of cases of acute hemolytic reactions are
diagnosed within minutes of transfusion and mortality is averted.

Go to the next page if you knew the correct answer, or click the link image(s)
below to further research the concepts in this question (if desired).

Research Concepts:
Transfusion, Reactions

We update eBooks quarterly and Apps daily based on user feedback.


Please tap flag to report any questions that need improvement.
Question 461: A febrile male with alcoholism has been admitted with a
diagnosis of a lung abscess. Cultures have been obtained and appropriate
antibiotics have been started. Four weeks after starting treatment, the patient is
afebrile but still has persistent sputum production. The chest x-ray is unchanged.
What is the next step?

Choices:
1. Continue present treatment
2. Percutaneous drainage
3. Advanced imaging
4. Thoracostomy
Answer: 1 - Continue present treatment
Explanations:
Generally, the chest x-ray lags behind clinical improvement. Advanced
imaging (CT, MRI) is unnecessary if there is no clinical deterioration.
If the patient is afebrile and there are no complications, it is best to continue
the present treatment.
Percutaneous drainage may be required for treatment of a lung abscess but
not as a first modality and never without prior advanced imaging.
Thoracostomy was previously the mainstay treatment but is seldom
required today.

Go to the next page if you knew the correct answer, or click the link image(s)
below to further research the concepts in this question (if desired).

Research Concepts:
Abscess, Lung

We update eBooks quarterly and Apps daily based on user feedback.


Please tap flag to report any questions that need improvement.
Question 462: Which of the following about bronchiectasis is false?
Choices:
1. Bronchiectasis is most frequently seen in the dependent parts of the lung,
including middle, lingula, and lower lobes
2. Bronchiectasis is bilateral in one third of cases
3. It frequently occurs in the main bronchi
4. Upper lobe bronchiectasis is usually congenital in origin
Answer: 3 - It frequently occurs in the main bronchi
Explanations:
Bronchiectasis occurs in distal segments (second-fourth order bronchi).

Go to the next page if you knew the correct answer, or click the link image(s)
below to further research the concepts in this question (if desired).

Research Concepts:
Bronchiectasis

We update eBooks quarterly and Apps daily based on user feedback.


Please tap flag to report any questions that need improvement.
Question 463: Which of the following is false about congenital cystic
adenomatoid malformation?

Choices:
1. The macrocystic variety has a ‘Swiss cheese’ configuration
2. Can present with pneumonia in young children
3. The predominant component may be either cystic or solid
4. Other cardiac lesions and anomalous vessels are common
Answer: 4 - Other cardiac lesions and anomalous vessels are common
Explanations:
This condition is characterized by an overgrowth of bronchioles and may be
either cystic or solid.
The condition usually presents in newborns and infants with respiratory
distress symptoms.
Air trapping can occur with resulting pneumothoraces. The condition can
be confused with congenital diaphragmatic hernia.
It is usually observed in the lower lobes. It is treated by surgery and
requires a lobectomy. These infants do not have associated anomalies.

Go to the next page if you knew the correct answer, or click the link image(s)
below to further research the concepts in this question (if desired).

Research Concepts:
Congenital Pulmonary Airway Malformation

We update eBooks quarterly and Apps daily based on user feedback.


Please tap flag to report any questions that need improvement.
Question 464: What is the most common symptom in patients with
unilateral diaphragmatic paralysis?

Choices:
1. Tachycardia
2. Tachypnea
3. Dyspnea
4. Nothing
Answer: 4 - Nothing
Explanations:
Most patients with unilateral diaphragmatic paralysis are asymptomatic.
A few patients will become symptomatic with daily living activities.
However, with intense exercise, most patients will become dyspneic.

Go to the next page if you knew the correct answer, or click the link image(s)
below to further research the concepts in this question (if desired).

Research Concepts:
Unilateral Diaphragmatic Paralysis

We update eBooks quarterly and Apps daily based on user feedback.


Please tap flag to report any questions that need improvement.
Question 465: In a 17-year-old HIV positive patient, which of the
following is the minimum amount of induration that would constitute a positive
tuberculosis (TB) test?

Choices:
1. 5 mm induration
2. 10 mm induration
3. 12 mm induration
4. 25 mm induration
Answer: 1 - 5 mm induration
Explanations:
A positive TB test in an HIV positive patient is 5 mm induration.
A 5 mm induration is also considered positive in organ transplant patients
and immunosuppressed patients.
A 15 mm induration is considered positive in patients with no risk factors or
prior exposure to TB.
Induration, or palpable firmness, should be measured perpendicular to the
long axis of the forearm where the intradermal test was applied. Erythema,
if present, should not impact test interpretation.

Go to the next page if you knew the correct answer, or click the link image(s)
below to further research the concepts in this question (if desired).

Research Concepts:
Tuberculosis, Screening

We update eBooks quarterly and Apps daily based on user feedback.


Please tap flag to report any questions that need improvement.
Question 466: When is the best time to perform chest percussion in an
infant with cystic fibrosis?

Choices:
1. Before meals
2. During meals
3. Between meals
4. After meals
Answer: 3 - Between meals
Explanations:
Chest percussion is performed between meals for infants with cystic
fibrosis.
Cystic fibrosis is an autosomal recessive, genetic disorder primarily
affecting the lungs but also the pancreas, liver, kidneys, and intestines.
Cystic fibrosis is characterized by abnormally thick mucous resulting in
blockages that cause lung remodeling, pancreatic damage from
accumulated digestive enzymes, and intestinal obstruction from thickened
feces.
Cystic fibrosis (CF) is part of newborn screening, and if suspected
confirmed by a sweat test. Sweat testing is the primary diagnostic test for
CF but CF can be diagnosed by identifying mutations in the CFTR gene.
Currently, cystic fibrosis (CF) is not curable. Treatment includes respiratory
therapy (mechanical agitation, bronchodilators, anticholinergics,
mucolytics, and inhaled corticosteroids), antibiotics for acute and chronic
lung infections, pancreatic enzyme supplementation, nutritional support,
and an active lifestyle.

Go to the next page if you knew the correct answer, or click the link image(s)
below to further research the concepts in this question (if desired).

Research Concepts:
Cystic Fibrosis

We update eBooks quarterly and Apps daily based on user feedback.


Please tap flag to report any questions that need improvement.
Question 467: Which is not true regarding obstructive sleep apnea?
Choices:
1. Apneic episodes are associated with narrow heart rate variations
2. Apneic episodes are associated with oxygen desaturation
3. Apneic episodes are associated with arousals on EEG
4. Males are more commonly affected
Answer: 1 - Apneic episodes are associated with narrow heart rate variations
Explanations:
Apneic episodes are associated with wide heart rate variations.
Obstructive sleep apnea (OSA) is characterized by episodes of complete
collapse of the airway upon entering deep sleep. This leads to oxygen
desaturation and arousal, resulting in nonrestorative sleep.
Muscle tone decreases during rapid eye movement sleep. An episode of
apnea in OSA is caused by at least 90% anterior to posterior collapse of the
airway for more than 10 seconds as the patient transitions into a deep sleep.
Nighttime polysomnography is the gold standard test for the diagnosis of
OSA.

Go to the next page if you knew the correct answer, or click the link image(s)
below to further research the concepts in this question (if desired).

Research Concepts:
Apnea, Obstructive Sleep Apnea

We update eBooks quarterly and Apps daily based on user feedback.


Please tap flag to report any questions that need improvement.
Question 468: Barotrauma during mechanical ventilation is MOST
commonly seen in patients with which diagnosis?

Choices:
1. Bronchiectasis
2. Emphysema
3. Lung abscess
4. ARDS
Answer: 4 - ARDS
Explanations:
Barotrauma is a well known complication of mechanical ventilation.
Barotrauma is most commonly seen after ventilating patients with ARDS.
Barotrauma is also common in patients with asthma, COPD, interstitial lung
disease, and PCP.

Go to the next page if you knew the correct answer, or click the link image(s)
below to further research the concepts in this question (if desired).

Research Concepts:
Ventilation, Barotrauma And Mechanical

We update eBooks quarterly and Apps daily based on user feedback.


Please tap flag to report any questions that need improvement.
Question 469: Which of the following does not cause elevated levels of
sweat chloride?

Choices:
1. Cystic fibrosis
2. Adrenal insufficiency
3. Hypothyroidism
4. Diabetes mellitus
Answer: 4 - Diabetes mellitus
Explanations:
There are many causes of elevated sweat chloride besides cystic fibrosis.
The other causes include Addison disease, glycogen storage disease,
hypothyroidism, ADH resistant diabetes insipidus, malnutrition, and
familial cholestasis.
Atopic dermatitis is a common skin disorder associated with elevated sweat
chloride.

Go to the next page if you knew the correct answer, or click the link image(s)
below to further research the concepts in this question (if desired).

Research Concepts:
Sweat Testing

We update eBooks quarterly and Apps daily based on user feedback.


Please tap flag to report any questions that need improvement.
Question 470: Which of the following statements regarding skin testing for
allergic reactivity is not true?

Choices:
1. Antihistamines given prior to testing may inhibit the reaction
2. Positive skin test results by intradermal testing correlate better than results by
puncture tests with clinical symptoms
3. Intradermal tests are more sensitive than puncture tests
4. The reaction peaks within approximately 20 minutes and usually resolves over
20 to 30 minutes
Answer: 2 - Positive skin test results by intradermal testing correlate better
than results by puncture tests with clinical symptoms

Explanations:
Skin testing in children is usually first performed using the prick/puncture
technique.
With this technique, a small drop of allergen is applied to the skin surface,
and a tiny amount is introduced into the epidermis by lightly pricking or
puncturing the skin through the drop of extract with a small needle. When
the prick/puncture skin test result is negative, and the history is suggestive,
selective skin testing using the intradermal technique may be performed.
This technique involves using a 26-gauge needle to inject 0.01 to 0.02 mL
of a dilute allergen extract into the dermis of the arm. This technique is
more sensitive than the prick/puncture technique, and the allergen extracts
used are 1,000- to 100-fold less concentrated than extracts used for
prick/puncture testing. Intradermal skin tests are not recommended for use
with food allergens because of the risk of triggering anaphylaxis. Irritant
rather than allergic reactions can occur with intradermal skin testing if
higher concentrations of extracts, such as 1:100 weight: volume, are used.
Although prick/puncture testing is less sensitive than intradermal skin
testing, positive prick/puncture skin test results tend to correlate better with
symptoms on natural exposure to the allergen.

Go to the next page if you knew the correct answer, or click the link image(s)
below to further research the concepts in this question (if desired).

Research Concepts:
Allergy, Testing

We update eBooks quarterly and Apps daily based on user feedback.


Please tap flag to report any questions that need improvement.
Question 471: Which of the following diseases is described by the
Mendelson syndrome?

Choices:
1. Hospital-acquired pneumonia
2. Community-acquired pneumonia
3. Aspiration pneumonia
4. Atypical pneumonia
Answer: 3 - Aspiration pneumonia
Explanations:
Mendelson syndrome is characterized by reactive, non-cardiogenic,
pulmonary edema resulting from aspiration of gastric contents.
Disease severity is correlated with aspirate acidity.

Go to the next page if you knew the correct answer, or click the link image(s)
below to further research the concepts in this question (if desired).

Research Concepts:
Mendelson Syndrome

We update eBooks quarterly and Apps daily based on user feedback.


Please tap flag to report any questions that need improvement.
Question 472: What is the single best test to detect pulmonary artery
stenosis following a lung transplant?

Choices:
1. Nuclear perfusion study
2. MRI
3. Echo
4. Angiography
Answer: 4 - Angiography
Explanations:
Pulmonary angiogram can be used to detect postoperative pulmonary artery
stenosis.
Transesophageal echocardiography has been used in some centers.
Cardiac MRI would likely be the best test to evaluate for pulmonary vein
stenosis.

Go to the next page if you knew the correct answer, or click the link image(s)
below to further research the concepts in this question (if desired).

Research Concepts:
Transplantation, Lung

We update eBooks quarterly and Apps daily based on user feedback.


Please tap flag to report any questions that need improvement.
Question 473: Which of the following about bronchiolitis obliterans is
true?

Choices:
1. Produces wheezing
2. Complication of pneumoconiosis
3. Does not respond to steroids
4. Restrictive effect on spirometry
Answer: 2 - Complication of pneumoconiosis
Explanations:
Clinically speaking, bronchiolitis obliterans refers to a syndrome of chronic
airflow obstruction.
Bronchiolitis obliterans presents with radiographic hyper inflation caused
by pathological changes in the small airways.
It commonly occurs as a complication after a lung transplant.
There is decreased air flow with an obstructive pattern on spirometry due to
narrowing of the bronchiolar lumen. This is usually treated with
immunosuppression, including steroids, though it can be progressive.

Go to the next page if you knew the correct answer, or click the link image(s)
below to further research the concepts in this question (if desired).

Research Concepts:
Bronchiolitis Obliterans (Obliterative Bronchiolitis, Constrictive
Bronchiolitis)

We update eBooks quarterly and Apps daily based on user feedback.


Please tap flag to report any questions that need improvement.
Question 474: How are rickettsial infections typically diagnosed?
Choices:
1. Polymerase chain reaction (PCR)
2. Culture
3. Serology
4. Antigen detection
Answer: 3 - Serology
Explanations:
Rickettsial infections are generally diagnosed via serologic detection of
antibody to the infectious organism. PCR is not widely available currently
and the organism is very difficult to culture.
Because antibodies may not develop until 7 to 10 days after onset of
symptoms, it is important to treat based on clinical suspicion rather than
waiting for confirmatory testing.
With Rocky Mountain Spotted Fever, in particular, delays in treatment have
been associated with significant increases in mortality and should be
avoided.
The Weil-Felix test is an agglutination test was historically used to make a
diagnosis of rickettsial infections. The test is based on the presence of
antigenic cross-reactivity between Rickettsia and certain Proteus species.
Serum of patients with Rickettsia often agglutinates in the presence of
Proteus. This is no longer recommended routinely due to poor specificity
and sensitivity.

Go to the next page if you knew the correct answer, or click the link image(s)
below to further research the concepts in this question (if desired).

Research Concepts:
Rickettsial Infection

We update eBooks quarterly and Apps daily based on user feedback.


Please tap flag to report any questions that need improvement.
Question 475: Which of the following is not in the differential of chronic
cough?

Choices:
1. Postnasal drip
2. Asthma
3. Gastroesophageal reflux disease (GERD)
4. Eustachian tube dysfunction
Answer: 4 - Eustachian tube dysfunction
Explanations:
Allergic rhinitis, postnasal drip, and asthma are in the differential of chronic
cough.
Gastroesophageal reflux disease (GERD) is in the differential for a chronic
cough.
A more serious cause of chronic cough is congestive heart failure.
Eustachian tube dysfunction is unlikely to cause chronic cough.

Go to the next page if you knew the correct answer, or click the link image(s)
below to further research the concepts in this question (if desired).

Research Concepts:
Cough, Chronic

We update eBooks quarterly and Apps daily based on user feedback.


Please tap flag to report any questions that need improvement.
Question 476: Which of the following statements about spontaneous
pneumothorax is not correct?

Choices:
1. Spontaneous pneumothorax is common in tall, thin males
2. 10% of these individuals also develop a hemothorax from a torn branch of the
pulmonary artery
3. Apical blebs are seen in the majority of patients
4. First time spontaneous pneumothoraces do not require surgical therapy
beyond placement of a chest tube
Answer: 2 - 10% of these individuals also develop a hemothorax from a torn
branch of the pulmonary artery

Explanations:
A pneumothorax can occur in chronic obstructive pulmonary disease,
tuberculosis, cystic fibrosis, lung abscess, interstitial lung disease, and
asthma.
Most air leaks seal when adequately drained. Indicators for surgery include
a persistent air leak for longer than 5-7 days, bilateral pneumothorax,
failure to expand the lung, and contralateral pneumonectomy. Surgery may
also be indicated for cases involving scuba divers and pilots.
With spontaneous pneumothorax, a hemothorax occurs from torn adhesions
or avulsion of small blood vessels in the pleura.
About 5-7% of patients can have a significant hemothorax.

Go to the next page if you knew the correct answer, or click the link image(s)
below to further research the concepts in this question (if desired).

Research Concepts:
Pneumothorax, Spontaneous

We update eBooks quarterly and Apps daily based on user feedback.


Please tap flag to report any questions that need improvement.
Question 477: In a patient with granulomatosis with polyangiitis, what
antibody will be found in the serum?

Choices:
1. c-ANCA
2. p-ANCA
3. Anti-smooth muscle
4. Anti-mitochondrial
Answer: 1 - c-ANCA
Explanations:
Granulomatosis with polyangiitis is characterized by the presence of anti-
neutrophil cytoplasmic antibodies.
The disease often involves the upper airway, lung, and kidney.

Go to the next page if you knew the correct answer, or click the link image(s)
below to further research the concepts in this question (if desired).

Research Concepts:
Granulomatosis with Polyangiitis (GPA, Wegener Granulomatosis)

We update eBooks quarterly and Apps daily based on user feedback.


Please tap flag to report any questions that need improvement.
Question 478: A patient with an aspergilloma presents with his fifth
episode of hemoptysis. The mass is localized to the left upper lobe. His FEV1 is
1.9. What is the best treatment for this patient?

Choices:
1. Cavernostomy with muscle flap
2. Lobectomy
3. Bronchial artery embolization
4. Percutaneous instillation of amphotericin in the cavity
Answer: 2 - Lobectomy
Explanations:
Surgery is confined to patients with aspergillomas and hemoptysis.
The surgery is usually difficult due to adhesions and diffuse disease. In
patients who can tolerate surgery and have adequate pulmonary reserve,
surgery is the best procedure for recurrent hemoptysis.
Aspergillomas occur exclusively in patients with underlying pulmonary
disease such as tuberculosis, histoplasmosis, or sarcoid, and in
immunosuppressed patients.
Newer drugs like itraconazole may be helpful.

Go to the next page if you knew the correct answer, or click the link image(s)
below to further research the concepts in this question (if desired).

Research Concepts:
Aspergilloma

We update eBooks quarterly and Apps daily based on user feedback.


Please tap flag to report any questions that need improvement.
Question 479: Which of the following is not a risk factor for asthma
persisting in a child?

Choices:
1. Onset prior to 12 months of age
2. Recurrent viral illnesses
3. Other allergic conditions such as allergic rhinitis or atopic dermatitis
4. Family history
Answer: 2 - Recurrent viral illnesses
Explanations:
Asthma is common in children, and the presence of risk factors are well
documented.
Family history and early onset are some of the known risk factors.
Recurrent viral illnesses are not considered a significant risk factor.
Atopy also has been associated with an increased risk of asthma.

Go to the next page if you knew the correct answer, or click the link image(s)
below to further research the concepts in this question (if desired).

Research Concepts:
Asthma, Pediatric

We update eBooks quarterly and Apps daily based on user feedback.


Please tap flag to report any questions that need improvement.
Question 480: During ventilation of an infant with congenital
diaphragmatic hernia, what precaution should be exercised?

Choices:
1. Respiration rate
2. Ventilatory pressure
3. Increased PEEP
4. Minimal Fi02
Answer: 2 - Ventilatory pressure
Explanations:
All infants with congenital diaphragmatic hernia have some degree of
respiratory stress.
For those who require mechanical ventilation, mask ventilation should be
limited as this can cause insufflation of the stomach and bowel.
The ventilatory pressures should be no more than 5 to 10 mmHg because
the lungs are hypoplastic and can easily rupture.

Go to the next page if you knew the correct answer, or click the link image(s)
below to further research the concepts in this question (if desired).

Research Concepts:
Hernia, Diaphragmatic, Congenital

We update eBooks quarterly and Apps daily based on user feedback.


Please tap flag to report any questions that need improvement.
Question 481: Which of the following is not part of the triad for Lofgren
syndrome?

Choices:
1. Fever
2. Bilateral hilar adenopathy
3. Polyarthralgia
4. Hemoptysis
Answer: 4 - Hemoptysis
Explanations:
Lofgren syndrome consists of fever, polyarthralgia, and bilateral hilar
adenopathy.
Lofgren syndrome is more common in Northern Europeans and has an
excellent prognosis.
It is important to know that Lofgren syndrome is an acute but milder
clinical presentation of systemic sarcoidosis. It may present with fever,
general malaise, and cough.
The condition resolves spontaneously and may be treated with NSAIDs.

Go to the next page if you knew the correct answer, or click the link image(s)
below to further research the concepts in this question (if desired).

Research Concepts:
Lofgren Syndrome

We update eBooks quarterly and Apps daily based on user feedback.


Please tap flag to report any questions that need improvement.
Question 482: After starting anticoagulation therapy, when will 50% of
pulmonary emboli disappear?

Choices:
1. 7 days
2. 3 days
3. 14 days
4. 3 months
Answer: 4 - 3 months
Explanations:
Pulmonary emboli do resolve with anticoagulation therapy.
In most cases, 36% of lung defects resolve in 5 days and 50% resolve in 3
months.

Go to the next page if you knew the correct answer, or click the link image(s)
below to further research the concepts in this question (if desired).

Research Concepts:
Embolism, Pulmonary

We update eBooks quarterly and Apps daily based on user feedback.


Please tap flag to report any questions that need improvement.
Question 483: Which of the following lung cancers is often associated with
congenital pulmonary airway malformations?

Choices:
1. Squamous cell cancer
2. Oat cell cancer
3. Pleuropulmonary blastoma
4. Adenocarcinoma
Answer: 3 - Pleuropulmonary blastoma
Explanations:
There are five types of congenital pulmonary airway malformations
(CPAMs) historically called congenital cystic adenomatoid malformations.
Type 0 shows acinar dysplasia. Type I has one or more multiloculated cysts
that are greater than 2 cm. Type II has one or more uniform cysts that are <2
cm. Type III are solid.
Type 4 CPAM should be considered a malignant lesion because
approximately 30 percent of cases present in a purely cystic form that has
histology that is indistinguishable from nonmalignant cysts.

Go to the next page if you knew the correct answer, or click the link image(s)
below to further research the concepts in this question (if desired).

Research Concepts:
Congenital Pulmonary Airway Malformation

We update eBooks quarterly and Apps daily based on user feedback.


Please tap flag to report any questions that need improvement.
Question 484: A 1 week old infant born at 39 weeks by vaginal delivery
had no complications at delivery. The parents complain that the child's breathing
sounds funny and seems to get worse when he coughs or feeds. His breathing is
described as high pitched and noisy but does get better when he sleeps. What is
the best initial management of this infant?

Choices:
1. Reassurance and supportive therapy
2. Open surgery
3. Tracheostomy
4. Intubation
Answer: 1 - Reassurance and supportive therapy
Explanations:
Tracheomalacia is a rare cause of upper airway partial obstruction.
It is usually caused by weakness in tracheal ring cartilages.
Snoring is often observed in such infants.
This problem is usually self limiting/benign and outgrown by 2 years of
age.

Go to the next page if you knew the correct answer, or click the link image(s)
below to further research the concepts in this question (if desired).

Research Concepts:
Tracheomalacia

We update eBooks quarterly and Apps daily based on user feedback.


Please tap flag to report any questions that need improvement.
Question 485: An asymptomatic patient from the southern U.S. moves to
New York. Three years later, a routine chest x-ray shows isolated calcific lesions
visible along the mediastinum. What is the next step in the management of this
patient?

Choices:
1. Bronchoscopy with biopsy
2. CT guided biopsy
3. Mediastinoscopy
4. Observation
Answer: 4 - Observation
Explanations:
Histoplasmosis presents later with calcific lesions.
Occasionally it presents with hemoptysis due to erosion of the nodes into
the bronchus. It may produce esophageal fistulas and the superior vena cava
syndrome.
For patients with lymph nodes eroding into the bronchus, the management
is a lobectomy rather than a bronchoscopy-based procedure. This patient is
asymptomatic and can be observed. Remember there are no active
histoplasma in the calcific lesions.
The superior vena cava syndrome generally does not require treatment. The
lesion grows slowly and there is adequate time for collateral formation to
occur and decompress the superior vena cava.

Go to the next page if you knew the correct answer, or click the link image(s)
below to further research the concepts in this question (if desired).

Research Concepts:
Histoplasmosis

We update eBooks quarterly and Apps daily based on user feedback.


Please tap flag to report any questions that need improvement.
Question 486: Which is true regarding aspergillus?
Choices:
1. Aspergillosis is a bacterial infection that occurs in patients who are
immunosuppressed
2. Allergic bronchopulmonary aspergillosis (ABPA) is a hypersensitivity
reaction to Aspergillus in patients with asthma and cystic fibrosis
3. ABPA does not have permanent sequelae
4. Chronic necrotizing pulmonary aspergillosis is a subacute pneumonia that
responds to antibacterial therapy
Answer: 2 - Allergic bronchopulmonary aspergillosis (ABPA) is a
hypersensitivity reaction to Aspergillus in patients with asthma and cystic
fibrosis

Explanations:
Aspergillus is a ubiquitous mold, which is inhaled in the form of spores.
It affects mostly the immunosuppressed. In patients with cystic fibrosis
(CF), defects in immune and mechanical clearance make them more
vulnerable to developing Allergic bronchopulmonary aspergillosis (ABPA).
ABPA is seen in asthma and CF. Fungal sensitization elicits a robust
hypersensitivity response characterized by elevated total serum IgE and
Aspergillus-specific IgE and IgG antibodies.
Recurrent ABPA can cause fibrotic lung disease and atelectasis. Chronic
necrotizing pulmonary aspergillosis is a subacute pneumonia and does not
respond to antibacterial therapy.

Go to the next page if you knew the correct answer, or click the link image(s)
below to further research the concepts in this question (if desired).

Research Concepts:
Aspergillus Fumigatus

We update eBooks quarterly and Apps daily based on user feedback.


Please tap flag to report any questions that need improvement.
Question 487: A female found unconscious near the scene of a fire is
hypotensive and comatose. Her arterial pH is 7.0, with carboxyhemoglobin of
45%. Which treatment is not indicated emergently?

Choices:
1. Sodium nitrite
2. Sodium bicarbonate
3. Endotracheal intubation and hyperventilation
4. Crystalloid infusion
Answer: 1 - Sodium nitrite
Explanations:
The patient has carbon monoxide toxicity and should be treated with
intubation and hyperventilation. Fluids and sodium bicarbonate should be
given.
There may be associated cyanide toxicity. If available, hydroxocobalamin
should be given.
Sodium nitrate is contraindicated with very high carboxyhemoglobin levels
because the methemoglobinemia produced will further reduce oxygen-
carrying capacity.

Go to the next page if you knew the correct answer, or click the link image(s)
below to further research the concepts in this question (if desired).

Research Concepts:
Toxicity, Carbon Monoxide

We update eBooks quarterly and Apps daily based on user feedback.


Please tap flag to report any questions that need improvement.
Question 488: An otherwise healthy 17 year old presents to the emergency
department following a submersion accident in his pool. The patient appears to
be doing fine, recalls the event, and has an occasional cough, though no other
difficulties with breathing. His breath sounds are clear bilaterally. Chest x-ray is
obtained and is found to be negative for acute findings. He is monitored for 6
hours and continues to have clear breath sounds and no longer has a cough.
Which of the following is appropriate disposition of this patient?

Choices:
1. Admit for observation
2. Discharge home with antibiotics
3. Discharge home with return precautions
4. Discharge home with steroids
Answer: 3 - Discharge home with return precautions
Explanations:
Patients who have minimal symptoms and clear breath sounds, typically are
able to be discharged home after a 6 to 8 hour observation period.
Routine use of steroids or antibiotics are not indicated unless the patient
was submerged in grossly contaminated water.
Admission likely is unwarranted in this patient who is doing well.
Initial chest radiograph may be negative. It may also show opacities that the
provider may be inclined to treat with antibiotics. However, this is
discouraged until the patient exhibits signs of infection, such as
leukocytosis, fever, or productive cough.

Go to the next page if you knew the correct answer, or click the link image(s)
below to further research the concepts in this question (if desired).

Research Concepts:
Drowning

We update eBooks quarterly and Apps daily based on user feedback.


Please tap flag to report any questions that need improvement.
Question 489: Which of the following is FDA approved for the treatment
of coccidioidomycosis?

Choices:
1. Doxycycline
2. Ketoconazole
3. Penicillin
4. Clindamycin
Answer: 2 - Ketoconazole
Explanations:
Coccidioidomycosis is the most common pulmonary mycotic infection in
the southwestern US.
Ketoconazole is the only azole antifungal approved by the FDA for the
treatment of coccidioidomycosis.
Although not approved by the FDA, itraconazole and fluconazole are
sometimes used for treatment.
Amphotericin B remains the mainstay of treatment for severe cases of the
disease.

Go to the next page if you knew the correct answer, or click the link image(s)
below to further research the concepts in this question (if desired).

Research Concepts:
Coccidioidomycosis

We update eBooks quarterly and Apps daily based on user feedback.


Please tap flag to report any questions that need improvement.
Question 490: What is the clinical significance of the epiglottic vallecula
for airway management?

Choices:
1. It helps identify the carina
2. It is a marker for the midline of the pharynx
3. It prevents injury to the vocal cords
4. Helps with intubation

Photo:Contributed by T. Silappathikaram
Answer: 4 - Helps with intubation
Explanations:
The location of the vallecula is vital for all healthcare workers who perform
intubation.
It is an important anatomical landmark during oral intubation of the trachea.
In order to visualize the vallecula, the blade of the Macintosh laryngoscope
is placed on to the vallecula and depressed in order to see the glottis.
If the vallecula is not visualized during intubation, the risk of esophageal
intubation is high.

Go to the next page if you knew the correct answer, or click the link image(s)
below to further research the concepts in this question (if desired).

Research Concepts:
Airway, Management

We update eBooks quarterly and Apps daily based on user feedback.


Please tap flag to report any questions that need improvement.
Question 491: Ketamine can cause which hemodynamic effect?
Choices:
1. Bradycardia
2. Hypertension
3. Hypotension
4. Atrioventricular dissociation
Answer: 2 - Hypertension
Explanations:
Ketamine produces hypertension and tachycardia.

Go to the next page if you knew the correct answer, or click the link image(s)
below to further research the concepts in this question (if desired).

Research Concepts:
Ketamine

We update eBooks quarterly and Apps daily based on user feedback.


Please tap flag to report any questions that need improvement.
Question 492: Which of the following structures is at the highest risk for
causing a hemorrhagic complication from percutaneous lung lesion biopsy?

Choices:
1. A lesion along a peripheral pulmonary artery
2. A lesion along a bronchiectatic airway
3. A lesion on the far side of a bulla
4. A lesion along a pulmonary vein
Answer: 2 - A lesion along a bronchiectatic airway
Explanations:
The bronchial arteries travel along the airways and can increase in caliber
with bronchiectasis.
The bronchial artery has a higher pressure than a pulmonary artery or a
pulmonary vein.
Puncturing a vessel under high pressure is more likely to result in continued
bleeding than puncturing a vessel under lower pressure.
Biopsying a bulla is likely to result in pneumothorax but not bleeding.

Go to the next page if you knew the correct answer, or click the link image(s)
below to further research the concepts in this question (if desired).

Research Concepts:
Transthoracic Needle Biopsy

We update eBooks quarterly and Apps daily based on user feedback.


Please tap flag to report any questions that need improvement.
Question 493: A 2-year-old child is suspected of having aspirated a
radiolucent foreign body but the chest radiograph is normal. Select the correct
statement.

Choices:
1. There could still be a foreign body
2. There is minimal chance there is a foreign body
3. CT of the chest must be done
4. Contrast bronchography is the next step
Answer: 1 - There could still be a foreign body
Explanations:
More than half of children with a foreign body in the airway have a normal
CXR.
Bronchoscopy can be used to locate foreign bodies in the upper airways.
In some cases, air trapping can be seen on CXR distal to the obstruction.
CT of the chest can detect foreign bodies not seen on CXR, but one would
have to weigh the risks and benefits of the radiation associated with it.

Go to the next page if you knew the correct answer, or click the link image(s)
below to further research the concepts in this question (if desired).

Research Concepts:
Foreign Body Aspiration

We update eBooks quarterly and Apps daily based on user feedback.


Please tap flag to report any questions that need improvement.
Question 494: In a neonate with bronchial atresia, what usually happens to
the distal lobe?

Choices:
1. Shrinks
2. Hyperinflates
3. Loses air
4. Atrophies
Answer: 2 - Hyperinflates
Explanations:
In neonates, any condition, which narrows the bronchus, results in air
trapping and a distal, hyperinflated lung.
Congenital bronchial atresia is a rare lung abnormality caused by focal
interruption of the proximal or distal bronchus. It is usually associated with
mucous impaction (mucocele) and associated with hyperinflation of the
obstructed lung segment
There is a debate on the management of bronchial atresia. Some surgeons
resect all lesions whereas others observe asymptomatic individuals.
The diagnosis of bronchial atresia is made by a CT scan. It will reveal a
bronchocele/mucocele, associated with emphysematous changes.

Go to the next page if you knew the correct answer, or click the link image(s)
below to further research the concepts in this question (if desired).

Research Concepts:
Bronchial Atresia

We update eBooks quarterly and Apps daily based on user feedback.


Please tap flag to report any questions that need improvement.
Question 495: What is the primary toxicity of bleomycin?
Choices:
1. Nephritis
2. Hemorrhagic cystitis
3. Lung fibrosis
4. Neuropathy
Answer: 3 - Lung fibrosis
Explanations:
Bleomycin is a glycopeptides and often used to treat lymphomas.
Bleomycin is felt to work by preventing incorporation of thymidine into
DNA strands. Bleomycin chelates metal iron and produces free radicals
which damage the DNA.
The most serious side effects of bleomycin is lung fibrosis and impaired
lung function. The lung toxicity is related to oxygen free radicals.
Other side effects of bleomycin include alopecia, dermatographism and
raynaud phenomenon.

Go to the next page if you knew the correct answer, or click the link image(s)
below to further research the concepts in this question (if desired).

Research Concepts:
Bleomycin

We update eBooks quarterly and Apps daily based on user feedback.


Please tap flag to report any questions that need improvement.
Question 496: In an infant with suspected congenital diaphragmatic hernia,
which one of the following should be performed prior to a chest x-ray?

Choices:
1. Place infant in Trendelenburg position
2. Insert NG tube
3. Remove all ECG leads
4. Hold the infant upright
Answer: 2 - Insert NG tube
Explanations:
An orogastric tube should be inserted prior to a chest x-ray.
If the stomach has herniated into the chest, then the orogastric tube will be
seen in the chest.
Sometimes one can mistake lung cysts for abdominal organs.

Go to the next page if you knew the correct answer, or click the link image(s)
below to further research the concepts in this question (if desired).

Research Concepts:
Hernia, Diaphragmatic, Congenital

We update eBooks quarterly and Apps daily based on user feedback.


Please tap flag to report any questions that need improvement.
Question 497: Which radiographic finding is MOST COMMON in chronic
pulmonary histoplasmosis?

Choices:
1. Calcified hilar adenopathy
2. Cavitation
3. Pleural effusion
4. Pneumothorax
Answer: 1 - Calcified hilar adenopathy
Explanations:
Hilar and mediastinal adenopathy is common with histoplasmosis and these
lymph nodes often calcify on healing, a permanent feature.
Cavitation is uncommon but does occur, and can simulate TB.
The chest radiograph may be normal, even more true in acute infections.
One form of the disease is disseminated, via the hematogenous route, with
renal, hepatosplenic, skeletal, and cerebral involvement.

Go to the next page if you knew the correct answer, or click the link image(s)
below to further research the concepts in this question (if desired).

Research Concepts:
Histoplasmosis

We update eBooks quarterly and Apps daily based on user feedback.


Please tap flag to report any questions that need improvement.
Question 498: After a symptomatic air embolism, which factor is least
important for the outcome?

Choices:
1. Health status of patient
2. Amount of air
3. Pulmonary function
4. Site of air entry
Answer: 4 - Site of air entry
Explanations:
The amount of air and the status of the patient are the most important
factors in determining the outcome.
Large amounts of gas that enter on the venous side can be symptomatic if
they overwhelm the pulmonary filter or pass through a patent foramen
ovale or a pulmonary arteriovenous malformation.
If a gas embolism is symptomatic, the prognosis depends on the degree of
symptomatology and status of the patient, not the initial site of entry.
A venous gas embolism is more likely to be asymptomatic and can often be
filtered by the lungs.

Go to the next page if you knew the correct answer, or click the link image(s)
below to further research the concepts in this question (if desired).

Research Concepts:
Embolism, Venous Gas

We update eBooks quarterly and Apps daily based on user feedback.


Please tap flag to report any questions that need improvement.
Question 499: Which of the following is a common etiology of chronic
cough in children?

Choices:
1. Cystic fibrosis
2. Pneumonia
3. Allergic rhinitis
4. Bronchiectasis
Answer: 3 - Allergic rhinitis
Explanations:
Allergic rhinitis is a common cause of chronic cough in children.
Other causes of chronic cough in children include chronic sinusitis and
enlarged adenoids.
Pneumonia, viral and chlamydial, is a rare etiology of chronic cough.
Bronchiectasis and cystic fibrosis are also rare causes and should be
considered after ruling out other more common causes.

Go to the next page if you knew the correct answer, or click the link image(s)
below to further research the concepts in this question (if desired).

Research Concepts:
Cough, Chronic

We update eBooks quarterly and Apps daily based on user feedback.


Please tap flag to report any questions that need improvement.
Question 500: Which of the following is false about acute rejection in lung
transplant?

Choices:
1. Rejection is highest in the first month after transplant
2. It presents with specific signs and symptoms
3. The chest x-ray may be normal
4. It may be confused with bacterial infection
Answer: 2 - It presents with specific signs and symptoms
Explanations:
Lung rejection can present with dyspnea, fever, elevated WBC, and diffuse
infiltrates.
Unfortunately, acute rejection is a difficult entity to diagnose.
Except for a transbronchial lung biopsy, there are no specific or symptoms.
It may be confused with infection, sepsis, and pulmonary edema.
The single best diagnosis of rejection is to administer steroids. Rejection
episodes clear up within 6 to 12 hours and a dramatic clinical improvement
is seen.

Go to the next page if you knew the correct answer, or click the link image(s)
below to further research the concepts in this question (if desired).

Research Concepts:
Transplantation, Lung, Rejection

We update eBooks quarterly and Apps daily based on user feedback.


Please tap flag to report any questions that need improvement.
Question 501: A 59-year old patient with alcoholism has been admitted
with purulent sputum, cough, and fever and weight loss. Work up reveals that he
has a large lung abscess in the right lung. Which of the following is not an
indication for surgery in this patient?

Choices:
1. Persistent fever for 2 weeks
2. Bronchopleural fistula
3. Massive hemoptysis
4. Large pleural effusion
Answer: 1 - Persistent fever for 2 weeks
Explanations:
Persistent fever is not a case for surgery in lung abscess.
The patient may need bronchoscopy, new cultures, x-ray, and newer
antibiotics. If no complications, continue present management.
Indications for surgery for lung abscess include an increase in the size of
the abscess, empyema, bronchial obstruction or suspicion of cancer.
When surgery is performed for a lung abscess, a double-lumen endotracheal
tube is required and a lobectomy is preferred.

Go to the next page if you knew the correct answer, or click the link image(s)
below to further research the concepts in this question (if desired).

Research Concepts:
Abscess, Lung

We update eBooks quarterly and Apps daily based on user feedback.


Please tap flag to report any questions that need improvement.
Question 502: Which of the following surgical procedures is not used to
prevent aspiration?

Choices:
1. Tracheostomy
2. Cricopharyngeal myotomy
3. Laryngeal suspension
4. Vocal cord medialization
Answer: 1 - Tracheostomy
Explanations:
Several surgical procedures are used to prevent recurrent aspiration. The
most common include cricopharyngeal myotomy, vocal cord medialization,
laryngeal suspension, and partial cricoid resection.
Tracheostomy is not useful for the prevention of aspiration.
The presence of a tracheotomy tube can increase aspiration risk as it may
limit the laryngeal elevation and anterior-superior excursion during
swallowing.
A cuffed tracheotomy tube can further increase the risk of aspiration and the
risks of tracheal mucosal ulceration, tracheal stenosis, and tracheal cartilage
-chondritis.

Go to the next page if you knew the correct answer, or click the link image(s)
below to further research the concepts in this question (if desired).

Research Concepts:
Aspiration

We update eBooks quarterly and Apps daily based on user feedback.


Please tap flag to report any questions that need improvement.
Question 503: What is the most common site for tuberculous
lymphadenitis?

Choices:
1. Groin
2. Axilla
3. Mediastinum
4. Neck
Answer: 4 - Neck
Explanations:
Tuberculous lymphadenitis often presents in the neck.
The infection is known as a scrofula and is found just along the
sternocleidomastoid muscle.
Scrofula is unilateral and painless.
Advanced cases of scrofula may develop draining sinus tracts.

Go to the next page if you knew the correct answer, or click the link image(s)
below to further research the concepts in this question (if desired).

Research Concepts:
Scrofula

We update eBooks quarterly and Apps daily based on user feedback.


Please tap flag to report any questions that need improvement.
Question 504: Which of the following about a pneumothorax in children is
correct?

Choices:
1. In children with a pneumothorax, symptoms are rarely present
2. Recurrence is greater than in adults
3. Simple observation is adequate in the majority of children
4. Besides cystic fibrosis and asthma, there are many more conditions that can be
a cause
Answer: 2 - Recurrence is greater than in adults
Explanations:
A pneumothorax in childhood occurs predominantly from cystic fibrosis
and asthma.
Most children are symptomatic and require thoracostomy.
Unlike adults, very few children can be observed.
The recurrence in children is much higher than in adults.

Go to the next page if you knew the correct answer, or click the link image(s)
below to further research the concepts in this question (if desired).

Research Concepts:
Pneumothorax, Spontaneous

We update eBooks quarterly and Apps daily based on user feedback.


Please tap flag to report any questions that need improvement.
Question 505: Which of the following is the least likely complication of
obstructive sleep apnea in children?

Choices:
1. Congestive heart failure
2. Gastroesophageal reflux
3. Systemic hypertension
4. Neurocognitive findings
Answer: 2 - Gastroesophageal reflux
Explanations:
Children with untreated obstructive sleep apnea can develop congestive
heart failure, cor pulmonale, and systemic hypertension.
Chronic hypercarbia and hypoxemia result in both pulmonary and systemic
hypertension.
Attention and behavior problems also may occur.
Tonsillectomy and adenoidectomy is one possible treatment but may lead to
weight gain.

Go to the next page if you knew the correct answer, or click the link image(s)
below to further research the concepts in this question (if desired).

Research Concepts:
Apnea, Obstructive Sleep Apnea

We update eBooks quarterly and Apps daily based on user feedback.


Please tap flag to report any questions that need improvement.
Question 506: Which statement is true about the use of steroids in sepsis?
Choices:
1. Low-dose corticosteroids can benefit patients with adrenal insufficiency
2. Corticosteroid administration in septic shock has been shown to decrease
mortality
3. Corticosteroids should be administered at the first signs of septic shock to be
effective
4. High-dose corticosteroids are of more benefit than low-dose steroids
Answer: 1 - Low-dose corticosteroids can benefit patients with adrenal
insufficiency

Explanations:
The role of steroids in septic shock is still undetermined. Corticosteroids
have been shown to increase mortality in sepsis and thus their use is
limited.
Low-dose corticosteroids can be of some benefit to individuals who have
adrenal insufficiency.
The Corticus trial showed that corticosteroids may help resolve shock but
do not affect mortality rate. The survival sepsis guidelines indicate that
corticosteroids only should be administered to individuals with septic shock
whose hypotension is not responsive to fluids or vasopressors.
While low doses of corticosteroids have been shown to reduce mortality in
some studies, there has been no clear benefit of high-dose steroids in septic
shock.

Go to the next page if you knew the correct answer, or click the link image(s)
below to further research the concepts in this question (if desired).

Research Concepts:
Sepsis, Bacterial

We update eBooks quarterly and Apps daily based on user feedback.


Please tap flag to report any questions that need improvement.
Question 507: When leaving the room of a patient on droplet precautions,
which order of action is correct?

Choices:
1. Remove the gloves, then goggles, then gown, leave the room, remove the
mask, and wash hands last.
2. Remove the gown, remove the gloves, wash the hands, exit the room, and
remove the mask.
3. Exit the room, remove the mask and gown, remove the gloves, and wash the
hands
4. Remove the gloves, remove the mask and gown, wash the hands, and exit the
room
Answer: 1 - Remove the gloves, then goggles, then gown, leave the room,
remove the mask, and wash hands last.

Explanations:
Remove the gloves first as they are most likely contaminated. Then remove
the goggles or face shield.
Next the gown is removed. Leave the room and then mask or respirator is
removed.
Hand washing depends on sink availability. An alcohol-based cleanser may
be used if the hands are not visibly soiled.
The infected material should be left in the room in a container by the door.

Go to the next page if you knew the correct answer, or click the link image(s)
below to further research the concepts in this question (if desired).

Research Concepts:
Precautions, Droplet

We update eBooks quarterly and Apps daily based on user feedback.


Please tap flag to report any questions that need improvement.
Question 508: Which of the following is NOT a feature of congenital lobar
emphysema?

Choices:
1. Affects the right side more often
2. Affects the upper lobe more often
3. May involve the whole lung
4. Results in progressive respiratory failure
Answer: 1 - Affects the right side more often
Explanations:
Congenital lobar emphysema often affects the left upper lung and may
progress to respiratory failure.

Go to the next page if you knew the correct answer, or click the link image(s)
below to further research the concepts in this question (if desired).

Research Concepts:
Emphysema, Lobar, Congenital

We update eBooks quarterly and Apps daily based on user feedback.


Please tap flag to report any questions that need improvement.
Question 509: In a patient with a pneumothorax following a stab wound,
the chest tube is best inserted at which level?

Choices:
1. Between the second and third intercostal spaces
2. Between the eighth and ninth intercostal spaces
3. Between the fourth and fifth intercostal spaces, just anterior to the mid-
axillary line
4. Just below the clavicle

Photo:Contributed by Wikimedia Commons,"Medical gallery of Mikael Häggström 2014" (Public Domain)


Answer: 3 - Between the fourth and fifth intercostal spaces, just anterior to
the mid-axillary line

Explanations:
Pneumothorax can be treated with insertion of a chest tube between the
fourth and fifth intercostal spaces, just anterior to the mid-axillary line.
Placement of the chest tube too high can lead to damage to subclavian
vessels. Placement too low can result in damage to the liver or spleen.
Chest tubes with trocars should be avoided due to the risk of injuring the
heart or lung.
In a pregnant patient, the chest tube should be placed one or two levels
higher as the diaphragm is higher in pregnancy.

Go to the next page if you knew the correct answer, or click the link image(s)
below to further research the concepts in this question (if desired).

Research Concepts:
Pneumothorax, Tension And Traumatic

We update eBooks quarterly and Apps daily based on user feedback.


Please tap flag to report any questions that need improvement.
Question 510: A female from Mexico presents with a generalized macular
rash and erythema nodosum. She has had a persistent cough with sputum
production. What is the best test for diagnosing her condition?

Choices:
1. Lung biopsy
2. Biopsy of the skin nodules
3. Serologic testing
4. Stool culture
Answer: 3 - Serologic testing
Explanations:
Coccidioidomycosis is seen in the southwestern U.S. It is inhaled as an
arthrospore, which matures into spherules.
Most adults have minimal symptoms. The severity of the infection is
determined by skin testing and serology.
On a histology examination, the endospores can easily be seen in the
spherule. The infection can present with a generalized skin rash in the
inguinal folds and erythema nodosum. Chest x-ray will reveal thin-walled
cystic cavities. The infection can involve the meninges.
Amphotericin is required for severe cases.

Go to the next page if you knew the correct answer, or click the link image(s)
below to further research the concepts in this question (if desired).

Research Concepts:
Coccidioidomycosis

We update eBooks quarterly and Apps daily based on user feedback.


Please tap flag to report any questions that need improvement.
Question 511: A gram-negative organism is found in unusually high
numbers in the mucus of a cystic fibrosis patient. Which of the following
virulence factors is most important in colonization and maintenance of the
organism in the lungs?

Choices:
1. Exotoxin A
2. Polysaccharide slime
3. Pyocyanin
4. Endotoxin
Answer: 2 - Polysaccharide slime
Explanations:
Staphylococcus aureus and Pseudomonas aeruginosa are two pulmonary
colonizers that cause pneumonia in cystic fibrosis patients.
Their slimy material causes poor penetration of antibiotics to the site and
resistance to phagocytic killing making these infections virulent.
Those of Northern European ancestry are the most commonly affected by
cystic fibrosis (CF). In this population, one per 3,000 newborns is
diagnosed annually, typically prior to six months of age. CF occurs equally
in females and males.
Currently, cystic fibrosis (CF) is not curable. Treatment includes respiratory
therapy (mechanical agitation, bronchodilators, anticholinergics,
mucolytics, and inhaled corticosteroids), antibiotics for acute and chronic
lung infections, pancreatic enzyme supplementation, nutritional support,
and an active lifestyle.

Go to the next page if you knew the correct answer, or click the link image(s)
below to further research the concepts in this question (if desired).

Research Concepts:
Cystic Fibrosis

We update eBooks quarterly and Apps daily based on user feedback.


Please tap flag to report any questions that need improvement.
Question 512: The most common cause of fever on postoperative day one
following surgery under general anesthesia should be managed by which of the
following actions?

Choices:
1. Obtain blood cultures
2. Instruct the patient to inhale deeply 10 times per hour or to use an incentive
spirometer
3. Change the dressing and culture the incision site
4. Obtain an abdominal CT scan
Answer: 2 - Instruct the patient to inhale deeply 10 times per hour or to use
an incentive spirometer

Explanations:
Mild temperature elevations postoperatively are most commonly due to
atelectasis from anesthesia.
Post operative respiratory infections typically present within the first 48
hours following surgery.

Go to the next page if you knew the correct answer, or click the link image(s)
below to further research the concepts in this question (if desired).

Research Concepts:
Atelectasis

We update eBooks quarterly and Apps daily based on user feedback.


Please tap flag to report any questions that need improvement.
Question 513: For a first-time spontaneous pneumothorax, which of the
following scenarios does not require surgery?

Choices:
1. Persistent air leak after 10 days
2. Failure of the lung to expand
3. Ipsilateral pneumothorax 3 months ago, now presenting with a first-time
contralateral pneumothorax
4. Pneumothorax with hemothorax
Answer: 3 - Ipsilateral pneumothorax 3 months ago, now presenting with a
first-time contralateral pneumothorax

Explanations:
Generally, a first-time pneumothorax needs conservative care. Most would
treat an asynchronous, first-time pneumothorax with chest tube drainage.
Surgery for pneumothorax involves resection of blebs and bulla,
obliteration of the pleural space, or wedge resection of blebs.
Segmentectomy and lobectomy are generally not done. Less than 5%
present with a hemothorax and, if bleeding is persistent, surgery is required.
A large hemothorax is rare after a pneumothorax, but can lead to
fibrothorax/empyema if not evacuated.

Go to the next page if you knew the correct answer, or click the link image(s)
below to further research the concepts in this question (if desired).

Research Concepts:
Pneumothorax, Spontaneous

We update eBooks quarterly and Apps daily based on user feedback.


Please tap flag to report any questions that need improvement.
Question 514: A 5-year-old child presents with a barking cough, stridor,
and fever. The child is treated with racemic epinephrine without response. What
is the most likely diagnosis?

Choices:
1. Bacterial tracheitis
2. Acute epiglottitis
3. Croup
4. Pneumonia
Answer: 1 - Bacterial tracheitis
Explanations:
Barking cough and stridor with a fever that does not respond to standard
croup treatment most likely indicates bacterial tracheitis.
Bacterial tracheitis is commonly caused by Staphylococcus aureus.
Treatment should include antibiotics and a low threshold for intubation.
Intubation should occur in the operating room or an intensive care setting.

Go to the next page if you knew the correct answer, or click the link image(s)
below to further research the concepts in this question (if desired).

Research Concepts:
Tracheitis, Bacterial

We update eBooks quarterly and Apps daily based on user feedback.


Please tap flag to report any questions that need improvement.
Question 515: Which of the following is diagnostic of bronchiolitis
obliterans?

Choices:
1. Decreased tidal volume (TV)
2. Decreased forced expiratory volume (FEV1)
3. Decreased total lung volume (TLV)
4. Decreased vital capacity (VC)
Answer: 2 - Decreased forced expiratory volume (FEV1)
Explanations:
All these parameters indicate decreases in airway flow/volume but FEV1 is
the most diagnostic.
Decreased air flow due to narrowing of the bronchiolar lumen is seen in
bronchiolitis obliterans (BO).
Hyperinflation with an increase in total lung volume or residual volume
may also be seen.
Obstructive pattern with decreased FEV1/FVC ratio may also be seen.

Go to the next page if you knew the correct answer, or click the link image(s)
below to further research the concepts in this question (if desired).

Research Concepts:
Bronchiolitis Obliterans (Obliterative Bronchiolitis, Constrictive
Bronchiolitis)

We update eBooks quarterly and Apps daily based on user feedback.


Please tap flag to report any questions that need improvement.
Question 516: In which of the following conditions is diaphragmatic
pacing not indicated?

Choices:
1. Chronic obstructive pulmonary disease
2. Sleep apnea
3. A patient with myasthenia gravis whose diaphragm is not functional
4. A patient with diabetes mellitus and high quadriplegia
Answer: 3 - A patient with myasthenia gravis whose diaphragm is not
functional

Explanations:
The two most common uses of diaphragm pacing are high cord paralysis
and sleep apnea, also known as central alveolar ventilation.
It is also indicated for those with severe chronic obstructive pulmonary
disease.
Diaphragm pacing is contraindicated in patients whose diaphragm or
phrenic nerve is not functional.
When the diaphragm is paced, a permanent tracheostomy is needed to
prevent upper airway obstruction from uncoordinated diaphragmatic and
laryngeal muscle activity.

Go to the next page if you knew the correct answer, or click the link image(s)
below to further research the concepts in this question (if desired).

Research Concepts:
Diaphragmatic Pacing

We update eBooks quarterly and Apps daily based on user feedback.


Please tap flag to report any questions that need improvement.
Question 517: What breathing pattern has a classic crescendo-decrescendo
form?

Choices:
1. Central sleep apnea
2. Cheyne-Stokes respirations
3. Obstructive sleep apnea (OSA)
4. Asthma
Answer: 2 - Cheyne-Stokes respirations
Explanations:
Cheyne-Stokes respirations have a classic crescendo-decrescendo form.
Apneic and hypopneic episodes are characteristic of obstructive sleep
apnea.
Central sleep apnea is associated with obstructive sleep apnea.
Cheyne-Stokes respirations can be associated with central sleep apnea.

Go to the next page if you knew the correct answer, or click the link image(s)
below to further research the concepts in this question (if desired).

Research Concepts:
Cheyne Stokes Respirations

We update eBooks quarterly and Apps daily based on user feedback.


Please tap flag to report any questions that need improvement.
Question 518: Which of the following statements about prenatal
presentations of congenital pulmonary airway malformations of the lung is
FALSE?

Choices:
1. The appearance of CPAMs on prenatal ultrasound ranges from incidental
findings of cystic-appearing lesions to massive pulmonary involvement
2. Fetal hydrops develops in as many as 40 percent of cases
3. Lesions regress during the course of gestation in as many as 59 percent of
cases
4. Complete spontaneous resolution in the postnatal period is very common
Answer: 4 - Complete spontaneous resolution in the postnatal period is very
common

Explanations:
The appearance of CPAMs on prenatal ultrasound ranges from incidental
findings of cystic-appearing lesions to massive pulmonary involvement.
Fetal hydrops develops in as many as 40 percent of cases.
Lesions regress during the course of gestation in as many as 59 percent of
cases.
Complete spontaneous resolution in the postnatal period has been reported
in a small number of cases.

Go to the next page if you knew the correct answer, or click the link image(s)
below to further research the concepts in this question (if desired).

Research Concepts:
Congenital Pulmonary Airway Malformation

We update eBooks quarterly and Apps daily based on user feedback.


Please tap flag to report any questions that need improvement.
Question 519: Which of the following lung volumes cannot be measured
by simple spirometry?

Choices:
1. Vital capacity
2. Inspiratory reserve
3. Tidal volume
4. Functional residual capacity
Answer: 4 - Functional residual capacity
Explanations:
Functional residual capacity is the volume remaining in the lungs after a
tidal exhalation.
It is the combination of expiratory reserve capacity that can be measured
spirometrically and residual volume that cannot.
Functional residual capacity can be measured with a plethysmograph.
Spirometry is dependent on patient cooperation and effort. It must be
repeated at least three times to ensure reproducibility.

Go to the next page if you knew the correct answer, or click the link image(s)
below to further research the concepts in this question (if desired).

Research Concepts:
Physiology, Lung

We update eBooks quarterly and Apps daily based on user feedback.


Please tap flag to report any questions that need improvement.
Question 520: A patient with a chest tube has bubbling in the water seal
drainage during expiration. What does this indicate?

Choices:
1. Pneumothorax
2. Emphysema
3. Lung expansion
4. Air leak
Answer: 4 - Air leak
Explanations:
Bubbling in the water seal chamber during expiration demonstrates that an
air leak exists. The larger the leak, the more bubbling that will result.
A water seal chest tube drainage system will allow for air to leave the
pleural space on exhalation and prevent air from entering the pleural cavity
or mediastinum on exhalation. Sterile water is normally instilled up to 2 cm
line in a water seal chamber. The chest tube drainage system must be kept
upright at all times and water seal chamber monitored to check for
evaporation of the fluid in the water seal chamber.
Valves are built into most systems to allow for high negative pressure and
maintaining of the water seal. High negative pressure situations can include
patients with vigorous coughing, respiratory distress, milking of tube
aggressively or disconnected/decreased suction.
Normally wall suction is set on 80-100 mmHg, to provide a gentle bubbling
in the suction control chamber. Usually - 20 cm of suction. Otherwise,
aggressive bubbling causes evaporation of the water in the suction control
chamber and ultimate decrease in suction pressure. Dry suction drainage
systems have no bubbling and no evaporation of water and decrease suction
risks. A regulator is used to control the amount of suction by turning a knob
or dial to the appropriate level. A red stripe will appear in the window near
the prescribed suction level. The dry drainage system comes preset at -20
cm for ease of use and can be adjusted. The wall suction will usually need
to be set to 80-100 mmHg. An orange float will appear in the indicator
window. Enough suction must be applied for the float to work. Drainage
systems have built in positive pressure valves as well to protect against
positive pressure complications. If a chest tube is clamped, a tension
pneumothorax can result.

Go to the next page if you knew the correct answer, or click the link image(s)
below to further research the concepts in this question (if desired).

Research Concepts:
Chest Tube
We update eBooks quarterly and Apps daily based on user feedback.
Please tap flag to report any questions that need improvement.
Section 9

Question 521: In which of the following group of tuberculosis patients


should surgery not be readily undertaken?

Choices:
1. Irreversibly destroyed lung that is subject to repeat bacterial infections
2. An open negative cavity of 4 cm with positive sputum cultures
3. Atypical resistant organisms after 3 months of therapy
4. Presence of an asymptomatic lung nodule
Answer: 2 - An open negative cavity of 4 cm with positive sputum cultures
Explanations:
Most patients with an open cavity and only sputum cultures do respond to
triple antibiotic therapy.
Whenever a lung is severely damaged, there is little chance that medical
therapy can help. In such cases, complete removal of the lung may be the
only option.
As long as the cavity remains open, there is a chance that medications may
enter it. Thus, some doctors will wait for 3 to 6 months before referring the
patient to a surgeon.
When a lung nodule is seen, one can wait but if there is any suspicion of
malignancy, it should be biopsied.

Go to the next page if you knew the correct answer, or click the link image(s)
below to further research the concepts in this question (if desired).

Research Concepts:
Tuberculosis

We update eBooks quarterly and Apps daily based on user feedback.


Please tap flag to report any questions that need improvement.
Question 522: What is true about high frequency ventilation?
Choices:
1. Respiratory rate ranges from 50 to 100/min
2. Tidal volume is 2.5 to 3.5 ml/kg
3. It decreases carbon dioxide diffusion
4. It prevents atelectasis in ARDS
Answer: 2 - Tidal volume is 2.5 to 3.5 ml/kg
Explanations:
In high frequency ventilation, low tidal volumes of 2.5 to 3.5 ml/kg help
prevent the barotrauma and decreased cardiac output associated with PEEP.

Go to the next page if you knew the correct answer, or click the link image(s)
below to further research the concepts in this question (if desired).

Research Concepts:
Ventilation, High Frequency

We update eBooks quarterly and Apps daily based on user feedback.


Please tap flag to report any questions that need improvement.
Question 523: Unilateral pulmonary edema is NOT commonly associated
with which of the following?

Choices:
1. Congestive heart failure
2. Prolonged postural positioning
3. Aspiration
4. Rapid expansion of pneumothorax
Answer: 1 - Congestive heart failure
Explanations:
Unilateral pulmonary edema is seen on the ipsilateral side of the
abnormality is cases such as bronchial obstruction, unilateral aspiration and
pulmonary contusion.
Contralateral pulmonary edema can be seen in Swyer-James syndrome,
proximal pulmonary artery obstruction and acute pulmonary embolism.
The downstream effects of CHF result in bilateral pulmonary edema.

Go to the next page if you knew the correct answer, or click the link image(s)
below to further research the concepts in this question (if desired).

Research Concepts:
Pulmonary Edema, Noncardiogenic

We update eBooks quarterly and Apps daily based on user feedback.


Please tap flag to report any questions that need improvement.
Question 524: A patient presents with signs and symptoms of an upper
respiratory tract infection. You discover that the bronchial lavage specimen
shows Levinthal Coles Lille bodies. The patient may have which condition?

Choices:
1. Legionella
2. Pontiac Fever
3. Psittacosis
4. Dengue
Answer: 3 - Psittacosis
Explanations:
Psittacosis is an infection acquired from birds.
The primary route of infection is inhalation. Subsequently, the organism
spreads in the blood.
The initial symptoms mimic an upper respiratory tract infection, but are
much more severe.
Histology will usually show interstitial pneumonitis with cytoplasmic
inclusion, Levinthal Coles Lille bodies, and non-caseating granulomas.

Go to the next page if you knew the correct answer, or click the link image(s)
below to further research the concepts in this question (if desired).

Research Concepts:
Psittacosis

We update eBooks quarterly and Apps daily based on user feedback.


Please tap flag to report any questions that need improvement.
Question 525: Which of the following is not true of positive end-expiratory
pressure (PEEP)?

Choices:
1. It decreases end diastolic volume
2. It decreases cardiac output
3. It decreases mixed venous oxygen
4. It decreases compliance of the left ventricle
Answer: 4 - It decreases compliance of the left ventricle
Explanations:
PEEP increases alveolar surface area for gas exchange, increases functional
residual capacity, and therefore increases arteriolar oxygen tension.
PEEP can decrease cardiac output and ventricular filling pressure.
PEEP does not affect compliance of the left ventricle.
Hemodynamic effects of PEEP can be reversed with fluids.

Go to the next page if you knew the correct answer, or click the link image(s)
below to further research the concepts in this question (if desired).

Research Concepts:
Positive End-Expiratory Pressure (PEEP)

We update eBooks quarterly and Apps daily based on user feedback.


Please tap flag to report any questions that need improvement.
Question 526: What is the most common cause of hospitalization for an
adult lower respiratory tract infection?

Choices:
1. Respiratory syncytial virus infection
2. Pneumococcal infection
3. Influenza infection
4. Adenovirus infection
Answer: 3 - Influenza infection
Explanations:
The most common cause of adult hospitalization for an adult lower
respiratory infection is influenza.
Influenza viruses are spread from person to person primarily through large-
particle respiratory droplet transmission.
Viral shedding occurs in two stages. The first stage is between 0 to 24 hours
and the second is between 5 to10 days after onset of symptoms.
Rates of infection are the highest among children. However, the risk for
complications is highest among young children and those who are 65 years
of age and older. Complications include hospitalization and death.

Go to the next page if you knew the correct answer, or click the link image(s)
below to further research the concepts in this question (if desired).

Research Concepts:
Influenza

We update eBooks quarterly and Apps daily based on user feedback.


Please tap flag to report any questions that need improvement.
Question 527: For which of these conditions is rigid bronchoscopy not
routinely appropriate?

Choices:
1. Removal of foreign body
2. Assessment of massive hemoptysis
3. Placement of stents
4. Biopsy of a tumor in the lung periphery
Answer: 4 - Biopsy of a tumor in the lung periphery
Explanations:
Rigid bronchoscopy can be used for hemoptysis, airway obstruction, laser
therapy, stenting, tracheal toilet, and in pediatric cases.
Both the operator and endoscopy personnel require training. Generally,
rigid bronchoscopy requires general anesthesia.
Without adequate experience, rigid bronchoscopy may also be associated
with increased complications than flexible bronchoscopy. These include
trauma to the oropharynx, neck, and esophagus.
Rigid bronchoscopy is not routinely used to perform sampling of a
peripheral lung lesion. Almost always this can be accomplished by flexible
bronchoscopy with adjunct tools such as radial probe ultrasound, confocal
endomicroscopy, or electromagnetic navigational (EMN) bronchoscopy.
Moreover, in the case of EMN, the metallic rigid scopes may interfere with
the magnetic field.

Go to the next page if you knew the correct answer, or click the link image(s)
below to further research the concepts in this question (if desired).

Research Concepts:
Bronchoscopy

We update eBooks quarterly and Apps daily based on user feedback.


Please tap flag to report any questions that need improvement.
Question 528: How is an individual with asthma classified with a peak
expiratory flow of less than 60% of predicted on more than one occasion?

Choices:
1. Mild persistent asthma
2. Moderate persistent asthma
3. Severe persistent asthma
4. None of the above
Answer: 3 - Severe persistent asthma
Explanations:
Peak flow measures can be used to assess but not diagnose asthma.
A peak expiratory flow (PEF) of less than 60% of predicted is indicative of
severe persistent asthma.
A PEF 60% to 80% of predicted is indicative of moderate persistent asthma.
A PEF of greater than 80% of predicted is indicative of mild persistent
asthma.

Go to the next page if you knew the correct answer, or click the link image(s)
below to further research the concepts in this question (if desired).

Research Concepts:
Asthma

We update eBooks quarterly and Apps daily based on user feedback.


Please tap flag to report any questions that need improvement.
Question 529: High levels of positive end-expiratory pressure can cause
barotrauma. What is the most sensitive test to identify all forms of pulmonary
barotrauma?

Choices:
1. Ultrasound
2. MRI
3. CT scan
4. Posterior-anterior chest x-ray
Answer: 3 - CT scan
Explanations:
A CT scan is diagnostic of barotrauma.
Barotrauma is a well-recognized complication for patients requiring
mechanical ventilation.
It is frequently seen in patients who are ventilated with acute respiratory
distress syndrome, but it can occur in any patient on a ventilator.
Barotrauma can present with varying features, ranging from interstitial air
or subcutaneous crepitus to severe respiratory distress from a tension
pneumothorax.

Go to the next page if you knew the correct answer, or click the link image(s)
below to further research the concepts in this question (if desired).

Research Concepts:
Ventilation, Barotrauma And Mechanical

We update eBooks quarterly and Apps daily based on user feedback.


Please tap flag to report any questions that need improvement.
Question 530: Which of the following statements about the treatment of
pertussis is FALSE?

Choices:
1. All patients with pertussis should be treated
2. Only symptomatic cases need treatment
3. Asymptomatic cases with polymerase chain reaction (PCR) proven pertussis
infection should be treated
4. Treatment is based on clinical suspicion rather than laboratory confirmation
Answer: 2 - Only symptomatic cases need treatment
Explanations:
Antimicrobial therapy for pertussis, when administered early in the course,
may shorten the duration of symptoms and decrease transmission to
susceptible contacts.
It is recommended to treat all patients with clinical pertussis, with or
without laboratory confirmation.
Macrolides are recommended for the treatment of pertussis.
Confirmatory laboratory tests include culture, polymerase chain reaction
(PCR), and serology.

Go to the next page if you knew the correct answer, or click the link image(s)
below to further research the concepts in this question (if desired).

Research Concepts:
Pertussis

We update eBooks quarterly and Apps daily based on user feedback.


Please tap flag to report any questions that need improvement.
Question 531: Which of the following is of no benefit for a patient who has
just aspirated during induction of anesthesia?

Choices:
1. Nasogastric tube placement and suction
2. Bronchoscopy
3. Corticosteroids
4. Hydration
Answer: 3 - Corticosteroids
Explanations:
Aspiration pneumonitis can occur during anesthesia and can add to
morbidity.
One can place a nasogastric tube to suction the stomach and the patient can
be placed on mechanical ventilation.
Hydration and maintenance of hemodynamics are vital.
Corticosteroids have not been shown to benefit patients who aspirate.

Go to the next page if you knew the correct answer, or click the link image(s)
below to further research the concepts in this question (if desired).

Research Concepts:
Pneumonia, Aspiration

We update eBooks quarterly and Apps daily based on user feedback.


Please tap flag to report any questions that need improvement.
Question 532: How could a decrease in lung compliance affect the
outcome of pressure support ventilation?

Choices:
1. Increased exhaled tidal volume
2. Decreased exhaled tidal volume
3. Increased respiratory rate
4. Decreased respiratory rate
Answer: 2 - Decreased exhaled tidal volume
Explanations:
Compliance is simply understood as how stiff the lungs present.
Non-compliant lungs may require higher levels of pressure support to
properly assist the patient in achieving optimal tidal volumes.
Since the lungs are stiff, they accept a reduced volume of gas.
For lungs that are stiff, the exhaled tidal volume would be reduced as a
result of the lungs volume capacity.

Go to the next page if you knew the correct answer, or click the link image(s)
below to further research the concepts in this question (if desired).

Research Concepts:
Pressure Support

We update eBooks quarterly and Apps daily based on user feedback.


Please tap flag to report any questions that need improvement.
Question 533: Which of the following is not an application of video-
assisted thoracic surgery?

Choices:
1. Lung biopsy in interstitial lung disease
2. Evaluation and drainage of pleural effusion
3. Evaluation of lymph nodes
4. Biopsy of a lung mass in a patient with a previous pneumonectomy
Answer: 4 - Biopsy of a lung mass in a patient with a previous
pneumonectomy

Explanations:
It is difficult to perform a biopsy on an inflated lung.
In patients with pneumonectomy, it is difficult to perform video-assisted
thoracic surgery because of serious hypoxia resulting from deflation of the
only lung.

Go to the next page if you knew the correct answer, or click the link image(s)
below to further research the concepts in this question (if desired).

Research Concepts:
Video-Assisted Thoracoscopy (VATs)

We update eBooks quarterly and Apps daily based on user feedback.


Please tap flag to report any questions that need improvement.
Question 534: In mechanical ventilation, how is PCO2 effected when the
respiratory rate is increased, assuming a constant tidal volume?

Choices:
1. Increases
2. Decreases
3. No change
4. Effect cannot be predicted
Answer: 2 - Decreases
Explanations:
The concentration of carbon dioxide (CO2) is inversely proportional to the
ventilation rate.
Increase respiratory rate to lower blood CO2 level; monitor pH through
arterial blood gasses to ensure a safe acid-base balance.
Decrease respiratory rate to raise blood CO2 level; perform arterial blood
gas to assure safe pH levels. If conscious or mildly sedated, one can trigger
the ventilator to deliver more breaths per minute than desired. This usually
indicates an appropriate respiratory drive to self-maintain pH levels in the
blood.
Add dead space to ventilator tubing by connecting 50 mL lengths of
corrugated tubing where the ventilator circuit connects to the endotracheal
tube to increase CO2 levels in conscious ventilator patients. The exhaled
air, which is high in CO2, will remain trapped in the dead space at the end
of exhalation and re-inhaled with the next breath.

Go to the next page if you knew the correct answer, or click the link image(s)
below to further research the concepts in this question (if desired).

Research Concepts:
Ventilation, Ventilator Management

We update eBooks quarterly and Apps daily based on user feedback.


Please tap flag to report any questions that need improvement.
Question 535: What is the gold standard for detecting lung transplant
rejection?

Choices:
1. Radiologic imaging
2. Pulmonary function test
3. CT scan
4. Lung biopsy
Answer: 4 - Lung biopsy
Explanations:
Post lung transplantation rejection is diagnosed by a lung biopsy.
Bronchoscopy with biopsy has replaced open lung biopsy for detecting lung
rejection.
In more than 50% of patients with rejection, the chest x-ray may be normal.
Even if signs are observed on the chest x-ray, they are non-specific and may
include interstitial disease or consolidation.
A CT scan of the chest may reveal septal thickening, ground-glass opacities,
consolidation, or nodules. Some physicians will treat the patient with
corticosteroids for 48 hours, and if these changes disappear, then acute
rejection is confirmed.

Go to the next page if you knew the correct answer, or click the link image(s)
below to further research the concepts in this question (if desired).

Research Concepts:
Transplantation, Lung

We update eBooks quarterly and Apps daily based on user feedback.


Please tap flag to report any questions that need improvement.
Question 536: Which of the following is the most common risk factor for
acute respiratory distress syndrome?

Choices:
1. Pulmonary contusion
2. Sepsis
3. Aspiration
4. Near drowning
Answer: 2 - Sepsis
Explanations:
Sepsis is the most common risk factor for acute respiratory distress
syndrome (ARDS).
ARDS is the most severe form of acute lung injury.
The partial pressure arterial oxygen and fraction of inspired oxygen ratio is
less than 200 in ARDS.
Cardiogenic pulmonary edema must be ruled out to make a diagnosis of
ARDS.

Go to the next page if you knew the correct answer, or click the link image(s)
below to further research the concepts in this question (if desired).

Research Concepts:
Acute Respiratory Distress Syndrome (ARDS)

We update eBooks quarterly and Apps daily based on user feedback.


Please tap flag to report any questions that need improvement.
Question 537: Which is true about a psychogenic cough?
Choices:
1. A cough followed by a "whoop"
2. A cough that is seasonal
3. A cough that is throat clearing
4. A cough that occurs during waking hours and ceases during sleep
Answer: 4 - A cough that occurs during waking hours and ceases during sleep
Explanations:
A psychogenic cough occurs during waking hours and ceases during sleep.
A cough related to allergic rhinitis is seasonal.
A cough related to postnasal drip is throat clearing.
A cough followed by a whoop is pertussis.

Go to the next page if you knew the correct answer, or click the link image(s)
below to further research the concepts in this question (if desired).

Research Concepts:
Psychogenic Cough

We update eBooks quarterly and Apps daily based on user feedback.


Please tap flag to report any questions that need improvement.
Question 538: What does a zone of induration greater than 15 mm mean in
interpreting a tuberculin skin test?

Choices:
1. Active infection with Mycobacterium tuberculosis
2. Active infection with any of the nontuberculous mycobacteria
3. Previous infection with M tuberculosis
4. Vaccination with bacillus Calmette-Guerin vaccine only
Answer: 3 - Previous infection with M tuberculosis
Explanations:
The tuberculin test indicates previous infection with Mycobacterium
tuberculosis or Mycobacterium bovis from several weeks up to 5 years.
It is not proof of current active infection with M. tuberculosis.
The tuberculin test detects cell-mediated immunity, not antibody.
If infection is with a nontuberculous strain of mycobacteria, the skin test is
generally smaller.

Go to the next page if you knew the correct answer, or click the link image(s)
below to further research the concepts in this question (if desired).

Research Concepts:
Tuberculosis, PPD Skin Test (Tuberculosis Skin Test)

We update eBooks quarterly and Apps daily based on user feedback.


Please tap flag to report any questions that need improvement.
Question 539: A pneumothorax in what part of the lung is hardest to
visualize on a plain chest radiograph?

Choices:
1. Inferior
2. Lateral
3. Apical
4. Medial
Answer: 3 - Apical
Explanations:
The majority of pneumothoraces that occur in the apex of the lung can be
seen a chest radiograph. This is where they are most likely found on upright
imaging.
Small pneumothoraces in the apex of the lung may not be seen.
In most cases, these small pneumothoraces may be visualized only on a
chest CT.
While it is true that pneumothoraces typically move with gravity, they may
sometimes be fixed in pleural position by fibrosis or inflammation.

Go to the next page if you knew the correct answer, or click the link image(s)
below to further research the concepts in this question (if desired).

Research Concepts:
Pneumothorax

We update eBooks quarterly and Apps daily based on user feedback.


Please tap flag to report any questions that need improvement.
Question 540: Which is true about flail chest breathing pattern?
Choices:
1. The mediastinum is pulled toward the affected side during expiration
2. The affected chest caves in during expiration
3. The affected chest bulges out during inspiration
4. The mediastinum is pulled toward the affected side during inspiration
Answer: 1 - The mediastinum is pulled toward the affected side during
expiration

Explanations:
In patients with flail chest, the affected area bulges out during expiration
and pulls the mediastinum toward the affected side.
Choices 2 and 3 describe what happens normally, the chest enlarging in
inspiration and shrinking in expiration.
A flail chest segment moves paradoxically, so during expiration while the
rest of the chest is shrinking, it is expanding.
This paradoxical movement of the chest wall will mechanically cause the
mediastinum to move toward the flail side during expiration and away
during inspiration.

Go to the next page if you knew the correct answer, or click the link image(s)
below to further research the concepts in this question (if desired).

Research Concepts:
Flail Chest

We update eBooks quarterly and Apps daily based on user feedback.


Please tap flag to report any questions that need improvement.
Question 541: Which is the initial sign of malignant hyperthermia?
Choices:
1. Increased end tidal CO2
2. Hypoxia
3. Tachycardia
4. Muscle rigidity
Answer: 3 - Tachycardia
Explanations:
The first feature in many patients who develop malignant hyperthermia is
unexplained tachycardia.
This is often followed by a steady rise in end-tidal CO2 and metabolic
acidosis.
Muscle rigidity and fever soon follow. Malignant hyperthermia is life
threatening and is usually caused by a general anesthetics or depolarizing
agent like succinylcholine.
If untreated, vascular collapse is common and death is certain. The disorder
is inherited in an autosomal dominant fashion.

Go to the next page if you knew the correct answer, or click the link image(s)
below to further research the concepts in this question (if desired).

Research Concepts:
Malignant Hyperthermia

We update eBooks quarterly and Apps daily based on user feedback.


Please tap flag to report any questions that need improvement.
Question 542: What is the first step in a positive inspiratory pressure leak
with laryngeal mask airway?

Choices:
1. Inflate cuff
2. Adjust tube position
3. Change to a larger laryngeal mask airway
4. Increase flow rate
Answer: 2 - Adjust tube position
Explanations:
The peak inspiratory pressure with a laryngeal mask airway (LMA) should
be kept under 30 cmH2O.
Any air leak with an LMA should be adjusted by pushing it downwards.
The optimal head position for an LMA insertion is the "sniffing" position.
The LMA is a supraglottic airway device designed as a method of elective
ventilation and an alternative to bag-valve-mask ventilation. It frees the
provider's hands and results in less gastric distention. The LMA was
initially used in the operating room; however, more recently it is used by
emergency services personnel for the management of the difficult airway.

Go to the next page if you knew the correct answer, or click the link image(s)
below to further research the concepts in this question (if desired).

Research Concepts:
Airway, Laryngeal Mask

We update eBooks quarterly and Apps daily based on user feedback.


Please tap flag to report any questions that need improvement.
Question 543: What oxygen supply system is most ideal for travel?
Choices:
1. Liquid
2. Concentrated
3. Solid
4. Frozen
Answer: 1 - Liquid
Explanations:
Liquid oxygen contains large supply of oxygen.
It is also lightweight for travel.

Go to the next page if you knew the correct answer, or click the link image(s)
below to further research the concepts in this question (if desired).

Research Concepts:
Home Oxygen Therapy

We update eBooks quarterly and Apps daily based on user feedback.


Please tap flag to report any questions that need improvement.
Question 544: Which of the following criteria is associated with the highest
risk of complications following a thoracotomy?

Choices:
1. Forced expiratory volume in one second less than 50% predicted
2. Maximum voluntary ventilation less than 45% predicted
3. Diffusing capacity of the lungs for carbon monoxide less than 40% predicted
4. Maximal oxygen consumption less than 12 mL/kg/min
Answer: 4 - Maximal oxygen consumption less than 12 mL/kg/min
Explanations:
Induced maximal oxygen consumption less than 10 mL/kg/min is
considered the best predictor of complications after surgery.
If the pulmonary vascular resistance is greater than 190 dynes, there also is
an increased risk of complications.
The major cause of perioperative morbidity and mortality after thoracic
surgery is respiratory complications, primarily pneumonia, atelectasis, and
respiratory failure.
Many scoring tools have been developed, but they are often subjective and
lack sensitivity. The best tool is the surgeon's clinical preoperative workup
and judgment.

Go to the next page if you knew the correct answer, or click the link image(s)
below to further research the concepts in this question (if desired).

Research Concepts:
Thoracotomy

We update eBooks quarterly and Apps daily based on user feedback.


Please tap flag to report any questions that need improvement.
Question 545: Use of which of the following medications should be
monitored with routine lung function testing?

Choices:
1. Adenosine
2. Amiodarone
3. Lidocaine
4. Quinidine
Answer: 2 - Amiodarone
Explanations:
Long term use of amiodarone is associated with lung fibrosis.
Patients taking amiodarone should undergo regular lung function testing.
Hyper/hypothyroidism is another common complication of this drug.
Quinidine can cause hemolytic anemia.

Go to the next page if you knew the correct answer, or click the link image(s)
below to further research the concepts in this question (if desired).

Research Concepts:
Amiodarone

We update eBooks quarterly and Apps daily based on user feedback.


Please tap flag to report any questions that need improvement.
Question 546: Which of the following is most characteristic of
bronchiolitis obliterans?

Choices:
1. Decreased expiratory reserve volume (ERV)
2. Decreased forced expiratory volume/1 sec (FEV1)
3. Decreased total lung capacity (TLC)
4. Decreased forced vital capacity (FVC)
Answer: 2 - Decreased forced expiratory volume/1 sec (FEV1)
Explanations:
Bronchiolitis obliterans (BO) is characterized by progressive airflow
limitation and fibroproliferative thickening of bronchiolar walls. It occurs
as a manifestation of rejection in lung transplant recipients as well as in
response to inhalational injuries. Therapy for BO is limited.
For transplant patients with BO, some recommend augmentation of
immunosuppression, steroids, and cytolytic therapy.The majority of
individuals contract opportunistic infections. This is the most common
cause of death.
When pulmonary function tests are performed, there is decreased air flow
during expiration, and reflects as a decreased FEV1, due to the narrowing
of the bronchiolar lumen from the fibro proliferative thickening of the
walls. Decreased FEV1 is the most diagnostic of BO.
Spirometry classically shows obstructive flow pattern with decreased
FEV1/FVC as well.

Go to the next page if you knew the correct answer, or click the link image(s)
below to further research the concepts in this question (if desired).

Research Concepts:
Bronchiolitis Obliterans (Obliterative Bronchiolitis, Constrictive
Bronchiolitis)

We update eBooks quarterly and Apps daily based on user feedback.


Please tap flag to report any questions that need improvement.
Question 547: Which of the following claims about sequestrations is false?
Choices:
1. With an intralobar sequestration, an AV shunt can occur
2. Extralobar sequestrations are more common on the left
3. Extralobar sequestration may present with symptoms of an infection in
childhood
4. Intralobar sequestrations may be associated with a Morgagni hernia
Answer: 4 - Intralobar sequestrations may be associated with a Morgagni
hernia

Explanations:
Sequestrations are non-functioning lung masses and receive blood supply
from the thoracic or abdominal aorta. There are two types, intralobar (75%)
and extralobar (25%). Intralobar sequestrations are evenly divided by
gender, but extralobar sequestrations are more common in males.
The venous drainage for intralobar sequestrations is via the pulmonary
veins. Venous drainage for extra lobar sequestration is via the azygous or
IVC. Neither form with the large bronchi. They may show air fluid levels if
infected. If the blood supply is large, CHF may occur due to high output
CHF from shunting.
Extra lobar sequestrations are associated with congenital anomalies, e.g.,
congenital diaphragmatic hernia. Extralobar sequestration usually presents
in infancy due to respiratory compression, but may present in childhood
with respiratory distress. Intralobar sequestration more commonly presents
in adults as recurrent chronic infections.
Chest x-ray is not diagnostic. CT can show the abnormal arterial supply
post-natally while US can show them pre-natally (fetal US). Extralobar
variant is treated by resection of the mass whereas intralobar type requires a
lobectomy. One has to be aware of aberrant vessels when excising these
masses since massive hemorrhage can result.

Go to the next page if you knew the correct answer, or click the link image(s)
below to further research the concepts in this question (if desired).

Research Concepts:
Pulmonary Sequestration

We update eBooks quarterly and Apps daily based on user feedback.


Please tap flag to report any questions that need improvement.
Question 548: Which of the following is the most common cause of
chronic stridor in children?

Choices:
1. Croup
2. Laryngomalacia
3. Choanal atresia
4. Foreign body

Photo:Contributed by Gray's Anatomy Plates


Answer: 2 - Laryngomalacia
Explanations:
Laryngomalacia is the most common cause of chronic stridor in children.
It is characterized by laxness or congenital deformities of the cartilage that
make up the larynx.
Collapse of the laryngeal walls occurs on inspiration resulting in stridor.
No specific therapy is needed in most cases, and the condition generally
resolves by 2 years of age.

Go to the next page if you knew the correct answer, or click the link image(s)
below to further research the concepts in this question (if desired).

Research Concepts:
Laryngomalacia

We update eBooks quarterly and Apps daily based on user feedback.


Please tap flag to report any questions that need improvement.
Question 549: What size of induration is considered positive for a purified
protein derivative (PPD) skin test in a low-risk patient?

Choices:
1. 1 millimeter
2. 4 millimeters
3. 9 millimeters
4. 15 millimeters
Answer: 4 - 15 millimeters
Explanations:
A purified protein derivative (PPD) skin test of 15 millimeters or more is
considered positive in low risk, healthy patients.
This cutoff size is smaller for at-risk groups.
False negatives are common if the patient has infectious mononucleosis,
sarcoidosis, Hodgkin disease, or is malnourished.
The interferon-gamma release assay can be used instead of the PPD skin
test.

Go to the next page if you knew the correct answer, or click the link image(s)
below to further research the concepts in this question (if desired).

Research Concepts:
Tuberculosis, PPD Skin Test (Tuberculosis Skin Test)

We update eBooks quarterly and Apps daily based on user feedback.


Please tap flag to report any questions that need improvement.
Question 550: What is the drug of choice for the prevention of respiratory
syncytial virus in high-risk children?

Choices:
1. Ribavirin
2. Palivizumab
3. Acyclovir
4. Epinephrine
Answer: 2 - Palivizumab
Explanations:
Palivizumab is an IgG directed against an epitope in the A antigenic site of
the F protein of respiratory syncytial virus (RSV). Palivizumab targets the
fusion protein of RSV and inhibits its entry into the cell for reproduction,
thus preventing infection. Dosing is once a month during the RSV season,
November through April, and is administered intramuscularly. Do not use if
particles are present, if the vial is cracked or damaged in any way, or if the
solution is cloudy or discolored.
Palivizumab dosing is at 15mg/kg and should be used for no more than 5
monthly doses according to the American Academy of Pediatrics
guidelines.
Children with hemodynamically significant congenital heart disease,
children born prematurely born at less than or equal to 29 weeks gestation,
children with chronic lung disease of prematurity who were born at less
than or equal to 32 weeks gestation are all considered high risk and eligible
for palivizumab prophylaxis.
Ribavirin is an antiviral with limited positive effects that has been linked to
adverse effects for patients. It also poses a risk to healthcare staff
administering the aerosolized medication.

Go to the next page if you knew the correct answer, or click the link image(s)
below to further research the concepts in this question (if desired).

Research Concepts:
Respiratory Syncytial Virus Infection (RSV)

We update eBooks quarterly and Apps daily based on user feedback.


Please tap flag to report any questions that need improvement.
Question 551: Which of these conditions is least likely to cause respiratory
distress in a newborn?

Choices:
1. Intralobar sequestration
2. Extralobar sequestration
3. Congenital lobar emphysema
4. Congenital diaphragmatic hernia
Answer: 1 - Intralobar sequestration
Explanations:
Intralobar sequestration generally is seen in the older child or adult and may
be entirely asymptomatic or may present with bronchiectasis or chronic
obstructive pulmonary disease.
The other conditions mentioned will typically present in infancy with
failure to thrive, dyspnea, cyanosis, and respiratory distress.
Extralobar pulmonary sequestration accounts for 15% to 25% of pulmonary
sequestrations. It is covered by its own pleura that are more commonly seen
in the lower lobes.
Congenital diaphragmatic hernias are associated with a variable degree of
pulmonary hypoplasia. There are often alterations of the surfactant system.
Congenital lobar emphysema is a potentially reversible condition. It may
require surgery

Go to the next page if you knew the correct answer, or click the link image(s)
below to further research the concepts in this question (if desired).

Research Concepts:
Neonatal Respiratory Distress Syndrome

We update eBooks quarterly and Apps daily based on user feedback.


Please tap flag to report any questions that need improvement.
Question 552: Which of the following is not true of inverse ratio
ventilation?

Choices:
1. It is used with pressure-controlled ventilation
2. Mean airway pressure will increase with prolongation of the expiratory time
3. Air trapping can occur (auto-PEEP)
4. It requires heavy sedation for use
Answer: 2 - Mean airway pressure will increase with prolongation of the
expiratory time

Explanations:
Inverse ratio ventilation is now used for acute respiratory distress syndrome
but can be associated with air trapping.
Mean airway pressure is directly elevated by prolongation of the inspiratory
time.
Patients have to be sedated and paralyzed for this type of ventilation.
As its associated with significant air trapping thus it can lead to auto-PEEP
and hemodynamic instability. Randomized clinical trials are still awaited
for inverse ratio ventilation.

Go to the next page if you knew the correct answer, or click the link image(s)
below to further research the concepts in this question (if desired).

Research Concepts:
Ventilation, Inverse Ratio

We update eBooks quarterly and Apps daily based on user feedback.


Please tap flag to report any questions that need improvement.
Question 553: What is the biggest advantage of using an armored
endotracheal tube?

Choices:
1. No kinking
2. No splitting
3. No tearing
4. No decaying
Answer: 1 - No kinking
Explanations:
An armored endotracheal tube is not likely to kink.

Go to the next page if you knew the correct answer, or click the link image(s)
below to further research the concepts in this question (if desired).

Research Concepts:
Intubation, Endotracheal Tube, Armored

We update eBooks quarterly and Apps daily based on user feedback.


Please tap flag to report any questions that need improvement.
Question 554: What is true regarding the Jarisch-Herxheimer reaction?
Choices:
1. Seen when an antibiotic is used to treat a susceptible infection typically
caused by a spirochete
2. It has a high mortality
3. Indicates a resistant strain of bacteria
4. All of the above
Answer: 1 - Seen when an antibiotic is used to treat a susceptible infection
typically caused by a spirochete

Explanations:
The Jarisch-Herxheimer reaction is seen when an antibiotic is used to treat a
susceptible infection typically caused by a spirochete.
There is not a high mortality.
It is thought that the reaction, which includes fever, myalgias, and
worsening rash, is a result of toxins released from the death of bacteria.
In syphilis, the symptoms last hours, in Lyme disease-sometimes months or
years. It has been seen with syphilis, Lyme, tick-borne relapsing fever,
borreliosis, Q fever, cat-scratch fever, and others.

Go to the next page if you knew the correct answer, or click the link image(s)
below to further research the concepts in this question (if desired).

Research Concepts:
Jarisch Herxheimer Reaction

We update eBooks quarterly and Apps daily based on user feedback.


Please tap flag to report any questions that need improvement.
Question 555: What is the best position for preoxygenation in preparation
for a rapid sequence intubation?

Choices:
1. Supine
2. Head down 20 degrees
3. Head up 20 degrees
4. Either position is fine
Answer: 3 - Head up 20 degrees
Explanations:
Preoxygenation is very important and should be done for a period of at least
3 minutes. I should be preferably be done with a non-rebreather mask.
Ideally, patients should be placed with the head up for better
preoxygenation of the alveoli without the pressure of the abdomen on the
lungs.
Studies have shown that the head up position helps with avoiding
microaspiration.
Head down position could be detrimental to a patient with a full stomach.

Go to the next page if you knew the correct answer, or click the link image(s)
below to further research the concepts in this question (if desired).

Research Concepts:
Intubation, Tracheal Rapid Sequence

We update eBooks quarterly and Apps daily based on user feedback.


Please tap flag to report any questions that need improvement.
Question 556: The major medical use of nitric oxide has been to decrease
which of the following?

Choices:
1. Inflammation
2. Peptic ulcer disease
3. Pulmonary artery pressure
4. Incidence of asthma
Answer: 3 - Pulmonary artery pressure
Explanations:
Nitric oxide has found use in cardiovascular surgery. It has been used
successfully when there is pulmonary hypertension.

Go to the next page if you knew the correct answer, or click the link image(s)
below to further research the concepts in this question (if desired).

Research Concepts:
Nitric Oxide

We update eBooks quarterly and Apps daily based on user feedback.


Please tap flag to report any questions that need improvement.
Question 557: When properly placed, the lateral portions of the laryngeal
mask are usually seated over which structure?

Choices:
1. Tonsils
2. Esophagus
3. Vallecula
4. Piriform fossa
Answer: 4 - Piriform fossa
Explanations:
The apex of the laryngeal mask should point towards the tongue and push
back towards the uvula.
The cuff should follow the natural end of the oropharynx and seat over the
pyriform fossa.
Once placed, the inflated cuff creates an air-tight seal.
The laryngeal mask airway (LMA) is an excellent alternative to the use of
the bag mask to avoid gastric inflation, decreasing the risk of aspiration.
This less invasive method should be utilized unless it is ineffective. The
LMA offers the clinician an airway aperture that allows for blind insertion
when the bag mask is ineffective, the patient has a short neck, a difficult
airway, requires high FiO2 levels, is experiencing apnea or poor respiratory
effort, or has a Mallampati score of III or IV.

Go to the next page if you knew the correct answer, or click the link image(s)
below to further research the concepts in this question (if desired).

Research Concepts:
Airway, Laryngeal Mask

We update eBooks quarterly and Apps daily based on user feedback.


Please tap flag to report any questions that need improvement.
Question 558: What is the best treatment for a patient with an
Echinococcal hydatid cyst in the lung that measures 5 x 5 cm?

Choices:
1. Aspiration under ultrasound
2. Antibiotics
3. Surgery
4. Radiation
Answer: 3 - Surgery
Explanations:
For a simple cyst located in the body, surgical removal is the safest and
quickest way to get rid of the parasite.
The patient should be treated with albendazole before and after surgery.
It is vital not to break the cysts or cause spillage, as an anaphylactic
reaction can occur.
New percutaneous methods have been devised which first heat-kill the
parasite, followed by aspiration of fluid.

Go to the next page if you knew the correct answer, or click the link image(s)
below to further research the concepts in this question (if desired).

Research Concepts:
Echinococcus Granulosus (Hydatid Cysts, Echinococcosis)

We update eBooks quarterly and Apps daily based on user feedback.


Please tap flag to report any questions that need improvement.
Question 559: In a patient with an echinococcosis lung infection, what is
the surgical procedure of choice?

Choices:
1. Enucleation of the intact cysts
2. Needle aspiration during open thoracostomy
3. CT-guided needle aspiration
4. Lobectomy
Answer: 1 - Enucleation of the intact cysts
Explanations:
Humans are an intermediate host for hydatid cyst. Humans come into
contact with dogs and accidentally ingests fecal material from dogs, which
contain the cysts.
The hydatid cyst may present as an incidental finding or patients may
present with bloody sputum, cough rash, urticaria, fever, and
bronchospasm.
The Casoni test is unreliable. For asymptomatic patients, there is more
emphasis now on drug therapy (albendazole).
One can instill hypertonic saline in the cyst at the time of enucleation to kill
cysts. Complications of spillage include anaphylactic shock, pleural
contamination, and possible death.

Go to the next page if you knew the correct answer, or click the link image(s)
below to further research the concepts in this question (if desired).

Research Concepts:
Echinococcus Granulosus (Hydatid Cysts, Echinococcosis)

We update eBooks quarterly and Apps daily based on user feedback.


Please tap flag to report any questions that need improvement.
Question 560: Massive hemoptysis temporally related to the insertion of a
Swan-Ganz catheter is most likely related to which of the following?

Choices:
1. Pulmonary emboli
2. Aortic dissection
3. Pulmonary artery rupture
4. Esophageal rupture
Answer: 3 - Pulmonary artery rupture
Explanations:
Pulmonary artery rupture is a documented complication of pulmonary
catheter placement and has a very high mortality rate.
Some surgeons have attempted to use stents to seal the rupture or a
pseudoaneurysm development, but survivors are still rare if there is massive
bleeding.
Use of the Swan-Ganz catheter in patients with pulmonary hypertension
should be with great caution.
The catheter balloon always should be inflated with air, not fluids. When
deflating the balloon, apply some traction on the catheter.

Go to the next page if you knew the correct answer, or click the link image(s)
below to further research the concepts in this question (if desired).

Research Concepts:
Catheterization, Pulmonary Artery

We update eBooks quarterly and Apps daily based on user feedback.


Please tap flag to report any questions that need improvement.
Question 561: Select the true statement about the diagnosis of
bronchiectasis.

Choices:
1. Bronchography is the diagnostic test of choice
2. High resolution CT scan is the diagnostic test of choice
3. The diagnosis is best made by bronchoscopy
4. It cannot be diagnosed during an acute pulmonary infection
Answer: 2 - High resolution CT scan is the diagnostic test of choice
Explanations:
High resolution CT has replace bronchography for diagnosis of
bronchiectasis.
Bronchiectasis is characterized by permanent abnormal dilation of the
bronchi.
It is usually caused by an infection. Tuberculosis is most common
worldwide but in the United States Staphylococcus aureus, Klebsiella,
adenovirus, and influenza virus are all causes.
It is usually seen in older women but those with underlying disease such as
cystic fibrosis may be affected at a much younger age.

Go to the next page if you knew the correct answer, or click the link image(s)
below to further research the concepts in this question (if desired).

Research Concepts:
Bronchiectasis

We update eBooks quarterly and Apps daily based on user feedback.


Please tap flag to report any questions that need improvement.
Question 562: A patient is shot in the right chest and has a chest tube
placed. The chest tube initially drains 900 ml of blood over the next 2 days. The
patient is discharged after 5 days but returns back 2 weeks later with spiking
fevers, diaphoresis, and dyspnea. What is the most likely cause of his fever?

Choices:
1. Endocarditis
2. Pneumonia
3. Empyema
4. Atelectasis
Answer: 3 - Empyema
Explanations:
After trauma and bleeding in the chest, there is always some blood retained
in the chest cavity.
If the blood is not adequately evacuated from the chest cavity, it can clot
and get infected, resulting in an empyema.
An infected hemothorax usually presents as a fever after a few weeks.
Chest x-ray and CT scan are diagnostic.
Delayed presentation means an open thoracotomy, but the surgeon may
attempt video-assisted thoracoscopic surgery to clean the chest.

Go to the next page if you knew the correct answer, or click the link image(s)
below to further research the concepts in this question (if desired).

Research Concepts:
Empyema

We update eBooks quarterly and Apps daily based on user feedback.


Please tap flag to report any questions that need improvement.
Question 563: Which of the following skin conditions is associated with
sarcoidosis?

Choices:
1. Erythema marginatum
2. Lupus pernio
3. Incontinentia pigmenti
4. Pemphigus vulgaris
Answer: 2 - Lupus pernio
Explanations:
Lupus pernio is a commonly seen manifestation of skin disease in
sarcoidosis.
It is characterized by red-to-purple or violaceous, indurated plaques and
nodules.
The facial area including the nasal skin is quite often involved.
The course is usually chronic. Severe cosmetic disfigurement may result.

Go to the next page if you knew the correct answer, or click the link image(s)
below to further research the concepts in this question (if desired).

Research Concepts:
Lupus Pernio

We update eBooks quarterly and Apps daily based on user feedback.


Please tap flag to report any questions that need improvement.
Question 564: Which of the following is the most common cause of
community-acquired empyema?

Choices:
1. Streptococcus
2. Klebsiella
3. Pseudomonas
4. Haemophilus
Answer: 1 - Streptococcus
Explanations:
Streptococcus pneumoniae is the most common cause of empyema.
Staphylococcus aureus is the most common organism in empyema
complicating previous surgery and acquired in the hospital. Methicillin-
resistant Staphylococcus aureus can cause up to 20% of cases.
Klebsiella, Pseudomonas, and Haemophilus are the most common aerobic
gram-negative organisms causing empyema.
Empyema typically requires tube thoracostomy in addition to antibiotics.

Go to the next page if you knew the correct answer, or click the link image(s)
below to further research the concepts in this question (if desired).

Research Concepts:
Empyema

We update eBooks quarterly and Apps daily based on user feedback.


Please tap flag to report any questions that need improvement.
Question 565: A patient with a pneumonectomy now presents 2 years later
with an empyema. What is the best management?

Choices:
1. Closed drainage
2. Clagett procedure
3. Debridement and re-closure
4. Thoracotomy with immediate muscle flap
Answer: 2 - Clagett procedure
Explanations:
Patients with an empyema will generally present with lethargy, anorexia,
and fever.
The most common cause of delayed empyema following pneumonectomy
is hematogenous spread. Diagnosis is easily made with bronchoscopy and a
CT scan.
Clagett is a 2-stage procedure. The first stage requires creating an open
window with drainage, followed by debridement and irrigation for 4 to 6
weeks. Once the cavity is sterilized, the second stage is undertaken.
During the second stage, the window is excised, and then the space is filled
with antibiotic solution and a muscle flap and closed.

Go to the next page if you knew the correct answer, or click the link image(s)
below to further research the concepts in this question (if desired).

Research Concepts:
Empyema

We update eBooks quarterly and Apps daily based on user feedback.


Please tap flag to report any questions that need improvement.
Question 566: What is the best test to evaluate patients who may be
deemed respectable of bullous disease?

Choices:
1. Arteriography
2. CT scan
3. Pulmonary function tests
4. Split nuclear studies
Answer: 2 - CT scan
Explanations:
CT is the ultimate imaging study for bullous disease.
It demonstrates size, location, and extent of the bulla, and better resolution
of the pulmonary arteries.
Patients with localized compression of the native lung by the bulla tend to
have the best results.
However, air leaks in the postoperative period and residual airspace disease
are not uncommon.

Go to the next page if you knew the correct answer, or click the link image(s)
below to further research the concepts in this question (if desired).

Research Concepts:
Pulmonary Bulla

We update eBooks quarterly and Apps daily based on user feedback.


Please tap flag to report any questions that need improvement.
Question 567: What is the difference between catheter-related bloodstream
infection (CRBSI) and Central Line Associated Blood Stream Infections
(CLABSI) ?

Choices:
1. CRBSI is from a central line only while CABSI is from any catheter
2. CRBSI incidents require less proof for the catheter being the source of
infection than CLABSI incidents
3. CRBSI occurs in immunocompromised patients while CLABSI can occur in
all patients
4. CRBSI is used clinically while CLABSI is used for surveillance
Answer: 4 - CRBSI is used clinically while CLABSI is used for surveillance
Explanations:
"Catheter-related bloodstream infection" is used when the central line is
identified as the source of the bloodstream infection. The central line is
identified as the source through laboratory testing, central line tip cultures,
quantitative blood cultures from the central line compared to peripheral
blood cultures, etc.
"Central Line Associated Blood Stream Infections (CLABSI) " is used
when the patient has a bloodstream infection, a catheter in place for greater
than 2 calendar days before the positive blood cultures were obtained, and
no obvious infection at another site.
Clinical definitions identify disease and treatment needs for an individual
patient. Surveillance definitions identify trends in a population for research
and development of prevention strategies.
CRBSI and CLABSI can occur in all patients, immunocompromised or not.
CRBSI requires more proof with laboratory evidence, not less, compared to
CLABSI. Both CRBSI and CLABSI are used for central catheters.

Go to the next page if you knew the correct answer, or click the link image(s)
below to further research the concepts in this question (if desired).

Research Concepts:
Central Line Associated Blood Stream Infections (CLABSI)

We update eBooks quarterly and Apps daily based on user feedback.


Please tap flag to report any questions that need improvement.
Question 568: A 5 month old infant is diagnosed with failure to thrive. The
patient was born at 28 weeks gestation and has been off oxygen for 8 weeks. The
only medications are albuterol and multivitamins. Caloric intake is adequate.
The physical exam is normal except for the weight. Chest x-ray shows
bronchopulmonary dysplasia. Select the next step in management.

Choices:
1. Night time tube feedings
2. Sleeping measurement of oxygen saturation
3. Sweat chloride testing
4. Restart oxygen
Answer: 2 - Sleeping measurement of oxygen saturation
Explanations:
The infant is most likely having apnea at night that would be detected by an
oxygen saturation monitor.

Go to the next page if you knew the correct answer, or click the link image(s)
below to further research the concepts in this question (if desired).

Research Concepts:
Bronchopulmonary Dysplasia

We update eBooks quarterly and Apps daily based on user feedback.


Please tap flag to report any questions that need improvement.
Question 569: Which one of the following prostacyclin synthetic analogs
are used for the treatment of primary pulmonary hypertension?

Choices:
1. Bosentan
2. Prostaglandin E
3. Alprostadil
4. Iloprost
Answer: 4 - Iloprost
Explanations:
Iloprost is a prostacyclin analog approved for treating primary pulmonary
hypertension.
Continuous, long-term intravenous epoprostenol infusion, for which semi-
permanent central venous catheter is required, is considered the most
effective therapy. It improves mortality but has a short half-life and is
expensive. For those who cannot tolerate intravenous infusion, inhaled or
subcutaneous prostanoids can be considered. Treprostinil can be used by
various routes such as intravenous, subcutaneous, and inhalation.
Bosentan also is also used in primary pulmonary hypertension, but it is not
a prostaglandin analog. Iloprost and treprostinil are inhaled.
Oral high dose calcium channel blockers, such as diltiazem and nifedipine,
are the first-line treatment. They only are used in those with vasoreactivity
testing positive for acute vasodilator response with short-acting pulmonary
vasodilators such as adenosine, nitric oxide, or epoprostenol. The criteria
for testing positive is a fall in pulmonary artery pressure to more than 10
mmHg with an increase or no difference in cardiac output. They should not
be used in non-responders to vasoreactivity test due to the risk of harm
being greater than potential improvement.

Go to the next page if you knew the correct answer, or click the link image(s)
below to further research the concepts in this question (if desired).

Research Concepts:
Pulmonary Hypertension, Primary

We update eBooks quarterly and Apps daily based on user feedback.


Please tap flag to report any questions that need improvement.
Question 570: Which of the following statements about BCG vaccination
for the prevention of tuberculosis is true?

Choices:
1. It uses a live vaccine
2. It is about 90 percent effective
3. Immunity persists with time
4. It is very safe
Answer: 1 - It uses a live vaccine
Explanations:
BCG is a live, attenuated strain of Mycobacterium bovis.
It may cause disseminated, fatal disease, usually in the
immunocompromised.
It is about 50 to 60 percent effective in causing immunity, but the immunity
is generally lost within ten years.

Go to the next page if you knew the correct answer, or click the link image(s)
below to further research the concepts in this question (if desired).

Research Concepts:
Bacillus Calmette Guerin (BCG), Skin Test

We update eBooks quarterly and Apps daily based on user feedback.


Please tap flag to report any questions that need improvement.
Question 571: Which of the following is the treatment for gastroesophageal
reflux disease (GERD)?

Choices:
1. Ipecac
2. Metoclopramide
3. Ciprofloxacin
4. Codeine
Answer: 2 - Metoclopramide
Explanations:
GERD is a common cause of persistent cough.
Pro-kinetics (metoclopramide), and life style modification are useful
measures.
Proton pump inhibitors and H2 blockers are used more commonly.

Go to the next page if you knew the correct answer, or click the link image(s)
below to further research the concepts in this question (if desired).

Research Concepts:
Gastroesophageal Reflux Disease

We update eBooks quarterly and Apps daily based on user feedback.


Please tap flag to report any questions that need improvement.
Question 572: Which of the following is not a cause of central apnea in
infants?

Choices:
1. Meningitis
2. Birth asphyxia
3. Inborn errors of metabolism
4. Pierre Robin sequence
Answer: 4 - Pierre Robin sequence
Explanations:
Normal breathing is regulated by the respiratory center located in the
medulla. Both hypoxia and hypercapnia increase the respiratory drive that
results in increased respiratory effort. Various conditions can blunt this
response and depress the respiratory center, resulting in central apnea.
Examples of conditions that cause central apnea are meningitis; intracranial
injury or hemorrhage caused by birth asphyxia; and, in inborn errors of
metabolism and toxin exposures, metabolic conditions such as
hypoglycemia, hypocalcemia, and acidosis are seen.
Pierre Robin sequence is a set of abnormalities affecting the head and face
and consisting of micrognathia, glossoptosis, and frequently cleft palate.
This facial anomaly can cause obstruction to the upper airway and result in
obstructive apnea.
Apnea of prematurity is a result of the immaturity of the respiratory control
that results in predominantly central apnea, but the poorly developed
thoracic cage and musculature cause obstructive apnea. Since both
components contribute to the apnea, it is characterized as mixed apnea.

Go to the next page if you knew the correct answer, or click the link image(s)
below to further research the concepts in this question (if desired).

Research Concepts:
Brief Resolved Unexplained Event (BRUE)

We update eBooks quarterly and Apps daily based on user feedback.


Please tap flag to report any questions that need improvement.
Question 573: Prominent dilated bronchi and the signet ring sign on chest
CT are signs of which of the following?

Choices:
1. Bronchiectasis
2. Interstitial pneumonia
3. Lobar pneumonia
4. Small cell carcinoma
Answer: 1 - Bronchiectasis
Explanations:
The signet ring sign is a dilated bronchus cut horizontally next to a
pulmonary artery.
Bronchiectasis is a abnormal dilation of bronchi due to damage to the
muscular and elastic elements of the bronchial wall and is usually due to
chronic lung disease.
It is diagnosed with high resolution CT.

Go to the next page if you knew the correct answer, or click the link image(s)
below to further research the concepts in this question (if desired).

Research Concepts:
Bronchiectasis

We update eBooks quarterly and Apps daily based on user feedback.


Please tap flag to report any questions that need improvement.
Question 574: An asthmatic patient is initiating orally inhaled
beclomethasone to control his asthma. Which of the following is a counseling
point?

Choices:
1. It should be used at the first sign of an asthma attack
2. It can be administered from one to three times daily
3. It replaces an albuterol inhaler
4. The patient should rinse his mouth after each administration
Answer: 4 - The patient should rinse his mouth after each administration
Explanations:
Beclomethasone is a glucocorticoid medication.
It is associated with oral candidiasis because of its effect on oral mucosa.
This can be mitigated by oral rinsing after administration.
It is an anti-inflammatory, not a bronchodilator.

Go to the next page if you knew the correct answer, or click the link image(s)
below to further research the concepts in this question (if desired).

Research Concepts:
Corticosteroids, Inhaled

We update eBooks quarterly and Apps daily based on user feedback.


Please tap flag to report any questions that need improvement.
Question 575: What is the most common complication associated with
hydrocarbon ingestion?

Choices:
1. Dysrhythmia
2. Pneumonitis
3. Hepatic failure
4. Coma
Answer: 2 - Pneumonitis
Explanations:
Hydrocarbon ingestion or inhalation can lead to severe pulmonary toxicity
with pneumonitis being the most common complication associated with
hydrocarbons. Pulmonary complications can occur from inhalation or
ingestion with subsequent aspiration. Hydrocarbons with increased
viscosity typically cause more severe damage as they can spread over the
alveoli. Hydrocarbon pneumonitis is caused by the destruction of alveolar
membranes as well as alteration of surfactant. Supportive care is the
mainstay of treatment.
Hydrocarbons can also cause severe dysrhythmias. This is thought to
secondary to "catecholamine sensitization." Sudden sniffing death
syndrome has been known to occur which is typically seen when a "huffer"
is excited shortly after using hydrocarbons leading to a surge of
catecholamines. If a patient with hydrocarbon exposure is having runs of
ventricular tachycardia, beta-blockers may be used.
Certain hydrocarbons may also cause hepatic damage-- most specifically
chlorinated hydrocarbons. Supportive care is the mainstay of treatment, and
prolonged observation should be considered for patients with exposures to
hydrocarbons that may lead to hepatic damage.
Central nervous system depression is a common symptom following
hydrocarbon use. However, coma is uncommon. Other common
complications from hydrocarbon use are dermatitis, renal tubular acidosis,
gastrointestinal irritation, or blood dyscrasias.

Go to the next page if you knew the correct answer, or click the link image(s)
below to further research the concepts in this question (if desired).

Research Concepts:
Toxicity, Hydrocarbon, Inhalation
We update eBooks quarterly and Apps daily based on user feedback.
Please tap flag to report any questions that need improvement.
Question 576: In a patient with a bronchopleural fistula, what is the best
form of ventilation?

Choices:
1. Face mask
2. Jet ventilation
3. Endotracheal intubation
4. High flow
Answer: 2 - Jet ventilation
Explanations:
High-frequency jet ventilation is approved by the FDA for ventilating
patients with bronchopleural fistulae.
When one has a bronchopleural fistula, any positive end-expiratory pressure
or high tidal volumes simply aggravate the leak.
In most cases, the bronchopleural fistula is well controlled. If it is severe,
one may have to place a unilateral bronchopleural blocker.
Another option is a double lumen tube and ventilation of only the normal
lung.

Go to the next page if you knew the correct answer, or click the link image(s)
below to further research the concepts in this question (if desired).

Research Concepts:
Fistula, Bronchopleural

We update eBooks quarterly and Apps daily based on user feedback.


Please tap flag to report any questions that need improvement.
Question 577: Select the true statement concerning sleep in children.
Choices:
1. Upper airway resistance doubles
2. Ventilatory drive increases
3. Functional residual capacity increases
4. Breathing is often erratic during stages I and II
Answer: 1 - Upper airway resistance doubles
Explanations:
Ventilatory drive decreases, functional residual capacity decreases, and
upper airway resistance increases during sleep in children.

Go to the next page if you knew the correct answer, or click the link image(s)
below to further research the concepts in this question (if desired).

Research Concepts:
Normal Sleep

We update eBooks quarterly and Apps daily based on user feedback.


Please tap flag to report any questions that need improvement.
Question 578: What is the most valid method of making a diagnosis of fat
embolic syndrome after a long-bone fracture?

Choices:
1. Clinical evaluation
2. ECG
3. Arterial blood gas
4. Chest x-ray
Answer: 1 - Clinical evaluation
Explanations:
There is not one, specific test to make a diagnosis of fat embolism, and
clinical suspicion is required.
An arterial blood gas may reveal an increasing pulmonary shunt fraction,
especially if it occurs within 24 hrs after injury.
Urine fat stains are not sensitive and should not be used to make a diagnosis
of fat embolism.
Other nonspecific features include anemia, thrombocytopenia, and
hypofibrinogenemia.

Go to the next page if you knew the correct answer, or click the link image(s)
below to further research the concepts in this question (if desired).

Research Concepts:
Embolism, Fat

We update eBooks quarterly and Apps daily based on user feedback.


Please tap flag to report any questions that need improvement.
Question 579: In which of the following groups of patients is the D-dimer
test most useful for ruling out pulmonary embolism?

Choices:
1. Patients with high clinical probability
2. Elderly patients
3. Pregnant women
4. Patients with low clinical probability
Answer: 4 - Patients with low clinical probability
Explanations:
D-dimer measurements are most reliable for excluding pulmonary
embolism in patients with low pre-test probability. There are many tools
available to classify patients into categories of risk.
D-dimer testing often yields false positives in patients who are elderly or
pregnant.
The D-dimer is a very non-specific measurement, and many comorbidities
can cause an elevation.
D-dimers are not useful in patients who have cancer because the patient is
already in a hypercoagulable state.

Go to the next page if you knew the correct answer, or click the link image(s)
below to further research the concepts in this question (if desired).

Research Concepts:
Embolism, Pulmonary

We update eBooks quarterly and Apps daily based on user feedback.


Please tap flag to report any questions that need improvement.
Question 580: Which of the following is the normal drug protocol for drug-
assisted intubation in a hypovolemic 8 kg pediatric patient?

Choices:
1. Atropine 0.1-0.5 mg then etomidate 0.2 mg/kg or midazolam 0.1 mg/kg then
succinylcholine chloride 2 mg/kg
2. Atropine 0.1-0.5 mg then etomidate 0.3 mg/kg or midazolam 0.3 mg/kg then
succinylcholine chloride 2 mg/kg
3. Atropine 0.1-0.5 mg then etomidate 0.1 mg/kg or midazolam 0.1 mg/kg then
succinylcholine chloride 1 mg/kg
4. Atropine 0.1-0.5 mg then etomidate 0.3 mg/kg or midazolam 0.3 mg/kg then
succinylcholine chloride 1 mg/kg
Answer: 1 - Atropine 0.1-0.5 mg then etomidate 0.2 mg/kg or midazolam 0.1
mg/kg then succinylcholine chloride 2 mg/kg

Explanations:
For a hypovolemic child, first use atropine 0.1-0.5 mg/kg followed by
etomidate 0.2 mg/kg or midazolam 0.1 mg/kg followed by succinylcholine
chloride 2 mg/kg.
If the patient is normovolemic, use 0.3 mg/kg of both etomidate and
midazolam.
If the patient is less than 10 kg, use succinylcholine 2 mg/kg. If the patient
is greater than 10 kg, use succinylcholine 1 mg/kg.
Vecuronium or rocuronium 0.6 mg/kg is used for longer paralysis.

Go to the next page if you knew the correct answer, or click the link image(s)
below to further research the concepts in this question (if desired).

Research Concepts:
Intubation, Anesthesia

We update eBooks quarterly and Apps daily based on user feedback.


Please tap flag to report any questions that need improvement.
Question 581: A 68-year-old patient has been receiving mechanical
ventilation with a balloon cuffed tracheostomy tube for the past 6 weeks. On
three occasions within 48 hours, about 10 ml of bright red blood has been
suctioned from the tracheostomy tube. What is the most likely source of the
bleeding?

Choices:
1. Peptic ulcer disease
2. Granulation tissue at tracheostomy site
3. Barotrauma
4. Eroded innominate artery
Answer: 4 - Eroded innominate artery
Explanations:
One of the most dramatic and fatal complications of tracheostomy is
massive hemorrhage.
The cause of massive hemorrhage is usually related to erosion of the
innominate artery although erosion of the right carotid artery has also been
reported.
The incidence of tracheoinnominate artery fistula has been reported to be
less than 1%.
The mMost frequently the site of fistula is the distal end of the
tracheostomy tube. Sometimes it may be due to a high innominate artery.

Go to the next page if you knew the correct answer, or click the link image(s)
below to further research the concepts in this question (if desired).

Research Concepts:
Innominate Artery Injury

We update eBooks quarterly and Apps daily based on user feedback.


Please tap flag to report any questions that need improvement.
Question 582: A 6-year-old HIV infected child was diagnosed with
pulmonary tuberculosis. Which of the following statements is TRUE regarding
management of this child?

Choices:
1. He should be started on HAART as soon as possible
2. He should be treated for TB first
3. TB treatment should be started right away and HAART can be initiated after
making sure that he tolerates the anti-tuberculosis medication
4. He should be treated only if has symptoms
Answer: 3 - TB treatment should be started right away and HAART can be
initiated after making sure that he tolerates the anti-tuberculosis medication

Explanations:
The wisest thing to do would be to start the TB medication right away.
After a few weeks of therapy liver function can be checked and HAART
can be started.
Starting both medication together it a bad idea as severe liver toxicity
would ensue.
Waiting 6 months till the Anti-TB medication is finished is also a bad idea
because the child might succumb due to other opportunistic infections.

Go to the next page if you knew the correct answer, or click the link image(s)
below to further research the concepts in this question (if desired).

Research Concepts:
Tuberculosis

We update eBooks quarterly and Apps daily based on user feedback.


Please tap flag to report any questions that need improvement.
Question 583: Which of the following about thoracoplasty is false?
Choices:
1. Progressive scoliosis may occur
2. Frozen shoulder symptoms may arise
3. Progressive pulmonary failure can occur
4. If an intrapleural drain is present, extrapleural drain may not be necessary
Answer: 4 - If an intrapleural drain is present, extrapleural drain may not be
necessary

Explanations:
Both extrapleural and intrapleural drains are required to achieved the
maximum desired effect of thoracoplasty.
The extrapleural drain also prevents excessive serous fluid from
accumulating.
The procedure is very debilitating and rarely used today.

Go to the next page if you knew the correct answer, or click the link image(s)
below to further research the concepts in this question (if desired).

Research Concepts:
Thoracoplasty

We update eBooks quarterly and Apps daily based on user feedback.


Please tap flag to report any questions that need improvement.
Question 584: A patient with acute respiratory distress syndrome who is
mechanically ventilated (TV 7 ml/kg, CPAP 12.5 cm H2O, and FIO2 75%)
becomes hypoxic (SaO2 40%) and hypotensive. The physical exam shows
jugular venous distension and breath sounds are absent over the left lung field
with right tracheal deviation. Which of the following actions should be taken
immediately?

Choices:
1. Increase FIO2 to 100%, get a portable chest radiograph, and await results
2. Pull the endotracheal tube back 2 centimeters
3. Assess endotracheal tube position, increase FIO2 to 100%, suction the airway,
obtain an emergent chest radiograph, and consider emergent chest tube
placement
4. Increase continuous positive airway pressure to 20 and start a dopamine drip
Answer: 3 - Assess endotracheal tube position, increase FIO2 to 100%,
suction the airway, obtain an emergent chest radiograph, and consider emergent
chest tube placement

Explanations:
The most likely diagnosis is tension pneumothorax.
The practitioner should rule out intubation of the right mainstem bronchus,
mucus plug obstructing the left bronchus, and left hemothorax.
A left chest tube should be placed if oxygen saturation of arterial
oxyhemoglobin and blood pressure do not rapidly respond.
The patient will likely have rapid deterioration and cardiovascular
compromise leading to death if the tension pneumothorax is not recognized
and corrected.

Go to the next page if you knew the correct answer, or click the link image(s)
below to further research the concepts in this question (if desired).

Research Concepts:
Pneumothorax, Tension And Traumatic

We update eBooks quarterly and Apps daily based on user feedback.


Please tap flag to report any questions that need improvement.
Question 585: Rasmussen aneurysms are associated with hemoptysis in
which disease?

Choices:
1. Bronchiectasis
2. Arteriovenous malformations
3. Tuberculosis
4. Lung abscess
Answer: 3 - Tuberculosis
Explanations:
Rasmussen aneurysms are found in tuberculosis and involve branches of the
pulmonary artery.

Go to the next page if you knew the correct answer, or click the link image(s)
below to further research the concepts in this question (if desired).

Research Concepts:
Tuberculosis

We update eBooks quarterly and Apps daily based on user feedback.


Please tap flag to report any questions that need improvement.
Section 10

Question 586: A 17-year-old pregnant female presents with no prenatal


care. She is at 40-weeks by dates and is in active labor. There is normal vaginal
delivery, but the neonate has respiratory distress. The exam shows decreased
breath sounds on the left. The chest radiograph shows dense lungs and shift of
the heart to the right. NG tube that has been inserted shows the stomach below
the diaphragm. Select the most likely diagnosis.

Choices:
1. Congenital pneumonia
2. Pneumothorax
3. Congenital pulmonary airway malformation
4. Bronchogenic cysts
Answer: 3 - Congenital pulmonary airway malformation
Explanations:
Congenital pulmonary airway malformations is secondary to abnormal
embryogenesis occurring before day 35 of development.
There is a cystic mass that is often diagnosed on prenatal ultrasound.
The affected lung may be compressed resulting in pulmonary hypoplasia.
Large lesions are treated surgically before birth.

Go to the next page if you knew the correct answer, or click the link image(s)
below to further research the concepts in this question (if desired).

Research Concepts:
Congenital Pulmonary Airway Malformation

We update eBooks quarterly and Apps daily based on user feedback.


Please tap flag to report any questions that need improvement.
Question 587: A 4-year-old child is diagnosed with aspiration pneumonia
and requires intubation and mechanical ventilation. The initial chest X-ray
showed left lower lobe pneumonia but when repeated the next day, show
bilateral upper and lower infiltrates. Her FIO2 is 100 percent with a PEEP of 5,
but her oxygen saturation is only 87 percent. What is the next step in
management?

Choices:
1. Increase the respiratory rate
2. Increase the PEEP
3. High-frequency oscillatory support
4. Increase the tidal volume
Answer: 2 - Increase the PEEP
Explanations:
Variables that can improve oxygenation are increasing the FIO2 or PEEP.
The FIO2 is already at 100 percent.
The PEEP can be increased.
High-frequency oscillatory support has been shown to be useful with ARDS
and may be then next option.

Go to the next page if you knew the correct answer, or click the link image(s)
below to further research the concepts in this question (if desired).

Research Concepts:
Acute Respiratory Distress Syndrome (ARDS)

We update eBooks quarterly and Apps daily based on user feedback.


Please tap flag to report any questions that need improvement.
Question 588: Which screening test is used to check for cystic fibrosis in
newborns?

Choices:
1. Sweat testing
2. Mutation testing
3. Immunoreactive trypsinogen levels
4. Meconium viscosity
Answer: 3 - Immunoreactive trypsinogen levels
Explanations:
Immunoreactive trypsinogen (IRT) levels should be checked to screen for
cystic fibrosis in all newborns.
Higher risk patients should have mutation and/or sweat testing.
Despite IRT testing, there are a significant number of patients that are not
diagnosed by screening.

Go to the next page if you knew the correct answer, or click the link image(s)
below to further research the concepts in this question (if desired).

Research Concepts:
Cystic Fibrosis

We update eBooks quarterly and Apps daily based on user feedback.


Please tap flag to report any questions that need improvement.
Question 589: Which of the following is false about high frequency jet
ventilation?

Choices:
1. Can be used for bronchopleural fistulas
2. Oxygen transport occurs by diffusion
3. Can be a closed or open system
4. Aspiration is eliminated
Answer: 4 - Aspiration is eliminated
Explanations:
HVJV offers an alternative when conventional mechanical ventilation fails
to adequately oxygenate the patient.
Small tidal volumes are delivered at high rates. So far, it has not been very
successful in ARDS.
A constant FiO2, adequate humidification, and PEEP is available on most
instruments.
It can oxygenate and eliminate CO2 but aspiration is not avoided.

Go to the next page if you knew the correct answer, or click the link image(s)
below to further research the concepts in this question (if desired).

Research Concepts:
Ventilation, High Frequency

We update eBooks quarterly and Apps daily based on user feedback.


Please tap flag to report any questions that need improvement.
Question 590: Which of the following is a chronic form of rejection after
lung transplantation?

Choices:
1. Bronchiolitis obliterans syndrome (BOS)
2. Cystic fibrosis (CF)
3. Chronic obstructive pulmonary disease (COPD)
4. None of the above
Answer: 1 - Bronchiolitis obliterans syndrome (BOS)
Explanations:
Bronchiolitis obliterans (BO) is a lung disease resulting in the scarring of
small airways of the lung which can result from various etiologies.
Bronchiolitis obliterans syndrome (BOS) is a scarring that occurs due to a
chronic form of airway rejection post-lung transplantation.
Chronic airway rejection is more common than chronic vascular rejection
post-lung transplantation.
BO presents as dyspnea and cough. Pulmonary function tests reveal
obstructive pattern with decreased airflow. Chronic obstructive pulmonary
disease and cystic fibrosis also cause obstructive air flow pattern but are not
forms of lung transplant rejection.

Go to the next page if you knew the correct answer, or click the link image(s)
below to further research the concepts in this question (if desired).

Research Concepts:
Bronchiolitis Obliterans (Obliterative Bronchiolitis, Constrictive
Bronchiolitis)

We update eBooks quarterly and Apps daily based on user feedback.


Please tap flag to report any questions that need improvement.
Question 591: How long does it take to sterilize equipment when immersed
in glutaraldehyde solution?

Choices:
1. 20 minutes
2. 40 minutes
3. 1 hour
4. 10 hours
Answer: 4 - 10 hours
Explanations:
Glutaraldehyde solution can sterilize equipment with minimum immersion
time of 10 hours at 25 degrees C.

Go to the next page if you knew the correct answer, or click the link image(s)
below to further research the concepts in this question (if desired).

Research Concepts:
Glutaraldehyde Sterilization

We update eBooks quarterly and Apps daily based on user feedback.


Please tap flag to report any questions that need improvement.
Question 592: Which of the following can help differentiate non allergic
angioedema from anaphylaxis?

Choices:
1. Clinical exam
2. Skin testing
3. Levels of tryptase
4. Measurement of immunoglobulin E
Answer: 3 - Levels of tryptase
Explanations:
Levels of tryptase can help differentiate angioedema from anaphylaxis.
Elevated levels of tryptase indicate mast cell degranulation, but false
negatives are not uncommon.
Unlike allergies, there are no skin tests for angioedema.
One can assess pulmonary function tests to assess the severity of wheezing
or perform a challenge test with histamine to confirm airway
hypersensitivity.

Go to the next page if you knew the correct answer, or click the link image(s)
below to further research the concepts in this question (if desired).

Research Concepts:
Angioedema, Hereditary

We update eBooks quarterly and Apps daily based on user feedback.


Please tap flag to report any questions that need improvement.
Question 593: Which interleukin has been linked to development of
allergies and asthma?

Choices:
1. IL1
2. IL2
3. IL3
4. IL5
Answer: 4 - IL5
Explanations:
Interleukin 5 has been linked to severe allergic rhinitis and asthma.
The IL5 induces eosinophils to release their mediators.
Eosinophils appear to have IL5 receptors.

Go to the next page if you knew the correct answer, or click the link image(s)
below to further research the concepts in this question (if desired).

Research Concepts:
Asthma, Interleukin 5, Allergic (IL-5)

We update eBooks quarterly and Apps daily based on user feedback.


Please tap flag to report any questions that need improvement.
Question 594: Radiographic findings of mediastinal widening, patchy
opacities and large effusions in addition to CT findings of hemorrhagic
mediastinal and hilar lymph nodes, peribronchial thickening and pleural
effusions are suggestive of which of the following infections?

Choices:
1. Klebsiella pneumoniae
2. Tuberculosis
3. Inhalation anthrax
4. Lymphoma
Answer: 3 - Inhalation anthrax
Explanations:
These findings are indicative of inhalation anthrax wherein alveolar and
lymph node hemorrhage and bloody effusions are classic diagnostic clues.
CT findings of TB typically include low attenuation lymph nodes initially
with calcifications in lung, hila or mediastinum in healed state. Radiograph
typically demonstrates airspace opacities in one lobe (primary stage of
disease), diffuse nodules (miliary stage of disease), or upper lobe fibrosis
(chronic stage of disease).
CT findings of Klebsiella most often shows consolidation of the right
middle or lower lobe.
CT findings of lymphoma include enlarged discrete homogenous soft tissue
attenuation bulky lymphadenopathy. Chest radiograph findings include
anterior and/or middle mediastinal stripes and lines widening and opacities.

Go to the next page if you knew the correct answer, or click the link image(s)
below to further research the concepts in this question (if desired).

Research Concepts:
Anthrax

We update eBooks quarterly and Apps daily based on user feedback.


Please tap flag to report any questions that need improvement.
Question 595: What is the most common complication after repair of a
newborn tracheoesophageal fistula?

Choices:
1. Reflux
2. Peptic ulcer disease
3. Stricture
4. Hiatal hernia
Answer: 1 - Reflux
Explanations:
Long-term studies have shown motility problems in babies who underwent
surgery for tracheoesophageal fistula.
There has been disordered peristalsis with reflux noted.
These babies need a barium swallow study or pH monitoring to evaluate
reflux.
Reflux generally presents as recurrent pneumonias and/or strictures.

Go to the next page if you knew the correct answer, or click the link image(s)
below to further research the concepts in this question (if desired).

Research Concepts:
Fistula, Tracheoesophageal

We update eBooks quarterly and Apps daily based on user feedback.


Please tap flag to report any questions that need improvement.
Question 596: Prone position for ventilation is used in patients with which
of the following disorders?

Choices:
1. Pneumothorax
2. Acute respiratory distress syndrome
3. Sepsis
4. Diaphragmatic hernia
Answer: 2 - Acute respiratory distress syndrome
Explanations:
Prone position to ventilate patients with acute respiratory distress syndrome
(ARDS) has been tried. The position does improve functional residual
capacity and postural drainage of secretions but makes no difference in
overall mortality.
Moving and caring for patients in prone position also is very difficult and
may cause dislodgment of tubes and lines.
ARDS is treated with mechanical ventilation.
The possibilities of non-invasive ventilation are limited to the early period
of the disease or to prevention in individuals with atypical cases of
pneumonia, lung contusion, or after major surgery, who are at risk of
developing ARDS.

Go to the next page if you knew the correct answer, or click the link image(s)
below to further research the concepts in this question (if desired).

Research Concepts:
Acute Respiratory Distress Syndrome (ARDS)

We update eBooks quarterly and Apps daily based on user feedback.


Please tap flag to report any questions that need improvement.
Question 597: At what point is central sleep apnea significant in preschool
children?

Choices:
1. Greater than 15 seconds
2. Greater than 20 seconds
3. Greater than 25 seconds
4. Greater than 30 seconds
Answer: 2 - Greater than 20 seconds
Explanations:
Central apnea in preschool children is considered significant if it is longer
than 20 seconds or if there is bradycardia.
Obstructive and central apnea can coexist in the same patient.
In children, the most common cause of obstructive sleep apnea (OSA) is
enlarged tonsils and adenoids.
The primary treatment for OSA in a child is tonsillectomy and
adenoidectomy.

Go to the next page if you knew the correct answer, or click the link image(s)
below to further research the concepts in this question (if desired).

Research Concepts:
Apnea, Obstructive Sleep Apnea

We update eBooks quarterly and Apps daily based on user feedback.


Please tap flag to report any questions that need improvement.
Question 598: Following a left pneumonectomy, a patient suddenly
becomes dyspneic, cyanotic, and tachycardic in the middle of the night. The
anterior-posterior chest x-ray is unremarkable. What is the most likely
diagnosis?

Choices:
1. Pulmonary embolus
2. Myocardial infarction
3. Pneumonia
4. Cardiac herniation
Answer: 4 - Cardiac herniation
Explanations:
Cardiac herniation must be suspected. The anterior-posterior chest x-ray
may not show a herniation.
Onset typically occurs suddenly within 24 hours after surgery and may
occur after a brief cough or change in position.
When acute symptoms occur within a few hours after surgery, herniation
must be suspected. It is a surgical emergency and can result in death.
An echocardiogram or CT scan can be used to make the diagnosis.

Go to the next page if you knew the correct answer, or click the link image(s)
below to further research the concepts in this question (if desired).

Research Concepts:
Pneumonectomy

We update eBooks quarterly and Apps daily based on user feedback.


Please tap flag to report any questions that need improvement.
Question 599: An 18-month previously healthy male is diagnosed with
pneumonia with a parapneumonic effusion. The patient is hypoxic and appears
toxic. Thoracentesis is done and blood cultures are drawn. Which of the
following antibiotics should be started?

Choices:
1. Ampicillin and clarithromycin
2. Ceftriaxone
3. Ceftriaxone and clindamycin
4. Ceftriaxone and vancomycin
Answer: 4 - Ceftriaxone and vancomycin
Explanations:
Community-acquired pneumonia in a toddler that is toxic should include an
advanced generation cephalosporin and vancomycin.
Coverage should include S. pneumoniae and S. aureus.
Local rates of MRSA should be taken into account.
Gram stain of the pleural fluid may be helpful.

Go to the next page if you knew the correct answer, or click the link image(s)
below to further research the concepts in this question (if desired).

Research Concepts:
Pneumonia, Pediatric

We update eBooks quarterly and Apps daily based on user feedback.


Please tap flag to report any questions that need improvement.
Question 600: What is the next step in management for a near drowning
patient who appears normal on exam?

Choices:
1. Reassurance
2. Observation for at least 8 hours
3. Administration of charcoal
4. None of the above
Answer: 2 - Observation for at least 8 hours
Explanations:
Studies have shown that most, if not all patients who develop symptoms
from submersion injuries do so within 7 hours of the near-drowning event.
Admission and observation for signs of pulmonary injury or other end-
organ would be prudent and the next best step.
Reassurance would not be warranted in this situation.

Go to the next page if you knew the correct answer, or click the link image(s)
below to further research the concepts in this question (if desired).

Research Concepts:
Drowning

We update eBooks quarterly and Apps daily based on user feedback.


Please tap flag to report any questions that need improvement.
Question 601: How often must a PtcO2 electrode on a neonate be rotated?
Choices:
1. 2 hours
2. 5 hours
3. 12 hours
4. 24 hours
Answer: 1 - 2 hours
Explanations:
A PtcO2 electrode on a neonate must be rotated every two hours.
It is rotated every four hours on an adult.

Go to the next page if you knew the correct answer, or click the link image(s)
below to further research the concepts in this question (if desired).

Research Concepts:
PCO2 Electrodes

We update eBooks quarterly and Apps daily based on user feedback.


Please tap flag to report any questions that need improvement.
Question 602: Which of the following respiratory parameters can be used
to predict successful extubation?

Choices:
1. Minute ventilation
2. Tidal volume
3. Vital capacity
4. All of the above
Answer: 4 - All of the above
Explanations:
Minute ventilation <10 L/min is predictive of mechanical ventilation
weaning success.
Tidal volume >5 ml/kg is also predictive of success.
Vital capacity >15ml/kg is another parameter predictive of success.
Lastly, respiratory rate of <30 breaths/min can predict success.

Go to the next page if you knew the correct answer, or click the link image(s)
below to further research the concepts in this question (if desired).

Research Concepts:
Extubation

We update eBooks quarterly and Apps daily based on user feedback.


Please tap flag to report any questions that need improvement.
Question 603: What is the most common complication after repair of a
newborn tracheoesophageal fistula?

Choices:
1. Reflux
2. Peptic ulcer disease
3. Stricture
4. Hiatal hernia
Answer: 1 - Reflux
Explanations:
Long-term studies have shown motility problems in babies who underwent
surgery for TEF.
There has been disordered peristalsis with reflux noted.
These babies need a barium swallow study or pH monitoring to evaluate
reflux, which generally presents as recurrent pneumonias and/or strictures.

Go to the next page if you knew the correct answer, or click the link image(s)
below to further research the concepts in this question (if desired).

Research Concepts:
Fistula, Tracheoesophageal

We update eBooks quarterly and Apps daily based on user feedback.


Please tap flag to report any questions that need improvement.
Question 604: Simple pulmonary eosinophilia demonstrates which of
following findings on radiograph?

Choices:
1. Bilateral, nonsegmental, symmetric consolidation with peripheral and upper
lobe distribution
2. Patchy transient opacities consisting of interstitial and alveolar edema
3. Findings are similar to cardiogenic pulmonary edema in appearance, ground
glass opacities, septal thickening, and bilateral effusions
4. Bilateral alveolar consolidation, central 'bat-wing' pattern, and absent pleural
effusions
Answer: 2 - Patchy transient opacities consisting of interstitial and alveolar
edema

Explanations:
Bilateral, nonsegmental, symmetric consolidation is seen with chronic
eosinophilic pneumonia, an idiopathic condition marked by alveolar filling
with mixed inflammatory infiltrate that consists largely of eosinophils.
Patchy transient opacities are seen with simple pulmonary eosinophilia,
pneumonia due to eosinophilic infiltration. Infection is migratory and
rapidly evolving over days, and spontaneously clears within 1 month.
Ground glass opacities, septal thickening, and bilateral effusions are seen
with acute eosinophilic pneumonia, an idiopathic acute febrile illness with
rapid onset respiratory. failure due to pulmonary eosinophilia. Peripheral
eosinophilia is usually absent.
Bilateral alveolar consolidation is seen with pulmonary alveolar proteinosis,
a rare diffuse lung disease characterized by the accumulation of abundant
protein-rich and lipid-rich surfactant material in alveoli.

Go to the next page if you knew the correct answer, or click the link image(s)
below to further research the concepts in this question (if desired).

Research Concepts:
Pulmonary Eosinophilia

We update eBooks quarterly and Apps daily based on user feedback.


Please tap flag to report any questions that need improvement.
Question 605: An 18-month-old child is brought in with a 3-day history of
cough and fever. He has been taking fluids but is not hungry. The only
medication his parents have given him is acetaminophen. He has no significant
past medical or travel history. Exam shows temperature of 38.5 degrees Celsius,
clear mucosa, dullness to percussion over the left lung with rhonchi, but no
rashes or cardiac findings. Chest X-ray shows a left lower lobe infiltrate and
opacification of the left lower half of the lung. What should be the next step in
management?

Choices:
1. Intravenous antibiotics
2. Left lateral decubitus chest X-ray
3. Ultrasound of the left hemithorax
4. Nasopharyngeal swab for viral DFA
Answer: 3 - Ultrasound of the left hemithorax
Explanations:
The patient most likely has pneumonia with parapneumonic effusion.
A decubitus chest X-ray would show if the effusion is free flowing.
An ultrasound would also confirm this and assist in thoracentesis for
culture.
Blood cultures should also be obtained and antibiotics should be held until
cultures are obtained unless the clinical situation is severe.

Go to the next page if you knew the correct answer, or click the link image(s)
below to further research the concepts in this question (if desired).

Research Concepts:
Parapneumonic, Pleural Effusions And Empyema Thoracis

We update eBooks quarterly and Apps daily based on user feedback.


Please tap flag to report any questions that need improvement.
Question 606: Which of the following is true regarding the role of sputum
Gram stain and culture in the diagnosis of community-acquired pneumonia?

Choices:
1. It is recommended only for selected subgroups of hospitalized patients
2. It is recommended for all hospitalized patients
3. It should only be performed if the sputum is obtained by bronchoscopy
4. It is only useful in cases of atypical pathogens
Answer: 1 - It is recommended only for selected subgroups of hospitalized
patients

Explanations:
The use of Gram stain and culture of lower respiratory tract specimen is
recommended to guide the selection of appropriate antimicrobial therapy in
selected hospitalized patients: those who are intubated and on mechanical
ventilation, and those who have risk factors for organisms that would not be
covered with standard empiric therapy.

Go to the next page if you knew the correct answer, or click the link image(s)
below to further research the concepts in this question (if desired).

Research Concepts:
Pneumonia, Community-Acquired

We update eBooks quarterly and Apps daily based on user feedback.


Please tap flag to report any questions that need improvement.
Question 607: In a young child, what is the horizontal groove at the level
of the diaphragm that extends from the sternum to the midaxillary line?

Choices:
1. Sternal groove
2. Costal groove
3. Manubrial groove
4. Harrison groove
Answer: 4 - Harrison groove
Explanations:
In children, the Harrison groove is the horizontal groove in the rib cage at
the level of the diaphragm, extending from the sternum to the midaxillary
line.
It corresponds to the costal insertion of the diaphragm.
The groove is caused by chronic asthma or obstructive respiratory disease.
The groove also appears in rickets because of defective mineralization of
the bones by the calcium necessary to harden them. The diaphragm, which
is always in tension, pulls the soft bone inward.

Go to the next page if you knew the correct answer, or click the link image(s)
below to further research the concepts in this question (if desired).

Research Concepts:
Anatomy, Harrison Groove

We update eBooks quarterly and Apps daily based on user feedback.


Please tap flag to report any questions that need improvement.
Question 608: What is the advantage of using pressure-controlled
ventilation over assist-control ventilation?

Choices:
1. Decreased lung over-inflation
2. Assists spontaneous breathing
3. More effective in patients with decreased lung compliance
4. None of the above
Answer: 1 - Decreased lung over-inflation
Explanations:
Over-inflation and concomitant lung injury are prevented with pressure-
controlled ventilation (PCV) by delivering pressure cycled mechanical
breaths.
PCV is not suitable in cases of intrinsic pulmonary dysfunction because it
delivers variable inflation volumes depending on lung compliance.
Assist-control ventilation (ACV) also is known as continuous mandatory
ventilation. Each breath is either an assist or controlled breath, but they are
all of equal volume. The greater the volume, the more expiratory time
required. If the inspiratory to expiratory ratio is less than 1:2, progressive
hyperinflation may result. ACV is not desirable for patients who breathe
rapidly as it may induce hyperinflation and respiratory alkalosis.
PCV has less barotrauma compared to ACV and Synchronized intermittent
mechanical ventilation. PCV traditionally has been preferred for patients
with a neuromuscular disease but otherwise normal lungs.

Go to the next page if you knew the correct answer, or click the link image(s)
below to further research the concepts in this question (if desired).

Research Concepts:
Ventilation, Mechanical

We update eBooks quarterly and Apps daily based on user feedback.


Please tap flag to report any questions that need improvement.
Question 609: What will a biopsy show in a patient with herpes
pneumonitis?

Choices:
1. Dense lymphocytic infiltration
2. Formation of multinucleate giant cells
3. Caseating necrosis
4. Liquefactive necrosis
Answer: 2 - Formation of multinucleate giant cells
Explanations:
Herpes simplex causes cells to swell and degenerate.
A biopsy of infected organs will show the formation of multinucleated giant
cells.
The infected cells often will show eosinophil intranuclear inclusions and
marginated nuclear chromatin.
Both herpes simplex and herpes zoster viruses are capable of causing
abnormal cell division in epidermal cells. The result is the formation of
multinucleated giant cells.

Go to the next page if you knew the correct answer, or click the link image(s)
below to further research the concepts in this question (if desired).

Research Concepts:
Herpes, Pneumonitis

We update eBooks quarterly and Apps daily based on user feedback.


Please tap flag to report any questions that need improvement.
Question 610: Which of the following is a major disadvantage of volume
ventilation after patient intubation?

Choices:
1. Delivery of a constant tidal volume
2. Barotrauma
3. Lack of adequate alarms
4. None of the above
Answer: 2 - Barotrauma
Explanations:
2 types of ventilators are available-volume or pressure.
A major disadvantage of volume ventilation is barotraumas.
A major advantage of volume ventilation delivery of a constant tidal
volume.
All volume ventilators have adequate alarm systems.

Go to the next page if you knew the correct answer, or click the link image(s)
below to further research the concepts in this question (if desired).

Research Concepts:
Ventilation, Barotrauma And Mechanical

We update eBooks quarterly and Apps daily based on user feedback.


Please tap flag to report any questions that need improvement.
Question 611: Which of the following is characteristic of spontaneous
pneumothorax?

Choices:
1. Clinical exam reveals dullness to percussion
2. Ipsilateral shift of trachea
3. The patient typically presents with sudden onset of dyspnea and pleuritic chest
pain
4. Common cause is penetrating trauma to the lungs
Answer: 3 - The patient typically presents with sudden onset of dyspnea and
pleuritic chest pain

Explanations:
Spontaneous pneumothorax refers to the accumulation of air in the pleural
space that is not due to an obvious source, such as trauma or iatrogenic
causes, and often presents with dyspnea and pleuritic chest pain.
Idiopathic causes may be associated with rupturing of apical subpleural
blebs, which results in the loss of negative intrathoracic pressure and lung
collapse.
Additional causes include Marfan syndrome, scuba diving, and chronic
obstructive lung disease.
Due to the collection of air between the lung and chest wall, the physical
exam may reveal absent breath sounds, tympanic percussion, and tracheal
deviation to the contralateral side of total lung collapse.

Go to the next page if you knew the correct answer, or click the link image(s)
below to further research the concepts in this question (if desired).

Research Concepts:
Pneumothorax, Spontaneous

We update eBooks quarterly and Apps daily based on user feedback.


Please tap flag to report any questions that need improvement.
Question 612: Which of the following is false about T-tube stents in
tracheal pathology?

Choices:
1. T-tubes can get plugged and cause obstruction
2. T-tube placement in children is more difficult than in adults
3. T-tubes can palliate patients with failed tracheal anastomotic leaks
4. Requires a surgical procedure to place a T-tube
Answer: 4 - Requires a surgical procedure to place a T-tube
Explanations:
T-tubes do not require surgery for placement.
T-tubes can maintain airway, preserve voice and can be used to treat
strictures.
They may be placed through the tracheal stoma or via a rigid bronchoscope
through the mouth.
They can become obstructed and aspiration can occur. So constant vigilance
is required.

Go to the next page if you knew the correct answer, or click the link image(s)
below to further research the concepts in this question (if desired).

Research Concepts:
Trachea, Laryngeal And Tracheal Stents

We update eBooks quarterly and Apps daily based on user feedback.


Please tap flag to report any questions that need improvement.
Question 613: A newborn infant with a persistent cough is examined and
found to have a continuous murmur radiating to his back. A congenital heart
disorder has been ruled out. What is the most likely diagnosis?

Choices:
1. Congenital lobar emphysema
2. Congenital diaphragmatic hernia
3. Sequestration
4. Tracheoesophageal fistula
Answer: 3 - Sequestration
Explanations:
Pulmonary sequestration is a solid or cystic mass that consists of
nonfunctioning primitive tissue. There is no communication with the
airways and the structure has an anomalous systemic blood supply.
The sequestration may present with signs of a lung infection on
examination. The blood supply is usually from the systemic rather than the
pulmonary circulation. In some infants, there may be several feeding
vessels.
The intralobar sequestration is covered by normal lung tissue, whereas the
extralobar variety may have its own pleural covering.
Sequestrations may be associated with other congenital problems.

Go to the next page if you knew the correct answer, or click the link image(s)
below to further research the concepts in this question (if desired).

Research Concepts:
Pulmonary Sequestration

We update eBooks quarterly and Apps daily based on user feedback.


Please tap flag to report any questions that need improvement.
Question 614: Which of the following statement is TRUE regarding the
Aspergillus species?

Choices:
1. Patients with allergic bronchopulmonary aspergillosis (ABPA) have a high
IgE titer against Aspergillus antigens and are frequently symptomatic
2. Aspergillus species exist only as molds, which are dimorphic non-septate
hyphae
3. Transmission is by airborne conidia
4. Aspergillomas in the lungs present as radiolucent structures on chest
radiograph with no evident change when patient is moved from erect to supine
position
Answer: 2 - Aspergillus species exist only as molds, which are dimorphic
non-septate hyphae

Explanations:
Patients with APBA are frequently asymptomatic.
Aspergillus species exist only as molds; they are not dimorphic. They have
septate hyphae that form dichotomous branches.
These molds are widely distributed in nature; grow on decaying vegetation,
producing chains of conidia.
Aspergillomas in the lungs present as radiopaque structures on CXR which
change position when patient is moved from erect to supine position.

Go to the next page if you knew the correct answer, or click the link image(s)
below to further research the concepts in this question (if desired).

Research Concepts:
Aspergillosis

We update eBooks quarterly and Apps daily based on user feedback.


Please tap flag to report any questions that need improvement.
Question 615: Insertion of a nasopharyngeal airway is contraindicated in a
patient with which condition?

Choices:
1. Pneumothorax
2. Pleural effusion
3. Ecchymosis behind the ear
4. Perforated stomach
Answer: 3 - Ecchymosis behind the ear
Explanations:
A nasopharyngeal airway (NPA) does not extend into the lower airway and
may be used in certain situations.
Insertion of an NPA is contraindicated in patients with severe head trauma,
facial injuries, or evidence of a basilar skull fracture.
An NPA, also known as a nasal trumpet, is an airway adjunct designed to be
inserted into the nasal passageway to maintain an open airway. When a
patient is unconscious, the muscles in the jaw relax, and the tongue slides
back and obstructs the airway. This makes airway management necessary.
The flared end is designed to prevent the device from completely entering
the nose.
An NPA is used when an artificial form of airway maintenance is necessary,
but tracheal intubation is impossible or inadvisable. A nasopharyngeal
airway is often used in conscious patients where an oropharyngeal airway
would trigger the gag reflex.

Go to the next page if you knew the correct answer, or click the link image(s)
below to further research the concepts in this question (if desired).

Research Concepts:
Airway, Nasopharyngeal

We update eBooks quarterly and Apps daily based on user feedback.


Please tap flag to report any questions that need improvement.
Question 616: When asthma is precipitated by aspirin, how does it
typically present?

Choices:
1. Nose bleed
2. Nasal congestion
3. Coughing
4. Bronchial dilation
Answer: 2 - Nasal congestion
Explanations:
Aspirin-induced asthma may present with nasal congestion, rhinorrhea, and
acute asthmatic attack.

Go to the next page if you knew the correct answer, or click the link image(s)
below to further research the concepts in this question (if desired).

Research Concepts:
Salicylic Acid (Aspirin), Allergy

We update eBooks quarterly and Apps daily based on user feedback.


Please tap flag to report any questions that need improvement.
Question 617: What is the most common cause of massive hemoptysis
worldwide?

Choices:
1. Arteriovenous malformation
2. Bronchiectasis
3. Lung abscess
4. Tuberculosis
Answer: 4 - Tuberculosis
Explanations:
Tuberculosis is still the leading cause of hemoptysis worldwide. Rigid
bronchoscopy is invaluable in localizing the site of hemorrhage. If the site
is not visualized, the bronchoscope should not be removed and can be used
to clear blood. The patient should be placed with the site of hemorrhage
down to avoid spillage of blood to the normal lung.
The initial management should be to place the patient with the site of
bleeding down, secure the airway, maintain hemodynamic balance, perform
bronchoscopic maneuvers that can control the bleeding like ice-cold lavage
via the scope, balloon tamponade, epinephrine, and argon plasma
coagulation to coagulate bleeding tissue.
Bronchial artery embolization can be attempted and is usually successful. In
certain situations, such as aspergillomas, recurrent bleeding is common.
Once the site is localized, and if the patient has adequate pulmonary
reserve, the lesion can be resected.
Note that massive hemoptysis definition varies in the literature, classically
it is defined as 600 ml/24 hours, but in a patient with diseased lungs, a little
bleeding can cause major problems.

Go to the next page if you knew the correct answer, or click the link image(s)
below to further research the concepts in this question (if desired).

Research Concepts:
Hemoptysis

We update eBooks quarterly and Apps daily based on user feedback.


Please tap flag to report any questions that need improvement.
Question 618: Which is a very common cause of asthma and
hospitalizations in urban American children?

Choices:
1. Kwashiorkor
2. Marasmus
3. Cockroaches
4. Mosquitoes
Answer: 3 - Cockroaches
Explanations:
Kwashiorkor is a form of severe protein-energy malnutrition characterized
by edema, irritability, anorexia, ulcerating dermatoses, and an enlarged liver
with fatty infiltrates.
Marasmus is a condition primarily caused by a deficiency in calories and
energy, whereas kwashiorkor indicates an associated protein deficiency,
resulting in an edematous appearance.
Cockroach allergy and exposure to the insect is an important cause of
asthma and hospitalizations among inner city children.
Mosquitoes do not cause asthma-like illnesses.

Go to the next page if you knew the correct answer, or click the link image(s)
below to further research the concepts in this question (if desired).

Research Concepts:
Allergy

We update eBooks quarterly and Apps daily based on user feedback.


Please tap flag to report any questions that need improvement.
Question 619: Which of the following is not an indication for blood
transfusion?

Choices:
1. Stroke
2. Acute chest syndrome
3. Acute pain crisis
4. Splenic sequestration
Answer: 3 - Acute pain crisis
Explanations:
Stroke (CVAs) are treated with blood transfusions either simple or
exchange transfusions with the goal to reduce the proportion of HbS in the
circulating blood
Similarly, acute chest syndrome and aplastic crisis need to be treated with
blood transfusions in addition to meticulous supportive therapy.
Vasooclusive crisis usually does not require transfusions, rather judicious
and adequate analgesic medications along with other non-pharmacological
pain management modalities.
Splenic sequestration crisis can rapidly lead to shock and death if not
treated promptly with transfusion.

Go to the next page if you knew the correct answer, or click the link image(s)
below to further research the concepts in this question (if desired).

Research Concepts:
Sickle Cell Disease

We update eBooks quarterly and Apps daily based on user feedback.


Please tap flag to report any questions that need improvement.
Question 620: Which of the following about bronchiolitis obliterans is
NOT correct?

Choices:
1. Produces wheezing
2. Complication of lung transplantation
3. Respond to steroids
4. Restrictive effect on spirometry
Answer: 3 - Respond to steroids
Explanations:
Clinically speaking, bronchiolitis obliterans refers to a syndrome of chronic
airflow obstruction with radiographic over inflation caused by pathological
changes in the small airways.
It can be a complication after a lung transplant.
It can have symptoms of cough, shortness of breath, and wheezing.
Steroids are not effective treatment.

Go to the next page if you knew the correct answer, or click the link image(s)
below to further research the concepts in this question (if desired).

Research Concepts:
Bronchiolitis Obliterans (Obliterative Bronchiolitis, Constrictive
Bronchiolitis)

We update eBooks quarterly and Apps daily based on user feedback.


Please tap flag to report any questions that need improvement.
Question 621: Which neuromuscular blocking agent breaks down
spontaneously?

Choices:
1. Atracurium
2. Curare
3. Lidocaine
4. Hexamethonium
Answer: 1 - Atracurium
Explanations:
Atracurium breaks downs spontaneously in the plasma to form a metabolite
with a long half-life; the metabolite can enter the CNS and can cause
seizures.
Prolonged infusion of atracurium is generally avoided.
Atracurium is metabolized by ester hydrolysis and Hofmann elimination
which occurs independently of renal or hepatic function.

Go to the next page if you knew the correct answer, or click the link image(s)
below to further research the concepts in this question (if desired).

Research Concepts:
Neuromuscular Blocking Agents

We update eBooks quarterly and Apps daily based on user feedback.


Please tap flag to report any questions that need improvement.
Question 622: Which of the following is a true about latex allergies?
Choices:
1. At least 33% of health-care workers experience some form of allergic reaction
to latex
2. Latex is rarely found in catheters and other medical supplies
3. Persons allergic to latex usually are sensitive to fruits such as bananas, kiwis,
pears, pineapples, grapes, and papayas
4. Latex allergies affect those with spina bifida less commonly than others
Answer: 3 - Persons allergic to latex usually are sensitive to fruits such as
bananas, kiwis, pears, pineapples, grapes, and papayas

Explanations:
Latex allergy is a significant problem, as up to 17% of health-care workers
have allergic reactions to latex.
Reactions can vary from contact dermatitis to anaphylaxis.
Latex can be found in catheters, gloves, and many other medical supplies.
Those allergic to latex may be sensitive to kiwis, bananas, grapes,
pineapples, and papayas.

Go to the next page if you knew the correct answer, or click the link image(s)
below to further research the concepts in this question (if desired).

Research Concepts:
Latex Allergy

We update eBooks quarterly and Apps daily based on user feedback.


Please tap flag to report any questions that need improvement.
Question 623: What is the last sign of respiratory insufficiency?
Choices:
1. Increased bicarbonate
2. Tachypnea
3. Hypoxia
4. Hypoxemia
Answer: 1 - Increased bicarbonate
Explanations:
The increased of bicarbonate level is the compensatory response by the
kidneys.

Go to the next page if you knew the correct answer, or click the link image(s)
below to further research the concepts in this question (if desired).

Research Concepts:
Respiratory Insufficiency

We update eBooks quarterly and Apps daily based on user feedback.


Please tap flag to report any questions that need improvement.
Question 624: A patient with a chest tube has subcutaneous emphysema.
What is the next step?

Choices:
1. Leave chest tube to water seal
2. Apply suction to chest tube
3. Strip the chest tube
4. Reposition the chest tube
Answer: 4 - Reposition the chest tube
Explanations:
Leaving the chest tube to water seal does not resolve the issue. Worsening
subcutaneous emphysema means that chest tube is not managing the air
leakage from the lung appropriately and that it is accumulating in the soft
tissue.
Applying suction to the chest tube only works if the tube is in the correct
position, not clogged or obstructed, and the intrathoracic pressure is not
exceeding the chest tube system's capabilities.
Most of the time, if subcutaneous emphysema is accumulating it, is
secondary to chest tube malposition of clogging or obstruction of the chest
tube.
Subcutaneous emphysema in the patient with a chest tube usually means
that the chest tube needs repositioning.

Go to the next page if you knew the correct answer, or click the link image(s)
below to further research the concepts in this question (if desired).

Research Concepts:
Chest Tube

We update eBooks quarterly and Apps daily based on user feedback.


Please tap flag to report any questions that need improvement.
Question 625: What is the most common skin finding with tuberculosis?
Choices:
1. Erythema nodosum
2. Erythema annulare centrifugum
3. Erythema migrans
4. Erythema multiforme

Photo:Contributed by Wikimedia Commons, Norman Purvis Walker (Public Domain)


Answer: 1 - Erythema nodosum
Explanations:
The most common skin finding with tuberculosis (TB) is erythema
nodosum.
It is not commonly seen with TB and is more commonly seen with other
conditions.
Other conditions that can cause erythema nodosum include acute
Streptococcal infection, inflammatory bowel disease, fungal infections,
medication, enteric pathogens, and sarcoidosis.

Go to the next page if you knew the correct answer, or click the link image(s)
below to further research the concepts in this question (if desired).

Research Concepts:
Erythema, Nodosum

We update eBooks quarterly and Apps daily based on user feedback.


Please tap flag to report any questions that need improvement.
Question 626: Which of the following is true of peak expiratory flow
monitoring in asthma?

Choices:
1. Daily monitoring is not recommended
2. Individuals with intermittent asthma should monitor each morning
3. Goal is to recognize early signs of deterioration
4. None of the above
Answer: 3 - Goal is to recognize early signs of deterioration
Explanations:
The goal of daily peak expiratory flow (PEF) monitoring is to recognize
early signs of deterioration to initiate corrective treatment.
Daily monitoring has long been recommended for asthma. Peak flows are
effort dependent.
Current guidelines recommend individuals with persistent asthma measure
PEF daily.
Additional assessments should be carried out if less than 89% of personal
best PEF.

Go to the next page if you knew the correct answer, or click the link image(s)
below to further research the concepts in this question (if desired).

Research Concepts:
Peak Flow Rate Measurement

We update eBooks quarterly and Apps daily based on user feedback.


Please tap flag to report any questions that need improvement.
Question 627: A newborn is found to have a congenital diaphragmatic
hernia. What is the chief reason for the high morbidity in these patients?

Choices:
1. Abdominal contents pushing on the lung
2. Elevated pulmonary vascular resistance
3. Failure of the lung to expand
4. Mediastinal shift
Answer: 2 - Elevated pulmonary vascular resistance
Explanations:
Children born with a diaphragmatic hernia have a very poor outcome if the
pulmonary vascular resistance is high.
Infants with low pulmonary vascular resistance do not have the same high
morbidity as those with elevated pressure.
Other factors that worsen prognosis include persistent fetal circulation,
abdominal pressure, hypoplastic lung, and failure of the lung to expand.
Extracorporeal membrane oxygenation has been used to treat infants with
high pulmonary pressures, but the results are not always consistent.

Go to the next page if you knew the correct answer, or click the link image(s)
below to further research the concepts in this question (if desired).

Research Concepts:
Hernia, Diaphragmatic, Congenital

We update eBooks quarterly and Apps daily based on user feedback.


Please tap flag to report any questions that need improvement.
Question 628: What is the initial step in the management of a patient with
newly diagnosed primary pulmonary hypertension?

Choices:
1. Prescribe inhaled corticosteroids
2. Evaluate vasoreactivity
3. Order an echocardiogram
4. Prescribe long-acting calcium channel blockers
Answer: 2 - Evaluate vasoreactivity
Explanations:
Primary pulmonary hypertension (PPH) is defined as elevated pulmonary
artery pressure with no discernible cause and is most prevalent in women of
childbearing age.
Vasoreactivity is evaluated by assessing a patient's response to an IV or
inhaled vasodilator challenge. A positive test is a decrease in the mean
pulmonary artery pressure by at least 10 mmHg to reach an absolute value
of 40 mmHg or less without a decrease in cardiac output. This is important
as the patients who respond to these drugs can be treated with calcium
channel blockers and have a more favorable prognosis.
If the patient has a positive test, then an oral vasodilator challenge is
performed, with a catheter in the pulmonary artery to determine the dose
that causes a 20% reduction in pulmonary artery pressure.
Patients with an acute vasodilator response have a better prognosis on
calcium channel blockers only. They constitute about 20% of those with
PPH. Phosphodiesterase inhibitors are also used to treat this condition.

Go to the next page if you knew the correct answer, or click the link image(s)
below to further research the concepts in this question (if desired).

Research Concepts:
Pulmonary Hypertension, Primary

We update eBooks quarterly and Apps daily based on user feedback.


Please tap flag to report any questions that need improvement.
Question 629: What pressure support level is often used to decrease
resistance by overcoming ventilator accessory dead space?

Choices:
1. Less than 15 cm H2O
2. Less than 12 cm H20
3. Less than 5 cm H20
4. Less than 3 cm H20
Answer: 3 - Less than 5 cm H20
Explanations:
The end result of pressure support ventilation is determined by the pressure
setting.
Pressure support levels resulting in tidal volumes of 10 to 12 ml/kg result in
the ventilator assuming all of the work of breathing.
Lower pressures up to 5 cm H2O can alleviate resistance caused by
mechanical dead space.
It is important to note that not enough pressure support can result in an
increased work of breathing as the patient attempts to overcome mechanical
dead space.

Go to the next page if you knew the correct answer, or click the link image(s)
below to further research the concepts in this question (if desired).

Research Concepts:
Pressure Support

We update eBooks quarterly and Apps daily based on user feedback.


Please tap flag to report any questions that need improvement.
Question 630: Which of the following underlying medical conditions is
least likely associated with the development of secondary spontaneous
pneumothorax?

Choices:
1. Marfan Syndrome
2. Chronic obstructive pulmonary disease
3. Coccidioidomycosis
4. Asthma
Answer: 3 - Coccidioidomycosis
Explanations:
Coccidioidomycosis is the least likely cause of secondary spontaneous
pneumothorax and is considered rare.
Tuberculosis, chronic obstructive pulmonary disease (COPD), asthma, lung
abscess, cystic fibrosis, and acute respiratory distress syndrome are more
likely causes with COPD being the most common.
Other causes can include Marfan syndrome, catamenial, sarcoidosis,
neoplasm, and eosinophilic granuloma.
The most common pathology of secondary spontaneous pneumothorax is
the rupture of an apical subpleural bleb or bulla.

Go to the next page if you knew the correct answer, or click the link image(s)
below to further research the concepts in this question (if desired).

Research Concepts:
Pneumothorax, Spontaneous

We update eBooks quarterly and Apps daily based on user feedback.


Please tap flag to report any questions that need improvement.
Question 631: Most patients with congenital tracheal malformations
exhibit which of the following symptoms?

Choices:
1. Biphasic stridor with prolonged expiration
2. Biphasic stridor with shortened expiration
3. Inspiratory stridor with prolonged inspiration
4. Expiratory stridor with shortened inspiration
Answer: 1 - Biphasic stridor with prolonged expiration
Explanations:
The stridor in patients with congenital tracheal malformations is both
inspiratory and expiratory, or biphasic.
Because of the fixed obstruction, the expiratory phase is prolonged.
One also may have other airway-related symptoms such as wheezing,
cough, rales, and crackles.
In most children, one will get a history of repeated neck hyperextension, as
this allows the child to maximally open the upper airway.

Go to the next page if you knew the correct answer, or click the link image(s)
below to further research the concepts in this question (if desired).

Research Concepts:
Trachea, Congenital Tracheal Malformation

We update eBooks quarterly and Apps daily based on user feedback.


Please tap flag to report any questions that need improvement.
Question 632: Which of the following is true regarding idiopathic
pulmonary arterial hypertension?

Choices:
1. The most common presenting symptom is cough
2. It is associated with copious sputum production
3. It has been associated with antibiotics
4. It occurs most commonly in females of childbearing age
Answer: 4 - It occurs most commonly in females of childbearing age
Explanations:
Idiopathic pulmonary arterial hypertension (IPAH) is a rare disease of
increased pulmonary arterial pressure with no identifiable cause. It typically
is diagnosed in females of childbearing age.
The most common presenting symptom is dyspnea.
It leads to cor pulmonale and hepatic congestion. Physical exam findings
include pitting peripheral edema, hepatomegaly, jugular venous distension,
and hepatojugular reflux.
Echocardiography shows right ventricular dilatation and septal flattening.
Cardiac catheterization of the right heart to measure accurate pressures is
the standard gold test. It measures pulmonary artery pressure, left
ventricular filling pressure, and cardiac output. Patients with IPAH have
high mean pulmonary artery pressures (greater than 25 mmHg at rest and
greater than 30 mmHg with exercise). Pulmonary capillary wedge pressures
are normal. Pericardial effusion confers a poor prognosis.

Go to the next page if you knew the correct answer, or click the link image(s)
below to further research the concepts in this question (if desired).

Research Concepts:
Pulmonary Hypertension, Idiopathic

We update eBooks quarterly and Apps daily based on user feedback.


Please tap flag to report any questions that need improvement.
Question 633: Which of the following may indicate a bronchial tear?
Choices:
1. Pneumothorax
2. Hemothorax
3. The lung remains collapsed despite chest tube placement
4. No air leak
Answer: 3 - The lung remains collapsed despite chest tube placement
Explanations:
Bronchial tears usually result from rapid deceleration injuries. Patients are
often in respiratory distress and may have signs of a large pneumothorax.
Trauma patients who strike the steering wheel in an auto accident are at risk
for this type of injury. Blows to the neck or chokehold maneuvers can lead
to tracheobronchial injury. Trachea or mainstem bronchial injuries can
present with a hoarse voice, tachypnea, dyspnea, and crepitus palpable in
the upper chest and neck area. The airway will need close monitoring.
A bronchial tear is likely when the lung remains collapsed despite
placement of a chest tube. Also visible will be a very large air leak.
These injuries require urgent treatment. Single-lung ventilation is necessary
to protect the uninjured lung.
In the operating room, flexible bronchoscopy can confirm the diagnosis.
Stents make it possible to salvage some patients without major surgery.

Go to the next page if you knew the correct answer, or click the link image(s)
below to further research the concepts in this question (if desired).

Research Concepts:
Tracheobronchial Tear

We update eBooks quarterly and Apps daily based on user feedback.


Please tap flag to report any questions that need improvement.
Question 634: Which of the following drugs can most effectively suppress
a cough in a patient with hemoptysis?

Choices:
1. Dextromethorphan
2. Diphenhydramine
3. Codeine
4. Steroids
Answer: 3 - Codeine
Explanations:
Codeine is a weak opioid that can suppress a cough and prevent massive
hemoptysis.
Dextromethorphan is also an antitussive, but its efficacy is lower than
codeine.
Steroids do not have antitussive properties.

Go to the next page if you knew the correct answer, or click the link image(s)
below to further research the concepts in this question (if desired).

Research Concepts:
Hemoptysis

We update eBooks quarterly and Apps daily based on user feedback.


Please tap flag to report any questions that need improvement.
Question 635: Which of the following is not characteristic of acute
respiratory distress syndrome?

Choices:
1. Bilateral pulmonary opacities
2. Onset of respiratory symptoms within 1 week of the inciting event
3. Pulmonary capillary wedge pressure of less than 18 mm Hg
4. PaO2/FIO2 ratio less than 400
Answer: 4 - PaO2/FIO2 ratio less than 400
Explanations:
Acute respiratory distress syndrome (ARDS) is characterized by rapid
development of bilateral airspace disease and hypoxemia in the absence of
cardiogenic pulmonary edema. A PaO2/FIO2 on 5 cm of continuous
positive airway pressure must be less than or equal to 300 at a minimum.
ARDS is characterized by widespread lung inflammation. It is triggered by
trauma, pneumonia, and sepsis. The hallmark of ARDS is a diffuse injury to
alveolar barrier cells, surfactant dysfunction, activation of the innate
immune response, and abnormal coagulation. ARDS results in impaired gas
exchange within the lungs at the microscopic alveoli level. The syndrome
has a high mortality rate between 20% and 50%.
The signs and symptoms of ARDS can begin from within 2 hours to days of
the inciting event. Signs and symptoms include shortness of breath,
tachypnea, and hypoxia.
Generally, radiographic findings of pulmonary edema affecting both lungs
and unrelated to heart failure suggest ARDS. Ultrasound findings include
anterior subpleural consolidations, absence or reduction of lung sliding,
“spared areas” of normal parenchyma, pleural line abnormalities, and
nonhomogeneous distribution of B-lines.

Go to the next page if you knew the correct answer, or click the link image(s)
below to further research the concepts in this question (if desired).

Research Concepts:
Acute Respiratory Distress Syndrome (ARDS)

We update eBooks quarterly and Apps daily based on user feedback.


Please tap flag to report any questions that need improvement.
Question 636: Which class of drugs has been shown to be of some benefit
in patients with fat embolism syndrome?

Choices:
1. Heparin
2. Corticosteroids
3. Aspirin
4. Antibiotics
Answer: 2 - Corticosteroids
Explanations:
Corticosteroids have been widely used to treat patients with fat embolism
syndrome.
Although there are some studies showing benefit, the data are not
compelling to show their effectiveness in all patients.

Go to the next page if you knew the correct answer, or click the link image(s)
below to further research the concepts in this question (if desired).

Research Concepts:
Embolism, Fat

We update eBooks quarterly and Apps daily based on user feedback.


Please tap flag to report any questions that need improvement.
Question 637: A patient had a difficult left lung transplantation. What is the
best way to determine stenosis of the pulmonary vein?

Choices:
1. CT scan
2. MRI
3. Pulmonary angiogram
4. Transesophageal echocardiogram
Answer: 4 - Transesophageal echocardiogram
Explanations:
A transesophageal echocardiogram (TEE) is the best method to detect
stenosis of the pulmonary vein.
Based on TEE, a diameter of the pulmonary veins less than 0.5 cm, a peak
systolic flow velocity greater than 1 m/s, and a pulmonary vein-left atrial
pressure gradient greater than 10 to 12 mmHg should raise suspicion for
pulmonary venous obstruction.
A few studies show that the mortality rates tend to be much higher after
unilateral procedures and in females.
In some transplant programs, TEE is now routinely done within the first 24
to 48 hours after surgery to look for pulmonary vein stenosis.

Go to the next page if you knew the correct answer, or click the link image(s)
below to further research the concepts in this question (if desired).

Research Concepts:
Transplantation, Lung

We update eBooks quarterly and Apps daily based on user feedback.


Please tap flag to report any questions that need improvement.
Question 638: What is the narrowest portion of the upper airway in an
infant?

Choices:
1. Glottic opening
2. Inferior cricoid ring
3. Tracheal bifurcation
4. Subglottis
Answer: 2 - Inferior cricoid ring
Explanations:
The narrowest portion of the upper airway in an infant is the inferior cricoid
ring.
In adults the narrowest portion is at the glottic opening.

Go to the next page if you knew the correct answer, or click the link image(s)
below to further research the concepts in this question (if desired).

Research Concepts:
Anatomy, Neck, Cricoid Cartilage

We update eBooks quarterly and Apps daily based on user feedback.


Please tap flag to report any questions that need improvement.
Question 639: Hypoxemia and shortness of breath have become evident in
a patient 48 hours after lung transplantation. What is the most likely diagnosis?

Choices:
1. Infection
2. Post-transplant lymphoproliferative disorder
3. Reperfusion edema
4. Upper lobe fibrosis
Answer: 3 - Reperfusion edema
Explanations:
Pulmonary infections, most often from Pseudomonas and Klebsiella
species, are an intermediate complication postoperatively (8 days to 2
months).
Post-transplant lymphoproliferative disorder (PTLD) is a secondary late
complication, which manifests within the first year and is represented by a
spectrum of lymphoid neoplasms that are primarily of B cell origin.
Reperfusion edema is a noncardiogenic pulmonary edema that typically
occurs more than 24 hours after transplantation, peaks in severity on
postoperative day 4, and generally improves by the end of the week.
Possible causes include surgical trauma, donor lung ischemia, and
denervation of the donor lung.
Upper lobe fibrosis is a secondary late complication, which occurs 1 to 4
years after transplantation.

Go to the next page if you knew the correct answer, or click the link image(s)
below to further research the concepts in this question (if desired).

Research Concepts:
Reperfusion Edema

We update eBooks quarterly and Apps daily based on user feedback.


Please tap flag to report any questions that need improvement.
Question 640: An 8-year-old male is at the twenty-fifth percentile for
height and the ninety-fifth percentile for weight. His mother reports that he
snores loudly at night and his teacher complains that he often falls asleep in
class. He complains of headaches. What is the next step in evaluation?

Choices:
1. Tonsillectomy and adenoidectomy
2. Continuous positive airway pressure at night
3. Polysomnography
4. Oral corticosteroids to reduce adenoid and tonsillar hypertrophy
Answer: 3 - Polysomnography
Explanations:
The patient likely has obstructive sleep apnea (OSA).
This needs to be confirmed by polysomnography.
The treatment can be with nasal continuous positive airway pressure,
nasopharyngeal airway, or tonsillectomy and adenoidectomy.
Complications of untreated OSA in children include cor pulmonale, growth
abnormalities, and developmental abnormalities.

Go to the next page if you knew the correct answer, or click the link image(s)
below to further research the concepts in this question (if desired).

Research Concepts:
Apnea, Obstructive Sleep Apnea

We update eBooks quarterly and Apps daily based on user feedback.


Please tap flag to report any questions that need improvement.
Question 641: Which drug may be used for the treatment of severe
respiratory syncytial virus in immunocompromised children?

Choices:
1. Acyclovir
2. Oseltamivir
3. Amantadine
4. Ribavirin
Answer: 4 - Ribavirin
Explanations:
Current evidenced-based practice does not support the use of ribavirin.
Ribavirin is an antiviral with limited positive effects that has been linked to
adverse effects for patients. The risk of adverse effects to the patient and
provider likely outweigh any benefits of the drug in most situations.
However, in the case of immunocompromised patients with severe
respiratory syncytial virus (RSV), it may be considered.
It is used as an aerosol in the treatment of RSV.
It also has been shown to decrease mortality and viremia in Lassa fever.
It is used orally for hepatitis C infection.

Go to the next page if you knew the correct answer, or click the link image(s)
below to further research the concepts in this question (if desired).

Research Concepts:
Respiratory Syncytial Virus Infection (RSV)

We update eBooks quarterly and Apps daily based on user feedback.


Please tap flag to report any questions that need improvement.
Question 642: Which of the following is not an indication for surgical
repair in eventration of the diaphragm?

Choices:
1. Respirator dependence
2. Frequent respiratory tract infection
3. Adults with eventration
4. Failure to thrive
Answer: 3 - Adults with eventration
Explanations:
Asymptomatic adults with eventration do not need plication of the
diaphragm.
Young children generally do need treatment. They are prone to respiratory
tract infections and may be ventilator-dependent.

Go to the next page if you knew the correct answer, or click the link image(s)
below to further research the concepts in this question (if desired).

Research Concepts:
Diaphragm, Eventration

We update eBooks quarterly and Apps daily based on user feedback.


Please tap flag to report any questions that need improvement.
Question 643: What is the best way to differentiate acute respiratory
distress syndrome from congestive heart failure?

Choices:
1. Chest x-ray
2. Pulmonary artery catheter
3. Central venous line
4. Arterial line
Answer: 2 - Pulmonary artery catheter
Explanations:
The pulmonary artery catheter is essential for differentiating acute
respiratory distress syndrome from congestive heart failure.
In congestive heart failure, the pulmonary wedge pressure is usually
elevated. In acute respiratory distress syndrome, the pulmonary wedge
pressure is usually normal.
Most clinicians make a diagnosis of acute respiratory distress syndrome
based on clinical features and do not use a pulmonary artery catheter.

Go to the next page if you knew the correct answer, or click the link image(s)
below to further research the concepts in this question (if desired).

Research Concepts:
Heart Failure, Congestive (CHF)

We update eBooks quarterly and Apps daily based on user feedback.


Please tap flag to report any questions that need improvement.
Question 644: In comparison to intermittent mandatory ventilation and T-
piece, Pressure support ventilation differs in which of the following ways?

Choices:
1. Each breath is patient-triggered, pressure-limited, and flow-cycled
2. Each patient breath is supplemented with a set amount of positive pressure
3. In comparison to intermittent mandatory ventilation and T-piece trials,
pressure support has been shown to have more weaning success
4. All of the above
Answer: 4 - All of the above
Explanations:
Patients must be able to initiate a spontaneous breath in pressure support
mode which in turn the ventilator will produce a preset pressure to
supplement the patient’s effort.
Pressure may be adjusted to decrease the patient's work of breathing.
When utilized in weaning, pressure may be titrated up and down based on
the patient’s ability. Setting the pressure support on the lowest amount
necessary has been proven to strengthen the respiratory drive.
Studies have shown that intermittent mandatory ventilation (IMV) methods
do not reduce muscle fatigue.

Go to the next page if you knew the correct answer, or click the link image(s)
below to further research the concepts in this question (if desired).

Research Concepts:
Pressure Support

We update eBooks quarterly and Apps daily based on user feedback.


Please tap flag to report any questions that need improvement.
Question 645: Patients on synchronized intermittent mandatory ventilation
(SIMV) exhibit which of the following?

Choices:
1. Hypocarbia
2. Hypercarbia
3. Increased work of breathing
4. Hypoxia
Answer: 3 - Increased work of breathing
Explanations:
Presetting a respiratory rate can prevent hypercarbia in a patient on
synchronized intermittent mandatory ventilation (SIMV).
The workload of breathing is increased with SIMV by spontaneously
breathing through a high resistant circuit which increases the work of
respiratory muscles.
SIMV guarantees a specific number of breaths, but unlike assist-control
ventilation, patient breaths are partially their own which reduces the risk of
hyperinflation or alkalosis.
Pressure support in addition to spontaneous breaths may reduce the work of
breathing. SIMV decreases cardiac output in patients with left-ventricular
dysfunction.

Go to the next page if you knew the correct answer, or click the link image(s)
below to further research the concepts in this question (if desired).

Research Concepts:
Ventilation, Synchronized Intermittent Mandatory

We update eBooks quarterly and Apps daily based on user feedback.


Please tap flag to report any questions that need improvement.
Question 646: Which neuromuscular blocking agent can cause
hyperkalemia?

Choices:
1. Rocuronium
2. Atracurium
3. Norcurium
4. Succinylcholine
Answer: 4 - Succinylcholine
Explanations:
Succinylcholine is used for rapid sequence intubation as is very short-
acting. It is a depolarizing skeletal muscle relaxant, It bonds with motor
endplate cholinergic receptors producing depolarization.
The neuromuscular block causes flaccid paralysis after initial
fasciculations. Succinylcholine can worsen the effects of hyperkalemia.
It must be used with caution in patients with burns, traumatic wounds, crush
injuries, or with hyperkalemia. It also may cause problems with those
receiving digoxin or quinidine. Arrhythmias or cardiac arrest are possible.
Caution should be employed when using it on patients with renal,
pulmonary cardiovascular, hepatic or cardiac disease.

Go to the next page if you knew the correct answer, or click the link image(s)
below to further research the concepts in this question (if desired).

Research Concepts:
Neuromuscular Blocking Agents

We update eBooks quarterly and Apps daily based on user feedback.


Please tap flag to report any questions that need improvement.
Question 647: Which of the following is not a characteristic of
pneumothorax in HIV patients?

Choices:
1. Can be initial presentation of AIDS
2. Pneumothorax may be bilateral
3. Recurrences are common
4. The pneumothorax usually resolves with a thoracostomy tube
Answer: 4 - The pneumothorax usually resolves with a thoracostomy tube
Explanations:
Pneumothorax in HIV patients can be difficult to treat with conservative
management.
Most fail to respond to simple thoracostomy and pleurodesis.
VATs has been carried out with pleurodesis. Some success has been
achieved by resection of the destroyed lung.
Despite this, HIV patients have a high recurrence rate. Usually the
pneumothorax predicts a downhill course.

Go to the next page if you knew the correct answer, or click the link image(s)
below to further research the concepts in this question (if desired).

Research Concepts:
HIV, Disease

We update eBooks quarterly and Apps daily based on user feedback.


Please tap flag to report any questions that need improvement.
Question 648: A 5-year-old healthy boy has been exposed to
Mycobacterium tuberculosis by his aunt who is now hospitalized with cavitary
pulmonary tuberculosis comes to your office for evaluation. You place a
Mantoux test and order a chest radiograph. The Mantoux test is non-reactive but
the chest radiograph is abnormal, showing mediastinal adenopathy with a left
upper lobe segmental lesion. What is the most likely explanation for the negative
Mantoux skin test?

Choices:
1. Receipt of measles vaccine 16 weeks earlier
2. Malnutrition
3. Selective anergy to PPD
4. The child's young age
Answer: 3 - Selective anergy to PPD
Explanations:
A negative Mantoux test does not exclude the diagnosis of tuberculosis
disease such as pulmonary tuberculosis, or latent tuberculosis infection.
Approximately 10% of immunocompetent children with culture-proven
tuberculosis do not react initially to a Mantoux test.
Young age (younger than 1 year), malnutrition and receipt of measles
vaccine can increase the likelihood for a negative Mantoux test.
The effect of measles vaccine on tuberculin reactivity is temporary and
should not last for more than 4 to 6 weeks after vaccination.

Go to the next page if you knew the correct answer, or click the link image(s)
below to further research the concepts in this question (if desired).

Research Concepts:
Tuberculosis

We update eBooks quarterly and Apps daily based on user feedback.


Please tap flag to report any questions that need improvement.
Question 649: Which of the following groups of patients has the highest
risk of death while waiting for a transplant?

Choices:
1. Patients with chronic obstructive pulmonary disease
2. Patients with primary pulmonary hypertension
3. Patients with interstitial lung disease
4. Patients with emphysema
Answer: 2 - Patients with primary pulmonary hypertension
Explanations:
Patients with primary pulmonary hypertension have the highest mortality
while waiting for a transplant.
Depression of cardiac function and elevated right ventricular pressures are
predictors of death.
However, the shortage of organ donors means even longer waiting times
and a high mortality.

Go to the next page if you knew the correct answer, or click the link image(s)
below to further research the concepts in this question (if desired).

Research Concepts:
Transplantation, Lung

We update eBooks quarterly and Apps daily based on user feedback.


Please tap flag to report any questions that need improvement.
Question 650: A patient is brought to the ER in coma after being involved
in a MVA. He is posturing. Prior to being intubated his respiratory pattern
displays a group of quick shallow inspirations followed by regular or irregular
periods of apnea which is repeated. What is the most likely diagnosis?

Choices:
1. Central neurogenic hyperventilation
2. Kussmaul respiration
3. Biot respiration
4. Cheyne-Stokes respiration
Answer: 3 - Biot respiration
Explanations:
Biot respiration also called ataxic breathing is usually due some type of
damage to the medulla. The insult could be from superior pressure due to
edema and hematoma, stroke, or intrinsic damage from trauma or tumor.
Cheyne-Stokes respirations are progressively deeper and sometime faster
breathing followed by gradual decrease resulting in a temporary stop. It is
felt to be due to unstable feedback in the respiratory control center. Causes
could be heart failure, TBI, and stroke.
Central neurogenic hyperventilation is very rare. It is deep and rapid breaths
of greater than 25/min. It is generally a response to brainstem damage.
Though frequently associated with the pons, it can occur as a result of
injury to other areas.
Kussmaul respiration is deep labored hyperventilation due to a severe
metabolic acidosis.

Go to the next page if you knew the correct answer, or click the link image(s)
below to further research the concepts in this question (if desired).

Research Concepts:
Abnormal Respirations

We update eBooks quarterly and Apps daily based on user feedback.


Please tap flag to report any questions that need improvement.
About Stat Pearls

Stat Pearls develops Kindle eBooks for Medical Professionals.


Here is a list of other books available on Kindle. Please visit our website at
http://www.StatPearls.com for more information.
Allied Health Specialties

Alternative-Acupuncture and Point Location


Anesthesia Technician
Apheresis Technician
Athletic Training
Audiology
Lab-Blood Banking Technician
Cardiovascular Technician
Certified Clinical Nutritionist (CCN)
Certified Medical Assistant
Certified Nursing Assistant
Lab-Chemistry Technologist
Chiropractic-NBCE Part I
Neuropsychology
Hemodialysis Technologist
Registered Dietitian
EKG Technician
EMT-Basic
AEMT-Intermediate
EMT-Paramedic
Exercise Physiology
Hemapheresis Practitioner
Lab-Hematology Scientist
Holistic Medicine
Medical Office Manager
Medical Billing and Coding
Lab-Medical Assistant
Lab-Medical Technologist
Lab-Medical Technician
Medical Physics
Mental Health Technician
Lab-Microbiology Technologist
Lab-Molecular Pathology
Occupational Therapist
Optometry-Clinical Science
Orthotics And Prosthetics
Patient Care Technician
Personal Training
Lab-Phlebotomy Technician
Physical Therapy
Psychologist
Radiation Technology
Radiology Technology
Radiology Tech-Densitometry
Radiology Tech-Cardiovascular
Radiology Technology-CT
Radiology Technology-Dosimetry
Radiology Tech-Mammography
Radiology Technology-MRI
Radiology Tech-Nuclear Medicine
Radiology Technology-Quality
Radiology Tech-Cardiac US
Radiology Technology-Vascular
Respiratory Therapist
Social Work
Speech Pathology
Radiology Tech-Ultrasound
EMT-Flight Paramedic
EMT-Critical Care Paramedic
EMT-Tactical Paramedic
Lab-Molecular Biology Tech
PT-Cardiovascular/Pulmonary
Physical Therapy-Neurology
Physical Therapy-Geriatrics
Physical Therapy-Orthopaedics
Physical Therapy-Pediatrics
Physical Therapy-Sports
PT-Women's Health
Healthcare Technology Manager
Medication Aide Certification
Lab-Chemistry Specialist
Lab-Blood Banking Specialist
Lab-Microbiology Specialist
Lab-Hematology Specialist
Lab-Microbiology Technician
Chiropractic-NBCE Part II
EMR-First Medical Responder
EMT-Community Paramedic
Certified Medical Administrative Assistant
Certified Electronic Health Record Specialist
Lab-Clinical Scientist
Lab-Clinical Technician
Occupational Therapy Assistant
Certified Surgical 1st Assistant
Psychiatric Technician
Chiropractic-NBCE SPEC

Clinical Nurse Specialties


CNS-Adult Health
CNS-Adult Psychiatric Health
CNS-Child/Adolescent Psych
CNS-Diabetes Management
CNS-Adult-Gerontology
CNS-Home Health
CNS-Pediatric
CNS-Public Community Health
CNS-Oncology

Dental Doctor Specialties

NBDE Part II-Operative Dentist


NBDE Part II-Pharmacology
NBDE Part II-Behavioral Science
NBDE Part II-Public Health
NBDE Part I-Anatomy
NBDE Part I-Embryology
NBDE Part I-Histology
NBDE Part I-Biochemistry
NBDE Part I-Physiology
NBDE Part I-Microbiology
NBDE Part I-Pathology
NBDE Part I-Occlusion Anatomy
Dental-Pediatrics
Dental-Prosthodontics

Graduate Specialties

ACLS
PALS
International Trauma-ITLS

Medical Doctor Specialties

Adolescent Medicine
Aerospace Medicine
Allergy and Immunology
Ambulatory and Urgent Care
Anesthesiology
Anesthesiology-Cardiothoracic
Nurse Anesthetist
Anesthesiology-Pediatric
Advanced Trauma-Support
Cardiology
Cardiology-Electrophysiology
Cardiology-Failure/Transplant
Cardiology-Interventional
Critical Care
Dermatology
Emergency Medicine
Endocrinology
Family Medicine
Gastroenterology
Genetics
Geriatrics
Hematology
Hospice/Palliative Medicine
Infectious Disease
Internal Medicine
Nephrology
Neurology
Neurology-Headache
Neurology-Neurodevelopmental
Neurology-Neuromuscular
Obesity Medicine
Occupational Medicine
Oncology
Pain Medicine
Pathology-Anatomic
Pediatric
Pediatric-Cardiology
Pediatric-Child Abuse
Pediatric-Critical Care
Pediatric-Developmental
Pediatric-Emergency Medicine
Pediatric-Endocrinology
Pediatric-Gastroenterology
Pediatric-Hematology/Oncology
Pediatric-Infectious Diseases
Pediatric-Neonatal/Perinatal
Pediatric-Nephrology
Pediatric-Neurology
Pediatric-Pulmonology
Pediatric-Rehabilitation
Pediatric-Rheumatology
Physical Medicine
Psychiatry
Psychiatry-Addiction
Psychiatry-Child/Adolescent
Psychiatry-Forensic
Psychiatry-Geriatric
Psychosomatic Medicine
Public and Preventive Medicine
Pulmonary
Radiation Oncology
Radiology
Radiology-Abdominal
Radiology-Musculoskeletal
Radiology-Neuroradiology
Nuclear Medicine
Radiology-Pediatric
Radiology-Physics
Radiology-Interventional
Rheumatology
SPEX
Spinal Cord Injury
Sports Medicine
Surgery-Cardiac
Surgery-Colon and Rectal
Surgery-Complex Oncology
Surgery-Craniofacial
Surgery-General
Surgery-Gynecologic Oncology
Surgery-Hand
Surgery-Maternal and Fetal
Surgery-Neurosurgery
Surgery-Obstetrics/Gynecology
Surgery-Ophthalmology
Surgery-Oral
Surgery-Orthopaedic
Surgery-Otolaryngology
Surgery-Pediatric
Surgery-Plastic
Surgery-Reproductive
Surgery-Thoracic
Surgery-Urologic
Surgery-Urologic Pediatric
Surgery-Vascular
Toxicology
Transplant Hepatology
Undersea and Hyperbaric
Radiology-Reproductive
Radiology-Cardiac
Radiology-Gastrointestinal
Radiology-Breast
Radiology-Radioisotope Safety
Radiology-Urinary
Surgery-Podiatry APMLE Part 1
Surgery-Podiatry APMLE Part 2
Surgery-Podiatry APMLE Part 3
Radiology-Thoracic
Radiology-Ultrasound
Emergency Ultrasound
Emergency-Medical Services
UK Professional-Assessment Part I
Australian Medical-Adaptive
National Assessment-Canada
Medical Canada-Evaluating Exam
Medical Canada-Qualifying Pt I
Compliance Physician
Clinical-Tropical Medicine
Clinical-Informatics
Dermatopathology
Dermatology-Pediatric
Dermatology-Procedural
Pathology-Laboratory Medicine
Pathology-Transfusion Medicine
Pathology-Chemistry
Pathology-Cytopathology
Pathology-Forensics
Pathology-Hematology
Pathology-Microbiology
Pathology-Molecular Genetic
Pathology-Neuropathology
Pathology-Pediatric
Phlebology
Neurology-Critical Care
Pediatric-Hospital Medicine
Neurology-Epilepsy
Neurology-Neurophysiology
Wound
Surgery-Oculoplastics
Brain Injury Medicine
Neurology-Vascular
Surgery-Podiatry Cert Medicine
Surgery-Podiatry Cert
Optometry-Basic Science
Optometry-Advanced Medical Care
Osteopathic Neuromusculoskeletal Medicine
Fitness, Diet, and Nutrition
Environmental and Wilderness

Medical Student Specialties

USMLE Step 1
USMLE Step 2
USMLE Step 3
COMLEX Level 1
COMLEX Level 2
COMLEX Level 3
Adult Ambulatory-Medical Student
Clinical Neurology-Medical Student
Family Medicine-Medical Student
Internal Medicine-Med Student
Ob Gyn-Medical Student
Pediatrics-Medical Student
Psychiatry-Medical Student
Surgery-Medical Student
Emergency Medicine-Med Student
Medicine Advanced-Med Student
Pediatrics Advanced-Med Student
Surgery Advanced-Med Student
Basic Science-Medical Student
Clinical Science-Medical Student
Clinical Diagnosis-Medical Student
India Medical Exam (AIPGMEE)
Dermatology-Medical Student
Clinical Skills-Medical Student
Anatomy-Medical Student
Behavioral Sci-Medical Student
Biochemistry-Medical Student
Embryology-Medical Student
Genetics-Medical Student
Histology-Medical Student
Microbiology-Medical Student
Neuroscience-Medical Student
Path Clin Lab-Medical Student
Pathology-Medical Student
Pharmacology-Medical Student
Physiology-Medical Student
Philippine Physician Exam (PLE)

Nurse Practitioner Specialties

NP-Adult Acute Gerontology


NP-Adult
NP-Diabetes Management
NP-Family
NP-Psychiatric/Mental Health
NP-Gerontology Primary Care
NP-Midwife
NP-Pediatric Primary Care
NP-Emergency Medicine
NP-Women's Health Care
NP-Acute Care Pediatric
NP-Neonatal
NP-Dermatology
NP-Addiction
NP-Genetics
NP-Nephrology
NP-Oncology
NP-Orthopedic
NP-Advanced Health Assessment
NP-Advanced Pharmacology
NP-Advanced Pathophysiology
NP-Advanced Physiology

Physician Assistant Specialties

PA-PANCE
PA-PANRE
PA-Cardiovascular/Thoracic
PA-Emergency Medicine
PA-Hospital Medicine
PA-Nephrology
PA-Orthopedic
PA-Pediatric
PA-Psychiatry

Pharmacist Specialties

Pharmacy-NAPLEX Review
Pharmacy-Technician (PTCB)
Pharmacy-Ambulatory Care
Pharmacy-Critical Care
Pharmacy-Nuclear Medicine
Pharmacy-Nutrition Support
Pharmacy-Oncology
Pharmacy-Pediatric
Pharmacy-Pharmacotherapy
Pharmacy-Psychiatric
Pharmacy-Infectious Disease
Pharmacy-Geriatric
Pharmacy-Cardiac
Pharmacy-Transplant
Pharmacy-Applied Toxicology

Nursing Specialties

Nurse-LPN/LVN NCLEX PN
Nurse-NCLEX RN
Nurse-Addiction (CARN)
Nurse-Administrator NE/NEA
Nurse-AIDS/HIV (ACRN/AACRN)
Nurse-Ambulatory Care
Nurse-Perianesthesia CAPA/CPAN
Nurse-Anticoagulation
Nurse-Bariatric (CBN)
Nurse-Breast Care
Nurse-Cardiovascular CVN/CVRN
Nurse-Case Management
Nurse-Continence Care
Nurse-Corrections (CCN)
Nurse-Critical Care (CCRN)
Nurse-Dermatology (DNC)
Nurse-Developmental Disability
Nurse-Diabetes Educator (CDE)
Nurse-Emergency Certification
Nurse-Flight Registered (CFRN)
Nurse-Foot/Nail Care (CFCN)
Nurse-Gastroenterology (CGRN)
Nurse-Genetics Clinical (GCN)
Nurse-Gerontology (NCA)
Nurse-Hemodialysis (CHN)
Nurse-Holistic
Nurse-Hospice/Palliative CHPN
Nurse-Hyperbaric (CHRN)
Nurse-Infection Control (CIC)
Nurse-Informatics
Nurse-Infusion (CRNI)
Nurse-Lactation (CLA)
Nurse-Low Risk Neonatal (LRN)
Nurse-Managed Care (CMCN)
Nurse-Maternal Newborn (MNN)
Nurse-Medical/Surgical
Nurse-Neonatal Intensive Care
Nurse-Nephrology (CNN)
Nurse-Neuroscience (CNRN)
Nurse-Occupational (COHN)
Nurse-Oncology (OCN)
Nurse-Operating Room (CNOR)
Nurse-Ophthalmology (CRNO)
Nurse-Orthopedic (ONC)
Nurse-Otolaryngology (CORLN)
Nurse-Pain Management (PMCN)
Nurse-Pediatric (CPN)
Nurse-Pediatric Critical Care
Nurse-Pediatric Emergency CPEN
Nurse-Pediatric Hem Onc (CPHON)
Nurse-Inpatient Obstetric RNC
Nurse-Peritoneal Dialysis (CPDN)
Nurse-Plastic Surgery (CPSN)
Nurse-Professional Development
Nurse-Healthcare Quality
Nurse-Progressive Care (PCCN)
Nurse-Psychiatric Health
Nurse-Radiologic (CRN)
Nurse-Rehabilitation Registered
Nurse-Risk Management (CPHRM)
Nurse-School (NCSN)
Nurse-Sexual Assault (SANE)
Nurse-Transplant (CCTN)
Nurse-Transport (CTRN)
Nurse-Trauma Certification
Nurse-Urologic Registered
Nurse-Wound Care (CWCN)
Nurse-Eating Disorders (CEDRN)
Nurse-Neonatal Transport C-NPT
Nurse-Trauma (TCRN)
Nurse-Emergency Pediatric Certification
Nurse-Certified Educator
Nurse-Prehospital (PA PHRN)
Nurse-Mobile Intensive Care MICN
Nurse-Forensic (CFN)
Nurse-Stroke Certified (SC-RN)
Nurse-Community Health
Nurse-Adult Medical Surgical RN
Nurse-Maternal Newborn PN
Nurse-Child Health RN
Nurse-Mental Health RN
Nurse-Fundamentals
Nurse-Mental Health PN
Nurse-Pharmacology RN
Nurse-Professional and Ethics
Nurse-Calculations
Nurse-Maternal Newborn RN
Nurse-Pharmacology PN
Nurse-Adult Medical Surgical PN
Nurse-Child Health PN
Nurse-Anatomy
Nurse-Physiology
Nurse-Microbiology
Nurse-Preceptor Practice
Nurse-Wound Care and Ostomy (OMS)
Nurse-Wound, Ostomy, and Continence (CWOCN)
Nurse-Clinical Nurse Leader
Nurse-Nutrition
Nurse-Laboratory Testing
Nurse - Elder Adult Care
Nurse - Critical Care/Rhythms
Nurse-Pathophysiology
Nurse-Health Assessment

You might also like